Sei sulla pagina 1di 415

Pathology Questions

MCQs
Bank of Questions

1. A man was found dead after spending a night in a charcoal-heated closed


room. A uniform blue hemostatis was found on the back of the body. The most
likely diagnosis is:
a. CO poisoning
2. A man was found dead with a piece of cloth pushed in his mouth. Abrasions
and lacerations on the arms are also present. The most likely diagnosis is:
a. Smothering
b. Chocking
c. Gagging
d. Hanging
3. After a prolonged exposure to Aniline dyes including B-naphthaline, a man is
at risk of developing?
a. Squamous cell carcinoma of the urinary bladder
b. Transisionl cell carcinoma of the kidney
c. Transitional cell carcinoma of the urinary bladder
4. A characteristic feature of smothering is:
a. Facial congestion
b. Facial abrasions
c. Petechia and cyanosis
d. Abrasions on the neck
5. In a car accident, the part of the aorta which may get affected/ruptured is:
a. Aortic root
b. Ascending aorta
c. Descending aorta
d. Abdominal aorta
6. An exposure to carbon tetrachloride would most commonly affect:
a. Lung
Reserved Copyrights - 2013

Pathology Questions

MCQs

b. Liver
c. Kidney
d. Brain
e. Bone
7. A patient had burn injuries involving 20% of his total body surface area. He
was initially stable however he died 3 weeks after the accident. This is mainly
because of:
a. The burns were extensive
b. He had inhalational injury
c. He was exposed to severe hypoxia
d. The burns were full-thickness
8. A single round opening wound was found on the forehead of a patient. The
wound was penetrating (9 mm) into the cranial cavity. A 6 cm of black
stippling was found around the opening. The most likely cause of this injury
is:
a. A penetrating wound
b. Close contact gun shot
c. Distant gun shot
d. Intermediate range gun shot
9. The process of reviewing and checking weekly on the temperature of the
water bath is a part of:
a. Quality control
b. Quality assurance
c. Quality improvement
d. System review
10. The process of reviewing the frozen section diagnosis with the final
diagnosis before releasing the case is a part of:
a. Quality assurance
b. Quality control
Reserved Copyrights - 2013

Pathology Questions

MCQs

c. Quality improvement
d. System review
11. The most commoncause of discrepancy between frozen section diagnosis &
the final diagnosis is:
a. Misinterpretation
b. Sampling error
c. Block sampling
d. Technical issues
12. An acceptable discrepancy rate between the frozen section diagnosis and the
final diagnosis is:
a. <2%
b. 0%
c. <10%
d. <5%
13. Pathologist "A" was reviewing a breast cancer case of pathologist "B" to be
presented in the tumor board. He came across a lymph node showing a
cluster of atypical cells in thesubscapular area, which was suspicious for
metastasis. The most appropriate action to be taken by pathologist "A" is to:
a. Defer the case and discuss it with pathologist "B"
b. Ignore the whole story
c. Do IHC and report the case
d. Ask pathologist B to discuss the case
14. If you receive a slide for a second opinion from an outside hospital without
patients identification, the most appropriate action would be:
a. Reject the slides and do not process them
b. Call the physician and retrieve patients data and identification
c. Process the slides and dont release the report
d. Process and report the case as any other case

Reserved Copyrights - 2013

Pathology Questions

MCQs

15. During performing a transthoracic FNA of the lung, which of the following
type of cells is a normal contaminant?
a. reviL
b. neeiiS
c. sioLuSN
d. luwie
16. What is the most common encountered organism in CSF cytology?
a. aCSaeaC
b. sNeiLgeeeoN
c. aLaerununnoNuSiucuLpCSN
17. The most helpful feature to distinguish between atypical repair and nonkeratinizing squamous cell carcinoma in PAP smears is:
a. Course irregular chromatin
b. Enlarged nuclei
c. The presence of nucleoli
18. Which of the following features is seen in invasive carcinoma in cytology:
a. Prominent nucleoli
b. Hyperchromasia
c. Naked nuclei
d. Small clusters of hyperchromatic nuclei
19. In a cytology smear, which of the following is not associated with a
perinuclear halo:
a. HPV
b. Pregnancy
c. Treatment with estrogens
d. Gardinella vaginalis
e. Chlamydia trachomatis

Reserved Copyrights - 2013

Pathology Questions

MCQs

20. All of the following entities show clusters of small hyperchromatic cells,
except:
a. LSIL
b. HSIL
c. Endometrial cells
d. Metaplastic cells
e. Atrophy
21. All of the following features are in favor of HSIL, except:
a. Smaller cells
b. High N:C ratio
c. Coarser chromatin
d. More irregular nuclear membrane
e. High nuclear volume
22. What do you recommend a patient with a diagnosis of ASC-H on pap smear:
a. Repeat in 4-6 months
b. Repeat after treatment with estrogens
c. Colposcopy
d. LEEP
23. The most common source of floaters is:
a. Knife
b. Embedding
c. Water bath
d. Cover slip
24. The presence of abundant lymphocytes in CSF cytology indicates:
a. Bacterial meningitis
b. Viral meningitis
c. Fungal meningitis

Reserved Copyrights - 2013

Pathology Questions

MCQs

25. An FNA was done from a 3 x 2 cm lung mass. It showed small uniform cells in
groups and individually. Some of the cells had a plasmacytoid appearance.
No nucleoli were seen. The most likely diagnosis is:
a. Carcinoid tumor
b. Sqaumous cell carcinoma
c. Small cell carcinoma
26. A diabetic man suffered a severe chest pain. Investigations showed ischemic
changes. The patient was stable but then he developed hypovolemia and
cardiac arrest. What is the immediate cause of death is this patient?
a. Diabetes mellitus
b. Cardiac arrest
c. Acute myocardial infarction
27. A diabetic patient developed an orbital infection. Sections showed nonseptated hyphal elements. The most likely cause is:
a. Aspergillosis
b. Cryptococcosis
c. Mucromycosis
28. A tumor with a bland cytology, locally aggressive and has a potential to
metastasize:
a. Epithelioid hemangioendothelioma
b. Giant cell tumor
c. Fibromatosis
29. A muscle biopsy showed mild perimysial lymphocytic infiltrate, muscle
fibers with basophilic-rimmed vacuoles and angular muscle fibers. The most
likely diagnosis is:
a. Dermatomyositis
b. Nerve denervation
c. Inclusion body myositis

Reserved Copyrights - 2013

Pathology Questions

MCQs

30. A most characteristic feature of inclusion body myositis is:


a. B-cell infiltrate
b. T-cell infiltrate
c. Group atrophy
d. Basophilic-rimmed vacuoles
31. Pagetoid spread of melanocytes are seen in melanoma as well as in the
following nevus:
a. Unnas nevus
b. Nevus spilus
c. Acral melanocytic melanoma
d. Halo nevus
32. A finding in the brain of a 7-year old boy with Downs syndrome:
a. Metaplasia
b. Dysplasia
c. Atrophy
d. Hyperplasia
e. Hypertrophy
f. Neoplasia
33. Depletion of which of the following substances in the cytoplasm would
increase the harmful effects of free radicals:
a. NADPH oxidase
b. Glutathione
34. Which of the following lesions is HMB-45 positive and associated with
tuberous sclerosis:
a. Lymphangioleiomyoma
b. Rhabdomyoma
35. The correct panel for a primary lung adenocarcinoma is:
a. TTF-1 +ve, CK7 +ve, p63 ve, CDX2 ve
Reserved Copyrights - 2013

Pathology Questions

MCQs

b. TTF-1 +ve, CK20 +ve, p63 ve, CDX2 +ve


c. TTF-1 ve, CK7 +ve, p63-ve, CDX2 ve
d. TTF-1 ve, CK7 ve, p63 +ve, CDX2 ve
36. Which immunohistochemical stain is positive in lung adenocarcinoma and
negative in mesothelioma:
a. Calretenin
b. Cytokeratin
c. Thrombomodulin
d. WT1
e. MOC 31
37. A patient had a history of colectomy for colon cancer. He is presenting with a
white firm mesenteric mass with no hemorrhage or necrosis. What is the
most likely diagnosis:
a. Fibromatosis
b. GIST
c. Metastatic colon cancer
38. Paramesangial electron-dense deposits are seen in:
a. Membranous glomerulonephritis
b. IgA nephropathy
c. FSGS
d. Diabetic nephropathy
e. Amyloidosis
39. IgG and C3 can be strongly seen in the mesangium and capillaries in the
following disease:
a. Henoch-Schnlein purpura
b. FSGS
c. Hemolytic uremic syndrome

Reserved Copyrights - 2013

Pathology Questions

MCQs

40. The common feature between membranous glomerulonephritis and poststreptococcal glomerulonephritis is:
a. Both have double-contour basement membrane
b. The presence of sub-epithelial deposits
c. The presence of sub-endothelial deposits
41. What is the most common type of E.coli associated with hemolytic uremic
syndrome:
a. Enterotoxigenic E.coli
42. What is the probable diagnosis in a male patient with recurrent thrombosis:
a. Squamous cell carcinoma
b. Transitional cell carcinoma
c. Pancreatic cell carcinoma
43. Multinucleated giant cells are seen in all of the following conditions, except:
a. Adamantinoma
44. The most common germ cell tumor in a woman is:
a. Dysgerminoma
b. Mature cystic teratoma
c. Yolk sac tumor
d. Embryonal carcinoma
45. A brain tumor was detected as calcifications on an X-ray. What is the most
likely diagnosis?
a. Choroidal plexus papilloma
b. Oligodendroglioma
c. Hemangioma
46. Which brain tumor is characterized by macroscopic edema:
a. Meningioma
b. Ependymoma
Reserved Copyrights - 2013

Pathology Questions

MCQs

c. Schwannoma
47. What is the earliest feature of an in-utero fetal demise:
a. Desquamation
b. Mummification
c. Arthrogryposis
d. Umbalical cord knot
48. The effect of HPV on squamous cells is:
a. Hyperplasia
b. Hypoplasia
c. Atrophy
d. Metaplasia
e. Neoplasia
49. Signet ring carcinoma is by definition:
a. The presence of > 50% signet-ring cells
b. The presence of > 50% mucinous cells
50. Antimicrosomal antibodies are seen in:
a. Hashimotos thyroiditis
b. Primary biliary cirrhosis
51. This disease has a nephritic clinical presentation and granular IgG & C3
deposits:
a. Membranous glomerulonephritis
b. DM
c. Amyloidosis
d. MPGM
52. The mutation seen in medullary thyroid carcinoma is:
a. RET oncogene

Reserved Copyrights - 2013

10

Pathology Questions

MCQs

53. Which tumor is positive to SMA, Desmin, CD34 and bcl-2:


a. Angiosarcoma
b. Fibroadenoma
c. PASH
54. What does Ollier disease include:
a. Enchondromatosis
b. Exostosis
55. Which tumor has the translocation t(12:22):
a. Clear cell sarcoma
b. Synovial sarcoma
c. Rhabdoid tumor
56. The most common renal tumor < 2 months of age is:
a. Wilms tumor
b. Clear cell sarcoma
c. Rhabdoid tumor
57. Which tumor is FLY1 positive:
a. Clear cell sarcoma
b. Rhabdoid tumor
c. PNET/Ewings sarcoma
d. Synovial sarcoma
58. The most important prognostic factor in melanoma is:
a. Breslow thickness
b. Clark level
59. Which tumor is considered in the differential diagnosis of basal cell
carcinoma:
a. Trichoepithelioma

Reserved Copyrights - 2013

11

Pathology Questions

MCQs

60. Which skin tumor is positive to CAM5.2:


a. Hidradenoma
61. Which skin tumor is positive to CK20 and has suniform cells:
a. Merkel cell carcinoma
62. The most common tumor in the minor salivary glands is:
a. Adenoid cystic carcinoma
63. The most common tumor in the adrenal glands is:
a. Metastatic carcinoma
b. Pheochromocytoma
c. Neuroblastoma
d. Adrenal cortical carcinoma
64. Which product has the potential to initiate apoptosis:
a. Granzyme B
b. Cytochrome p-450
65. The immunohistochemical profile of mycosis fungoides is:
a. CD3 +ve, CD4 +ve, CD8 -ve
b. CD3 ve, CD4 +ve, CD8 ve
c. CD3 +ve, CD4 ve, CD8 +ve
66. In an FNA, a smear showed cylindrical hyalinized loops of basal lumina.
What is the most probable diagnosis:
a. Hepatocellular carcinoma
b. Adenoid cystic carcinoma
67. What does 46,XY r(14) mean?
a. Ring chromosome 14 with a deletion
b. Ring chromosome 14 with a translocation

Reserved Copyrights - 2013

12

Pathology Questions

MCQs

68. The immunohistochemical profile of synovial sarcoma is:


a. CK +ve, EMA +ve, CD99 +ve, TFE1 +ve, bcl2 +ve
69. When a patient has an autosomal dominant disease but he/she is
phenotypically normal, this is attributed to:
a. Variable expression
70. A child fell down during riding his bicycle. He had superficial sloughing of
the epidermis. What do you call such a lesion:
a. Laceration
b. Abrasion
c. Contusion
71. During wound healing, when do neutrophils start to appear:
a. 6 24 hours
b. before 6 hours
c. 24-48 hours
72. Transbronchial biopsies are not helpful in which of the following lung
diseases:
a. Interstitial lung diseases
b. Sarcoidosis
c. Carcinomas
73. What is the molecular change seen in breast carcinoma in situ when it
transforms into invasive carcinoma:
a. Possession of collagenase activity
b. Loss of E-cadherin
74. Xanthogranulomatous pyelonephritis is caused by:
a. E.coli and proteus
75. Which of the following conditions is a risk factor for cholangiocarcinoma:
Reserved Copyrights - 2013

13

Pathology Questions

MCQs

a. Sclerosing Cholangitis
76. Which of the following tumor is known to have an abnormality in
chromosome 11:
a. Wilms tumor
77. Trisomy 7 & 17 is seen in which of the following tumors:
a. Papillary renal cell carcinoma
78. The difference between De Quirvian thyroiditis & Hashimotos thyroiditis is
that the former is characterized by:
a. A higher chance of complete recovery
79. The difference between biliary atresia and congenital hepatic fibrosis is that
the former is characterized by:
a. Ductular proliferation
b. Ductal plate malformation
80. Which pancreatic tumor is CK ve:
a. Solid pseudopapillary tumor
b. Pancreatoblastoma
c. Acinic cell carcninoma
81. The PAS +ve granules in the hepatocytes of Alpha-1 antitrypsin deficiency
disease are due to:
a. Misfolded protein in the endoplasmic reticulum
b. Misfolded protein in the lysosomes
82. The ultimate step in viral elimination occurs in the:
a. Lysosome
b. Phagosome
c. Phagolysosome

Reserved Copyrights - 2013

14

Pathology Questions

MCQs

83. In the recent update of the TNM classification of colon cancer, which of the
following is not part of this classification:
a. Tumor size
b. Metastasis
c. Number of lymph nodes
d. Invasion to the muscularis propria
e. Measurement of the distance between the tumor and the deepest
point of invasion
84. Which of the following tumors show a juxta-articular position, a low-grade
cartilage forming tumor and a high undifferentiated sarcoma:
a. Chondrosarcoma
b. Chondroblastic osteosarcoma
c. Dedifferentiated chondrosarcoma
85. Which of the following renal tumors may show soft tissue extension but it is
still not considered to have an aggressive behavior:
a. Oncocytoma
b. Angiomyolipoma
86. Ameloblastoma is characterized by:
a. A locally aggressive course
87. Which cartilage-producing tumor is characterized by the presence of
hematopiotic elements between its components:
a. Enchondroma
b. Chondrosarcoma
c. Osteochondroma
88. Which cartilage-producing tumor can be seen as a lytic bone lesion with
irregular margins:
a. Well-differentiated chondrosarcoma

Reserved Copyrights - 2013

15

Pathology Questions

MCQs

89. Which of the following tumors occurs most commonly in long bones of
children or young adults and it is characterized by a spindle cell sarcomatous
component with osteoid formation. This component is seen intimately
admixed with a cartilaginous component of an equivalent grade:
a. Chondroblastic osteosarcoma
90. What is the most common jaw lesion and shows multinucleated giant cells:
a. Central giant cell granuloma
b. Aneurysmal bone cyst
c. Osteoclastoma
d. Brown tumor

91. A patient is complaining of multiple skin nodules which was composed of


many osteoclast-like giant cells, the most likely diagnosis is:
a. Giant cell tumor of tendon sheath
92. Which of the following entities has a striking female predominance:
a. Collagenous colitis
93. What is the most common cause for urinary tract infections:
a. E.coli
94. Which soft tissue tumor is characterized by areas of high grade
dedifferentiation:
a. Pleomorphic MFH
b. Rhabdomyosarcoma
95. Which of the following entities is positive for HMB45 and negative for S100,
and CK:
a. Angiomyolipoma

Reserved Copyrights - 2013

16

Pathology Questions

MCQs

96. Which skin lesion is characterized by granulomas with central basophilic


degenerated collagen:
a. Granuloma annulare
97. Which of the following features is a favorable factor in neuroblastoma
a. Age < 1 year
b. The presence of N-Myc
98. What pancreatic lesion is associated with VHL syndrome:
a. Microcystic serous cystadenoma
99. Cowadry A inclusions are seen in:
a. HSV
100.

Multinucleaion, nuclear molding and chromatin margination are seen in:


a. HSV

101.

Drying artifact in cytology causes:


a. Nuclear englargement
b. Vacuolated cytoplasm
c. Hyperchromatic nuclei

102.

What does the abbreviation MOSD stand for:


a. Method of sheet data

103.

Which disease occurs in Orientals and associated with high levels of IgE,
recurrent infections, lymphadenopathy and peripheral eosinophilia:
a. Kimuras disease
b. Castelman disease
c. Hodgkin lymphoma
d. Langerhans histiocytosis
e. Epitheliod hemangioma

Reserved Copyrights - 2013

17

Pathology Questions
104.

MCQs

Which lesion is characterized by the presence of blood vessels lined by


epithelioid endothelial cells:
a. Kimura disease
b. Castelman disease
c. Hodgkin lymphoma
d. Langerhans histiocytosis
e. Epithelioid hemangioma

105.

Which lymphoma as 2 cell populations, germinal centres & activated Bcells:


a. MALTomas
b. SLL/CLL
c. Follicular lymphoma
d. DLBCL
e. Mantle cell lymphoma

106.

Which lymphoma is characterized by t(11:14):


a. Mantle cell lymphoma

107.

Transolocation t(11:18) is seen in:


a. MALTomas/MZL

108.

Which tumor is associated with Peutz-Jeghers syndrome:


a. Sex cord tumor with annular differentiation
b. Fibroma

109.

Which tumor totally lacks any differentiation:


a. Adenosarcoma
b. MMMT
c. Adenofibroma
d. Undifferentiated uterine sarcoma

Reserved Copyrights - 2013

18

Pathology Questions
110.

MCQs

Which stain is used to differentiate atypical teratoid/rhaboid tumor from


choroidal plexus carcinoma:
a. EMA
b. GFAP
c. CK
d. Synaphtophysin
e. Vimentin

111.

The most common source for ovarian squamous cell carcinoma is:
a. Endometriosis
b. Mature teratoma
c. Endometriod adenocarcinoma

112.

The major difference between classic Hodgins lymphoma and nodular


lymphocyte predominant Hodgkins lymphoma is:
a. Nodularity
b. Inflammatory background
c. Positivity of tumor cells to CD30 and CD15

113.

Which tumor is positive to CD5 & CD117


a. Thymoma
b. Thymic carcinoma
c. Systemic mastocytosis

114.

Which tumor is characteristically located in the neck of males:


a. Spindle cell lipoma

115.

What is the typical immunochemical profile of Hodgkin lymphoma:


a. CD30+ve, CD45-ve, CD15+ve

116.

Which entity is characterized by a positive staining for CD20 and Bcl2:


a. Follicular lymphoma

Reserved Copyrights - 2013

19

Pathology Questions
117.

MCQs

What is the most common clinical presentation of respiratory


papillomatosis:
a. Post obstruction pneumonia
b. Change in voice
c. Cavitary lung lesions

118.

The most common cause for respiratory papillomatosis:


a. HPV 16
b. HPV 11
c. HPV 18
d. HPV 1

119.

Which renal tumor is characterized by the presence of a central scar and


prominent nucleoli:
a. Oncocytoma

120.

Which of the following tumors has a villoglandular architecture and it


shows positivity for ER and PR:
a. Endometriod endometrial adenocarcinoma

121.

Birbeck granules are seen in which of the following entities:


a. Langerhans histiocytosis

122.

Which tumor shows neurosecretory granules and synaptic vesicles under


EM:
a. Neuroblastoma
b. Pheochromocytoma

123.

Which of the following tumors has the genetic abnormality t(7:17):


a. Endometrial stromal sarcoma

124.

Which of the following oral cysts has the greatest tendency to recur:
a. Odontogenic keratocyst

Reserved Copyrights - 2013

20

Pathology Questions

125.

MCQs

Which of the following cystic renal diseases is characterized by cysts at


the cortico-medullary junction:
a. Medullary cystic kidney disease
b. Medullary sponge kidney
c. Polycystic kidney disease
d. Renal dysplasia

126.

Which disease is characterized by hepatomegally, neurological symptoms


and the presence of cells with a PAS +ve fibrillary cytoplasm:
a. Gaucher disease

127.

Which cells are responsible for the production of IgE in type 1


hypersensitivity reactions:
a. CD4 lymphocytes
b. Mast cells
c. Neutrophils
d. Macrophages

128.

How does the infection reach the brain in a case of HIV:


a. Infected macrophages

129.

What does the window period mean in a case of HIV:


a. The interval between the time of infection and the development of
detectable antibodies

130.

What is the best method for diagnosing amyloidosis:


a. Congo red special stain

131.

What is a large area of a subcutaneous blood collection called:


a. Ecchyomosis

132.

Why is there no light-chain restriction in Hodgkins lymphoma:

Reserved Copyrights - 2013

21

Pathology Questions

MCQs

a. The neoplastic cells represent < 1% of the tumor cells


133.

Which organism characteristically causes suppurative inflammation and


microabscesses:
a. Staph. aureus
b. C. neoformans

134.

Which tumor is -catenin +ve and focally +ve to CD117:


a. Fibromatosis
b. GIST

135.

The most common clinical presentation of Digeorge syndrome is:


a. Defects in the heart and blood vessels
b. Clinical hypocalcemia
c. Facial anomalies

136.

Fragile X syndrome is characterized by all of the following, except:


a. Tandem repeat of trinucleotides
b. Affects only males
c. Mental retardation is more severe in consecutive generations

137.

What is the mechanism of neutrophilic killing of bacteria:


a. NADPH oxidative reaction

138.

The most important cells to be present in the diagnosis of temporal


arteritis are:
a. Neutrophils
b. Giant cells
c. Lymphocytes
d. Plasma cells

139.

The indication of performing frozen section in breast specimens is to:


a. Assess the margins

Reserved Copyrights - 2013

22

Pathology Questions

140.

MCQs

The most common cause of a bloody nipple discharge is:


a. Intraductal papilloma

141.

All of the following are true regarding intraductal papillomas, except:


a. Nipple retraction
b. Bloody nipple discharge
c. Growth in the lactiferous ducts
d. Multiple lesions

142.

The turn around time (TAT) for frozen section according to the CAP is:
a. 100% of the frozen blocks must be completed by 20 min
b. 90% of the frozen blocks must be completed by 10 min
c. 90% of the frozen blocks must be completed by 20 min
d. 50% of the frozen blocks must be completed by 20 min

143.

Which study can be either retrospective or prospective and in which the


investigators are interested in rare exposures?
a. Case control study
b. Cohort study
c. Prevalence study
d. Interventional trial

144.

The most favorable location for carcinoid tumor is:


a. Appendix
b. Colon
c. Stomach

145.

Nutmeg liver associated with:


a. Chronic passive congestion
b. Portal vein thrombosis

Reserved Copyrights - 2013

23

Pathology Questions
146.

MCQs

A patient suffered from multiple pleural nodules and pleural thickening


which composed of spindle cells, CK +ve and Calretenin +ve, the most
likely diagnosis is:
a. Synovial sarcoma
b. Malignant mesothelioma, sarcomatoid type

147.

Which tumor with the translocation t(X:18):


a. Synovial sarcoma

148.

Pre analytical review of control management of immunohistochemistry


includes:
a. New reagent lots
b. Antibody validity
c. Procedure and protocol
d. Tissue fixation

149.

A 10-y old boy had fever with sore throat. His mother gave him aspirin
after which the child became comatosed and his liver enzyme elevated.
What is the most likely diagnosis:
a. Reyes syndrome
b. Alpha1 antitrypsin deficiency
c. Biliary atresia
d. Wilson disease

150.

An 11-y old male developed dementia, hemiballismus and elevated liver


enzymes. CT scan showed liver cirrhosis. What other finding is expected
to be present in this patient:
a. Antimitochondrial antibodies
b. PAS postive esinophilic cytoplasmic globules
c. Kayser-Fleischer ring in the cornea

Reserved Copyrights - 2013

24

Pathology Questions
151.

MCQs

A 12-y old boy presented with an abdominal mass. The mass was found to
be +ve for CD20, CD43, CD10, Bcl 6 & -ve for Bcl 2. Which translocation he
most likely has:
a. t (8:14)
b. t (2:4)
c. t (11:14)
d. t (14:18)

152.

Calculate the mean of the following weights 78, 80, 83, 85, 86, 87, and 90:
a. 84.1
b. 80
c. 87

153.

The investigators want to study the effect of exercise program on death.


They found 30 deaths in 100 persons in the control group (not taking the
exercise program) and 50 deaths in 100 persons in the study group
(taking the exercise program). What is the relative risk of death in the
study group compared to the control group:
a. 0.2
b. 0.3
c. 0.5
d. 0.6

154.

What does Rhodococci infection cause:


a. Malakoplakia
b. Pseudoinflammatory tumor
c. Pseudotuberculosis

155.

Where do you see antimitochondrial antibodies:


a. Primary biliary cirrhosis

156.

What are the most common sites for secondary ovarian metastases:
a. Gastrointestinal tract (colon or stomach)

Reserved Copyrights - 2013

25

Pathology Questions

MCQs

b. Breast
c. Uterus
157.

What is the most common testicular tumor in children:


a. Embryonal carcinoma
b. Yolk sac tumor
c. Seminoma

158.

What is the most specific immunohistochemical stain of glomus tumor:


a. SMA
b. CD34
c. CD31
d. Desmin

159.

What is the most specifin immunostain of Langerhans cells:


a. CD68
b. CD20
c. CD 1a

160.

What is the tumor marker for yolk sac tumor:


a. Alpha fetoprotein
b. CEA

161.

B2-nephthylamine is considered a risk factor for cancer in which of the


following organs:
a. Urinary bladder

162.

On performing an FNA of the thyroid, which of the following, according to


the Bethesda, renders the diagnosis insufficient:
a. Only colloid, no follicular cells
b. 2 follicular cells with macrophages
c. 3 follicular cell groups only
d. Abundant Hurthle cells

Reserved Copyrights - 2013

26

Pathology Questions

MCQs

e. 7follicular cell groups, each containing 10 to 12 cells


163.

A 30-y old female has a breast mass. FNA showed three-dimensional


clusters with high N/C ratio and nuclear atypia, also there were bipolar
naked nuclei within the epithelial group. What is the most likely
diagnosis:
a. Fibroadenoma
b. Ductal carcinoma
c. Lobular carcinoma

164.

The mot common location of malakoplakia in urinary tract is:


a. Renal pelvis
b. Urinary bladder
c. Urethra
d. Testis
e. Epididymis

165.

The most common location of peptic ulcer:


a. Gastric antrum
b. Second part of duodenum
c. First part of duodenum
d. GEJ

166.

The most common location of polymorphous low-grade adenocarcinoma:


a. Parotid glands
b. Submandibular glands
c. Submental glands
d. Palate

167.

Which lymphoma is commonly associated with immunocompromised


patient(AIDS):
a. Diffuse large B cell lymphoma
b. Follicular lymphoma

Reserved Copyrights - 2013

27

Pathology Questions

MCQs

c. Hodgkin lymphoma
168.

Which of the following lesions commonly occur in the posterior


mediastinum:
a. Bronchogenic cyst
b. Lymphoma
c. Thymoma
d. Sarcoidosis
e. Schwanoma

169.

Which of the following thymic lesions is commonly associated with


myasthenia gravis:
a. Thymic follicular hyperplasia
b. Thymoma
c. Thymitis

170.

Which of the following is associated with karyorrhexis and cell


fragmentation:
a. Viral hepatitis
b. Chronic alcoholic liver disease
c. Renal graft rejection
d. Barbiturate overdose
e. Brown atrophy of the heart

171.

The benefit of literature review in a study is:


a. Precision
b. Applicability

172.

Rhodinin stain is used for:


a. Iron
b. Copper
c. Lead
d. Fat

Reserved Copyrights - 2013

28

Pathology Questions

MCQs

e. Zinc
173.

The purpost of MHC II matching before an allograft transplantation is to


prevent:
a. Cytotoxic CD8 T-cell activation
b. CD4 T-helper cell activation
c. Graft vs host disease

174.

Which gene is considered as a gate keeper of the cell cycle:


a. P16
b. RB
c. BRAC1

175.

Which tumor in the endocervix is considered a low grade malignancy with


a good prognosis:
a. Endocervical adenocarcinoma, usual type
b. Endomeriod adenocarcinoma of the uterus involving the cervix
c. Papillary serous adenocarcinoma of the uterus
d. Villoglandular papillary adenocarcinoma of cervix

176.

Which of the following is true regarding uterine papillary serous


adenocarcinoma:
a. Has BRAC1 gene
b. Has P53 mutation
c. Occurs with unopposed estrogen
d. Has K-Ras mutation
e. Occurs in the women at the 2nd and 3rd decade

177.

An HIV patent had cough and dyspnea. His lung biopsy showed intraalveolar frothy eosinophilic material. Which special stain may be useful:
a. PAS
b. Modified Methamin silver stain
c. Mucicarmin

Reserved Copyrights - 2013

29

Pathology Questions

178.

MCQs

Which of the following is a risk factor for hepatic angiosarcoma:


a. Vinyl chloride
b. Asbestos

179.

Which interstitial lung disease is strongly associated with smoking:


a. Usual interstitial pneumonia (UIP)
b. Desquamative interstitial pneumonia (DIP)
c. Non specific interstitial pneumonia (NSIP)

180.

A patient presented with coughing. His lung biopsy showed patchy


interstitial mononuclear cell infiltrate. What is the most likely causative
organism:
a. E.coli
b. Aspergillus
c. Mycobacterium tuberculosis
d. Mycoplasma pneumonia
e. Adenovirus

181.

What is the most common salivary gland lesion in HIV patients:


a. Epidermoid cyst
b. Lymphoepithelial cyst
c. Lymphoepithelioma-like carcinoma

182.

Which ovarian tumor occurs in younger women with vitalization


symptoms:
a. Steroid secreting tumor
b. Sertoli- Leydig cell tumor

183.

Which ovarian tumor occurs in postmenopausal women and it presents


bilaterally with pareneoplastic hormonal effect:
a. Small cell carcinoma of the ovary, pulmonary type
b. Small cell carcinoma of the ovary, hypercalcemic type

Reserved Copyrights - 2013

30

Pathology Questions

184.

MCQs

Which of the following ovarian tumors occurs in young women and it


metastasizes by the time of its diagnosis:
a. Adult granulosa cell tumor
b. Sex cord tumor with annular differentiation
c. Small cell carcinoma of the ovary, hypercalcemic type
d. Clear cell carcinoma
e. Serous cystadenocarcinoma

185.

The most commonly readily diagnosed entity of lymph node FNA is:
a. Metastatic carcinoma
b. Hodgkin lymphoma
c. Non-Hodgkin lymphoma
d. Reactive lymph node
e. Follicular lymphoma

186.

A 30-y African female, domestic worker, presented with painful inguinal


lymphadenopathy. Microscopically showed granulomas. The most likely
diagnosis is:
a. Treponema palladium
b. Gardinella vaginalis
c. HSV
d. Candida infection

187.

A female patient underwent a bone marrow transplantation. Her CBC


results were normal but she developed a skin rash. What is the most likely
cause:
a. GVHD

188.

A female patient had a car accident and broke a femur and an arm. In few
days, she developed dyspnea, What is the cause of her new symptom:
a. Fat embolism
b. ICU stay

Reserved Copyrights - 2013

31

Pathology Questions

189.

MCQs

A lung biopsy showed alveolar hyaline membrane formation without


inflammatory cells infiltrate. What is the most likely diagnosis:
a. Diffuse alveolar damage

190.

Which of the following organisms is an obligate intracellular microbe:


a. Pseudomonas aeruginosa
b. N. Meningitidis
c. Borrelia burgdorferi
d. Yersenis
e. Rickettsia

191.

Which of the following is the most common glioma in the brain:


a. Oligodendroglioma
b. Low grade astrocytoma
c. Glioblastoma multiform
d. Piliocytic astrocytoma
e. Ependymoma

192.

What does the fried-egg appearance of oligodendrocytes represent:


a. Cell cytoplasm
b. Delayed fixation artifact
c. Lipid in the cytoplasm
d. Glycogen in the cytoplasm

193.

A newborn was found to have atransluminating mass at the lower lumber


spinal region. He also has flattening of the skull. This patient is at risk for
developing:
a. Hydrocephalus
b. Medulloblastoma
c. Cerebellar vermis agenesis

194.

A zellen-ballen architecture is characteristic of which lesion:

Reserved Copyrights - 2013

32

Pathology Questions

MCQs

a. Schwannoma
b. Chordoma
c. Paraganglioma
195.

Which of the following thyroid tumors is positive for CK19:


a. Follicular carcinoma
b. Follicular adenoma
c. Hurthle cell neoplasm
d. Papillary carcinoma

196.

Which tumor is commonly associated with VHL syndrome:


a. Clear cell renal cell carcinoma
b. Papillary renal cell carcinoma

197.

Which tumor has the following immunohistochemical profile: PLAP+ve,


OCT4+ve, CD99+ve, CD30+ve, CK +ve, CD117-ve:
a. Emberyonal carcinoma
b. Yolk sac tumor
c. Dysgerminoma

198.

All of the following are associated with nephrotic syndrome except:


a. Proteinurea
b. Hypoalbuminemia
c. Hypertension
d. Hyperlipidemia

199.

The abuse of which of the following drugsis associated with myocardial


infarction:
a. Opiates
b. Heroin
c. Cocaine

Reserved Copyrights - 2013

33

Pathology Questions
200.

What

is

the

MCQs
microscopic

description

of

rapidly

progressive

gloumerolonephritis:
a. Crescent formation
b. Subepithelial IgG deposition
c. Hyaline nodules
201.

Which of the following is least likely to be associated with hematurea:


a. Rapidly progressive glomerulonephritis
b. Minimal change disease
c. Membrenoproliferative glomerulonephritis

202.

A tall young male patient presented with a ruptured aortic aneurysm.


Which gene mutation he probably has:
a. FMR
b. Fibrilin

203.

A hypersensitivity reaction with pronounced eosinophillia is seen after


the exposure to:
a. Inhaled dust
b. Liver flukes

204.

Which of the following prevents collagen synthesis during wound healing:


a. Infection
b. Diabetes
c. Vitamin C deficiency

205.

Which of the following predicts the behavior of malignant phylloids tumor


of the breast:
a. Tumor diameter
b. Ki67 index
c. Stromal over growth
d. P16

Reserved Copyrights - 2013

34

Pathology Questions
206.

MCQs

The vast majority of bronchoalveolar mucinous carcinomas are classified


now according to the new classification as:
a. Invasive mucinous carcinoma
b. Minimally invasive carcinoma

207.

A patient has multiple meningiomas and brain tumors with chromosome


22 abnormality. The most likely diagnosis is:
a. NF1
b. NF2

208.

The cell of origin in mycosis fungoides is:


a. CD4
b. CD8
c. Dendritic cells

209.

Which of the following viruses is associated with nasopharyngeal


carcinoma:
a. EBV
b. HPV
c. CMV

210.

Which of the following is considered to be an oncogenic infectious agent:


a. T-cell lymphotropic virus

211.

Which of the following cutaneous lesions has prominent nuclei &


macronucleoli:
a. Spitz nevus
b. Blue nevus
c. Melanoma

212.

What type of nevus has prominent nucleoli and mild atypia:


a. Spitz nevus

Reserved Copyrights - 2013

35

Pathology Questions
213.

MCQs

Which of the following procedures will help to avoid metastasis in


melanoma:
a. Early diagnosis and complete excision
b. Removal of sentinel lymph node
c. Chemotherapy
d. Reduce exposure of UV light

214.

What is the most common type of uterine tumors that is associated with
endometriosis:
a. Yolk sac tumor
b. Clear cell carcinoma
c. Endometriod tumor, secretary type

215.

What is the most common mediastinal lymphoma in children:


a. Lymphoblastic lymphoma
b. Nodular sclerosis HL
c. Mixed cellularity HL
d. MCL
e. DLBCL

216.

Which of the following tumors most resembles adrenocortical carcinoma:


a. Renal clear cell carcinoma
b. Breast ductal carcinoma

217.

What is the most common cause of obstructive jaundice in children:


a. Bile duct hamartoma
b. Mesenchymal hamartoma
c. Choledocal cyst
d. Biliary atresia

218.

An autopsy showed an atrophied pancreas (fibrosis & inspissated


secretions in the ducts) and normal islet cells. What is the most common
cause:

Reserved Copyrights - 2013

36

Pathology Questions

MCQs

a. Cholelithiasis
b. Alcohol
219.

What is the feature seen in a baby of a diabetic mother:


a. Insulinoma
b. Nesidioblastosis

220.

All of the following are features of extrahepatic obstruction, except:


a. Bile duct proliferation
b. Bile duct with inspissated bile
c. Neutrophilic infiltration
d. Portal tract edema
e. Hepatocyte and canalicular without bile

221.

What is the latest hematolymphoid neoplasms classification:


a. WHO 2010
b. WHO 2008
c. WHO 2010
d. REAL classification
e. RAPPAPORT classification

222.

The majority of patients with post-infectious glomerulonephritis will have


the following outcome:
a. Complete recovery
b. RPGN
c. MPGN
d. Membranous glomerulonephritis

223.

In a case of uncontrolled diabetes mellitus, glycosylation of the amino


acids will cause:
a. Cataract
b. Peripheral neuropathy
c. Retinal microaneurysm

Reserved Copyrights - 2013

37

Pathology Questions

MCQs

d. Atherogenesis
e. Amyloid islets
224.

The most common organism causing infective endocarditis is:


a. Staph. aureus
b. Strept. viridins
c. Strept. Pyogenes

225.

Which one of the following organs is affected by red infarction:


a. Spleen
b. Heart
c. Liver
d. Kidney
e. Lung

226.

The most common organism causing rheumatic heart disease:


a. Strept. pyogenes

227.

The accumulation of fluid in inflammation is due to:


a. Endothelial contraction
b. Lymphatic obstruction
c. Arteriolar vasoconstriction

228.

Injury to the middle meningeal artery will lead to:


a. Epidural hemorrhage
b. Subdural hemorrhage
c. Intracranial hemorrhage

229.

Which AR disorder has no grossly identified cysts but hepatic microcysts


are present:
a. Congenital hepatic fibrosis
b. Caroli disease
c. Caroli syndrome

Reserved Copyrights - 2013

38

Pathology Questions

230.

MCQs

Which of the following is considered a risk factor for aortic dissection:


a. Diabetes mulletis
b. Hypertension
c. Hyperlipidemia

231.

WaterhouseFriderichsen syndrome is due to:


a. Meningococcal infection
b. H. Influenza
c. E. Coli

232.

What is the most common site for atypical teratoid/rhabdoid tumor:


a. Cerebropontine angle (Posterior fossa)
b. Cerebellum
c. 3rd ventricle

233.

An example of antigen cross reactivity is:


a. Rheumatic heart disease

234.

A urine cytology showed multinucleated giant cells, the most probable


cause is:
a. BCG therapy
b. Urothelial Ca
c. Cystitis

235.

Which tumor is positive for CD1a and S100 and has nuclear grooves:
a. Kimura disease
b. Castelman disease
c. Hodgkin lymphoma
d. Langerhan's cell histiocytosis
e. Epitheliod hemangioma

Reserved Copyrights - 2013

39

Pathology Questions
236.

MCQs

Which of the following presents as multiple liver nodules with


heterogeneous blood flow:
a. FNH
b. Fibro-lamellar HCC
c. Hepatic adenoma
d. Regenerative nodules

237.

The grading of a teratoma depends on:


a. The amount of immature neuroepithelium

238.

Which of the following is a subserosal uterine tumor and positive for


calretinin and CK:
a. Adenomatoid tumor

239.

Which of the following hepatic tumors that its prognosis depends on


histology & mitosis:
a. Teratoma
b. Hepatoblastoma
c. Poorly differentiated rhabdomyosarcoma
d. Mesenchymal hamartoma

240.

Which of the following of PTC subtypes has a poor prognosis:


a. Macrofollicular
b. Nodular facitis like stroma
c. Oncocytic
d. Columnar cell variant

241.

Which ovarian tumor can cause pseudoprecocious puberty:


a.

242.

Juvenile granulosa cell tumor

P-value is significant if it is:


a. 0.5
b. >0.05

Reserved Copyrights - 2013

40

Pathology Questions

MCQs

c. <0.05
d. <0.5
243.

Which immunostain help to differentiate between angiomyolipoma and


leiomyoma:
a. HMB45

244.

What do lymphocytes attack in graft versus host disease in the liver:


a. Hepatic bile ducts

245.

What is the immunofluoresence patter of IgG in Goodpastures syndrome:


a. Linear IgG

246.

Which of the following tumors has been linked to a viral etiology:


a. Angiosarcoma

247.

Which

of

the

following

ovarian

lesions

is

characterized

by

hyperestrogenism:
a. Adult granulosa cell tumor
248.

Which of the following tumors does not recur after complete excision.
Sections show epithelioid cells, which have a tendency to grow around
small blood vessels:
a. Angiomyofibroblastoma
b. Epthelioid sarcoma
c. Aggressive angiomyxoma
d. Leiomyosarcoma

249.

Sections in a thyroid showed a diffuse involvement of both lobes of the


gland with papillary and hyperplastic follicles lined by bland looking cells.
What is the most likely diagnosis:
a. Graves disease

Reserved Copyrights - 2013

41

Pathology Questions
250.

MCQs

A child presented with skull lesions, which showed prominent eosinophils


and spindle cells positive for CD 117. The most likely diagnosis is:
a. Histiocytosis X
b. Systemic mastocytosis

251.

Which product is responsible for the pain, fever and vasodilatation that
occurs in a case of acute appendicitis:
a. Bradykinin
b. Thrombaxane
c. Prostaglandin
d. NO

252.

Regarding identification, the following is true:


a. There is no way to identify people with severe postmortem
changes
b. Tattoos are the most credible way to identify a person
c. Visual identification is rarely used
d. Muslims never use tattoos
e. A positive identification means that the person is identified
"Beyond a Reasonable Doubt

253.

Given the postmortem examination reveals a right lower quadrant scar


and a history of an arm fracture. We would most likely confirm the
identity of the person by the following, except:
a. Visually
b. Identify the person through imaging the arm and comparing it to
the imaging available in his electronic medical chart
c. Asking the wife to identify the body
d. Asking the family to identify the body
e. Asking the boss to identify his body

Reserved Copyrights - 2013

42

Pathology Questions
254.

MCQs

Given the postmortem examination reveals a right lower quadrant scar


and a history of an arm fracture. We would most likely confirm the
identity of the person by the following:
a. Visually
b. Identify the person through imaging the arm and comparing it to
the imaging available in his electronic medical chart
c. Identify the person through imaging the bodys teeth and
comparing it to his dental records
d. Sampling skin and cartilage for DNA analysis and compare to a
family member
e. Fingerprinting the deceased and comparing to that present filed in
the national database

255.

We can best confirm the sex of the person by the following:


a. Visually
b. By examining the skull and pelvis
c. By identifying scars on the body
d. Sampling skin and cartilage for DNA analysis and compare to a
family member
e. Through imaging the skeleton or teeth and comparing it to the
imaging available in the electronic medical chart

256.

We would most likely confirm the identity of the body by the following,
except:
a. Visually, through national ID
b. Asking the family to identify the body
c. Asking his coworker to identify the body
d. Asking his boss to identify his body
e. Identify the person through imaging the bodys teeth and
comparing it to his dental records

257.

The identified body may show:


a. Rigor mortis

Reserved Copyrights - 2013

43

Pathology Questions

MCQs

b. Livor mortis in the hands and feet only


c. Skin slippage
d. Livor mortis on the chest and abdomen
e. Multiple cut wounds
258.

We most likely would estimate or confirm the age of the above mentioned
person by the following, except:
a. Visually
b. Specific features and measurements of the skeleton and teeth, by
gross (visual) examination or imaging
c. Asking the friends
d. Asking the family
e. Asking the boss
f. National identification
g. Drivers license
h. Medical records

259.

Regarding the post-mortem examination (PME), the following applies:


a. Radiology is essential.
b. No PME is needed.
c. We should meet with the family at their wish (via phone or in
person) to discuss the findings and suggestions.
d. The injuries are due to assault prior to death.
e. There will be multiple bruises, abrasions and lacerations to the
head and abdomen and limbs.

260.

A 30-yr old man from an underdeveloped country complained of lower


limb edema and draining sinuses in the foot. The micro-organism is
filamentous and positive for Gram stain and partially for AFB. What is the
most likely causative agent of this infection:
a. Actinomyces Esraieli
b. Cryptococcosis
c. Nocardia

Reserved Copyrights - 2013

44

Pathology Questions

MCQs

d. Diphtheria
e. Pneumocystic carinii
261.

What is the causative organism of atypical pneumonia:


a. Mycoplasma pneumonia

262.

What is the causative organism of sinonasal papilloma:


a. HPV
b. HIV
c. EBV
d. RSV

263.

In a sputum cytology specimen, Creola bodies are seen in which of the


following conditions:
a. Aspergillosis
b. Sarcoidosis
c. Squamous cell carcinoma
d. Adenocarcinoma

264.

Which of the following features is not seen in sarcoidosis:


a. Giant cells
b. Schumann bodies
c. Creola bodies
d. Asteroid bodies
e. Epithelioid cells

265.

Ferruginous bodies are composed of:


a. Pollen
b. Iron
c. Asbestos
d. Lead

266.

Which of the following findings is helpful in diagnosing aspergillosis:

Reserved Copyrights - 2013

45

Pathology Questions

MCQs

a. Asteroid bodies
b. Oxalate crystals
267.

Which of the following organisms can cause an eosinophilic pneumonia,


granulomas palisading a central eosinophilic material and brochoalveolar
destruction:
a. GMS +ve aspergillosis
b. Geimsa +ve leishmaniasis
c. AFB +ve TB
d. GMS +ve Fungi

268.

What is the function of caspases


a. Apoptosis
b. Coagulative necrosis
c. Caseaus necrosis
d. Pinocytosis
e. Phagocytosis

269.

Which of the following is an acute-phase protein:


a. IL-6
b. Bradykinin
c. Interferon
d. Histamine

270.

Which of the following is a pro-apoptotic gene:


a. P53
b. Bcl-2
c. Bcl-x

271.

Which of the following tumors is typically seen within the peritoneal


cavity of young males:
a. Desmoplastic small round cell tumor

Reserved Copyrights - 2013

46

Pathology Questions
272.

MCQs

All of the following cells can undergo from G0 to G1 after a trauma in the
abdomen, except:
a. Skeletal muscles
b. Smooth muscles
c. Liver cells
d. Endothelial cells
e. Fibroblasts

273.

During wound healing, when does a wound return to its normal strength:
a. 3 months
b. Never returns to normal
c. 1 month
d. 1 year

274.

What tumor is most likely to co-express CK5/6 & WT-1 and desmin in a
dot-like pattern on immunohisochemical examination: (PIP 2012 B)
a. Dysgerminoma
b. Epithelioid sarcoma
c. Inflammatory myofibroblastic tumor
d. Intra-abdominal desmoplastic small round cell tumor
e. Renal clear cell carcinoma

275.

Which of the following muscle diseases shows a positive ATPase stain:


a. Myotonica dystrophica
b. Duchanne muscular dystrophy
c. Degenerative diseases
d. Mitochondrial myopathies

276.

A young female presented with neurological deficit, visual disturbances


and peripheral numbness. Which of the following vasculitis is the most
likely cause of her symptoms:
a. Takayasu vasculitis
b. Kawasaki disease

Reserved Copyrights - 2013

47

Pathology Questions

MCQs

c. Temporal arteritis
d. Polyangitis nodosa
e. Wegners granulomatosis
277.

Which of the following is the most common type of aneurysms caused by


hypertension:
a. Saccular aneurysms
b. Atherosclerotic aneurysms
c. Berry aneurysms
d. Charcot-Bouchard aneurysms

278.

Which of the following is least likely to be associated with a significant


ductular reaction:
a. Primary biliary cirrhosis
b. Primary sclerosing cholangitis
c. Chronic hepatitis B
d. Large bile duct obstruction due to gallstones
e. Massive liver necrosis

279.

Which of the following thyroid tumors is most likely to undergo clear cell
change:
a. Papillary carcinoma
b. Follicular carcinoma
c. Medullary carcinoma
d. Spindle cell anaplastic carcinoma
e. Hurthle cell tumors

280.

What is the most common type of lymphoma in the brain:


a. Diffuse large B-cell lymphoma

281.

Anaplastic carcinoma of the thyroid is characterized by:


a. Metastasis at the time of presentation

Reserved Copyrights - 2013

48

Pathology Questions
282.

MCQs

What is the most common type of breast cancer occurring in young


females:
a. Secretory carcinoma

283.

Which of the following lesions occurs usually in the vulva? It is well


circumscribed and highlighted by perivascular cuffs of epithelioid
desmin-positive cells. It does not usually recur after excision:
a. Angiomyofibroblastoma

284.

CDX2 is negative in which of the following tumors:


a. Colon cancer
b. Carcinoid of appendix
c. Gastric adenocarcinoma
d. Adenocarcinoma of the ampulla of vater
e. Cholangiocarcinoma

285.

Which organ is most commonly affected in cat scratch disease:


a. Liver
b. Spleen
c. Lymph nodes

286.

Which of the following vulval lesions is poorly circumscribed, myxoid,


large and typically recurs after excision:
a. Aggressive angiomyxoma

287.

Which of the following liver lesions has no gender predilection:


a. Fibrolamellar HCC
b. Focal nodular hyperplasia
c. Bile duct hamartoma
d. Hepatic adenoma
e. Nodular regenerative hyperplasia

288.

Cholangiocarcinoma is associated with which of the following conditions:

Reserved Copyrights - 2013

49

Pathology Questions

MCQs

a. Caroli disease
289.

A renal biopsy from a patient with a history of renal transplant showed


arteritis and tubulitis. What is the most likely diagnosis:
a. Acute rejection
b. Chronic rejection
c. Acute tubular necrosis
d. GVHD

290.

What is the most common malignant germ cell tumor in females:


a. Dysgerminoma
b. Immature teratoma
c. Choriocarcinoma
d. Granulosa cell tumor

291.

What is the most common soft tissue tumor arising in the first and second
decades of life:
a. Rhabdomyosarcoma
b. Neuroblastoma
c. Ewing sarcoma
d. Lymphoblastic lymphoma

292.

What is the most common location for minor salivary glands tumors:
a. Oral cavity and hard palate

293.

Which of the following is a grade I meningioma:


a. Papillary
b. Microcystic
c. Rhabdoid
d. Clear cell
e. Chordoid

Reserved Copyrights - 2013

50

Pathology Questions
294.

MCQs

A 50-yr old male presented with a 3-day history of dementia and he


eventually died. What is the most likely cause:
a. Prion protein misfolding (spongiotic encephalitis)

295.

Which of the following conditions is a risk factor for developing squamous


cell carcinoma:
a. Xeroderma pigmentosa
b. Ionizing radiation
c. HPV type 3
d. EBV

296.

Which of the following tumors would show long thin branching microvilli
ultrastructurally:
a. Mesothelioma

297.

Which of the following renal tumors would show a large number of


mitochondria ultrastructurally:
a. Oncocytoma

298.

What does apoptosis include:


a. Release of cytochrome C into the cytoplasm

299.

Which of the following renal tumors would show cytoplasmic lysosomes:


a. Chromophobe RCC
b. Clear cell RCC
c. Papillary RCC
d. Oncocytoma

300.

Which of the following renal tumors arise from the intercalated ducts and
show cytoplasmic microvesicles:
a. Chromophobe RCC
b. Clear cell RCC
c. Papillary RCC

Reserved Copyrights - 2013

51

Pathology Questions

MCQs

d. Oncocytoma
301.

What is the embedding media for electron microscopy:


a. Resin
b. Paraffin
c. Gelatin
d. Non-paraffin wax

302.

What is the most suitable fixative for electron microscopy examination:


a. Gluteraldehyde

303.

Where do you see the deposition of immune complexes in the cutaneous


manifestations of systemic lupus erythromatosus:
a. Dermo-epidermal junction
b. Tips of the dermal papillae
c. Dermis
d. Perivascular
e. Perineural

304.

HLA-B27 is seen in association of which of the following conditions:


a. Ankylosing spondylitis

305.

What is the most important inflammatory mediator in rheumatoid


arthritis:
a. TNF
b. IL-6
c. Histamine

306.

A deletion in 3p is seen in which of the following tumors:


a. Small cell carcinoma

307.

Which of the following tumors shows BRAF and KRAS mutations and may
arise from a benign lesion:

Reserved Copyrights - 2013

52

Pathology Questions

MCQs

a. Endometriod endometrial adenocarcinoma


b. Serous adenocarcinoma, low grade
c. Serous adenocarcinoma, high grade
d. Clear cell carcinoma
e. Ovarian endometriod adenocarcinoma
308.

Which uterine carcinoma has the PTEN mutation:


a. Papillary serous carcinoma
b. Endometriod endometrial adenocarcinoma
c. Transitional cell carcinoma
d. Mucinous carcinoma

309.

A patient presented with multiple skin nodules, an adrenal mass with high
urine catecholamines and a 4-cm mass in the temporoparietal area of the
brain. What is the most likely diagnosis:
a. N-Myc
b. P53
c. KRAS
d. Rb
e. NF1

310.

What is the expected mutation in a woman who is diagnosed for the first
time with breast cancer and has no family history:
a. BRCA1
b. P53
c. BRCA2
d. PTEN

311.

Which of the following pancreatic tumors is associated with VHL


syndrome:
a. Pancreatic serous cystadenoma

312.

Which syndrome is characterized by multiple meningiomas:

Reserved Copyrights - 2013

53

Pathology Questions

MCQs

a. NF-2
b. NF-1
c. Li-Fraumeni Syndrome
d. Tuberous sclerosis
e. VHL syndrome
313.

Which of the following familial conditions is characterized by


hemangioblastomas in the retina and brain, cysts in the kidney, pancreas,
liver and increased risk of clear cell renal cell carcinoma:
a. VHL syndrome

314.

Which ovarian syndrome is characterized by a P53 and BRCA1 mutation:


a. Clear cell carcinoma
b. Serous carcinoma, high grade
c. Serous carcinoma, low grade
d. Endometriod carcinoma

315.

A female patient with history of uterine fibroids has developed multiple


pulmonary nodules. Which of the following sentences is correct:
a. Both tumors have the same clone.

316.

A patient was found to have signet ring carcinoma which had the
following immunostain pattern: MUC1 ve, MUC2 +ve, CDX2 +ve, MUC5AC
+ve, PSA ve, TTF1 ve, CK7 ve and CK20 +ve. What is the most likely
primary site of this tumor:
a. Lung
b. Prostate
c. Pancreas
d. Colon
e. Breast

317.

The discohesiveness and diffuse infiltration in signet ring carcinoma is


due to:

Reserved Copyrights - 2013

54

Pathology Questions

MCQs

a. Loss of E-cadherin
318.

Which of the following tumors strongly expresses ALK-1:


a. Inflammatory myofibroblastic tumor

319.

Giant cells in pancreatic undifferentiated carcinoma with osteoclast-like


giant cells are typically:
a. CD68 +ve and CK ve
b. CD68 ve and EMA +ve

320.

Which lymphoma is negative for CD5, CD23, CD10 & cyclin D1:
a. MALT lymphoma

321.

Which lymhoma is positive for CD5 and CD23:


a. CLL/SLL

322.

Which lymphoma is positive for CD5 and cyclin D1:


a. Mantle cell lymphoma

323.

Which tumor is positive for CD99, CD43, CD34 and CD79a:


a. Lymphoblastic lymphoma
b. Ewings sarcoma/PTEN

324.

Which immunostain is helpful in differentiating thymoma from thymic


carcinoma:
a. CD5 immunostain

325.

Which type of lung carcinoma is typically peripheral in location:


a. Squamous cell carcinoma
b. Adenocarcinoma
c. Carcinoid tumor

Reserved Copyrights - 2013

55

Pathology Questions
326.

MCQs

All of the following conditions are characterized by cytotoxic T-cells


infiltration, except:
a. Drug eruption
b. Urticaria
c. GVHD
d. Erythema multiform

327.

Which of the following tumors shares the immunohistochemical profile of


Ewings sarcoma:
a. Lymphoblastic lymphoma

328.

What is the stage for a patient with invasive ductal carcinoma of the
breast and infiltration of the dermal lymphatics:
a. T1
b. T2
c. T3
d. T4d

329.

What is the CAP recommendation time for frozen section:


a. 15 min
b. 20 min
c. 25 min
d. 30 min

330.

What is the most common type of gastric polyps:


a. Fundic gland polyp
b. Adenomatous polyp
c. Hyperplastic polyp

331.

What is the most common benign liver tumor:


a. Cavernous hemangioma

332.

What is the most common type of stones seen in gallstones:

Reserved Copyrights - 2013

56

Pathology Questions

MCQs

a. Cholesterol stones
b. Calcium stones
c. Mixed stones
333.

A female patient presented with watery discharge. Pap showed clusters of


malignant glandular cells in a clear background:
a. Fallopian tube carcinoma
b. Transitional zone carcinoma
c. Endometrial carcinoma
d. Endocervical carcinoma

334.

Red bags are used for the disposal of:


a. Biological waste products and contaminated containers
b. Sharp objects
c. Biologically contaminated towels, papers and containers
d. Biochemical containers

335.

What would a physician do if he/she injured himself/herself with a knife


or needle prick:
a. Squeeze the blood and wash with water
b. Go to ER immediately
c. Write an incidence report

336.

What is the most common cause of discrepancy between the frozen


section and the permanent diagnosis:
a. Overwhelming work
b. Incomplete history and clinical data
c. Inadequate sampling of the specimen

337.

Frozen section can not be used to assess which of the following:


a. Margins
b. Diagnosis
c. Grade

Reserved Copyrights - 2013

57

Pathology Questions

338.

MCQs

What is the most common cause of error in the pathology reports:


a. Overwhelming work
b. Incomplete history and clinical data
c. Inadequate sampling of the specimen

339.

Sections of a lymph node showed monocytoid B-cells and reactive


follicular hyperplasia with epithelioid histiocytic infiltrations. What is the
most likely diagnosis:
a. HIV
b. Reactive lymph node
c. Cat scratch disease
d. Toxoplasma infection

340.

Which of the following liver diseases show ground-glass appearance:


a. HBV

341.

Which of the following lymphomas involves the splenic red pulp and
spares the white pulp:
a. Hairy cell leukemia

342.

What is the 2nd most common malignancy of the major salivary glands:
a. Acinic cell carcinoma

343.

A cerebellar lesion showed hairy-like cells. What is the most likely


diagnosis:
a. Pilocytic astrocytoma

344.

What is the definition of spongiosis:


a. Intercellular edema in the epidermis
b. Intercellular edema in the keratinocytes
c. Edema of the dermis

Reserved Copyrights - 2013

58

Pathology Questions
345.

MCQs

What do you call the obstruction of the hepatic venous blood flow:
a. Budd Chiari syndrome
b. Crigler Najjar syndrome

346.

What is the most likely diagnosis in a young male with a tumor in the
peritoneal cavity:
a. Ewings sarcoma/Primitive neuroectodermal tumor
b. Desmoplastic round cell tumor
c. Alveolar soft part sarcoma
d. Embryonal rhabdomyosarcoma

347.

What is the most common cardiac cause for cyanosis:


a. Tetralogy of Fallot

348.

Gardner syndrome is associated with which of the following tumors:


a. Elastofibroma
b. Dermatofibroma
c. Spindle cell lipoma
d. Nuchal fibroma
e. Fibrolipoma

349.

P53 overexpression is seen in which of the following breast tumors:


a. Basal-like
b. Luminal A
c. Luminal B
d. Unclassified
e. Her 2 neu

350.

A female patient presented with an irregular mass and calcifications on


the mammogram. Sections showed foamy cells. What is the most likely
diagnosis:
a. Trauma

Reserved Copyrights - 2013

59

Pathology Questions
351.

MCQs

Which of the following renal tumors shows a prominent desmoplastic


reaction:
a. Collecting duct carcinoma

352.

A 16-yr old female presented with a breast lesion. Sections showed


nodules with stromal and ductular proliferation and adipose tissue:
a. Sclerosing adenosis
b. Fibroadenoma
c. Hamartoma
d. Tubular adenoma
e. Fibrocystic disease

353.

A lung adenocarcinoma has a lipoid pattern without stromal or vascular


invasion. Cells are columnar with apical mucin. What is the most likely
diagnosis:
a. BAL, mucinous type

354.

Which tumor is positive for SMA and HMB-45:


a. PEComas

355.

INI1/BAF47 is used for the diagnosis of which of the following tumors:


a. Epitheliod sarcoma

356.

Which of the following viral genes is mostly linked to the pathogenesis of


cancer:
a. LCR
b. E3
c. E4
d. E6

357.

Secondary hyperparathyroidism is related to which of the following


conditions:
a. Chronic renal disease

Reserved Copyrights - 2013

60

Pathology Questions

358.

MCQs

Hypertension is part of all of the following conditions, except:


a. Acromegally
b. Addisons disease
c. Cushing disease
d. Pheochromocytoma
e. Conns disease

359.

Which of the following entities can be diagnosed by a peripheral blood


smear:
a. CML

360.

Which immunostain can help you differentiate between follicular


lymphoma and follicular hyperplasia:
a. Bcl-2

361.

A skin disease shows dermal papillae microabscesses and IgA positive on


IF, which of the following entities is usually associated with this disease:
a. Celiac disease

362.

Which of the following associations is linked to MENIIb:


a. Pancreas and parathyroid

363.

T-cell enteropathy associated lymphoma is related to which of the


following conditions:
a. Celiac disease
b. Crohns disease
c. Whipples disease

364.

A lady with endometrial hyperplasia and an ovarian tumor. What is the


most likely diagnosis of the ovarian lesion:
a. Granulosa cell tumor
b. Yolk sac tumor

Reserved Copyrights - 2013

61

Pathology Questions

365.

MCQs

All of the following are causes for chronic pancreatitis, except:


a. Alcohol
b. Trauma
c. Stones
d. Biliary diseases

366.

What is the underlying cause of erythema nodosum:


a. Autoimmune
b. Virus

367.

What is the most common mutation seen in osteosarcoma:


a. P53 gene

368.

What are the common features in extrinsic allergic alveolitis:


a. Esinophils in the bronchial walls
b. Fibrosis and non-caseating granulomas

369.

What is the most common pancreatic tumor in females:


a. Mucinous cystaderenoma

370.

What do you see in NF1:


a. Plexiform schwannomas
b. Plexiform neurofibromas

371.

Which pleural tumor has a nodular or diffuse architecture and a positivity


for CD34, Bcl2 and CK:
a. Malignant mesothelioma
b. Solitary fibrous tumor
c. Synovial sarcoma

372.

Ulcerative colitis is associated with:


a. Primary sclerosing cholangitis

Reserved Copyrights - 2013

62

Pathology Questions

373.

MCQs

What is the immunoflouresence pattern of IgG in post-streptococcal


glomerulonephritis:
a. Linear
b. Course granular
c. Fine granular

374.

What is the causative organism for bacillary angiomatosis:


a. Bartonella species

375.

Which part of the kidney is affected in minimal change disease:


a. Mesangium
b. Epithelium
c. Blood vessels
d. Interstitium

376.

What is the most common glomerular disease associated with HCV:


a. Memberenoproliferative glomerulonephritis

377.

In which cardiac disease do you see Aschoff bodies:


a. Rheumatic heart disease

378.

Long-standing lymphatic stasis after mastectomy is a risk factor to


develop which of the following conditions:
a. Angiosarcoma

379.

What tumor has the following genetic abnormality t(X:17):


a. Alveolar soft part sarcoma

380.

What is the natural course for intraosseous ameloblastoma:


a. Recurrence
b. Transformation
c. Local invasion/local aggressiveness

Reserved Copyrights - 2013

63

Pathology Questions

381.

MCQs

Which tumor shows rhombdoid crystals on EM:


a. Alveolar soft part sarcoma

382.

Which of the following is considered a pancreatic tumor marker:


a. CA19-9
b. CEA
c. CA15-3
d. AFP

383.

B2 nephthalamine is a carcinogenic of which of the following tumors:


a. Urothelial carcinoma of the urinary bladder
b. Squamous cell carcinoma of the urinary bladder

384.

What is the most common cause of infections in patients with sickle cell
anemia:
a. Salmonella
b. Staph. Aureus

385.

What is the pathogenesis of osteoarthritis:


a. Articular degeneration of the cartilage
b. Decreased amounts of the synovial fluid

386.

Koilocytosis in a Pap smear indicates the following:


a. CIN II
b. HPV infection

387.

All of the followings are risk factors for cervical cancer, except:
a. Early marriage
b. Multiple partners
c. IUCD

Reserved Copyrights - 2013

64

Pathology Questions
388.

MCQs

What is the most likely underlying cause of a female presenting with


infertility and galactorrhea:
a. Pituitary adenoma

389.

All of the following are true about papillary thyroid carcinoma, except:
a. It can be multifocal
b. Radiation is a risk factor
c. A hematogenous spread is more common than a lymphatic spread

390.

A patient presented with tremors and exophthalamous. A thyroid nuclear


scans showed an increased diffuse uptake. What is the most likely
diagosis:
a. Graves disease
b. Multinodular goiter

391.

Formation of fistulas is part of which of the following entities:


a. Ulcerative colitis
b. Diverticulosis
c. Crohns disease

392.

Linitis plastica refers to which of the following:


a. An infiltrating tumor within the gastric wall

393.

Which of the following is true regarding HAV:


a. It is a ds-DNA virus
b. Spreads parenterally
c. Spreads via a fecal-oral route

394.

What is the most likely diagnosis for a patient presenting with a


mediastinal mass, granulomas and calcinuria:
a. Sarcoidosis
b. Tuberculosis

Reserved Copyrights - 2013

65

Pathology Questions
395.

MCQs

What does empyema characterize with:


a. High levels of sugar
b. High levels of protein

396.

What characterizes emphysema:


a. Valve destruction

397.

What characterizes liver cirrhosis:


a. Degenerative nodules
b. Fibrosis
c. Piecemeal necrosis

398.

What is the most common of pyogenic osteomyelitis:


a. Staph. Aureus

399.

Which of the following is a common complication of TB osteoarthritis:


a. Malignancy
b. Amyloidosis

400.

What is the most common site for prostatic adenocarcinoma:


a. Peripheral prostate
b. Central zone prostate
c. Transitional zone prostate

401.

What is the most common site for endometriosis:


a. Ovary
b. Ligaments
c. Cervix

402.

In regards to lobular carcinoma in situ of the breast, the following is


correct:
a. Other sites are likely to be involved
b. Estrogen receptor is usually negative

Reserved Copyrights - 2013

66

Pathology Questions

403.

MCQs

Cell death in hepatitis A is due to:


a. DNA damage
b. Stimulation of Bcl2
c. Stimulation of Bcl-x
d. An ATP-dependent process
e. Free radicals

404.

Which of the following features is seen in necrosis:


a. Inflammation in the surrounding tissue

405.

Which of the following findings can be seen in a renal biopsy from a


patient with diabetes and hypertension:
a. Fibrinoid necrosis
b. Glomeriolosclerosis/arteriosclerosis

406.

What is the cause of edema around brain tumors:


a. Lymphatic obstruction
b. Increased hydrostatic pressure
c. Decreased osmotic pressure
d. Mass effect

407.

Snap freezing can be used for all of the following tests, except:
a. EM
b. H&E
c. Muscle/nerve biopsy
d. Frozen section
e. Flow cytometry

408.

The optimal freezing temperature for breast tissue is:


a. -20c

409.

Anti-centromere antibodies are seen in which of the following conditions:

Reserved Copyrights - 2013

67

Pathology Questions

MCQs

a. CREST syndrome
b. Hashimotos thyroiditis
c. Reiter disease
410.

Anti-Smith antibodies are seen in which of the following conditions:


a. SLE
b. Ascending cholangitis
c. Sjogerns syndrome

411.

A smoking woman is on oral contraceptive pills, which of the following


risks is the greatest in this patient:
a. Cardiovascular disease/ischemic heart disease
b. Thromboembolism/thrombophlebitis
c. Breast carcinoma
d. Ovarian carcinoma
e. Endometrial carcinoma

412.

Working in the industry of batteries increases the risk of which of the


following conditions:
a. Renal toxicity
b. Lung cancer
c. Mesothelioma

413.

A tumor with spindle cell morphology, high N:C ratio, marked


pleomorphism, negativity for keratin and positivity for vimentin. What is
the most likely diagnosis:
a. 15-yr male with a lower limb tumor and lung metastasis
b. 5-yr old boy with a renal mass
c. 25-yr old boy with an enlarged left testis
d. An old lady with multiple ovarian tumors and massive ascitis
e. 35-yr old woman with left breast mass and enlarged lymph nodes

Reserved Copyrights - 2013

68

Pathology Questions
414.

MCQs

A 6-yr old by developed facial rash that has the appearance of a slap. The
rash, which is composed of small red spots, extended to the upper and
lower limbs. The boy had also arthralgia and he suddenly developed a lifethreatening aplastic crisis. The most likely cause of this presentation is:
a. Rhinovirus
b. Parvovirus
c. Parainfluenza virus
d. Measels
e. Rubella virus

415.

Which of the following statements is true:


a. Prostaglandins and cytokines are cell-derived mediators
b. Prostaglandins and cytokines are long-lived

416.

All of the following are para-neoplastic syndromes, except:


a. Cancer cachexia
b. Fever
c. Clubber
d. Cushings disease
e. Non-bacterial thrombotic endocarditis

417.

What is the most common brain location for CMV infection in a patient
with AIDS:
a. Periventricular/Intraventricular

418.

What is the most common complication of colon amebiasis:


a. Ulceration and perforation of the colon

419.

A baby of a diabetic mother is complaining of a bloody diarrhea. What is


the most probable diagnosis:
a. Abdominal radiograph showing submucosal gas bubbles

420.

When is the maximum wound strength reached:

Reserved Copyrights - 2013

69

Pathology Questions

MCQs

a. 3 months
b. 7 months
c. 1 year
421.

Which of the following is an autosomal recessiv disorder:


a. Spinal muscle atrophy

422.

What is the definition of genomics:


a. The study of genes and their interactions
b. The study of a single gene
c. The study of proteins and their interactions

423.

Which of the following is an example of a stop codon:


a. -thalassemia

424.

Which of the following is an example of a granulomatous disease:


a. Leprosy

425.

Which of the following may reduce the effect of toxic metabolites:


a. Glutathione

426.

Which of the following vitamins has an anti-toxic effect:


a. Vitamin K
b. Vitamin E
c. Vitamin C

427.

Which of the following is an inherited mutant gene in FAP:


a. APC gene

428.

What is the definition of grading:


a. The amount of differentiation

429.

Where is the amyloidosis of aging deposited:

Reserved Copyrights - 2013

70

Pathology Questions

MCQs

a. Liver
b. Lung
c. Heart
d. Brain
430.

Which of the following cell types is unable to proliferate:


a. Neuronal cells

431.

One of the following is an effect of an oncovirus:


a. T-cell leukemia

432.

What is the reason behind a negative immunostain in the centre of the


section:
a. Fixation problem
b. Antigen retrieval method

433.

What is the reason for the black pigment on a slide:


a. Formalin artifact

434.

What is the reason for bubbles on the slide:


a. Cover slip

435.

The following is a feature of autosomal recessive diseases:


a. They tend to be more uniform than autosomal dominant diseases

436.

The following is a feature of autosomal dominant diseases:


a. Some patients do not show the disease
b. It has reduced penetrance
c. Patients should have affected parents
d. The onset of the clinical features are early

437.

What are the advantages of a synaptic report:


a. Leads to an independent thinking

Reserved Copyrights - 2013

71

Pathology Questions

MCQs

b. Delay the turn-around time


c. It includes the diagnosis, prognostic factors, grade and stage
438.

What is the cause of thrombo-embolic vegetations over an area of a


myocardial infarction:
a. Endothelial injury
b. A change in the blood flow
c. Hypercoagulability
d. Statis of blood
e. Heart failure

439.

What are the changes seen in the breast during lactation:


a. Lobular hyperplasia

440.

What are the changes seen in the lower esophygus in a case of reflux:
a. Goblet cell metaplasia

441.

What is the common feature seen in a case of vasculitis:


a. Neutrophilic infiltration

442.

All of the following are features of acute inflammation, except:


a. Mononuclear inflammatory cell infiltrate

443.

Cytokines are mainly released from which of the following types of cells:
a. Monocytes and lymphocytes
b. Neutrophils

444.

What is the cause of hepatic fatty change in a case of alcoholism:


a. Impaired lipoprotein secretion from the liver
b. Increase trasport of lipid from the periphery

445.

What is the most common feature seen after a blood transfusion:


a. Accumulation of iron in the liver

Reserved Copyrights - 2013

72

Pathology Questions

446.

MCQs

Which mediator functions as an opsonin:


a. C3b
b. C5a

447.

Which type of inflammation is characterized by the presence of


esinophilic fiber or esinophilic meshwork thread:
a. Serous inflmmation
b. Fibrinous inflammation
c. Suppurative inflammation

448.

Which of the following mediators can cause acute phase reactants:


a. IL-1 and TNF

449.

Which of the following may prevent the process of wound healing:


a. Inflammation
b. Abnormal immune system
c. Infection

450.

What is the EM finding in a case of renal oncocytoma:


a. Mitochondria

451.

What are the most common causes of death in children between 1 4


years old:
a. Accidents, congenital malformations, chromosomal anomalies and
malignancies

452.

All is true regarding measles, except:


a. Recurrence is common
b. Multinucleated giant cells
c. Kopliks spots

Reserved Copyrights - 2013

73

Pathology Questions
453.

MCQs

The most common type of cancer to develop in a child with a previous


history of neck radiation is:
a. Thyroid cancer

454.

What does triple repeat mutation mean:


a. An amplification of a trinucleotide sequence

455.

Which one of the following is part of class MHC II:


a. It includes HLA-DP

456.

What do you the occurrence of a flat face, hypoplastic lung and deformed
feet:
a. Sequence
b. Syndrome
c. Malformation
d. Disruption

457.

A patient with acute myocardial infarction received anti-thrombotic


agents. After a while the levels of creatinine kinase increased. What is the
most common complication that has possibly occurred:
a. Reperfusion injury

458.

During inflammation, killing of viral elements occur in the:


a. Phagolysosome

459.

What is the pathogenesis of cystic fibrosis:


a. Abnormally folded protein with subsequent loss or degradation of
protein

460.

Which of the following is expected to be seen on an old rheumatic heart


valve:
a. Dystrophic calcification
b. Amyloidosis

Reserved Copyrights - 2013

74

Pathology Questions

MCQs

c. Metastatic calcification
461.

What is the definition of chemotaxis:


a. Migration of leukocytes to the site of inflammation

462.

Which of the following conditions can be seen after a burn injury and
consisted of elevated nodules containing fibroblasts:
a. Keloid

463.

What is the definition of a granuloma:


a. An aggregate of activated macrophages

464.

Which of the following is an example of a granulomatous disease:


a. Sarcoidosis

465.

All of the following are features of chronic inflammation, except:


a. Purulent exudate

466.

Which of the following is a feature of chronic inflammation:


a. Destruction to tissue and stroma

467.

What is the most probable cause of death in an obese lady during her
post-op hospital course:
a. Pulmonary embolism

468.

What do you call the blue lesions seen in alkaptonuria:


a. Ochronosis

469.

Phenylketunuria is an example of:


a. Autosomal recessive disease
b. X-link recessive
c. Autosomal dominant disease

Reserved Copyrights - 2013

75

Pathology Questions
470.

MCQs

Which of the following is an example of lysosomal storage disease:


a. Niemann pick disease

471.

A male patient with mental retardation, motor dysfunction and cherry red
spot in the retina. What would the most probable diagnosis be:
a. Tay-sachs disease

472.

Which of the following factors delay wound healing:


a. Steroids

473.

A newborn presented with clinical hypocalcemia and opportunistic


infections. What is the pathogenesis of this condition:
a. Defects in the 3rd and 4th pharyngeal pouches

474.

What is the mechanism of radiotherapy in eliminating cancer cells:


a. The release of free radicals

475.

A patient with SLE and renal disease would show what type of immune
complexes:
a. Subendothelial immune complex

476.

What is the pathogenesis behind a disease that presents with frequent


eyelid drops and muscle weakness:
a. Antibodies against the neuromuscular junction

477.

What is the mechanism of developing rash after a bee sting:


a. IgE production

478.

A reaction to helminthes is due to the production of:


a. Histamine

479.

A patient presented with a 4 cm breast carcinoma mass and metastases to


lymph nodes and distant organs. What is the correct stage of this patient:

Reserved Copyrights - 2013

76

Pathology Questions

MCQs

a. T2N1M1
480.

What is the site of action of histamine?


a. Arterioles
b. Precapillary venules
c. Venules
d. Postcapillaery venules
e. Capillaries

481.

Where is histamine released from?


a. Lymphocytes
b. Mast cells
c. Machrophages

482.

Where do cytotoxic T-cells act?


a. MHC Class I
b. MHC Class II

483.

How do cytotoxic Tcells kill organisms?


a. Granzyme b

484.

What is the meaning of a tumor with grade 1, stage 1:


a. A well differentiated and a localized tumor

485.

What is the most likely diagnosis of a slowly growing uterine mass of a 4yr duration:
a. Leiomyoma
b. Leiomyosarcoma

486.

A woman presented with vaginal discharge. A smear showed


multinucleated cells with a ground-glass appearance. What is the most
probable causative organism:
a. CMV

Reserved Copyrights - 2013

77

Pathology Questions

MCQs

b. HSV
c. Fungi
d. Dysplasia
487.

The target antibodies used for the diagnosis of HIV:


a. Gp120
b. Gp41
c. P24
d. Reverse transcriptase

488.

Which of the following compounds is deposited in the pancreas in a case


of hemochromatosis:
a. Iron
b. Copper
c. Lead

489.

Which of the following is an acute phase reactant:


a. Amyloid associated protein

490.

What do you call a leukocyte-induced injury:


a. Sarcoidosis
b. Interstitial fibrosis
c. Vasculitis
d. Tuberculosis

491.

How do macrophages destroy the extracellular matrix:


a. IL-2
b. Proteases
c. Catalases

492.

What is the most common cause of mental retardation:


a. Downs syndrome

Reserved Copyrights - 2013

78

Pathology Questions
493.

MCQs

What does RT-PCR mean:


a. Reverese transcriptase polymerase chain reaction

494.

The deficiency of which of the following enzymes can cause neuropathy,


glossitis, dermatitis and cheliosis:
a. Riboflavin (Vitamin B2)

495.

The deficiency of which of the following enzymes can cause contusions


and bleeding tendencies:
a. Vitamin K
b. Vitamin E

496.

A patient with Prader-Willi syndrome and an abnormal chromosome 15.


What is the most common complication of this disease:
a. Hypogonadism
b. Tall stature
c. Mental retardation
d. Ataxia

497.

Which of the following specimens can be diagnosed by only gross


examination:
a. Spleen in a case of sickle cell anemia
b. Non-union bone fracture

498.

Where do you see the following karyotype 46Xr(X):


a. Turners syndrome

499.

The single most important criterion to differentiate between benign and


malignant tumors is:
a. Invasion
b. Metastasis

Reserved Copyrights - 2013

79

Pathology Questions
500.

MCQs

Which of the following terminologies can not be used in a gross


description:
a. Lymphatic invasion

501.

Which of the following organs do not need to be inked:


a. The surface of the ovary
b. The radial margin of the colon
c. The capsule of the kidney

502.

What is the most common cause of systemic emboli:


a. Left ventricular heart failure
b. Left atrial emboli
c. Right ventricular heart failure

503.

Which of the following conditions is associated with pancreatic cancer:


a. Recurrent venous thrombi

504.

A patient with a history of small cell carcinoma of the lung has presented
with face puffiness and high blood pressure. His symptoms are caused by
the release of which of the following compounds:
a. ACTH
b. TSH
c. PTH

505.

Which of the following statements is incorrectly paired:


a. Anti ds-DNA polyarteritis nodosa
b. SS-B Sjogerens disease
c. Anti Smith antibodies SLE
d. Scl70 Scleroderma

506.

A patient in the FNA clinic has refused the procedure. What will you do
next:
a. Leave the room and give her time to think

Reserved Copyrights - 2013

80

Pathology Questions

MCQs

b. Try to convince her


507.

Which of the following compounds is a risk for squamous cell carcinoma:


a. Radon
b. Asbestos
c. Silica
d. Exposure to birds
e. Inorganic substances

508.

What is the most common cause of premature labor:


a. Premature rupture of membranes

509.

What is the cause of atopic dermatitis:


a. A local immediate hypersensitivity reaction

510.

What is the pathogenesis of a renal disease showing subendothelial


deposits on a renal biopsy:
a. Immune complex deposition

511.

Which of the following is a mutant gene in colon cancer:


a. RAS oncogene

512.

Which of the following is a stain for lipids:


a. Sudan Black

513.

A baby with trisomy 18 had microcephaly, abnormal kidneys, rocky feet


and overlapping fingers. What is this syndrome called:
a. Edwards syndrome

514.

What is the best prognostic factor in breast carcinoma:


a. Negative lymph node metastasis
b. Positive ER immunostain

Reserved Copyrights - 2013

81

Pathology Questions
515.

MCQs

What is the most common complication of anti-tuberculosis medication:


a. Vitamin B6 deficiency

516.

What is the pathogenesis of systemic lupus erythromatosus:


a. Type III hypersensitivity reaction

517.

All of the following is a differential diagnosis of Ewings sarcoma, except:


a. Rhabdomyosarcoma
b. Leiomyosarcoma
c. Neuroblastoma

518.

What is the most common site of cardiac myxoma:


a. Left atrium

519.

What is the most important feature in diabetic mastopathy:


a. Lymphocytic infiltrate and dense fibrosis

520.

Which of the following conditions has the immunostain profile: CD45+ve,


CD30+ve, CD20-ve:
a. Anaplastic T-cell lymphoma
b. Diffuse large B-cell lymphoma
c. Peripheral T-cell lymphoma

521.

Lacunar cells, caused by formalin artifact, in Hodgkin lymphoma are:


a. CD30 +ve
b. CD30 ve
c. Seen mainly in mixed cellularity variant

522.

What is the most common likely diagnosis of a mass in an old lady


between the scapula and the chest wall:
a. Elastofibroma

523.

All of the following are features of DM type I, except:

Reserved Copyrights - 2013

82

Pathology Questions

MCQs

a. Amyloid deposition
524.

Which of the following tumors is positive to CD20 and CDX2:


a. Colon cancer

525.

What is the most common paratesticular tumor in children:


a. Spindle cell variant of rhabdomyosarcoma

526.

Linear deposition of anti-GBM antibodies is seen in which of the following


conditions:
a. Goodpastures syndrome

527.

A positive stain for CD10 and Bcl6 is seen in:


a. Grade II follicular lymphoma
b. Diffuse large B-cell lymphoma

528.

How do neutrophils kill streptococci organims:


a. Hydrogen peroxide

529.

Which of the following statements is correct regarding Reed-Sternberg


cells:
a. The clearing of the lacunar variant is an artifact

530.

Which of the following is considered a prognostic factor in pancreatic


cancer:
a. Invasion to the peripancreatic tissue
b. Vimentin
c. Synaptophysin
d. GFAP

531.

In post lymphoproliferative disorders, all of the following are true, except:


a. It depends on the type of transplant
b. It regresses after stopping the immunosuppression

Reserved Copyrights - 2013

83

Pathology Questions

MCQs

c. EBV is negative in the early stages


d. It is associated with EBV infection
532.

Which of the following tumors has PTEN mutation:


a. Endometrial adenocarcinoma

533.

Which of the following tumors is TTF1 ve & CDX2 ve:


a. Primary lung adenocarcinoma
b. Mucinous bronchoalveolar carcinoma
c. Non-mucinous bronchoalveolar carcinoma
d. Metastatic colon adenocarcinoma

534.

How would you know that a burn patient was alive at the time of fire:
a. The presence of carbon/suet in his/her lungs

535.

Which of the following tumors has a tubulovillous architecture and ER/PR


positive immunostains:
a. Endometroid adenocarcinoma

536.

Which of the following is a consequence of long-standing lymphedema


post mastectomy:
a. Angiosarcoma

537.

What is the most characteristic ultrastructural finding in the myocardium:


a. Intercalating discs

538.

What is the most common skin tumor in HIV patients:


a. Kaposi sarcoma

539.

Which of the following conditions can be diagnosed with a peripheral


blood smear?
a. Chronic myeloid leukemia

Reserved Copyrights - 2013

84

Pathology Questions
540.

MCQs

All of the following are features of glioblastoma multiforme, except:


a. Proliferating blood vessels
b. Palisading necrosis
c. Rosenthal fibers

541.

The most common somatic tumor arising from a teratoma is which of the
following:
a. Squamous cell carcinoma

542.

Which of the following interstitial lung diseases is characterized by a


variegated appearance on low power:
a. Desquamative interstitial pneumonia
b. Usual interstitial pneumonia
c. Organizing pneumonia

543.

Which of the following immunostain panels is characteristic to a primary


lung adenocarcinoma:
a. CK7 +ve, TTF1 +ve, CDX2 ve, P63 ve

544.

Which of the following conditions occurs exclusively in women during


their reproductive age:
a. Lymphangioleomatosis

545.

A patient with a 12 years history of dyspnea and bilateral pulmonary


infiltrate on X-ray. What is the most possible cause:
a. Inorganic dust
b. Organic particles
c. Bird dust
d. Radon

546.

A patient with a peripheral lung mass was found to have atypical cells in
a sputum smear. A diagnosis of squamous cell carcinoma was made.
Which of the following is considered a risk factor:

Reserved Copyrights - 2013

85

Pathology Questions

MCQs

a. Radon gas
b. Immunodeficiency
547.

What would you see in a patient with myasthenia graves:


a. Thymic follicular hyperplasia
b. Thymoma
c. Thymolipoma
d. Thymic carcinoma

548.

What is the most common site for laryngeal carcinoma:


a. Epiglottis
b. Glottis
c. Supraglottis

549.

Which of the following salivary gland tumors has no myoepithelial cell


layer:
a. Adenoid cystic carcinoma
b. Canalicular adenoma
c. Polymorphous low grade adenocarcinoma
d. Basal cell adenoma
e. Pleomorphic adenoma

550.

What is the most common salivary gland malignancy in children:


a. Mucoepidermoid carcinoma

551.

Which of the following tumors has the translocation t(2:5):


a. Anaplastic large cell lymphoma

552.

Which of the following tumors is least likely to metastasize to lymph


nodes:
a. Leiomyosarcoma
b. Synovial sarcoma
c. Rhabdomyosarcoma

Reserved Copyrights - 2013

86

Pathology Questions

MCQs

d. Osteosarcoma
e. Epithelioid sarcoma
553.

Which cardiac disease can be diagnosed by a biopsy from another less


invasive body site:
a. Hypertrophic cardiomyopathy
b. Dilated cardiomyopathy
c. Leoffler carditis
d. Restrictive cardiomyopathy

554.

What will constrictive pericarditis cause in the liver:


a. Bile duct proliferation
b. Sinosidal dilatation

555.

What stain can be used to differentiate lipofuscin and hemosideren:


a. Iron
b. Calcium
c. Lipid

556.

Which of the following carcinomas is associated with EBV:


a. Lymphoepithelial carcinoma

557.

The morphology of post infectious glomerulonephritis includes:


a. Hypercellular acute inflammatory cells infiltrate
b. Double contour

558.

The following ultrastructural features (pinocytic vacules, lipid bodies and


cytoplasmic filaments) are found in which of the following tumors:
a. Leiomyosarcoma
b. Fibrosarcoma
c. SCC
d. Adenocarcinoma
e. Synovial sarcoma

Reserved Copyrights - 2013

87

Pathology Questions

559.

MCQs

Where do you see premelanosomes under EM examination:


a. Angiomyolipoma

560.

A pap smear showed hyperchromatic columnar cells with intact cilia and
central nuclei. What is the probable diagnosis:
a. Tubal metaplasia
b. Adenocarcinoma in situ
c. SCC

561.

A pap smear showed a cluster of multinucleation cells with centrally


located nuclei. What is the probable diagnosis:
a. Syncitiotrophoblasts
b. SCC
c. Cytotrophoblasts

562.

All of the following features are seen in cytology specimen of a carcinoid,


except:
a. Nucleoli

563.

Which tumor shows cytoplasmic glycogen on EM examination:


a. Ewing sarcoma

564.

Which combination of the following cause episomal replication:


a. HPV 6 & 11
b. HPV 30 & 33
c. HPV 16 & 18

565.

The least common site of bone metastais is:


a. Skull
b. Ribs
c. Pelvis
d. Femur
e. Hand

Reserved Copyrights - 2013

88

Pathology Questions

566.

MCQs

A 60-year old male with a lytic lesion in the right iliac bone with spotty
calcifications and extentions to the surrounding soft tissue. What is the
probable diagnosis:
a. Chondrosarcoma
b. Pagets disease

567.

The least tumor metastesing to the bone is:


a. Squamous cell carcinoma of skin
b. Renal cell carcinoma
c. Breast carcinoma
d. Prostate carcinoma
e. Thyroid carcinoma

568.

In vaginal normal pH, what type of organisms can be seen?


a. Actinomyces
b. Trichomonas vaginalis
c. Lactobacilli acidophilic

569.

In acute inflammation, the interstitial edema is due to:


a. Neutrophils
b. Lymphatic obstruction
c. Endothelial retraction
d. Arteriolar contriction
e. Adhesion molecules

570.

Spongiform encephalitis is seen in:


a. Prion protein

571.

What would you seen in the brain of a patient with Alzehiemer disease:
a. Hypertrophy
b. Hyperplasia
c. Dysplasia

Reserved Copyrights - 2013

89

Pathology Questions

MCQs

d. Atrophy
572.

An elderly woman, with no known medical illnesses, was visited by her


son. Four hours later she was found dead in her bathtub with frontal
bruises and subarachnoid hemorrhage. What is the cause of death?
a. Fall
b. Strangulation
c. Drowning

573.

A 70-year old patient with a history of GBM for 1 year. Ten days after an
unremarkable check up with his family physician, he was found dead. The
autopsy report should state the following:
a. The cause of death is cardiac arrest
b. Smoking should be written under associated conditions
c. GBM is the underlying cause of death

574.

The least tumor positive to CD34 is:


a. Synovial sarcoma

575.

A patient with clinically apparent cutaneous crops which regress


spontaneously has probably the following diagnosis:
a. Sezary syndrome
b. Lymphomatoid papulosis
c. MALT lymphoma

576.

Which of the following is a risk factor for diabetes type II:


a. Loss of B-cells
b. Increase insulin resistance
c. Chronic renal failure
d. Amyloid deposition

577.

What is the most common site for osteochondroma:


a. Metaphysis

Reserved Copyrights - 2013

90

Pathology Questions

578.

MCQs

What is the most common glomerular disease world-wide?


a. IgA nephropathy

579.

ATPase pattern of staining would show the following in neuronal atrophy:


a. Group atrophy
b. Type two fibers
c. Internalization of nuclei

580.

A rupture of a berry aneurysm in a circle of wills will cause the following:


a. Subarachnoid hemorrhage

581.

Which of the following is an active process that augments blood to the


arterioles?
a. Hyperemia
b. Cyanosis
c. Congestion
d. Edema

582.

What is the site of action of p53?


a. G1-S
b. G2-M
c. G0-S

583.

Which of the following describes a familial disease due to a structural


protein defect appearing in adulthood?
a. Autosomal dominant
b. Autosomal recessive
c. X-linked recessive
d. X-linked dominant
e. Sporadic

584.

What is the cause of blackening of the joints and urine?

Reserved Copyrights - 2013

91

Pathology Questions

MCQs

a. Homogentesic acid deficeincy


b. Hexaminase deficeincy
c. Protein c defect
d. Glucose 6 phosphatase
e. Fibrillin defect
585.

Which of the following factors inhibit cell growth in repair?


a. Heparin sulfate
b. Interferon gama
c. TGF-
d. Cyclic AMP

586.

A pateint with a history of multiple myeloma presented with multiple lytic


bone lesions. What is the most common cause of the lytic lesions?
a. TGF
b. TNF
c. PDGF
d. IL-2
e. IL-6

587.

Which of the following is the most important action to reduce error in


pathology?
a. Vigorous cleaning of all instrument after each case
b. Changing the blade of the microtome every day
c. Clean the water bath by the end of each day

588.

Which of the following parameters is not important to be mentioned in a


breast pathology report?
a. Number of lymph node mets
b. Perineural invasion
c. Size
d. Tumor cell type

Reserved Copyrights - 2013

92

Pathology Questions
589.

MCQs

What should a report of an osteosarcoma include?


a. Percentage of tumor necrosis

590.

Which of the following variants of basal cell carcinoma has the highiest
rate of recurrence?
a. Macronodular BCC
b. Morphia-like BCC
c. Pigmented BCC
d. Glandular BCC

591.

18-skin, the lichened reaction around skin adnexa:


a. Lichen planus
b. Lichen nitidus
c. Lichen striatus
d. Lichen planus pigmentosus
e. Lichenoid drug eruption

592.

In lung, the grade of neuroendocrine tumor depends on:


a. Size
b. Pleomrphoism
c. Mitoses

593.

A patenit with diarrhea, fever, flushing and a plaque on the tricuspid


valve. What is the most probable cause?
a. RHD
b. Hypothyroidism
c. Amyloidosis
d. Carcinoid heart disease

594.

Which of the following is not normal in urine cytology?


a. Umbrella cells
b. Squamous cells
c. Sperms
d. Multinucleated giant cells

Reserved Copyrights - 2013

93

Pathology Questions

MCQs

e. Renal tubular epithelium


595.

A bronchial lavage shows groups of monotonous plasmacytoid small


hyperchromatic cells in tubules with inconspicuous nucleoli. No cilia are
seen. What is the most probable diagnosis?
a. Reserve cell hyperplasia
b. Small cell carcinoma
c. Carcinoid
d. Poorly differentiated squamous carcinoma

596.

Which of the following variants of papillary thyroid carcinoma occurs


more commonly in children?
a. Solid
b. Microfollicular
c. Nodular fasciitis-like
d. Macrofllicular
e. Encapsulated

597.

What is the most likely breast tumor to stain with estrogen receptor?
a. Mucinous
b. Lobular
c. Ductal
d. Medullary

598.

Which of the following solutions dissolve fat?


a. Bouins
b. Carnoys
c. B5
d. Formaline

599.

Which type of hemangiomas is reactive with GLUT1?


a. Infantile hemangioma
b. Lobular capillary hemangioma

Reserved Copyrights - 2013

94

Pathology Questions

600.

MCQs

What is Whipple triad?


a. Hypoglycemia, signs of hypoglycemia, response to sugar intake
b. Hypocalcaemia, signs of hypocalcaemia, response to calcium intake
c. Steatorrhea, diarrhea, DM

601.

Where do you see Raccoon eye sign?


a. Basal skull fracture

602.

What would be the effect of a lesion occupying the ventromedial aspect of


the hypothalamus?
a. Increase appetite
b. Decrease appetite
c. Tunnel vision
d. Increase urination
e. Paralysis of extraocular muscle

603.

A pateint presented with a history of smoking (20 packs/yr),


hyponaterima, weight gain and hyperosmolar urine. What is the most
probable diagnosis?
a. SIADH

604.

A lady has presented with a whitish patch in vulva which was diagnosed
as lichened sclerosis. What would the histology show?
a. Atrophic epidermis and sclerosed dermis
b. SCC
c. Hyperplasia with atypia

605.

What is the most common site for glomus tumor?


a. Finger and toe

606.

Which of the following tumors is characterized by a local aggressive


behavior and late metastasis?

Reserved Copyrights - 2013

95

Pathology Questions

MCQs

a. Epitholod hemangioendothlioma
b. GCT
c. Fibromatosis
607.

Where do acral lentigenous melanomas occur?


a. Palm and sole

608.

What is the underlying cause of a pateint with SLE and and presenting
with a cardiac valve disease?
a. Anticardiolipin antibodies
b. Bacteria
c. Turbulent blood flow
d. Increased blood viscosity

609.

What is the microscopic picuture of BOOP?


a. Proliferation of fibroblasts within the alveolar walls

610.

Which of the following malignanies increased in children after the the war
of 1968?
a. Thyroid carcinoma
b. Breast carcinoma

611.

A 20-year old young adult came to the ER with respiratory distress and a
medistinal mass. What is the most probable diagnosis?
a. T-cell lymphoma
b. B-cell lymphoma
c. Thymoma

612.

Which of the following tissues is able to regenerate?


a. Liver
b. Dermis
c. Skeletal muscles
d. Cardiac muscles

Reserved Copyrights - 2013

96

Pathology Questions

MCQs

e. Cornea
613.

Which of the following factors favors a diagnosis of hepatoblastoma over


a hepatoceller adenoma:
a. Age <10 years
b. A large mass of mature liver cell
c. Increased -fetoprotein levels

614.

Which of the following is a common feature of membranous


glomerulonephritis and post-strepotococcal glomeonephritis?
a. Both occur in the young age group
b. Both have subepithelial deposits
c. Both resolve spontaneously
d. Both present with nephrotic syndrome

615.

Which of the following diseases show acid fast bacilli around peripheral
nerves?
a. Leprosy
b. TB

616.

Which of the following cells is responsibale for liver cirrhosis?


a. Sinusoidal endothelial cells
b. Ita cells

617.

What is the hyaline cytoplasmic material seen in alcoholic hepatitis?


a. Mallory bodies

618.

A 30-year old male presented with a mediastinal mass showing hyalinized


lymphoid follicles & abundant plasma cells. What is the probable
diagnosis?
a. Castleman disease
b. Hodgkins diseasee
c. Plasmacytoma of lymph node

Reserved Copyrights - 2013

97

Pathology Questions

619.

MCQs

Progressive transformation of germinal centers is associated with which


of the following lymphomas?
a. NLPHL
b. Classic HL, nodular sclerosis
c. Follicular lymphoma

620.

What is the most common lymphoma in HIV patients?


a. Burkitt lymphoma
b. DLBCL

621.

A 30-year old HIV patient presented with cough & dyspnea. A lung biopsy
showed frothy alveolar material. Which stain is best used to identify the
causative organism?
a. Modified methelamin silver stain

622.

Which of the following ovarian tumors is most often associated with


pseuoprecocious puberty?
a. Juvenile granulosa cell tumor

623.

Which of the following ovarian tumors has the following profile: PLAP+,
OCT4+, CD30+, CK+, CD117- ?
a. Embryonal carcinoma

624.

Which of the following uterine tumors have a tubuloalveolar appearance


& PTEN mutation?
a. Endometrioid endometrial adenocarcinoma

625.

A 6-year old boy presented with an abdominal mass which showed:


CD20+, CD10+, BCL-6+, CD19+, CD43+, BCL-2-, and >90% proliferation
index. What is the genetic mutation underlying this tumor?
a. t(8:14)

Reserved Copyrights - 2013

98

Pathology Questions
626.

MCQs

A 50-year old female with history of prolonged diarrhea, nausea,


vomiting, gastric hyperacidity and multiple duodenal ulcers will probably
have which of the following conditions?
a. Gastrin secreting tumor of pancreas
b. Insulinoma

627.

A female patient is known to have a prolactin-secreting anterior pituitary


tumor; what will be her clinical manifestation?
a. 30 years old, with amenorrhea, galactorrhea & -ve pregnancy test

628.

What is the appropriate follow up of a patient with ASC-H on PAP smear?


a. Colposcopy

629.

A patient presented with hemoptysis, rapidly progressive renal failure


and cresent formation in a renal biopsy with linear immune complex
deposit. What is the most probable diagnosis?
a. Goodpasture syndrome

630.

What is the most common causative organism of acute pyelonephritis?


a. E coli
b. Strep. group A
c. Staph aureus

631.

What is the most common genetic abnormality in clear cell RCC?


a. Deletion of chromosome 3
b. Abnormality in chromosome 12
c. Loss of Y chromosome
d. Loss of 1 chromosome

632.

What is the cause of renal papillary necrosis?


a. DM
b. Interstesial nephritis

Reserved Copyrights - 2013

99

Pathology Questions
633.

MCQs

What is the most likely diagnosis of a destructive metaphysical cortical


lesion in the distal femur with soft tissue extention in a 15 years old boy?
a. Osteosarcoma

634.

What is the first antibody that appears in acute inflammation and can not
cross the placenta?
a. IgM
b. IgG
c. IgA

635.

Which of the following stains is positive in adrenocortical tumors?


a. Melan A
b. Cytokeratin

636.

Which of the following uterine tumors is associated with unaopposed


estrogen effect and preceeded by endometrial hyperplasia?
a. Endometroid adenocarcinoma
b. Papillary serrous carcinoma

637.

Which of the following conditions is characterized by: hypoalbunimia,


hyperlipidemia, hypertension, fat casts and lipid vacuoles?
a. Nephritic syndrome
b. Nephrotic syndrome

638.

Which of the following is the most sensitive testicular tumor to


radiotherapy?
a. Seminoma

639.

Which of the following liver disease show ducropenia?


a. HBV
b. HCV
c. PBC
d. HCC

Reserved Copyrights - 2013

100

Pathology Questions
640.

MCQs

For each of the following questions choose the most likely diagnosis from
the following diagnostic set (an answer may be used once, more than
once, or not at all) (PIP 2000 A)
Diagnostic set:
a. Infiltrating duct carcinoma.
b. Infiltrating lobular carcinoma.
c. Langerhan's cell histiocytosis.
d. Malignant non-Hodgkin's lymphoma, small cell diffuse type.
e. Metastatic gastric carcinoma to the breast.
f. Sclerosing adenosis.
Which most often demonstrates a sinus catarrh pattern of metastasis to
axillary lymph nodes?
b. Infiltrating lobular carcinoma.
Which lesion typically shows negative reactions for cytokeratin and S100
immunostains?
d. Malignant non-Hodgkin's lymphoma, small cell diffuse type.
Which lesion often demonstrates Her2/neu overexpression?
a. Infiltrating duct carcinoma.

641.

For each of the following questions choose the most likely diagnosis from
the following diagnostic set (an answer may be used once, more than
once, or not at all) (PIP 2000 A)
Diagnostic set:
a. Burkitt's lymphoma.
b. Extramedullary myeloid tumor.
c. Large cell non-Hodgkin's lymphoma.
d. Lymphoblastic lymphoma.
e. MALT lymphoma.
f. Mantle cell lymphoma.
g. Small cell carcinoma.

Reserved Copyrights - 2013

101

Pathology Questions

MCQs

Which tumor is associated with eosinophilic myelocytes?


b. Extramedullary myeloid tumor.
Which tumor typically expresses CD3 and TdT?
d. Lymphoblastic lymphoma.
Which tumor cells demonstrate a clonal t(11;14) translocation?
f. Mantle cell lymphoma.
The gastrointestinal presentation of which tumor often resembles
multiple polyps:
f. Mantle cell lymphoma.
642.

Classic Wilm's tumor has which of the following characteristics? (PIP


2000 A)
a. Triphasic histology with epithelial, blastemal, and mesenchymal
components.
b. Prognosis favorable, with greater than 80 percent cure rate.
c. Stage is based on extent of tumor, extent of resection, and
presence of metastasis.
d. Association with dysmorphic syndromes.
e. All of the above.

643.

Malignant rhabdoid tumor of kidney has which of the following


characteristics?(PIP 2000 A)
a. Favorable prognosis, with greater than 50 percent cure rate.
b. Immunohistochemichal profile identical to rhabdomyosarcoma.
c. Association with primary or metastatic central nervous system
neoplasms.
d. Association with Von Hipple-Lindau syndrome.
e. Tendency for metastasis to bone.

Reserved Copyrights - 2013

102

Pathology Questions
644.

MCQs

Clear cell sarcoma of kidney has which of the following characteristics?


(PIP 2000 A)
a. Deceptively bland histologic appearance.
b. Arcading vascular pattern.
c. Tendency for metastasis to bone.
d. Peak incidence between 6 months and 2 years of age.
e. All of the above.

645.

Which of the following ovarian neoplasms would be least likely to exhibit


immunoreactivity for inhibin? (PIP 2000 A)
a. Adult granulosa cell tumor.
b. Endometrioid adenocarcinoma.
c. Fibrothecoma.
d. Juvenile granulosa cell tumor.
e. Sertoli-Leydig cell tumor.

646.

Which of the following ovarian neoplasms is most frequently associated


with virilism? (PIP 2000 A)
a. Strumal carcinoid.
b. Sertoli-Leydig cell tumor.
c. Adult granulosa cell tumor.
d. Immature teratoma.

647.

Which of the following is the most common ovarian sex cord-stromal


tumor? (PIP 2000 A)
a. Endometrioid carcinoma.
b. Gynadroblastoma.
c. Granulosa cell tumor.
d. Sertoli-Leydig cell tumor.
e. Teratoma.

Reserved Copyrights - 2013

103

Pathology Questions
648.

MCQs

For each of the following questions choose the most likely diagnosis from
the following diagnostic set (an answer may be used once, more than
once, or not at all) (PIP 2000 A)
Diagnostic set:
a. Acinar cell carcinoma.
b. Ductal adenocarcinoma.
c. Microcytic adenoma.
d. Pancreatic endocrine tumor.
e. Pancreatoblastoma.
f. Solid and pseudopapillary carcinoma.
Which is characterized histologically by a mixture of glandular, acinar,
endocrine and squamoid elements?
e. Pancreatoblastoma.
Which is characterized histologically by PAS positive diastase-resistant
cytoplasmic granules?
a. Acinar cell carcinoma.
Which occurs primarily in young women?
f. Solid and pseudopapillary carcinoma.

649.

For each of the following questions choose the most likely diagnosis from
the following diagnostic set (an answer may be used once, more than
once, or not at all) (PIP 2000 A)
Diagnostic set:
a. Adrenal cortical carcinoma.
b. Angiomyolipoma.
c. Angiomyolipoma.
d. Atypical lipoma.
e. Leiomyosarcoma
f. Lipoma.
g. Liposarcoma.

Reserved Copyrights - 2013

104

Pathology Questions

MCQs

h. Oncocytoma.
i.

Renal cell carcinoma.

Which lesion exhibits HMB45-positive, keratin-negative perivascular


epithelioid cells?
c. Angiomyolipoma
Which lesion is the most common malignant stomal tumor of the kidney?
e. Leiomyosarcoma
Which lesion is most commonly associated with tuberous sclerosis
complex?
c. Angiomyolipoma
650.

Which of the following features is associated with recurrence of desmoid


tumor (abdominal fibromatosis)? (PIP 2000 A)
a. Positive resection margins.
b. Abundant centrally located slit-like vessels in the mass.
c. Familial adenomatous polyposis.
d. Younger age.
e. All of the above.

651.

Familial adenomatous polyposis coli is associated with which of the


following tumors? (PIP 2000 A)
a. Inflammatory myofibroblastic tumor.
b. Nodular fasciitis.
c. Desmoid tumor.
d. Fibrosarcoma.
e. Neurofibroma.

652.

Which of the following features distinguish reactive fibrosis from desmoid


tumor? (PIP 2000 A)
a. Recent and/or old hemorrhage.

Reserved Copyrights - 2013

105

Pathology Questions

MCQs

b. Abundant collagen.
c. Circumscription.
d. Cellular atypia.
e. Abundant fibroblasts.
653.

For each of the following questions choose the most likely diagnosisfrom
the following diagnostic set (an answer may be used once, more than
once, or not at all) (PIP 2000 A)
Diagnostic set:
a. Adenosarcoma.
b. Malignant mixed mllerian tumor crcinosarcoma, heterologous
type.
c. Poorly differentiated endometrial carcinoma with spindle cell
metaplasia.
d. Primary uterine osteogenic sarcoma(heterologous sarcoma of
uterus).
e. Teratoma with malignant transformation.
A large malignant uterine neoplasm typically containing cartilage:
b. Malignant mixed mllerian tumor crcinosarcoma, heterologous
type.
Squamous cell carcinoma is the most common histologic component in
this neoplasm:
e. Teratoma with malignant transformation.
This neoplasm is characterized by sarcomatous collarettes compressing
tubular glands:
a. Adenosarcoma.

654.

For each of the following questions choose the most likely diagnosis from
the following diagnostic set (an answer may be used once, more than
once, or not at all) (PIP 2000 A)

Reserved Copyrights - 2013

106

Pathology Questions

MCQs

Diagnostic set:
a. Choristoma.
b. Dermoid cyst.
c. Immature teratoma.
d. Mature teratoma.
e. Polyembryoma.
f. Teratoma with malignant transformation.
g. Teratocarcinoma.
Which testicular tumor is much more likely to occur in prepubertal
children than adults?
d. Mature teratoma.
Adenocarcinoma occuring in a teratoma is designated:
f. Teratoma with malignant transformation
A primitive neuroendocrine tumor (PNET) is most likely to occur in:
c. Immature teratoma.
655.

For each of the following questions choose the most likely diagnosis from
the following diagnostic set (an answer may be used once, more than
once, or not at all) (PIP 2000 A)
Diagnostic set:
a. Adenovirus.
b. Behet's syndrome.
c. Candida albicans.
d. Chemichal esophagitis.
e. Cytomegalovirus.
f. Herpes simplex.
g. Reflux esophagitis.
h. Squamous cell carcinoma.

Reserved Copyrights - 2013

107

Pathology Questions

MCQs

Granular, eosinophilic cytoplasmic inclusions are useful in the


identification of which agent?
e. Cytomegalovirus.
Esophagitis associated with inflammation of the oral cavity and ocular
and genital areas is characteristic of which entity?
b. Behet's syndrome.
Which infectious agent commonly involves epithelial cells, endothelial
cells, smooth muscle and ganglion cells of the gastrointestinal tract?
e. Cytomegalovirus.
656.

For each of the following questions choose the most likely diagnosis from
the following diagnostic set (an answer may be used once, more than
once, or not at all) (PIP 2000 B)
Diagnostic set:
a. Glioblastoma multiforme.
b. Gliomatosis cerebri.
c. Muultiple sclerosis.
d. Post-encephalitic gliosis.
e. Viral encephalitis.
Multiple lobes of the brain are diffusely involved by neoplastic astrocytes
in which condition?
b.Gliomatosis cerebri.
Relative axonal preservation is a characteristic feature of which
condition?
c. Muultiple sclerosis.
Pseudopalisading with central necrosis is the classic histological finding
in which lesion?
a. Glioblastoma multiforme.

Reserved Copyrights - 2013

108

Pathology Questions

657.

MCQs

The most important fact predicting survival in patients with mucinous


tumor of borderline malignancy (mucinous tumor of low malignant
potential of the ovary)is: (PIP 2000 B)
a. Architectural complexity.
b. Tumor size.
c. Tumor stage.
d. Patient age.
e. Nuclear grade.

658.

Which of the following staetments about mucinous tumor of borderline


malignancy (mucinous tumor of low malignant potential of the ovary) is
true: (PIP 2000 B)
a. Bilateral ovarian involvement is seen in the majority of patients.
b. It accounts for the majority of ovarian mucinous tumors.
c. The tumor cells resemble endometrial cells.
d. Destructive stromal invasion can be present.
e. None of the above is true.

659.

Which of the following is most likely in carcinoid tumor consistent with


thymic origin: (PIP 2000 B)
a. Immunoreactivity

for

chromogranin

and

cytokeratin,

and

identification of neurosecretory granules by electron microscopy.


b. Immunoreactivity for leukocyte common antigen CD45 and CD20
(L26).
c. Immunoreactivity for placental alkaline phosphatase and Leu-7,
and negative cytokeratin stains.
d. Immunoreactivity for chromogranin, negative cytokeratin stains,
and identification of neurosecretory granules by electron
microscopy.
e. Immunoreactivity for CD15 (LeuM1) and CD30 (BerH2),
andnegative leukocyte common antigen.

Reserved Copyrights - 2013

109

Pathology Questions
660.

MCQs

Multiple endocrine neoplasia (MEN) syndromes are most likely


associated with which of the following mediastinal tumors? (PIP 2000 B)
a. Thymoma.
b. Teratoma, mature.
c. Carcinoid tumor.
d. Malignant lymphoma.
e. Seminoma.

661.

All of are true about sarcomatoid renal cell carcinoma except: (PIP 2000
B)
a. Survival rate usually less than one year.
b. High-rate extrarenal spread and metastasis at time of surgery.
c. HMB-45 is usually positive.
d. Usually arises in the renal cortex.
e. May have foci of osteosarcoma or chondrosarcoma.

662.

All of the following are true about angiomyolipma except: (PIP 2000 B)
a. HMB-45 is usually positive.
b. Strong association with tuberous sclerosis.
c. Clinical findings (flank pain and hematuria) similar to renal cell
carcinoma.
d. Most common renal tumor of early infancy.
e. May be multifocal and bilateral, but rarely malignant.

663.

For each of the following questions choose the most likely diagnosis from
the following diagnostic set (an answer may be used once, morethan once,
or not at all). (PIP 2000 B)
Diagnostic set:
a. Acute necrotizing eosinophilic myocarditis.
b. Chagas disease.
c. Giant cell myocarditis.
d. Hypereosinphillic syndrome.
e. Hypersensitivity/eosinophilic myocarditis.

Reserved Copyrights - 2013

110

Pathology Questions

MCQs

f. Lymphocytic myocarditis.
g. Large cell lymphoma.
h. Toxoplasmosis.
Which disease is characterized by seroiginous areas of necrosis with
numerous multinucleated histiocytes?
c. Giant cell myocarditis.
Which disease is characterized by an infiltrate of histiocytes and
eosinophils around vessels and in cardiac connective tissue planes?
e. Hypersensitivity/eosinophilic myocarditis.
Which disease is characterized by the presence of cardiac mural
thrombi?
d. Hypereosinphillic syndrome.
664.

Which of the following pulmonary infections is characterized by a


necrotizing vasculitis? (PIP 2000 B)
a. Legionella pneumonia.
b. Thoracic actinomycosis.
c. Psuedomonas pneumonia.
d. Cryptococosis.
e. Cytomegalovirus.

665.

Granulomatous inflammation is commonly associated with which of the


following? (PIP 2000 B)
a. Legionella pneumonia.
b. Cryptococosis.
c. Pulmonary tularemia.
d. Cytomegalovirus infecton.
e. Thoracic actinomycosis.

666.

Which of the following statements is true? (PIP 2000 B)

Reserved Copyrights - 2013

111

Pathology Questions

MCQs

a. Viral culture is more sensitive than histologic examination of


tissue in diagnosing CMV pneumonia.
b. Legionella pneumonia is caused only by legionella pnemophilia
species.
c. Pulmonary tularemia is characteristically associated with acute
necrotizing vasculitis.
d. Legionella bacteria are easily identified in tissue sections stained
with Gram's stain.
e. Actinomycosis commonly stains with an acid fast stain.
667.

For each of the following questions choose the most likely diagnosis from
the following diagnostic set (an answer may be used once, more than
once, or not at all). (PIP 2000 B)
Diagnostic set:
a. Hairy cell leukemia.
b. Hepatosplenic T-cell leukemia.
c. Mantle cell lymphoma.
d. Marginal zone lymphoma.
e. Prolymphocytic leukemia.
f. Small lymphocytic lymphoma/ chronic lymphocytic leukemia.
g. Systemic mastocytosis.
Which lymphoproliferative disorder typically co-expresses CD5 and CD23
in addition to pan B-cell markers?
f. Small lymphocytic lymphoma/ chronic lymphocytic leukemia.
Which B-cell neoplasm shows predominantly red pulp infiltration in the
spleen.
a. Hairy cell leukemia.
Which neoplasm is cytologically and immunophenotypically similar to
hairy cell leukemia but shows a different pattern of involvement in the
spleen?

Reserved Copyrights - 2013

112

Pathology Questions

MCQs

d. Marginal zone lymphoma.


668.

For each of the following questions choose the most likely diagnosis from
the following diagnostic set (an answer may be used once, more than
once, or not at all). (PIP 2000 B)
Diagnostic set:
a. Acinar cell carcinoma.
b. Ductal adenocarcinoma.
c. Nesidioblastosis.
d. Pancreatic endocrine tumor.
e. Pancreatoblastoma.
f. Serous (microcytic) adenoma.
g. Solid and pseudopapillary epithelial neoplasm.
h. Undifferentiated (anaplastic) carcinoma.
Which lesion is most commonly seen in young children and characterized
by squamoid corpuscles?
e. Pancreatoblastoma.
Which lesion is most commonly associated with subcutaneous fat
necrosis?
a. Acinar cell carcinoma
Which lesion is seen most often in patients with Zollinger-Ellison
syndrome?
d. Pancreatic endocrine tumor.

669.

For each of the following questions choose the most likely diagnosis from
the following diagnostic set (an answer may be used once, more than
once, or not at all). (PIP 2000 B)
Diagnostic set:
a. Adenomatoid tumor.
b. Adenomyoma.

Reserved Copyrights - 2013

113

Pathology Questions

MCQs

c. Adenosarcoma.
d. Carcinosarcoma (malignant mixed mllerian tumor.
e. Endometrial stromal nodule.
f. Endometrial stromal sarcoma.
g. Leiomyoma.
h. Leiomyosarcoma.
i.

Stromomyoma.

Which lesion is characterized by an admixture of benign glandular


epithelium and malignant stroma?
c. Adenosarcoma.
Which lesion is the most frequent benign tumor of the fallopian tube?
a. Adenomatoid tumor.
Which lesion consists of a circumscribed, nodular mass of benign smooth
muscle with interspersed islands of benign endometrial glands and
stroma?
b. Adenomyoma.
670.

For each of the following questions choose the most likely diagnosis from
the following diagnostic set (an answer may be used once, more than
once, or not at all). (PIP 2000 B)
Diagnostic set:
a. Discoid lupus erythematosus.
b. Discoid (nummular) eczema.
c. Erythema annulare centrifigum.
d. Granuloma annulare.
e. Lichen palnus.
f. Lichen sclerosus et atrophicus.
g. Psoriasis.
h. Tinea.

Reserved Copyrights - 2013

114

Pathology Questions

MCQs

Which lesion typically presesnts as violaceous flat topped papules with


saw-tooth epidermal hyperpalsia, hypergranulosis, and prominent
Civatte bodies?
e. Lichen palnus.
Which lesion is associated with localized scleroderma (morphea)?
f. Lichen sclerosus et atrophicus.
Which lesion typically shows granular deposits of immunoglobulin and
complement along the dermal-epidermal hunction in sun-exposed,
clinically affected skin?
a. Discoid lupus erythromatosus.
671.

For each of the following questions choose the most likely diagnosis from
the following diagnostic set (an answer may be used once, more than
once, or not at all). (PIP 2000 C)
Diagnostic set:
a. Adrenal cortical adenoma.
b. Angiomyolipoma
c. Extramedullary hematopoiesis.
d. Granulocytic sarcoma (chloroma).
e. Lipoma.
f. Liposarcoma.
g. Myelolipoma.
h. Pheochromocytoma.
Which

adrenal

lesion

is

almost

always

associated

with

hepatosplenomegaly and abnormal bone marrow findings?


c. Extramedullary hematopoiesis.
Which lesion is most commonly located in the adrenal gland, shows an
admixture of mature adipose tissue and hematopoietic elements and is
most often an incidental, asymptomatic lesion?
Reserved Copyrights - 2013

115

Pathology Questions

MCQs

g. Myelolipoma.
Which lesion may be associated with tuberous sclerosis complex?
b. Angiomyolipoma.
672.

For each of the following questions choose the most likely diagnosis from
the following diagnostic set (an answer may be used once, more than
once, or not at all). (PIP 2000 C)
Diagnostic set:
a. Atypical ductal hyperplasia.
b. Atypical lobular hyperplasia.
c. Ductal carcinoma in-situ.
d. Ductal hyperplasia.
e. Lobular carcinoma in-situ.
Which lesion is typically non-palpable, may be bilateral and multicentric,
and characteristically is composed of cells that are noncohesive,
monotonus with round cytologically bland nuclei, absent nucleoli and
scant cytoplasm, that fill and expand acinar spaces?
e. Lobular carcinoma in-situ.
Which lesion is nonpalpable and shows an increase in the cellularity of
the duct epithelium that is characterized by a mixed cell population with a
streaming appearance in which the nuclei are oriented parallel to the long
axis of the cells with nuclear crowding?
d. Ductal hyperplasia.
Which lesion is frequently nonpalpable and characterized by an epithelial
proliferation that fills the duct with neoplastic cells a single cell type with
well defined cell borders, and may show bridges and punched out spaces?
c. Ductal carcinoma in-situ.

Reserved Copyrights - 2013

116

Pathology Questions
673.

MCQs

All the following are characteristic of low-grade MALT lymphoma except:


(PIP 2000 C)
a. May have prominent reactive-appearing lymphoid follicles.
b. Neoplastic cells have large lobulated nuclei that may mimic ReedSternberg cells.
c. May have groups of palsma cells below the surface epithelium.
d. Lymphoepithelial lesions are characteristic.
e. Neoplastic cells have the phenotype of marginal zone B cells.

674.

The most common site for extranodal lymphoma is: (PIP 2000 C)
a. Skin.
b. Liver.
c. Brain.
d. Gastrointestinal tract.
e. Bone.

675.

The most common primary site for a Krkenberg tumor is: (PIP 2000 C)
a. Breast.
b. Colon.
c. Lung.
d. Pancreas.
e. Stomach.

676.

Which of the following statements about clear cell adenocarcinoma of the


ovary is/are true? (PIP 2000 C)
a. Cytoplasm is glycogen rich.
b. Mucin is present in tubules and cysts.
c. It may contain hyaline bodies.
d. Immunoreactivity for alpha fetoprotein may be present.
e. All of the above are true.

Reserved Copyrights - 2013

117

Pathology Questions
677.

MCQs

For each of the following questions choose the most likely diagnosis from
the following diagnostic set (an answer may be used once, more than
once, or not at all). (PIP 2000 C)
Diagnostic set:
a. Embryonal rhabdomyosarcoma.
b. Granulosa cell tumor.
c. Malignant lymphoma.
d. Serous adenocarcinoma.
e. Small cell carcinoma of ovary.
Immunoreactivity for and desmin and smooth muscle actin:
a. Embryonal rhabdomyosarcoma.
Associated with estrogenic effects:
b. Granulosa cell tumor.
Immunoprofile of keratin positive, parathyroid hormone-related protein
(PTH-rP) positive, LCA negative:
e. Small cell carcinoma of ovary.

678.

For each of the following questions choose the most likely diagnosis from
the following diagnostic set (an answer may be used once, more than
once, or not at all). (PIP 2000 C)
Diagnostic set:
a. Anaplastic large cell lymphoma.
b. Diffuse large B-cell lymphoma.
c. Inflammatory pseudotumor.
d. Metastatic carcinoma.
e. Nodular sclerosis Hodgkin's disease.
f. Seminoma.
g. Thymoma.

Reserved Copyrights - 2013

118

Pathology Questions

MCQs

Which lesion characteristically has a CD45-, CD15+, CD30+ and EMAimmunophenotype?


e. Nodular sclerosis Hodgkin's disease.
Which lesion is found to have a t(2:5) translocation?
a. Anaplastic large cell lymphoma.
Lacunar cells are characteristic of which lesion?
e. Nodular sclerosis Hodgkin's disease.
679.

For each of the following questions choose the most likely diagnosis from
the following diagnostic set (an answer may be used once, more than
once, or not at all). (PIP 2000 C)
Diagnostic set:
a. Cholangiocarcinoma.
b. Combined hepatocellular-cholangiocarcinoma.
c. Fibrolamellar hepatocellular carcinoma.
d. Focal nodular hyperplasia.
e. Hepatocellular carcinoma, NOS.
f. Liver cell (hepatocellular adenoma).
Which malignant liver tumor has the most favorable prognosis?
c. Fibrolamellar hepatocellular carcinoma.
Which hepatic lesion is most strongly associated with cirrhosis?
e. Hepatocellular carcinoma, NOS.
In which hepatic lesion is elevation of serum alpha-fetoprotein most
common?
e. Hepatocellular carcinoma, NOS.

Reserved Copyrights - 2013

119

Pathology Questions
680.

MCQs

For each of the following questions choose the most likely diagnosis from
the following diagnostic set (an answer may be used once, more than
once, or not at all). (PIP 2000 C)
Diagnostic set:
a. Anaplastic large cell lymphoma.
b. Blastomyces dermatiditis.
c. Halogenoderma.
d. Loboa lobi.
e. Sporothrix schenckii.
f. Squamous carcinoma.
Fungal organism which to date has not been cultured to synthetic media?
d. Loboa lobi.
Cutaneous lesion resulting from ingestion of bromides or iodines?
c. Halogenoderma.
Fungus whose cigar-shaped yeast form is thought to be diagnostic in
tissue sections:
e. Sporothrix schenckii.

681.

For each of the following questions choose the most likely diagnosis from
the following diagnostic set (an answer may be used once, more than
once, or not at all). (PIP 2000 C)
Diagnostic set:
a. Adult granulosa tumor.
b. Endometrioid stromal sarcoma.
c. Primary ovarian lymphoma.
d. Sertoli leydig cell tumor.
e. Small cell carcinoma.
f. Undifferentiated carcinoma.
Which is most likely to simultaneous involve both uterus and ovary?

Reserved Copyrights - 2013

120

Pathology Questions

MCQs

b. Endometrioid stromal sarcoma.


Which is most often associated with virilizing symptoms?
d. Sertoli leydig cell tumor.
Which is associated with hypercalcemia?
e. Small cell carcinoma.
682.

For each of the following questions choose the most likely diagnosis from
the following diagnostic set (an answer may be used once, more than
once, or not at all). (PIP 2000 C)
Diagnostic set:
a. Dermatofibrosarcoma protuberans.
b. Diffuse neurofibroma.
c. Elephantiasis neuromatosa.
d. Localized neurofibroma.
e. Plexiform neurofibroma.
f. Spindle cell liposarcoma.
Which is the most common tumor found in neurofibromatosis, type I?
d. Localized neurofibroma.
Which lesion arises as a plaque-like leison of the dermis and
subcutaneous tissue, and has abundant Meissner-like bodies?
b. Diffuse neurofibroma.
Which typically occurs in the arms and shoulders of middle aged to older
adults?
f. Spindle cell liposarcoma.

683.

Which of the following is not characteristic of pilocytic astrocytoma? (PIP


2000 D)
a. Cerebellar location.

Reserved Copyrights - 2013

121

Pathology Questions

MCQs

b. Childhood presentation.
c. Circumcription.
d. Cyst formation.
e. Tendency for malignant transformation.
684.

Histologic features of pilocytic astrocytoma include: (PIP 2000 D)


a. Homer Wright rosettes.
b. Perivascular pseudorosettes.
c. Psammoma bodies.
d. Rosenthal fibers.
e. Verocay bodies.

685.

An immunostain that may be most helpful in differentiating pilocytic


astrocytoma from other astrocytic neoplasm is: (PIP 2000 D)
a. Epithelial membrane antigen(EMA).
b. Glial fibrillary astrocytic protein (GFAP).
c. Neurofilament protein (NFP).
d. S-100 protein.
e. Synaptophysin.

686.

For each of the following questions choose the most likely diagnosis from
the following diagnostic set (an answer may be used once, more than
once, or not at all). (PIP 2000 D)
Diagnostic set:
a. Acute interstitial pneumonia.
b. Adenovirus pneumonia.
c. Giant cell interstitial pneumonia.
d. Hantavirus pneumonia.
e. Measles pneumonia.
f. Mycoplasma pneumonia.
g. Respiratory syncytial virus pneumonia.
Intracytoplasmic inclusions only:

Reserved Copyrights - 2013

122

Pathology Questions

MCQs

g. Respiratory syncytial virus pneumonia.


Intracytoplasmic inclusions and intranuclear inclusions:
e. Measles pneumonia.
Intrabronchiolar neutrophilic exudate with plasma cell rich chronic
inflammation in bronchiolar wall:
f. Mycoplasma pneumonia.
687.

For each of the following questions choose the most likely diagnosis from
the following diagnostic set (an answer may be used once, more than
once, or not at all). (PIP 2000 D)
Diagnostic set:
a. Chorangiosis.
b. Chronic intervillositis.
c. Generalized villous dysmaturity.
d. Lymphoplasmacytic villitis.
e. Partial hydatidiform mole.
f. Recurrent bacterial villitis.
Histologic pattern associated with treponemal infection:
d. Lymphoplasmacytic villitis.
Lesion associated with triploid karyotype:
e. Partial hydatidiform mole.
Lesion characterized by villous hypervascularity and clinical diabetes and
preeclampsia:
a. Chorangiosis.

688.

For each of the following questions choose the most likely diagnosis from
the following diagnostic set (an answer may be used once, more than
once, or not at all). (PIP 2000 D)

Reserved Copyrights - 2013

123

Pathology Questions

MCQs

Diagnostic set:
a. Celiac disease.
b. Crohn's disease.
c. Eosinophilic gastoenteritis.
d. Giardia enteritis.
e. Mast cell disease.
f. Peripheral T-cell lymphoma.
g. Scleroderma.
Which may feature eosinophilia and atypical lymphoid cells?
f. Peripheral T-cell lymphoma.
Which may feature edema and relatively pure population of eosinophils
with minimal tissue damage?
c. Eosinophilic gastoenteritis.
Which will feature moderate eosinophilia admixed with tryptase-positive
mononuclear cell population by immunoperoxidase staining?
e. Mast cell disease.
689.

All of the following have proven clinical utility as prognostic factors,


except: (PIP 2000 D)
a. Tumor size.
b. Tumor grade.
c. Tumor necrosis.
d. Lymph node status.
e. Hormone receptor status.

690.

All of the following are classified as special type carcinomas with regard
to prognosis: (PIP 2000 D)
a. Classic infiltrating lobular carcinoma.
b. Apocrine carcinoma.
c. Mucinous carcinoma.

Reserved Copyrights - 2013

124

Pathology Questions

MCQs

d. Tubular carcinoma.
e. Cribriform carcinoma.
691.

Which predictive factor has been endorsed by the Tumor Panel of the
American Society of Clinical Oncology for routine clinical use inbreast
cancer? (PIP 2000 D)
a. Her2/neu.
b. p53.
c. bcl-2.
d. Estrogen receptor.
e. Epidermal growth factor receptor.

692.

For each of the following questions choose the most likely diagnosis from
the following diagnostic set (an answer may be used once, more than
once, or not at all). (PIP 2000 D)
Diagnostic set:
a. Ectopic deciduosis.
b. Fibroma.
c. Granulosa cell tumor, adult.
d. Hyperreactio luteinalis.
e. Leydig cell tumor.
f. Pregnancy luteoma.
g. Steroid cell tumor, not otherwise specified.
h. Stromal luetoma.
i.

Thecoma.

Which lesion characteristically contains crystalloids of Reinke?


e. Leydig cell tumor.
Which lesion is most commonly bilateral and multicystic, associated with
gestational trophoblastic tumors, and frequently involutes following
treatment?
d. Hyperreactio luteinalis.
Reserved Copyrights - 2013

125

Pathology Questions

MCQs

Which lesion is typically solid, white and fibrous on gross examination?


b. Fibroma.
693.

For each of the following questions choose the most likely diagnosis from
the following diagnostic set (an answer may be used once, more than
once, or not at all). (PIP 2000 D)
Diagnostic set:
a. Calcifying aponeurotic fibroma.
b. Congenital (infantile) fibrosarcoma.
c. Elastofibroma.
d. Embryonal rhabdomyosarcoma.
e. Fibrolipoma.
f. Fibrous hamartoma of infancy.
g. Infantile digital fibromatosis.
h. Infantile (desmoid type) fibromatosis.
Which

lesion

may

display

t(12;15)(13;q52)

chromosome

rearrangement with fusion of the ETV6 and NTRK3 (ETV6-NTRK3)


genes?
b. Congenital (infantile) fibrosarcoma.
Which lesion characteristically presents as slow-growing mass of the
connective tissue of the lower scapula and chest wall in elderly adults?
c. Elastofibroma.
Which lesion is a fibrous proliferation of infants that characteristically
involves the fingers and toes and demonstrates fibroblasts that contain
intracytoplasmic, eosinophilic inclusions on H&E stain?
g. Infantile digital fibromatosis.
694.

For each of the following questions choose the most likely diagnosis from
the following diagnostic set (an answer may be used once, more than
once, or not at all). (PIP 2000 D)

Reserved Copyrights - 2013

126

Pathology Questions

MCQs

Diagnostic set:
a. Inverted papilloma.
b. Papillary urothelail carcinoma, high grade.
c. Papillary urothelail carcinoma,low grade.
d. Papillary urothelail hyperplasia.
e. Papillary urothelail neoplasm of low malignant potential.
f. Urothelial carcinoma with inverted papilloma-like pattern
g. Urothelial papilloma.
A putative benign precursor lesion of low-grade papillary urothelial
neoplasm:
d. Papillary urothelail hyperplasia.
A papillary lesion that can be diagnosed cytologically in urine specimens
by the presence of 3-dimensional clusters of dysplastic urothelail cells
with altered polarity:
c. Papillary urothelial carcinoma, low grade.
A lesion which is not biologically aggressive but is associated with
recurrences in 40-50 percent of cases and carries a risk for progression
with each recurrence having a 10-20 percent change of being invasive:
e. Papillary urothelial neoplasm of low malignant potential.
695.

For each of the following questions choose the most likely diagnosis from
the following diagnostic set (an answer may be used once, more than
once, or not at all). (PIP 2000 D)
Diagnostic set:
a. Extraosseous Ewing's sarcoma.
b. Merkle cell carcinoma.
c. Metastatic small cell neuroendocrine carcinoma.
d. Non-Hodgkin's lymphoma.
e. Small cell malignant melanoma.
f. Small cell sweat gland carcinoma.

Reserved Copyrights - 2013

127

Pathology Questions

MCQs

Which tumor is typically positive for Cytokeartin 20(CK20)?


b. Merkle cell carcinoma.
Which tumor is typically shows strong membrane staining for CD99?
a. Extraosseous Ewing's sarcoma.
Which tumor may show positive staining for neuroendocrine markersand
be negative for Cytokeartin 20 (CK20)?
c. Metastatic small cell neuroendocrine carcinoma.
696.

Which of the following is true? (PIP 2000 D)


a. In Crohn's disease, biopsies of endoscopically normal mucosa may
display colitis.
b. Collagenous colitis occurs more often in men than women.
c. Approximately 50 percent of patients with chronic diarrhea and
normal endoscopic findings have either lymphocytic colitis or
collageneous colitis.
d. Diffuse mucosal lymphocytosis or lymphoplasmacytosis is
common in graft-versus-host disease.
e. Nonsteroidal anti-inflammatory drugs (NSAIDs) are associated
with diffuse neutrophilic cryptitis.

697.

Intraepithelial lymphocytosis is a prominent feature of colon biopsies in


all of the following except: (PIP 2000 D)
a. Collagenous colitis.
b. Chronic ulcerative colitis.
c. Lymphocytic colitis.
d. Refractory spure.
e. Brainerd diarrhea.

698.

In individuals with chronic diarrhea and no HIV infection, the most


common biopsy finding is: (PIP 2000 D)

Reserved Copyrights - 2013

128

Pathology Questions

MCQs

a. Melanosis coli.
b. Diffuse neutrophilic cryptitis.
c. Intraepithelial lymphocytosis.
d. Histologically normal mucosa.
e. Apoptosis of crypt epithelial cells.
699.

Which of the following tumors has the poorest diagnosis? (PIP 2001 A)
a. Alveolar adenoma.
b. Bronchioloalveolar carcinoma, Type 1(mucinous subtype).
c. Bronchioloalveolar carcinoma, Type 2(hobnail subtype).
d. Bronchioloalveolar carcinoma, mixed types 1 and 2
e. Bronchioloalveolar carcinoma, Type 3 (with central sclerosis).

700.

Which of the following immunohistochemichal staining is characteristic of


primary pulmonary adenocarcinoma? (PIP 2001 A)
a. CK20(+), CK7(-), CEA(+).
b. CK20(+), CK7(+), CA 19.9(+).
c.

CK20(-), CK7(+), TTF-1(+).

d. CK20(-), CK7(-), EMA(+).


e. Pancytokeratin(+), calretinin(+), TTF-1(-).
701.

Which of the following is considered most important in assessing the


prognosis of bronchioloalveolar carcinoma? (PIP 2001 A)
a. Degree of histologic atypia.
b. Extent of intrapulmonary spread.
c. Lymph node status.
d. Pulmonary function status.
e. Histology subtype 2 bronchioloalveolar carcinoma.

702.

For each of the following questions choose the most likely diagnosis from
the following diagnostic set (an answer may be used once, more than
once, or not at all). (PIP 2001 A)
Diagnostic set:

Reserved Copyrights - 2013

129

Pathology Questions

MCQs

a. Caroli's disease.
b. Cholangiocarcinoma.
c. Primary biliary cirrhosis.
d. Primary sclerosing cholangitis.
e. Recurrent pyogenic cholangitis (oriental cholangiohepatitis).
Which is a congenital liver abnormality that involves both lobes and is
associated with bile duct stones, cholangitis and renal cysts?
a. Caroli's disease.
Which liver abnormality shows preferential involvement of the left lobe,
intrahepatic bile duct stones, and cholangitis?
e. Recurrent pyogenic cholangitisoriental cholangiohepatitis.
Which liver abnormality most often arises in patients with inflammatory
bowel diseasae?
d. Primary sclerosing cholangitis.
703.

A balance translocation involving the EWS gene on chromosome 22 is


seen in: (PIP 2001 A)
a.

Neuroblastoma.

b. Acute lymphoblastic lymphomas and leukemias.


c.

Wilm's tumor.

d. Primitive neuroectodermal tumor.


e.
704.

Medulloblastoma.

Antbodies to MIC-2 gene product are seen in all except: (PIP 2001 A)
a. Ewing's sarcoma.
b. Neuroblastoma.
c. Acute lymphoblastic lymphomas and leukemias.
d. Wilm's tumor.
e. Primitive neuroectodermal tumor.

Reserved Copyrights - 2013

130

Pathology Questions
705.

feature

MCQs
distinguishing

Ewing's

sarcoma

from

primitive

neuroectodermal tumor is: (PIP 2001 A)


a. Electron microscopic findings.
b. EWS-FL1 gene translocation.
c. More favorable prognosis.
d. -microglobulin staining.
e. Elevated choline acetyltransferase activity.
706.

Oxyphil (oncocytic) adenomas of the parathyroid glands differ from chief


cell adenoma in which of the following: (PIP 2001 A)
a. Larger size.
b. More follicular pattern.
c. Lower serum calcium.
d. Decreased intracellular fat.
e. All of the above.

707.

Parathyroid carcinoma differs from parathyroid adenoma by: (PIP 2001


A)
a. Smaller size.
b. Invasion of neighboring structures.
c. Occurrence more frequently in women.
d. Occurrence in older patients.
e. Lower serum calcium levels.

708.

The frequency of hyperparathyroidism in MEN syndromes:


a. MEN I > MEN II.
b. MEN IIB > MEN II.
c. MEN II > MEN I.
d. MEN IIB > MEN I.
e. MEN I = MEN IIB.

Reserved Copyrights - 2013

131

Pathology Questions
709.

MCQs

For each of the following questions choose the most likely diagnosis from
the following diagnostic set (an answer may be used once, more than
once, or not at all). (PIP 2001 A)
Diagnostic set:
a. Anaplastic large cell lymphoma.
b. Erdheim-Chester disease.
c. Histoplasmosis.
d. Hodgkin's disease.
e. Inflammatory malignant fibrous histiocytoma.
f. Langerhans' cell histiocytosis.
g. Malacoplakia.
h. Malignant melanoma.
i.
Which

Rosai-Dorfman disease.
lesion

is

characterized

by

S-100

protein

and

CD

immunoreactivity and the ultrastuctural presence of Birbeck'sgranules?


f. Langerhans' cell histiocytosis.
Which lesion is thought to represent an unusual host response to
bacterial infection that is characterized by the presence of PAS-positive,
diastase-resistant intracytoplasmic calcospherities?
g. Malacoplakia.
Which is a malignant neoplasm that is characterized by an intense,
neutrophil-rich inflammatory infiltrate with intracytoplasmic neutrophils
often present within neoplastic or xanthoma cells?
e. Inflammatory malignant fibrous histiocytoma.
710.

For each of the following questions choose the most likely diagnosis from
the following diagnostic set (an answer may be used once, more than
once, or not at all). (PIP 2001 A)
Diagnostic set:
a. Beta hemolytic streptococcus.

Reserved Copyrights - 2013

132

Pathology Questions

MCQs

b. Coccidiodes immitus.
c. Cytomegalovirus(CMV).
d. Herpes simplex virus(HSV).
e. Parvovirus.
f. Toxoplasma gondii.
g. Varicella virus.
Which characteristically produces lymphoplasmacytic villitis, with villous
fibrosis, karyorrhexis, and hemosiderin deposition?
c. Cytomegalovirus CMV.
Infection with which organism shows characteristic intranuclear viral
inclusions in fetal red blood cells?
e. Parvovirus.
This agent characteristically infects the extraplacental membranes
without eliciting an inflammatory or desmoplastic reaction?
f. Toxoplasma gondii.
711.

For each of the following questions choose the most likely diagnosis from
the following diagnostic set (an answer may be used once, more than
once, or not at all). (PIP 2001 A)
Diagnostic set:
a. Follicle center cell lymphoma.
b. Hairy cell leukemia.
c. Hodgkin's lymphoma.
d. Langerhans' cell histiocytosis.
e. Malignant melanoma.
f. Mycobacterium avium intracellulare infection.
g. Splenic lymphoma with villous lymphocytes.
h. Systemic mastocytosis.
Which lesion exhibits dilated pseudosinuses within the red pulp?

Reserved Copyrights - 2013

133

Pathology Questions

MCQs

b. Hairy cell leukemia.


Which leison expresses CD 1a, S-100, vimentin, HLA-Dr and CD 68?
d. Langerhans' cell histiocytosis.
Which lesion contains toluidine blue positive cytoplasmic granules?
h. Systemic mastocytosis.
712.

For each of the following questions choose the most likely diagnosis from
the following diagnostic set (an answer may be used once, more than
once, or not at all). (PIP 2001 A)
Diagnostic set:
a. Congenital lobar emphysema.
b. Cystic adenomatoid malformation type II.
c. Micronodular pneumocyte hyperplasia.
d. Normal lung, 24 weeks of gestational age.
e. Perinatal pneumonia.
f. Pulmonary hyperplasia.
g. Pulmonary hypoplasia.
This lesion is associated with an abnormal reduction in weight and/or
volume of the lung:
g. Pulmonary hypoplasia.
This lesion is an orderly increase in size and/or number in the structural
components of the pulmonary lobules:
f. Pulmonary hyperplasia.
This lesion is a hamartoma associated with the tuberous sclerosis

complex:
c. Micronodular pneumocyte hyperplasia.

Reserved Copyrights - 2013

134

Pathology Questions
713.

MCQs

One of the factors associated with a good prognosis for solitary fibrous
tumor is: (PIP 2001 A)
a. Epithelioid histology.
b. Size greater than 10 cm in diameter.
c. Lack of keratin reactivity.
d. Complete surgical excision.
e. Age at presentation.

714.

Which of the following neoplasms is characteristically CD 34 positive?


(PIP 2001 A)
a. Calcifying fibrous pseudotumor.
b. Desmoid tumor.
c. Leiomyoma.
d. Sarcomatoid carcinoma.
e. Sarcomatous mesothelioma.
f. Solitary fibrous tumor.
g. Synovial sarcoma.

715.

Which is the most common location for extra-abdominal desmoids


tumors? (PIP 2001 A)
a. Shoulder.
b. Forearm.
c. Buttock.
d. Mesentry.
e. Neck.

716.

The most common malignant tumor of the gallbladder is:


a. Adenocarcinoma, intestinal type.
b. Adenocarcinoma, low grade (well to moderately differentiated).
c. Adenocarcinoma,

high

grade

(poorly

differentiated

to

undifferentiated).
d. Adenosquamous carcinoma.
e. Clear cell adenocarcinoma.
Reserved Copyrights - 2013

135

Pathology Questions
717.

MCQs

All of the following are associated with an increased risk of gallbladder


carcinoma except: (PIP 2001 A)
a. Cholelithiasis.
b. Anomalous pancreaticobiliary duct union.
c. Gastroesophegeal reflux disease.
d. Sclerosing cholangitis.
e. Native American heritage.

718.

Which of the following is true about TNM staging of gallbladder


carcinoma?
a. Most gallbladder carcinomas are pT1N0 or pT2N0 at diagnosis.
b. The peripancreatic (head of pancreas) and periduodenal lymph
nodes are the ones most commonly involved by metastasis.
c. When carcinoma in situ extensively involves Rokitansky-Aschoff
sinuses, it is classified as pT1.
d. Tumor invasion through the muscularis into perimuscular
connective tissue but not perforating serosa and not invading
liver is classified as pT2.
e. Metastasis to peripancreatic lymph nodes is categorized as pN1.

719.

Which of the following procedures would be most helpful for the


diagnosis of progressive multifocal leukoencephalopathy? (PIP 2001 B)
a. GFAP.
b. HAM-56.
c. In situ hybridization for JC virus.
d. Myelin stain (Luxol fast blue).
e. S-100 protien.

720.

Progressive multifocal leukoencephalopathy is most frequently associated


with: (PIP 2001 B)
a. Autoimmune reaction.
b. Chromosome 1p deletion.
c. Fibrinoid necrosis of blood vessels.

Reserved Copyrights - 2013

136

Pathology Questions

MCQs

d. Hypertension.
e. Immunosuppression.
721.

The typical clinical course of progressive multifocal leukoencephalopathy


seen in this case is: (PIP 2001 B)
a. Monophasic (single episode) with complete recovery.
b. Monophasic (single episode) wit significant sequelae.
c. Relapsing and remitting over many years.
d. Relentlessly progressive over several months.
e. Slowly progressive over several years.

722.

For each of the following questions choose the most likely diagnosis from
the following diagnostic set (an answer may be used once, more than
once, or not at all). (PIP 2001 B)
Diagnostic set:
a. Chromophil renal cell carcinoma.
b. Chromophobe renal cell carcinoma, eosinophilic variant.
c. Chromophobe renal cell carcinoma, typical type.
d. Clear cell renal cell carcinoma.
e. Metanephric adenoma.
f. Metanephric (nephrogenic) adenofibroma.
g. Renal adenoma.
h. Renal oncocytoma.
A cellular tumor focally hyalinized stroma composed of tightl packed,
small, round tubules lined by cells with small, uniform cytologically bland
nuclei and scant pale staining cytoplasm:
e. Metanephric adenoma.
A tumor having a nested and archipelaginous pattern and containing a
uniform population of cells with low grade nuclear features, granular
eosinophilic cytoplasm and patchy, weak peri-membranous staining with
Hale's colloidal iron:

Reserved Copyrights - 2013

137

Pathology Questions

MCQs

h. Renal oncocytoma.
A tumor having a sheet-like and trabecular architecture and containing
cells with grade 2 nuclear features, granular eosinophilic cytoplasm, and
uniformly strong diffuse reticular cytoplasmic staining with Hale's
colloidal iron:
b. Chromophobe renal cell carcinoma, eosinophilic variant.
723.

All of the following are accepted criteria for classic medullary carcinom of
the breast except: (PIP 2001 B)
a. Prominent syncytial growth pattern.
b. Intense lymphoblastic infiltrate.
c. High grade nuclei.
d. Tumor necrosis.
e. Circumscribed margins.

724.

For each of the following questions choose the most likely diagnosis
from the following diagnostic set (an answer may be used once, more
than once, or not at all). (PIP 2001 B)
Diagnostic set:
a. Chronic lymphocytic thyroiditis.
b. Follicular adenoma, Hrthle cell type.
c. Follicular carcinoma, Hrthle cell type.
d. Graves' disease.
e. Marginal zone lymphoma of MALT type.
f. Medullary thyroid carcinoma.
g. Papillary thyroid carcinoma.
Which thyroid lesion is characterized histologically by a lymphocytic
infiltrate in the thyroid gland with germinal center formation, oxyphilic
metaphasia of thyroid follicular colonization, absence of extension
outside the thyroid gland, and immunoreactivity with both B-cell and Tcell markers?

Reserved Copyrights - 2013

138

Pathology Questions

MCQs

a. Chronic lymphocytic thyroiditis.


Which thyroid lesion is characterized by a lymphocytic infiltrate in the
thyroid gland coupled with a laboratory evidence of hyperthyroidism,
immunoglobulin deposition within the thyroid, thymic enlargement, and
almost invariable presence of thyroid stimulating antibodies?
d. Graves' disease.
Which thyroid lesion is characterized by a lymphocytic infiltration with
follicular colonization, extension of the infiltrate beyond the thyroid
capsule into perithyroidal soft tissues, immunoreactivity withCD 20 but
not CD 45RO , and light chain restriction? (PIP 2001 B)
e. Marginal zone lymphoma of MALT type.
725.

For each of the following questions choose the most likely diagnosis from
the following diagnostic set (an answer may be used once, more than
once, or not at all). (PIP 2001 B)
Diagnostic set:
a. Acute myelogenous leukemia.
b. Angiosarcoma.
c. Cholangiocarcinoma.
d. EBV infection(mononucleosis).
e. Hairy cell leukemia.
f. Hepatocellular carcinoma.
g. Hepatosplenic T-cell lymphoma.
h. Large cell lymphoma, B-cell phenotype.
The majority of cases from angiomatous pseudosinuses in the liver and
spleen:
e. Hairy cell leukemia.
Displays

the

following

phenotype:

positive

for

CD43

and

myeloperoxidase; negative for CD 20 and CD3:


Reserved Copyrights - 2013

139

Pathology Questions

MCQs

a. Acute myelogenous leukemia.


Aggressive malignancy that most commonly expresses gamma-delta Tcell receptor:
g. Hepatosplenic T-cell lymphoma.
726.

All the following are true about mantle cell lymphoma except: (PIP 2001
B)
a. Moderately aggressive with a survival rate of approximately five
years.
b. May involve multiple sites including lymph nodes, spleen,
peripheral blood, bone marrow and gastrointestinal tract.
c. May present as intestinal polyps.
d. Characterized

by

clusters

of

prolymphocytes

and

paraimmunoblasts.
e. Blastoid transformation denotes a more aggressive course.
727.

The most common non-Hodgkin's lymphoproliferative disorder involving


the spleen:
a. Mantle cell lymphoma.
b. Diffuse large cell lymphoma.
c. Chronic lymphocytic leukemia/small lymphocytic lymphoma.
d. Splenic marginal cell lymphoma.
e. Hairy cell leukemia.

728.

For each of the following questions choose the most likely diagnosis
from the following diagnostic set (an answer may be used once, more
than once, or not at all). (PIP 2001 B)
Diagnostic set:
a. Adult rhabdomyoma.
b. Alveolar rhabomyosarcoma.
c. Alveolar soft part sarcoma.
d. Chordoma, metastatic.

Reserved Copyrights - 2013

140

Pathology Questions

MCQs

e. Clear cell sarcoma.


f. Epithelioid sarcoma.
g. Granular cell tumor.
h. Melanoma, metastatic.
i.

Paraganglioma, metastatic.

j.

Pheochromocytoma, metastatic.

k. Renal cell carcinoma, metastatic.


Which lesion contains characteristic ultrastuctural rhomboid crystalline
material with a lattice pattern?
c. Alveolar soft part sarcoma.
Which lesion often occurs in the hand or forearm and may simulate
granulomatous inflammation?
f. Epithelioid sarcoma.
Which lesion often involves tendons or aponeuroses of the foot or ankle
and may contain melanin pigment?
e. Clear cell sarcoma.
729.

For each of the following questions choose the most likely diagnosis from
the following diagnostic set (an answer may be used once, more than
once, or not at all). (PIP 2001 B)
Diagnostic set:
a. Fibroma.
b. Fibromatosis.
c. Massive edema.
d. Myxoma.
e. Sclerosing stromal tumor.
f. Stromal hyperthecosis.
g. Thecoma.

Reserved Copyrights - 2013

141

Pathology Questions

MCQs

Which lesion is best characterized by the following features:Child or


young adult, large size, right side predominance, acute abdomen,
preserved ovarian architecture:
c. Massive edema.
Reproductive age patient, asymptomatic mass, low cellularity, colloidal
iron-rich stroma, effaced ovarian architecture:
d. Myxoma.
Young adult, menstrual irregularity, large size, pseudolobar histology,
hemangiopericytoma-like vasculature:
e. Sclerosing stromal tumor.
730.

For each of the following questions choose the most likely diagnosis from
the following diagnostic set (an answer may be used once, more than
once, or not at all). (PIP 2001 B)
Diagnostic set:
a. Aggressive angiomyxoma.
b. Angimyolipoma.
c. Atypical lipoma.
d. Malignant peripheral nerve sheath tumor.
e. Myxoid chondrosarcoma.
f. Myxoid liposarcoma.
g. Myxoid malignant fibrous histiocytoma.
h. Myxoma.
i.

Sarcinatiud renal cell carcinoma.

j.

Schwannoma.

k. Sclerosing lipogranuloma
Which entity is closely related to round cell liposarcoma?
f. Myxoid liposarcoma.

Reserved Copyrights - 2013

142

Pathology Questions
Which

lesion

MCQs
often

demonstrates

the

reciprocal

translocation

t(12;16)(q13;p11)?
f. Myxoid liposarcoma.
Which lesion shows an alcian blue-positive but hyaluronidase-resistant
mucopolysaccharide stroma?
e. Myxoid chondrosarcoma.
731.

For each of the following questions choose the most likely diagnosis from
the following diagnostic set (an answer may be used once, more than
once, or not at all). (PIP 2001 B)
Diagnostic set:
a. Capillary hemangioma.
b. Congenital nevus.
c. Deep penetrataing nevus.
d. Lentigo malignant melanoma.
e. Pigmented spindle cell nevus.
f. Spindle cell hemanioendothelioma.
g. Spitz nevus.
h. Superficial spreading melanoma.
Which lesion typically presents as a very dark, small (3 to 6mm), uniform
macule on the extremity of a young patient?
e. Pigmented spindle cell nevus.
Kamino bodies are a characteristic feature of which lesion?
g. Spitz nevus.
Which lesion is wedge shaped with fascicles of spindeled melanocytes and
admixed melanophages that infiltrate pilar muscle?
c. Deep penetrataing nevus.

Reserved Copyrights - 2013

143

Pathology Questions
732.

MCQs

For each of the following questions choose the most likely diagnosis
from the following diagnostic set (an answer may be used once, more
than once, or not at all). (PIP 2001 C)
Diagnostic set:
a. Brenner tumor.
b. Carcinoid tumor, insular/strumal.
c. Endometrioid adenocarcinoma, trabecular pattern.
d. Granulosa cell tumor.
e. Mature cystic teratoma.
f. Mucinous cystadenoma.
g. Sertoli-leydig cell tumor.
h. Sex-cord tumor with annular tubules.
Which lesion is characterized by Call-Exner bodies?
d. Granulosa cell tumor.
Which lesion is composed of transitional cells with pale, grooved nuclei
and may contain scattered benign mucinous glands?
a. Brenner tumor.
Which lesion may have sarcoma-like mural nodules?
f. Mucinous cystadenoma.

733.

Important elements of the final pathology report for rhabdomyosarcoma


resected following treatment include: (PIP 2001 C)
a. Presence and extent of cytodifferentiation.
b. Assessment of resecetion margins.
c. Measurement of tumor size in three dimensions.
d. Comparison of pre- and post- treatment histologic features.
e. All of the above.

734.

Which of the following types of rhabdomyosarcoma is considered an


unfavorable histologic type? (PIP 2001 C)

Reserved Copyrights - 2013

144

Pathology Questions

MCQs

a. Botryoid rhabdomyosarcoma.
b. Spindle cell rhabdomyosarcoma.
c. Embryonal rhabdomyosarcoma, not otherwise specified.
d. Alveolar rhabdomyosarcoma.
e. None of the above.
735.

Which of the following immunohistochemichal markers sre used for the


diagnosis of rhabdomyosarcoma? (PIP 2001 C)
a. Myogenin.
b. Myo-D1.
c. Muscle specific actin.
d. Desmin.
e. All of the above.

736.

For each of the following questions choose the most likely diagnosis from
the following diagnostic set (an answer may be used once, more than
once, or not at all). (PIP 2001 C)
Diagnostic set:
a. Adenocarcinoma.
b. Adenoid cystic carcinoma.
c. Basaloid squamous cell carcinoma.
d. Carcinoid tumor.
e. Small cell carcinoma.
f. Spindle cell carcinoma.
A polypoid esophegeal neoplasm composed of cytokeratin positive
spindle cells with or without a light microscopically identifiable
squamous component:
f. Spindle cell carcinoma.
A neoplasm in the esophegus that most often arises in the setting of
Barret's esophegus:
a. Adenocarcinoma.

Reserved Copyrights - 2013

145

Pathology Questions

MCQs

A tumor that exhibits strong immunoreactivity for chromogranin and


many neurosecretory granules by electron microscopy:
d. Carcinoid tumor.
737.

For each of the following questions choose the most likely diagnosis from
the following diagnostic set (an answer may be used once, more than
once, or not at all). (PIP 2001 C)
Diagnostic set:
a. Chronic lymphocytic thyroiditis.
b. Diffuse toxic goiter.
c. Dyshormonogenetic goiter.
d. Papillary hyperplastic nodule.
e. Papillary thyroid carcinoma.
Match the following patients with the most likely condition from the
diagnostic set:
An 11- year old girl with a 2.5cm, circumscribed centrally cystic thyroid
nodule. Histology showed an encapsulated nodule composed of papillae
and follicles with stromal edema and round, polarized nuclei:
d. Papillary hyperplastic nodule.
A 28- year old man with a 2.0 cm firm, poorly circumscribed,
intrathyroidal mass. Histology showed a mixed papillary and follicular
nodule with multifocal areas of nuclear enlargement and clearing:
e. Papillary thyroid carcinoma.
An 8 year-old deaf girl with a large goiter. Her developmental milestones,
which were similar to two older siblings, indicated she was slow. The
goiter was composed of encapsulated and nonencapsulated, variabely
sized nodules. Both nodular and non-nodular thyroid areas contained
atypical nuclei:

Reserved Copyrights - 2013

146

Pathology Questions

MCQs

c. Dyshormonogenetic goiter.
738.

Which of the following statements regarding amyloidosis is false? (PIP


2001 C)
a. Primary and secondary forms exist.
b. Metachromatic stains are useful in making the diagnosis.
c. Intracytoplasmic hyaline globules are characteristic.
d. Electron microscopy typically demonstrates 7.5-10 nm nonbranching filaments.
e. Nodular and diffuse forms of liver involvement have been
described.

739.

Which of the following hereditary diseases affecting the liver may present
in adulthood?
a. Alpha-1 antitrypsin deficiency.
b. Wilson's disease.
c. Hemochromatosis.
d. Hereditary amyloidosis.
e. All of the above.

740.

For each of the following questions choose the most likely diagnosis from
the following diagnostic set (an answer may be used once, more than
once, or not at all). (PIP 2001 C)
Diagnostic set:
a. B-cell

chronic

lymphocytic

leukemia/small

lymphocytic

lymphoma.
b. Diffuse large B-cell lymphoma.
c. Follicular lymphoma.
d. Hairy cell leukemia.
e. Mantle cell lymphoma.
f. Nodualr lymphocyte predominant Hodgkin's disease.
g. Splenic marginal zone B-cell lymphoma.

Reserved Copyrights - 2013

147

Pathology Questions

MCQs

Which lymphoma/ leukemia expresses CD 19, CD 20, CD 5, and FMC-7 but


not CD 23?
e. Mantle cell lymphoma.
Which lymphoma/ leukemia is associated with t:(14;18)?
c. Follicular lymphoma.
Which lymphoma/ leukemia most commonly presents with red pulp
involvement?
d. Hairy cell leukemia.
Which lymphoma/ leukemia is typically bcl-1 positive?
e. Mantle cell lymphoma
741.

For each of the following questions choose the most likely diagnosis from
the following diagnostic set (an answer may be used once, more than
once, or not at all). (PIP 2001 C)
Diagnostic set:
a. Acinar cell carcinoma.
b. Ductal adenocarcinoma.
c. Pancreatic endocrine neoplasm.
d. Pancreatoblastoma.
e. Serous (microcystic)cystadenoma.
f. Solid-pseudopapillary tumor.
Which tumor is a gland-forming malignancy that usually exhibits a
desmoplastic stroma?
b. Ductal adenocarcinoma.
Which tumor is associated with both clinical and immunohistochemichal
features of pancreatic enzyme secretion?
a. Acinar cell carcinoma.

Reserved Copyrights - 2013

148

Pathology Questions

MCQs

Which tumor is a low grade malignant neoplasm most likely to be


mistaken for a pancreatic pseudocyst?
f. Solid-pseudopapillary tumor.
742.

Calcification in this ovarian tumor characteristically occurs in the form of


stromal plaques? (PIP 2001 C)
a. Brenner tumor.
b. Carcinoid tumor, insular type.
c. Mucinous adenofibroma.
d. Sex cord tumor with annular tubules.
e. Serous adenofibroma.

743.

Which of the following statements about ovarian transitional cell tumor is


correct? (PIP 2001 C)
a. The epithelial cells are ultrastructurally identical to urothelial cells
of the urinary tract.
b. Squamous differentiation may be extensive in the tumor.
c. The epithelial cells are typically CK7(+) and CK20(+).
d. Malignant Brenner tumor contains a component of benign Brenner
tumor or Brenner tumor of low malignant potential.
e. Up to 20 percent of the epithelial cells in the transitional tumor
may demonstrate columnar mucinous differentiation.

744.

Which of the following statements about ovarian tumors is correct?


a. Insular granulosa cell tumor is commonly admixed with other
histologic patterns, such as microfollicular or gyriform.
b. The presence of neuroendocrine granules in the cytoplasm of
tumor cells excludes the diagnosis of an ovarian surface epithelial
cell tumor.
c. Mucinous adenofibroma often produces an estrogenic clinical
syndrome.
d. Priamary squamous cell carcinoma most commonly arises in a
Brenner tumor.

Reserved Copyrights - 2013

149

Pathology Questions

MCQs

e. Stromal leutinization is common in ovarian adenofibromas.


745.

For each of the following questions choose the most likely diagnosis from
the following diagnostic set (an answer may be used once, more than
once, or not at all). (PIP 2001 C)
Diagnostic set:
a. Angiolipoma.
b. Atypical lipomatous tumor(well differentiated liposarcoma).
c. Cellular angiofibroma.
d. Lipoma.
e. Shwannoma.
f. Spindle cell lipoma.
Which tumor characteristically occurs in the genital region of both men
and women?
c. Cellular angiofibroma.
Which tumor is characterized cytogenetically by ring chromosomes?
b. Atypical lipomatous tumor(well differentiated liposarcoma.
Which tumor shows a prediliction for the neck and shoulder of older
men?
f. Spindle cell lipoma.

746.

Which immunostain would be most helpful in the diagnosis of central


neurocytoma? (PIP 2001 D)
a. Cytokeratins.
b. Epithelial membrane antigen.
c. Glial fibrilary acidic protien GFAP.
d. S-100 protien.
e. Synaptophysin.

Reserved Copyrights - 2013

150

Pathology Questions
747.

MCQs

The most helpful ultrastructural feature in the diagnosis of central


neurocytoma is the presence of: (PIP 2001 D)
a. Desmosomes.
b. Intercellular junctional complexes.
c. Intermediate filaments.
d. Microvilli and/or cilia.
e. Neurosecretory granules.

748.

The most appropriate treatment for the lesion diagnosed in central


neurocytoma is: (PIP 2001 D)
a. Biopsy for diagnosis followed by whole brain radiation.
b. Combination chemotherapy with PCV (procarbazine, lomustine,
vincristine).
c. Gross total resection alone.
d. Gross total resection followed by local field radiation therapy.
e. Stereotactic radiosurgery.

749.

Which of the following immunohistochemichal markers is most specific


for mesothelial origin? (PIP 2001 D)
a. Cytokeratin.
b. Epithelial membrane antigen.
c. Calretinin.
d. MART-1 (Melan A).
e. CD15(Leu M1).

750.

Which ultrastructural finding most strongly supports an interpretation of


epithelial mesothelioma? (PIP 2001 D)
a. Tonofilaments.
b. Desmosomes.
c. Lysozomes.
d. Long thin microvilli.
e. Viral particles.

Reserved Copyrights - 2013

151

Pathology Questions
751.

MCQs

Which type of mesothelioma has the best prognosis? (PIP 2001 D)


a. Desmoplastic.
b. Multicystic.
c. Diffuse epithelial.
d. Lymphohistiocystoid.
e. sarcomatoid.

752.

For each of the following questions choose the most likely diagnosis from
the following diagnostic set (an answer may be used once, more than
once, or not at all). (PIP 2001 D)
Diagnostic set:
a. Clear cell sarcoma.
b. Mesoblastic nephroma, cellular type.
c. Mesoblastic nephroma, classic type.
d. Metanephric adenofibroma.
e. Metanephric stromal tumor.
f. Primitive neuroectodermal tumor.
g. Rhabdoid tumor.
h. Sarcomatoid renal cell carcinoma.
i.

Wilm's tumor.

Which tumor has a characteristic diffuse, linear membranous staining


pattern for CD99 (MIC2)?
f. Primitive neuroectodermal tumor.
Which tumor predominates in males, has a propensity for bone
metastasis, and rarely arises at external sites?
a. Clear cell sarcoma.
Which

tumor

is

composed

entirely

of

CD34(+)

spindle

cells

anddemonstrates intratumoral angiodysplasia, onion-skinning around


entrapped tubules and heterologous differentaition?
e. Metanephric stromal tumor.
Reserved Copyrights - 2013

152

Pathology Questions

753.

MCQs

For each of the following questions choose the most likely diagnosis from
the following diagnostic set (an answer may be used once, more than
once, or not at all). (PIP 2001 D)
Diagnostic set:
a. Gaucher's Disease.
b. Granular cell tumor.
c. Hibernoma.
d. Langerhans' cell histiocytosis.
e. Schwannoma.
f. Spindle cell lipoma.
g. Xanthogranuloma.
Which lesion characteristically shows immunopositivity for S-100 protein
and CD 1a?
d. Langerhans' cell histiocytosis.
Which lesion characteristically demonstrates Touton-type iant cells?
g. Xanthogranuloma.
Which of the above lesions shows S-100 positivity, positive stainingfor
lysosomal granules, and is ultrastructurally characterized by intracellular
septation into membrane limited compartments?
b. Granular cell tumor.

754.

Chronic rejection of the liver of the liver typically manifests as: (PIP 2001
D)
a. An overwhelming necro-inflammatory lesion of liver paranchyma.
b. Loss of small bile ducts wit an obliterative arteriopathy.
c. Extensive hepatic fibrosis.
d. None of the above.

Reserved Copyrights - 2013

153

Pathology Questions
755.

MCQs

Acute rejection of the liver may be distinguished from chronic hepatitis


by: (PIP 2001 D)
a. The presence of endotheliitis.
b. The presence of mixed portal inflammation (lymphocytes, plasma
cells, eosinophils) as opposed to a purely lymphocytic infiltrate.
c. The lack of piecemeal necrosis.
d. Extensive duct damage.
e. All of the above.

756.

How long after liver transplantation does chronic rejection typically


occur? (PIP 2001 D)
a. Within two months.
b. Within one year.
c. Three to four years.
d. After the fourth year.

757.

For each of the following questions choose the most likely diagnosis from
the following diagnostic set (an answer may be used once, morethan once,
or not at all). (PIP 2001 D)
Diagnostic set:
a. Cracinoma.
b. Carcinoid tumor.
c. Gastrointestinal stromal tumor.
d. Granular cell tumor.
e. Metastatic melanoma.
f. Paraganglioma.
g. Schwannoma.
h. Solitary fibrous tumor.
Immunoreactive panel includes positive staining for vimentin, CD34, and
CD117, but absence of neuron specific enolase, S-100 and PGP9.5 :
c. Gastrointestinal stromal tumor.

Reserved Copyrights - 2013

154

Pathology Questions

MCQs

Immunoreactive panel includes positive staining for vimentin, CD34, but


absence of CD117, neuron specific enolase, S-100 and PGP9.5 :
h. Solitary fibrous tumor.
Immunoreactive panel includes diffuse positive staining for cytokeratin
and synaptophysin, but absence of CD34, and CD117:
b. Carcinoid tumor.
758.

For each of the following questions choose the most likely diagnosis from
the following diagnostic set (an answer may be used once, more than
once, or not at all). (PIP 2001 D)
Diagnostic set:
a. Adenoid cystic carcinoma.
b. Basal cell adenoma.
c. Carcinoma ex pleomorphic adenoma.
d. Mucoepidermoid carcinoma.
e. Myoepithelioma.
f. Pleomorphic adenoma.
g. Polymorphous low-grade adenocarcinoma.
Which tumor is most prone to early recurrence and poor long-term
survival?
c. Carcinoma ex pleomorphic adenoma.
Which is the most common tumor arising within the parapharyngeal
space?
f. Pleomorphic adenoma.
Which is the most common malignant tumor of the minor salivary glands?
d.Mucoepidermoid carcinoma.

Reserved Copyrights - 2013

155

Pathology Questions
759.

MCQs

For each of the following questions choose the most likely diagnosis from
the following diagnostic set (an answer may be used once, more than
once, or not at all). (PIP 2001 D)
Diagnostic set:
a. Acute cholecystitis with secondary vasculitis.
b. Collagen vascular disease associated vasculitis.
c. Granulomatous

cholecystitis

associated

with

Churg-Strauss

syndrome.
d. Isolated gallbladder vasculitis.
e. Polyarteritis nodosa.
f. Wegener's granulomatosis.
Which vasculitic disease is commonly associated with a history of
asthma?
b. Granulomatous cholecystitis associated with Churg-Strauss
syndrome.)
A histologic picture of acute and healing arteritis with adjacent segments
of normal arteries and sparing viens and capillaries is considered
characteristic of which disease?
e. Polyarteritis nodosa.
The triad of necrotizing granulomatous lesions of the upper and lower
respiratory tract, focal segmental glomerulonephritis and necrotizing
vasculitis of small arteries and veins is characteristic of which entity?
f. Wegener's granulomatosis.
760.

Which ovarian tumor, which is seen more often in women under age
thirty, than in women older than fifty years? (PIP 2001 D)
a. Malignant mixed Mullerian tumor.
b. Yolk sac tumor.
c. Papillary serous carcinoma.
d. Endometrioid adenocarcinoma.

Reserved Copyrights - 2013

156

Pathology Questions

MCQs

e. Clear cell carcinoma.


761.

Which of the following features favors primary ovarian carcinomaover


metastatic colonic adenocarcinoma? (PIP 2001 D)
a. Cytokeratin 20 positivity, cytokeratin 7 negativity.
b. Carcinoembryonic antigen (CEA) positivity.
c. Normal serum CA-125.
d. Cribriform architecture with dirty necrosis.
e. Presence of squamous differentiation.

762.

Endometrioid carcinoma of the ovary may show all of the following


characteristics except: (PIP 2001 D)
a. Squamous differentiation.
b. Extracellular mucin.
c. High-grade micropapillae and numerous psammoma bodies.
d. Associated with endometriosis.
e. Associated with a synchronous endometrial primary.

763.

For each of the following questions choose the most likely diagnosis from
the following diagnostic set (an answer may be used once, more than
once, or not at all). (PIP 2001 D)
Diagnostic set:
a. Atypical endosalpingiosis.
b. Ectopic breast tissue.
c. Endometriosis.
d. Endosalpingiosis.
e. Hidradenoma papilliferum.
f. Metastatic adenocarcinoma.
g. Multilocular peritoneal inclusion cyst.
h. Urachal cyst.
Which lesion is congenital in origin and can present as an enlargingmass
in the labia?

Reserved Copyrights - 2013

157

Pathology Questions

MCQs

b. Ectopic breast tissue.


Which lesion is located in the lower midline abdomen and regarded as a
developmental malformation?
h. Urachal cyst.
Which lesion can manifest stromal decidual change?
c. Endometriosis
764.

Surgical excision is almost always indicated for: (PIP 2002 A)


a. Focal nodular hyperplasia.
b. Liver cell adenoma.
c. Both.
d. Neither.

765.

Oral contraceptive use has an etiologic association with: (PIP 2002 A)


a. Focal nodular hyperplasia.
b. Liver cell adenoma.
c. Both.
d. Neither.

766.

Chronic viral hepatitis B or C is often associated with: (PIP 2002 A)


a. Focal nodular hyperplasia.
b. Liver cell adenoma.
c. Both.
d. Neither.

767.

Typical features of aggressive angiomyxoma include all of the following


EXCEPT: (PIP 2002 A)
a. Infiltrative growth pattern.
b. High rate of local recurrence.
c. High rate of metastasis.
d. Origin in pelvis and peritonium.

Reserved Copyrights - 2013

158

Pathology Questions

MCQs

e. Predilection for females.


768.

A sharply circumscribed mass with a predilection for the peritonium, no


propensity for recurrence, and plump epithelioid tumor cells with a
predominantly perivascular arrangement are a feature of: (PIP 2002 A)
a. Superficial angiomyxoma.
b. Fibroepithelial polyp qith atypia.
c. Agressive angiomyxoma.
d. Nerve sheath myxoma.
e. Angiomyofibroblastoma.

769.

Helpful microscopic features in the differential diagnosis of pelvic myxoid


masses include all of the following EXCEPT: (PIP 2002 A)
a. An arborizing vascular pattern.
b. A nodular growth pattern.
c. Mature adipose tissue within the mass.
d. Multinucleated cells within the tumor.
e. Cytologic atypia and pleomorphism.

770.

The most common tumor of the heart in an adult is: (PIP 2002 A)
a. Cardiac fibroma.
b. Cardiac myxoma.
c. Cardiac rhabdomyoma.
d. Angiosarcoma.
e. Embryonal rhabdomyosarcoma.

771.

The most common tumor of the heart in a child is: (PIP 2002 A)
a. Cardiac fibroma.
b. Cardiac myxoma.
c. Cardiac rhabdomyoma.
d. Embryonal rhabdomyosarcoma.
e. Synovial sarcoma.

Reserved Copyrights - 2013

159

Pathology Questions
772.

MCQs

The most likely diagnosis in a young woman with spotty skin


pigmentation, a cardiac tumor, adrenal hyperplasia, and similar signs and
symptoms in family member: (PIP 2002 A)
a. Familial myxoma syndrome.
b. Tuberous sclerosis.
c. Metastatic adenocarcinoma.
d. Embryonal rhabdomyosarcoma.
e. Myxoid liposarcoma.

773.

Which of the following is NOT characteristic of atypical polypoid adenoma


(of Mazur)? (PIP 2002 A)
a. Proliferative or hyperplastic endometrial glands with metaplastic
changes.
b. May be misinterpreted as adenocarcinoma with myometrial
invasion on curettage specimen.
c. Multiple dilated cysts with intracystic, polypoid projections and
periglandular stromal consideration.
d. Stromal smooth muscle component may be quite prominent.
e. Stroma may contain occasional mitosis.

774.

Which of the following features is NOT found in low grade endometrial


stromal sarcoma: (PIP 2002 A)
a. Lymphovascular space invasion.
b. Typically contains less than 3 mitoses per 10 high power fields.
c. Viable glands adjacent to necrotic debris.
d. Nodular or diffuse premeation of myometrium.
e. Minimal nuclear pleomorphism.

775.

Adenosarcoma may show all of the following characteristics EXCEPT: (PIP


2002 A)
a. Cystic spaces with intraluminal projections.
b. Biphasic tumor with glandular and stromal componenets.
c. Cratilagenous and rhabdomyoblastic stromal differentiation.

Reserved Copyrights - 2013

160

Pathology Questions

MCQs

d. Periglandular stromal condensation.


e. Syncytiotrophoblastic stromal cells.
776.

For each of the following, select the most likely diagnosis from the
diagnostic set( an answer may be used once, more than once, or not at
all): (PIP 2002 A)
Diagnostic set:
a. Benign metastasizing leiomyoma.
b. Centrilobular emphysema.
c. Congenital pulmonary lymphangiectasis.
d. Endometrial stromal sarcoma, metastatic.
e. Interstitial pulmonary fibrosis.
f. Langerhans cell histiocytosis.
g. Leiomyosarcoma, metastatic.
h. Lymphangioleiomyomatosis.
i.

Lymphangiomatosis.

j.

Multifocal micronodular pneumocyte hyperpalsia.

k. Panacinar emphysema.
Which lesion is associated with tuberous sclerosis complex and is
typically negative for HMB-45?
j. Multifocal micronodular pneumocyte hyperpalsia.
Which lesion is cystic and occurs almost exclusively in women of
reproductive age?
h. Lymphangioleiomyomatosis.
Which lesion is associated with cigarette smoking and contains S100
positive cells?
f. Langerhans cell histiocytosis.
777.

All of the following afftect the pathalogic staging of an invasive breast


tumor EXCEPT: (PIP 2002 A)

Reserved Copyrights - 2013

161

Pathology Questions

MCQs

a. Tumor size.
b. Lymph node status.
c. Extranodal extension.
d. Dermal invasion.
e. Chest wall extension.
778.

Which invasive breast tumor has the worst prognosis? (PIP 2002 A)
a. Classic lobular carcinoma.
b. Metapalstic carcinoma.
c. Ductal carcinoma of no special type.
d. Tubular carcinoma.
e. Mucinous carcinoma.

779.

All of the following have been clinically validated as predictive and/or


prognostic factors in breast cancer EXCEPT: (PIP 2002 A)
a. HER2/neu.
b. Bcl-2.
c. PR.
d. ER.
e. Tumor size.

780.

For each of the following, select the most likely diagnosis from the
diagnostic set(an answer may be used once, more than once, or not at all):
(PIP 2002 A)
Diagnostic set:
a. Anaplastic carcinoma.
b. Hurthle cell neoplasm.
c. Hyalinizing trabecular adenoma.
d. Medullary thyroid carcinoma.
e. Metastatic neuroendocrine carcinoma.
f. Paraganglioma.
g. Plasmocytoma.
h. Poorly differentiated (insular) thyroid carcinoma.

Reserved Copyrights - 2013

162

Pathology Questions

MCQs

Monoclonal CEA immunoposistivity with a positive familial history is


supportive of this diagnosis:
d. Medullary thyroid carcinoma.
Is characterized by a multifocal growth pattern with subepithelial cell
balls protruding into thyroid follicles:
e. Metastatic neuroendocrine carcinoma.
Displays the following immunoreactivity: pan-neuroendocrine positive
tumor cells and S-100 protein positive sustentacular cells, and cytokeratin
negative tumor cells:
f. Paraganglioma.
781.

Tumor cells containing abundant mitochondria are most characteristic of:


(PIP 2002 A)
a. Granular cell tumor.
b. Pheochromocytoma.
c. Adrenocortical carcinoma.
d. Adrenocortical oncocytoma.
e. Alveolar soft part sarcoma.

782.

Pheochromocytomaa are part of the spectrum of neoplasms in each of the


following syndromes EXCEPT: (PIP 2002 A)
a. Multiple endocrine neoplasia type 1.
b. Multiple endocrine neoplasia type 2A.
c. Multiple endocrine neoplasia type 2B.
d. Von Hipple-Lindau disease.
e. Neurofibromatosis.

783.

In the absence of metastatic disease, malignancy in pheochromocytoma


can be predicted by: (PIP 2002 A)
a. Vascular invasion.

Reserved Copyrights - 2013

163

Pathology Questions

MCQs

b. Abnormal mitotic figures and necrosis.


c. Nuclear pleomorphism.
d. Pigmentation.
e. None of the above.
784.

Which of the following statements about thymic carcinoid tumor is TRUE?


(PIP 2002 A)
a. It

may

be

associated

with

symptomatic

neuromascular

abnormalities.
b. It is usually encapsulated and exhibits fibrous trabeculae.
c. It is associated with paraneoplastic phenomena in 90 percent of
cases.
d. It is commonly associated with the complete (classic) carcinoid
syndrome.
785.

The lymphocytes involved in thymic follicular hyperpalsia: (PIP 2002 A)


a. Are CD1a, CD2, CD3, and TdT positive.
b. Show rearrangement of the T-cell receptor gene.
c. Exhibit rearrangements of the immunoglobulin heavy chain gene.
d. Are CD20 and CD21 positive and TdT negative.

786.

The most important prognostic indicator for thymoma is: (PIP 2002 A)
a. Histologic subtype.
b. Clinicopathologic stage.
c. Ploidy status.
d. Presence of associated paraneoplastic phenomena.

787.

Which single feature accurately predicts the behavior of a uterine smooth


muscle tumor? (PIP 2002 A)
a. Intravenous growth.
b. Mitotic activity.
c. Tumor size.
d. Tumor necrosis.

Reserved Copyrights - 2013

164

Pathology Questions

MCQs

e. Cellular atypia.
f. None of the above.
788.

Which special stain accurately distinguishes endometrial stromal cells


from myometrial cells? (PIP 2002 A)
a. Cytokeratin.
b. Desmin.
c. Smooth muscle actin.
d. Vimentin.
e. Reticulin.
f. None of the above.

789.

Which feature is of least importance in evaluating an epithelioid smooth


muscle tumor? (PIP 2002 A)
a. Cellular atypia.
b. Necrosis.
c. Size.
d. Mitotic activity.

790.

Metastatic carcinoma, glioblastoma, cerebral abcess, primary central


nervous system lymphoma and demyelinating pseudotumor all may share
which

of

the

following

appearances

on

magnetic

resonance

imaging(MRI)? (PIP 2002 B)


a. Non-enhancing diffuse abnormality.
b. Non-enhancing circumscribed mass.
c. Solitary contrast-enhancing ring lesion.
d. T2-wheighted

abnormality

indicated

periventricular

demyelination.
e. T2-wheighted abnormality showing only cerebral white matter
edema.
791.

An important pathophysiologic mechanism in cerebral abscess is: (PIP


2002 B)

Reserved Copyrights - 2013

165

Pathology Questions

MCQs

a. Autoimmune attack against an endogenous myelin epitope.


b. Hematogenous dissemination.
c. Immunosuppression.
d. Loss of integrity of a normal physical barrier against infection.
e. Loss of mutation of a tumor suppressor gene.
792.

Granular mitoses and Creutzfeldt astrocytes are characteristic histologic


features that are most closely associated with: (PIP 2002 B)
a. Cerebral abcess.
b. Demyelinating disease.
c. Glioblastoma.
d. Metastatic brain tumors.
e. Toxoplasmosis.

793.

For each of the following, select the most likely diagnosis from the
diagnostic set (an answer may be used once, more than once, or not at
all): (PIP 2002 B)
Diagnostic set:
a. Adenocarcinoma.
b. Basaloid carcinoma.
c. Biphasic pulmonary blastoma.
d. Carcinoid tumor.
e. Malignant mesothelioma.
f. Mantle cell lymphoma.
g. Small cell carcinoma.
Which pulmonary lesion is characterized by solid cell nests, fusiform
tumor cells with peripheral palisading, and a CD56 (NCAM) negative, CK
5/6 positive phenotype? (PIP 2002 B)
b. Basaloid carcinoma.

Reserved Copyrights - 2013

166

Pathology Questions

MCQs

Which tumor typically shows prominent necrosis and increased mitotic


activity with strong membranous immunoposistivity for CD56 (NCAM)
and dot-like cytoplasmic staining for low molecular weight cytokeratin?
g. Small cell carcinoma.
Which tumor is combined with keratinizing squamous carcinoma in a
high percentage of cases?
b. Basaloid carcinoma.
794.

For each of the following, select the most likely diagnosis from the
diagnostic set (an answer may be used once, more than once, or not at
all): (PIP 2002 B)
Diagnostic set:
a. Immature teratoma.
b. Malignant mesodermal mixed tumor.
c. Mature cystic teratoma.
d. Mature solid teratoma.
e. Primitive neuroectodermal tumor.
Lacks neuroectodermal tissue and is composed of mullerian type
carcinoma and mesenchymal componenets:
b. Malignant mesodermal mixed tumor.
Predominantly composed of primitive neuroectodermal tissue with
interstitial type glands and immature cartilage:
a. Immature teratoma.
A cystic lesion lined entirely or partly by epithelium resembling the
epidermis with its appendages, and containing mesodermal, endodermal
and ectodermal elements:
c. Mature cystic teratoma.

Reserved Copyrights - 2013

167

Pathology Questions
795.

MCQs

For each of the following, select the most likely diagnosis from the
diagnostic set( an answer may be used once, more than once, or not at
all): (PIP 2002 B)
Diagnostic set:
a. Anaplastic large cell lymphoma, T-cell and null-cell type.
b. Angioimmunoblastic T-cell lymphoma.
c. Follicular lymphoma (follicle center lymphoma),grade 3.
d. Hodgkin lymphoma, mixed cellularity type.
e. Hodgkin lymphoma, nodular sclerosing type.
f. Large B-cell lymphoma, T-cell/histiocyte rich.
g. Metastatic lymphoepithelial-like carcinoma.
h. Metastatic melanoma.
i.

Seminoma.

Most of the large neoplastic cells are in sinuses, have abundant cytoplasm,
reniform to U-shapef nuclei, and small nucleoli, and are CD15(-), CD20(-),
CD30(+), CD45(+) and EMA(+):
a.Anaplastic large cell lymphoma, T-cell and null-cell type.
Most of the large neoplastic cells have amphophilic to basophilic
cytoplasm, round to ovoid nuclei, and large nucleoli, and are CD15(-),
CD20(+), CD30(-), CD45(+) and EMA(+):
f.Large B-cell lymphoma, T-cell/histiocyte rich.
Most of the large neoplastic cells have abundant clear cytoplasm, b-or
multilobulated nuclei, easily identified nucleoli, and are CD15(+),
CD30(+), CD45(-) and EMA(-). Architecture is nodular with dense
collagen bands:
e.Hodgkin lymphoma, nodular sclerosing type.)
796.

For each of the following, select the most likely diagnosis from the
diagnostic set( an answer may be used once, more than once, or not at
all): (PIP 2002 B)

Reserved Copyrights - 2013

168

Pathology Questions

MCQs

Diagnostic set:
a. Adenosarcoma.
b. Endometioid stromal sarcoma with sex-cord like differentiation.
c. Fibrosarcoma.
d. Granulosa cell tumor.
e. Immature teratoma.
f. Malignant mesodermal mixed tumor.
g. Sclerosing stromal tumor.
Which lesion characteristically shows hypercellular cuffing of atypical
stromal cells around benign glandular componenets?
a. Adenosarcoma.
Which lesion commonly shows immunohistochemical expression of
epithelial markers in malignant stromal cells?
f. Malignant mesodermal mixed tumor.
Which of the above is characterized by the presence of numerous small
arteries resembling those of late secretory type endometrium?
b.

Endometioid

stromal

sarcoma

with

sex-cord

like

differentiation.)
797.

For each of the following, select the most likely diagnosis from the
diagnostic set( an answer may be used once, more than once, or not at
all): (PIP 2002 B)
Diagnostic set:
a. Bronchogenic cyst.
b. Immature teratoma.
c. Mature cystic teratoma.
d. Nonteratomatous mixed germ cell tumor.
e. Seminoma.
f. Teratoma with malignant components.
g. Thymic cyst.

Reserved Copyrights - 2013

169

Pathology Questions

MCQs

Which entity is the most common primary mediastinal germ cell tumor?
c. Mature cystic teratoma.
Which entity includes a sarcomatous or carcinomatous componenet
associated with disorganized mature tissues?
f. Teratoma with malignant components.
Which

entity

includes

primitive

neuroepithelial

tubules

among

disorganized mature tissues?


b. Immature teratoma.
798.

For each of the following, select the most likely diagnosis from the
diagnostic set (an answer may be used once, more than once, or not atall):
(PIP 2002 B)
Diagnostic set:
a. Acinar cell carcinoma.
b. Chronic pancreatitis.
c. Ductal adenocarcinoma.
d. Intraductal papillary mucinous tumor.
e. Islet cell tumor.
f. Metastatic renal cell carcinoma.
g. Mucinous cystadenocarcinoma.
h. Pancreatoblastoma.
i.

Solid-pseudopapillary tumor.

The tumor shows proliferation of dysplastic epithelium with excess mucin


production involving the main pancreatic duct, associated with chronic
pancreatitis:
d. Intraductal papillary mucinous tumor.

Reserved Copyrights - 2013

170

Pathology Questions

MCQs

This aggressive tumor typically consists of infiltrating glands embedded


in dense desmoplastic stroma forming an ill-defined mass in the
pancreatic head:
c. Ductal adenocarcinoma.
This indolent tumor almost exclusively occurs in the pancreas of
adolescent or young adult women:
i. Solid-pseudopapillary tumor.
799.

For each of the following, select the most likely diagnosis from the
diagnostic set (an answer may be used once, more than once, or not at
all): (PIP 2002 B)
Diagnostic set:
a. Extraskeletal osteosarcoma.
b. Fibrodysplasia ossificans progressiva.
c. Heterotopic ossification.
d. Myositis ossificans.
e. Nodular fascitis.
f. Osteoblastic carcinoma.
g. Posttraumatic periositis.
Which condition typically presents in childhood and is hereditary
progressive debilitating disorder:
b. Fibrodysplasia ossificans progressiva.
Which condition is a self-limiting ossifying process that follows trauma
and morphologically demonstrates a zonal phenomenon with immature
tissues in the center of the lesion and mature bone at theperiphery?
d. Myositis ossificans.
Which condition typically presents in the 6th -7th decades of life and is a
primary malignancy of the soft tissue?
a. Extraskeletal osteosarcoma.

Reserved Copyrights - 2013

171

Pathology Questions

800.

MCQs

Ring chromosomes, characteristic of well differentiated liposarcoma, are


also commonly seen in: (PIP 2002 B)
a. Lipoma.
b. Rhabdomyosarcoma.
c. Round cell liposarcoma.
d. Dedifferentiated liposarcoma.
e. Uterine adenofibroma.

801.

For each of the following, select the most likely diagnosis from the
diagnostic set( an answer may be used once, more than once, or not at
all): (PIP 2002 B)
Diagnostic set:
a. Eosinophilic gastroenteritis.
b. Fundic gland polyp.
c. Gastrointestinal stromal tumor (GIST).
d. Inflammatory fibroid polyp.
e. Inflammatory myofibroblastic tumor.
Which lesion is a benign neoplastic mucosal excrescene characterized by
cystically dilated glands?
b. Fundic gland polyp.
Which lesion is consistently CD117 (c-kit) positive?
c. Gastrointestinal stromal tumor (GIST).
Which lesion is most strongly associated with peripheral eosinophilia?
a. Eosinophilic gastroenteritis.

802.

The four most common solitary circumscribed tumors of the lumbar


cistern are: (PIP 2002 D)
a. Hemangioblastoma, meningioma, schwannoma, subependymoma.
b. Meningioma, schwannoma, neurothekoma, paraganglioma.

Reserved Copyrights - 2013

172

Pathology Questions

MCQs

c. Meningioma, paraganglioma, ganglioneuroma, paciniomyolipoma.


d. Schwannoma,

meningioma,

myxopapillary

ependymoma,

meningioma,

myxopapillary

ependymoma,

paraganglioma.
e. Schwannoma,

subependymoma.
803.

Which of the following immunostains is most often positive in


myxopapillary ependymoma of the filum terminale: (PIP 2002 D)
a. Chromogranin.
b. Cytokeratins.
c. Epithelial membrane antigen.
d. S-100 protein.
e. Synaptophysin.

804.

The most helpful ultrastructural feature in the diagnosis of myxopapillary


ependymoma of the filum terminal would be the presence of: (PIP 2002
D)
a. Dense-core neurosecretory-type granules.
b. Desmosomes.
c. Intercellular lumens with microvilli and/or cilia.
d. Luse bodies (long-spacing collagen).
e. Neurotubule bundles.

805.

For each of the following, select the most likely diagnosis from the
diagnostic set (an answer may be used once, more than once, or not at
all): (PIP 2002 D)
Diagnostic set:
a. Choriocarcinoma.
b. Epithelioid leiomyosarcoma.
c. Epithelioid trophoblastic tumor.
d. Perivascular epithelioid cell tumor.
e. Placental site nodule.
f. Placental site trophoblastic tumor.

Reserved Copyrights - 2013

173

Pathology Questions

MCQs

g. Squamous cell carcinoma.


Immunopositive for pankeratin (AE1/AE3) with high Ki-67 index(>50%);
negative for inhibin- and CK-18; negative serum -hCG:
g. Squamous cell carcinoma.
Microscopic lesion with low cellularity, hyalinized stroma, and Ki-67
index less than 10 percent; immunoposistive for inhibin-, CK-18; focally
positive for hPL and CD146 (Mel-CAM):
e. Placental site nodule.
Immunopositive for inhibin-, AE1/AE3,CK-18; diffusely positive for hPL;
Ki-67 index less than 5 percent; low elevation of serum -hCG:
f. Placental site trophoblastic tumor.
806.

Which tumor typically presents as a painful subcutaneous mass? (PIP


2002 D)
a. Adult rhabdomyoma.
b. Angiolipoma.
c. Chondroid lipoma.
d. Granular cell tumor.
e. Hibernoma.

807.

Supernumerary

ring

chromosomes

of

chromosome

12

are

commonfeature of: (PIP 2002 D)


a. Adult rhabdomyoma.
b. Angiolipoma.
c. Hibernoma.
d. Lipoma.
e. Well differentiated liposarcoma (atypical lipomatous tumor).
808.

The most common site of which tumor is the head and neck region: (PIP
2002 D)

Reserved Copyrights - 2013

174

Pathology Questions

MCQs

a. Adult rhabdomyoma.
b. Angiolipoma.
c. Hibernoma.
d. Lipoma.
e. Well differentiated liposarcoma (atypical lipomatous tumor).
809.

For each of the following, select the most likely diagnosis from the
diagnostic set (an answer may be used once, more than once, or not at
all): (PIP 2002 D)
Diagnostic set:
a. Carcinoid tumor.
b. Gastrointestinal stromal tumor(GIST).
c. Invasive poorly differentiated adenocarcinoma.
d. Invasive well differentiated adenocarcinoma.
e. Lymphoepithelioma-like carcinoma.
f. Small cell neuroendocrine carcinoma.
Which lesion can arise in the setting of pernicious anemia as a response to
hypergastrnemia:
a. Carcinoid tumor.
Which lesion is consistently CD117 (c-kit) positive?
b. Gastrointestinal stromal tumor (GIST).

810.

Which lesion may be associated with Epstein-Barr infection?


e. Lymphoepithelioma-like carcinoma.

811.

For each of the following, select the most likely diagnosis from the
diagnostic set (an answer may be used once, more than once, or not at
all): (PIP 2002 D)
Diagnostic set:
a. Autoimmune hepatitis.
b. Chronic hepatitis B.

Reserved Copyrights - 2013

175

Pathology Questions

MCQs

c. Chronic hepatitis C.
d. Primary biliary cirrhosis.
e. Primary sclerosing cholangitis.
f. Sarcoidosis.
Which entity usually shows negative viral serology with positive
antinuclear

antibodies

(ANA)

and

moderately

elevated

alanine

aminotransferase (ALT) without evidence of significant biliary damage?


a. Autoimmune hepatitis.
Which entity usually shows negative viral serology with positive
antimitrochondrial

antibodies

(AMA),

mildly

elevated

alanine

aminotransferase (ALT) significant damage of biliary damage? (PIP 2002


D)
d. Primary biliary cirrhosis.
Which entity is usually diagnosed by the radiographic demonstration of
beaded bile ducts in the absence of other disease?
e. Primary sclerosing cholangitis.
812.

For each of the following, select the most likely diagnosis from the
diagnostic set (an answer may be used once, more than once, or not at
all): (PIP 2002 D)
Diagnostic set:
a. Acute myelogenous leukemia.
b. Anaplastic large cell lymphoma
c. Diffuse large B-cell lymphoma.
d. Hairy cell leukemia.
e. Hepatosplenic T-cell lymphoma.
f. Hodgkin lymphoma, classic type.
g. Malignant melanoma.
h. Metastatic adenocarcinoma.

Reserved Copyrights - 2013

176

Pathology Questions

MCQs

The neoplastic cells are typically positive for CD15 and CD30 and are
negative for CD45:
f. Hodgkin lymphoma, classic type.
Lymphoma which usually forms a mass when involving the spleen and
whose cells strongly express CD30, CD45:
b. Anaplastic large cell lymphoma
Agressive lymphoma with diffuse pattern of splenic involvement and
cytoplasmic TIA-1 expression:
e. Hepatosplenic T-cell lymphoma.
813.

For each of the following, select the most likely diagnosis from the
diagnostic set (an answer may be used once, more than once, or not atall):
(PIP 2002 D)
Diagnostic set:
a. Choriocarcinoma.
b. Dysgerminoma.
c. Embryonal carcinoma.
d. Endometrioid adenocarcinoma.
e. Juvenile granulosa cell tumor.
f. Metastatic hepatocellular adenocarcinoma.
g. Mixed malignant germ cell tumors.
h. Polyemberyoma.
i.

Sertoli-Leydig cell tumor.

j.

Yolk sac tumor (endodermal sinus tumor).

Composed of primordial germ cells with clear cytoplasm and distinct cell
membranes arranged in diffuse, insular trabecular and cord- like patterns.
The stroma consists of broad fibrous bands rich in lymphocytes, and
sarcoid-like granulomas can be seen. Tumor cells are immunoreactive for
placental like alkaline phosphatase.:
b. Dysgerminoma.
Reserved Copyrights - 2013

177

Pathology Questions

MCQs

Characterized by a wide variety of microscopic patterns that at least


focally includes a reticular pattern, shows Schiller-Duval bodies and
scattered intracellular hyaline bodies, and is immunoreactive for alphafetoprotein:
j.Yolk sac tumor (endodermal sinus tumor).
Occurs in younger patients and is composed of large, pleomorphic cells
that line glands and papillae and grow as solid sheets. Tumor cells are
occasionally immunoreactive for alpha-fetoprotein and may show human
chorionic gonadotropin and associated syncytiotrophoblastic cells:
c. Embryonal carcinoma.
814.

For each of the following, select the most likely diagnosis from the
diagnostic set (an answer may be used once, more than once, or not at
all): (PIP 2002 D)
Diagnostic set:
a. B-cell

chronic

lymphocytic

leukemia/small

lymphocytic

lymphoma.
b. Follicular lymphoma.
c. Hairy-cell leukemia.
d. Lymphoid hyperplasia.
e. Mantle cell lymphoma.
f. Splenic marginal zone lymphoma.
Which tumor is characterized by immunophenotypic profile of CD19(+),
CD5(+) and CD23(+):
a.

B-cell

chronic

lymphocytic

leukemia/small

lymphocyticlymphoma)
Which tumor has a characteristic chromosomal translocation of
t(11;14)(q13;q32), with resultant overexpression of cyclin D1:
e. Mantle cell lymphoma.
Reserved Copyrights - 2013

178

Pathology Questions

MCQs

Which tumor is characterized by immunop profile of CD19(+), CD5(-),


CD23(-), CD10(-), is tartrate resistant acid phosphatase (TRAP) negative,
and does not express cyclin D1:
f. Splenic marginal zone lymphoma.
815.

What is the commonly used diagnostic label to describe the infant


accompanied by Listeria monocytogenes infection of the placenta: (PIP
2002 D)
a. Hepato-adrenal necrosis.
b. Granulomatosis infantisepticum.
c. Group B streptococci species.
d. Congenital TORCH infection.
e. Perinatal herpes simplex infection.

816.

An expected maternal history in Listeria monocytogenes infection of the


placenta would be:(PIP 2002 D)
a. Genital herpes simplex infection.
b. Recent ingestion of packaged meat or cheese.
c. Recent immigration to the united states from a tropical country.
d. The presence of multiple household cats.
e. Sexual promiscuity.

817.

Which is the most appropriate comment? (PIP 2002 D)


a. The causative organism in Listeria monocytogenes infection of the
placenta is the most common cause of neonatal species.
b. The placental changes suggest that the infant should have been
treated with Acyclovir.
c. The

mother's

second

untoward

pregnancy

outcome

is

characteristic of the diagnosis of Listeria monocytogenes infection


of the placenta.
d. Successful detection of Listeria monocytogenes infection of the
placenta by placental exam may direct therapy.
e. None of the above.
Reserved Copyrights - 2013

179

Pathology Questions
818.

MCQs

For each of the following, select the most likely diagnosis from the
diagnostic set( an answer may be used once, more than once, or not at
all): (PIP 2002 D)
Diagnostic set:
a. Active chronic gastitis helicobacter pylori associated (Type B
gastritis, chronic antral gastritis).
b. Chronic atrophic gastritis (Type A gastritis, diffuse corporal
gastritis, autoimmune gastritis).
c. Eosinophilic gastroenteritis.
d. Extranodal marginal zone B-cell lymphoma(MALT lymphoma).
e. Gastric candidiasis.
f. Lmphocytic gastritis (Varioliform gastritis).
g. Reactive gastropathy (Chemical gastritis).
Which entity is most commonly associated with duodenal ulcers?
a. Active chronic gastitis helicobacter pylori associated (Type B
gastritis, chronic antral gastritis).
Which entity is associated with anti-intrinsic factor antibodies?
b. Chronic atrophic gastritis (Type A gastritis, diffuse corporal
gastritis, autoimmune gastritis).
Which entity is associated with non-steroidal anti-inflammatory agents?
g. Reactive gastropathy (Chemical gastritis).

819.

For each of the following questions choose the most likely diagnosis from
the following diagnostic set (an answer may be used once, more than
once, or not at all). (PIP 2003 B)
Diagnostic set:
a. Clear cell epedymoma.
b. Demyelinating disease.
c. Diffuse astrocytoma.
d. Dysembryoplastic neuroepithelial tumor.

Reserved Copyrights - 2013

180

Pathology Questions

MCQs

e. Extraventicular neurocytoma.
f. Ganglioglioma.
g. Oligodendroglioma.
h. Progressive multifocal leukoencephalopathy.
In which lesion is allelic loss of chromosomal arms 1p and 19q strongly
associated with chemosensitivity?
g. Oligodendroglioma.
Which lesion features a columnar arrangement of small round cells,
delicate fibrillary processes, and fine capillaries surrounded by pools of
mucin with floating neurons?
d. Dysembryoplastic neuroepithelial tumor.
Which lesion is associated with JC virus infection?
h. Progressive multifocal leukoencephalopathy.
820.

This condition is more common in the elderly, is typically acute and


segmental, and features superficial mucosal necrosis as the hallmark
histologic finding: (PIP 2003 B)
a. Ischemic colitis.
b. Crohn's colitic.
c. Acute self-limited bacterial colitis.
d. Fulminant phase of chronic ulcerative colitis.
e. Amebic colitis.

821.

The fulminant phase of chronic ulcerative colitis may be associated with


which one of the following? (PIP 2003 B)
a. Epithelioid granulomas bordering the muscularis propria.
b. Fissuring ulceration of the colon.
c. Segmental involvement of the jejunum.
d. Intervening segments of microscopically normal colonic mucosa.
e. Dense lymphoid aggregates of the deep submucosa and serosa.

Reserved Copyrights - 2013

181

Pathology Questions

822.

MCQs

The major short term implication of distinguishing Crohn's colitis from


chronic ulcerative colitis at the time of colectomy for fulminant phase of
colitis is: (PIP 2003 B)
a. Assessment of malignant potential.
b. Selection of appropriate medication.
c. Selection of appropriate dietary restriction.
d. Selection of appropriate surgical reconstruction.
e. Determining how stringently to search for cytomegalovirus
inclusions.

823.

For each of the following questions choose the most likely diagnosis from
the following diagnostic set (an answer may be used once, more than
once, or not at all). (PIP 2003 B)
Diagnostic set:
a. Adult granulosa cell tumor.
b. Carcinoid tumor.
c. Endometrioid carcinoma.
d. Leydig cell tumor.
e. Malignant mixed Mllerian tumor.
f. Metastatic carcinoma.
g. Sclerosing stromal tumor.
h. Sertoli-Leydig cell tumor.
Which is the most common inhibin-positive tumor?
a. Adult granulosa cell tumor.
Squamous differentiation would be most commonly encountered in
which tumor?
c. Endometrioid carcinoma.
Which tumor is most likely to show bilateral ovarian involvement?
f. Metastatic carcinoma.

Reserved Copyrights - 2013

182

Pathology Questions
824.

MCQs

Which of the following is most likely to be associated with development of


the hemolytic-uremic syndrome? (PIP 2003 B)
a. Fulminant ulcerative colitis.
b. Clostridium difficile colitis.
c. Cytomegalovirus colitis.
d. Necrotizing herpes simplex colitis.
e. Hemorrhagic colitis due to E.coli 0157:H7.

825.

In a patient who is suspected to have CMV colitis, which of the following


assays performed on biopsy tissue or stool is least likely to confirm the
diagnosis? (PIP 2003 B)
a. Viral culture of fresh biopsy tissue.
b. Stool viral culture.
c. Identification of CMV in H&E sections.
d. Immunoperoxidase staining.
e. In-situ hybridization.

826.

A colorectal biopsy performed on an immunosuppressed patient with


colitis demonstrates intranuclear inclusions pedominantly affecting the
surface epithelium including goblet cells; the inclusions are amphophilic,
of crescent or targetoid shape and completely fill the affected nuclei.
Which of the following are they most likely to present? (PIP 2003 B)
a. Varicella-zoster virus.
b. Herpes simplex virus.
c. Adenovirus.
d. Cytomegalovirus.
e. Epsein-Barr virus.

827.

Which one of the following is not a characteristic feature of serous


borderline cystadenofibroma? (PIP 2003 B)
a. A 5mm or greater confluent area of micropapillary growth.
b. Extensive epithelial stratification.
c. Detachment of epithelial cell clusters.

Reserved Copyrights - 2013

183

Pathology Questions

MCQs

d. Epithelial cell atypia.


e. Mitoses usually less than 4 per 10 HPF.
828.

The most common surface epithelial tumor of the ovary is: (PIP 2003 B)
a. Serous borderline tumor.
b. Benign mucinous cystadenoma.
c. Benign serous cystadenoma.
d. Borderline serous tumor.
e. Serous carcinoma.

829.

For each of the following questions choose the most likely diagnosis from
the following diagnostic set (an answer may be used once, more than
once, or not at all). (PIP 2003 B)
Diagnostic set:
a. Angiosarcoma.
b. Endovascular papillary angioendothelioma (Dabska tumor).
c. Inflammatory carcinoma.
d. Kaposi sarcoma.
e. Malignant melanoma.
f. Metaplastic carcinoma.
g. Spindle cell hemangioma.
h. Stewart-Treves syndrome.
Associated with chronic lymphedema in post mastectomy patients:
h. Stewart-Treves syndrome.
Associated with human herpesvirus 8 (HHV8):
d. Kaposi sarcoma.
A low-grade malignant cutaneous neoplasm typically presenting in
children as diffuse swelling or intradermal tumor:
b. Endovascular papillary angioendothelioma (Dabska tumor).

Reserved Copyrights - 2013

184

Pathology Questions
830.

MCQs

For each of the following questions choose the most likely diagnosis from
the following diagnostic set (an answer may be used once, morethan once,
or not at all). (PIP 2003 B)
Diagnostic set:
a. Angimyolipoma, extra renal.
b. Abdominal desmoplastic small round cell tumor.
c. Clear cell myomelanocytic tumor.
d. Clear cell sarcoma of tendons and aponeuroses.
e. Gastrointestinal stromal tumor.
f. Leiomyosarcoma.
Which tumor is antigenically poly-phenotypic but does not express
HMB-45?
b. Abdominal desmoplastic small round cell tumor.
Which tumor is also known as malignant melanoma af soft parts?
d. Clear cell sarcoma of tendons and aponeuroses.
Which tumor typically expresses both CD117 and CD34?
e. Gastrointestinal stromal tumor.

831.

For each of the following questions choose the most likely diagnosis from
the following diagnostic set (an answer may be used once, more than
once, or not at all). (PIP 2003 B)
Diagnostic set:
a. Angioimmunoblastic T-cell lymphoma.
b. Castleman disease.
c. Follicular lymphoma.
d. HIV-related lymphadenopathy.
e. Hodgkin's lymphoma.
f. Mantle cell lymphoma.

Reserved Copyrights - 2013

185

Pathology Questions

MCQs

Vascular proliferation and an interfollicular or diffuse proliferation of


intermediate size atypical T-lymphocytes with clear cytoplasm and
prominent mitotic activity:
a. Angioimmunoblastic T-cell lymphoma
Effacement of the nodal architecture by mixed cell proliferation with
scattered atypical large cells with polylobated nuclei:
e. Hodgkin's lymphoma.
Prominent interfollicular proliferation of blood vessels and small
lymphocytes, obliteration of lymph node sinuses, and abnormal follicles
with regressive germinal centers and onion-scaling lymphocytes:
b. Castleman disease.
Which of the following is true regarding diffuse parenchymal amyloidosis
in the lung? (PIP 2003 B)
a. It is never associated with underlying systemic amyloidosis.
b. It almost always causes severe respiratory imparement.
c. The histologic appearance is essentially indistinguishable from
pulmonary involvement by systemic light chain disease.
d. Most cases are familial or associated with hemodialysis.
e. In most cases the type of amyloid deposits is AA.
832.

Which disease is characterized by diffuse pulmonary fibrosis without


temporal heterogeneity or hyaline membrane formation?
a. Diffuse parenchymal amyloidosis.
b. Non-specific interstitial pneumonia (NSIP), fibrosing type.
c. Usual interstitial pneumonia (UIP).
d. Diffuse alveolar damage (DAD), acute phase.
e. None of the above.

833.

Whcih disease is characterized by patchy interstitial fibrosis, which shows


temporal heterogeneity in the form of fibroblastic foci? (PIP 2003 B)

Reserved Copyrights - 2013

186

Pathology Questions

MCQs

a. Diffuse parenchymal amyloidosis.


b. Non-specific interstitial pneumonia (NSIP), fibrosing type.
c. Usual interstitial pneumonia (UIP).
d. Diffuse alveolar damage (DAD), acute phase.
e. None of the above.
834.

The

distinct

histologic

features

of

invasive

micropapillary

carcinomainclude all of the following except: (PIP 2003 B)


a. Spaces surrounding micropapillary structures.
b. Vascular papillary fonds.
c. Intermediate or high nuclear grade.
d. Frequent tumor emboli.
e. Numerous mitoses.
835.

The most common immunopheotype associated with micropapillary


carcinoma is: (PIP 2003 B)
a. ER(-) PR(-) HER2(-).
b. ER(+) PR(-) HER2(-).
c. ER(+) PR(+) HER2(-).
d. ER(+) PR(+) HER2(+).
e. ER(-) PR(-) HER2(+).

836.

For each of the following, select the most likely diagnosis from the
diagnostic set( an answer may be used once, more than once, or not at
all): (PIP 2003 C)
Diagnostic set:
a. Angiosarcoma.
b. Endometrial stromal sarcoma.
c. Intravenous leiomyomatosis.
d. Leiomyosarcoma.
e. Renal cell carcinoma, sarcomatoid type.

Reserved Copyrights - 2013

187

Pathology Questions

MCQs

Which tumor is typically composed of round cells with indistinct borders,


contains prominent arterioles, is typically negative for actin,desmin and
h-Caldesmon and is strongly positive for CD10?
b. Endometrial stromal sarcoma.
Which tumor is the most common primary tumor of the vena cava?
d. Leiomyosarcoma.
Which tumor is composed of spindled cells with moderately abundant
eosinophilic cytoplasm and centrally placed elongated nuclei with blunt
ends, shows a mitotic count greater than 5 per HPFs, is positive for actin,
desmin and h-Caldesmon and is negative for CD10?
d. Leiomyosarcoma.
837.

For each of the following, select the most likely diagnosis from the
diagnostic set (an answer may be used once, more than once, or not at
all): (PIP 2003 C)
Diagnostic set:
a. Giant fibrovascular polyp.
b. Inflammatory fibroid polyp.
c. Pseudoepitheliomatous hyperpalsia.
d. Squamous cell carcinoma.
e. Squamous papilloma.
The most common neoplasm of the middle esophagus, typically
associated with long-standing chronic esophagitis:
d. Squamous cell carcinoma.
Pedunculated esophegeal lesion that most commonly arises in the upper
third of the esophagus, predominantly occurs in men and is lined by
squamous epithelium:
a. Giant fibrovascular polyp.

Reserved Copyrights - 2013

188

Pathology Questions

MCQs

Typically a solitary exophytic esophegeal lesion that occurs in middle


aged men with the majority located in the lower esophagus:
e. Squamous papilloma.
838.

For each of the following, select the most likely diagnosis from the
diagnostic set (an answer may be used once, more than once, or not at
all): (PIP 2003 C)
Diagnostic set:
a. Adult granulosa cell tumor.
b. Carcinoid tumor.
c. Cellular fibroma.
d. Endometrioid carcinoma.
e. Juvenile granulosa cell tumor.
f. Metastatic breast carcinoma.
g. Small cell carcinoma(hypercalcemic type).
h. Thecoma.
Which neoplasm is most commonly associated with the formation of
microfollicles termed Call-Exner bodies?
a. Adult granulosa cell tumor.
Which primary ovarian neoplasm is highly aggressive, but usually occurs
in a younger age group?
g. Small cell carcinoma (hypercalcemic type).
Which entity is a low-grade malignancy showing indolent behavior with
potential for very late recurrences?
a. Adult granulosa cell tumor.

839.

Which is the most frequent benign uterine tumor in women? (PIP 2003 C)
a. Atypical polypoid adenomyoma.
b. Leiomyoma.
c. Adenomatoid tumor.

Reserved Copyrights - 2013

189

Pathology Questions

MCQs

d. Phyllodes tumor.
840.

Which uterine lesion is typically found in the clinical setting of diabetes,


hypertension and obesity? (PIP 2003 C)
a. Atypical polypoid adenomyoma.
b. Endometrial hyperpalsia.
c. Leiomyoma.
d. Adenomatoid tumor.

841.

For each of the following, select the most likely diagnosis from the
diagnostic set (an answer may be used once, more than once, or not at
all): (PIP 2003 C)
Diagnostic set:
a. Angiosarcoma
b. Embryonal(undifferentiated) sarcoma.
c. Embroynal rhabdomyosarcoma.
d. Hepatocellular carcinoma.
e. Mesenchymal hamartoma.
Which is the most common liver tumor in the 5-20 year old group?
d. Hepatocellular carcinoma.
Which hepatic neoplasm most commonly arises within large bileducts?
c. Embroynal rhabdomyosarcoma.
Which liver tumor occurs in the 5-20 year old age group, contains
pleomorphic spindle cells, displays eosinophilic inclusions, and has a
characteristic myxoid background?
b. Embryonal (undifferentiated) sarcoma.

842.

For each of the following, select the most likely diagnosis from the
diagnostic set (an answer may be used once, more than once, or not at
all): (PIP 2003 C)

Reserved Copyrights - 2013

190

Pathology Questions

MCQs

Diagnostic set:
a. Angiosarcoma.
b. Bacillary angiomatosis.
c. Littoral cell angioma.
d. Lymphangioma.
e. Peliosis.
f. Splenic hamartoma.
g. Splenic hemangioma.
Which tumor is highly aggressive and commonly metastasizes?
a. Angiosarcoma.
An unusual coexpression of endothelial (CD31) and histiocytic (CD68)
antigens characterizes this splenic lesion:
c. Littoral cell angioma.
Which splenic lesion exhibits a haphazard arrangement of blood-filled
spaces, but does not form a discrete mass?
e. Peliosis.
843.

Which neoplasm is characterized by t(x;18)(p11.2,q11.2)? (PIP 2003 C)


a. Cellular schwannoma.
b. Fibrosarcoma.
c. Hemangiopericytoma.
d. Leiomyosarcoma.
e. Synovial sarcoma.

844.

Cellular schwannomas are characterized by all of the following except:


(PIP 2003 C)
a. Lack of geographic necrosis.
b. Metastatic potential.
c. Predominance of Antoni A tissue.
d. Strong S-100 protein immunoreactivity.

Reserved Copyrights - 2013

191

Pathology Questions

MCQs

e. Well-defined capsule.
845.

Features of hemangiopericytoma include all of the following except: (PIP


2003 C)
a. CD34 positivity.
b. EMA positivity.
c. Pericellular reticulin.
d. Staghorn vascular pattern.

846.

For each of the following, select the most likely diagnosis from the
diagnostic set (an answer may be used once, more than once, or not at
all): (PIP 2003 C)
Diagnostic set:
a. Adrenal cortical carcinoma.
b. Angiolipoma.
c. Extramedullary hematopoietic tumor associated with essential
thrombocythemia.
d. Extramedullary plasmacytoma.
e. Liposarcoma.
f. Myeloid sarcoma (granulocytic sarcoma).
g. Myelolipoma.
Which lesion is typically seen in a subcutaneous location and often
associated with pain?
b. Angiolipoma.
Which neoplasm is most commonly seen in the upper respiratory tract?
d. Extramedullary plasmacytoma.
Which

benign

tumor

commonly

presents

as

an

incidental

adrenal/periadrenal mass?
g. Myelolipoma.

Reserved Copyrights - 2013

192

Pathology Questions
847.

MCQs

For each of the following, select the most likely diagnosis from the
diagnostic set (an answer may be used once, more than once, or not at
all): (PIP 2003 C)
Diagnostic set:
a. Inflammatory pseudotumor.
b. Invasive urothelial carcinoma with pseudosarcomatous stromal
reaction.
c. Leiomyosarcoma.
d. Postoperative spindle cell nodule.
e. Sarcomatoid urothelial carcinoma.
An exophytic lesion characterized by compact, haphazard fascicles of
mitotically active, cytologically bland spindle cells that are cytokeratin,
vimentin, muscle specific actin and desmin(+), but EMA (-):
d. Postoperative spindle cell nodule.
A polypoid,myxoid tumor composed of mitotically active, atypical spindle
cells that are cytokeratin(+),EMA(+) and vimentin(+):
e. Sarcomatoid urothelial carcinoma.
A myxoid mural mass with granulation tissue-type vascularity, a
polymorphous chronic inflammatory infiltrate, and widely spaced,
cytologically bland spindle cells that are muscle specific actin(+),
vimentin(+), and cytokeratin(-):
a. Inflammatory pseudotumor.

848.

Biphasic malignant melanoma may have all the below characteristics


except: (PIP 2003 C)
a. S-100 positivity.
b. HMB-45 positivity.
c. Intracytoplasmic mucin
d. Vimentin positivity.
e. Intracytoplasmic brown granules.

Reserved Copyrights - 2013

193

Pathology Questions

849.

MCQs

For the following list the best single criterion for separating biphasic
pulmonary blastoma from biphasic synovial sarcoma is: (PIP 2003 C)
a. Spindle cell keratin positivity.
b. Spindle cell vimentin positivity.
c. Epithelial/epithelioid cell keratin positivity.
d. Epithelial/epithelioid cell vimentin positivity.
e. Supranuclear and subnuclear vacuoles in the epithelial component.

850.

Which of the following is true regarding biphasic pulmonary blastoma?


(PIP 2003 C)
a. Tumor cells grow along preexisting alveoli without destruction of
the underlying architecture.
b. Found almost exclusively in the first and second decade of life.
c. May have areas of chondrosarcoma, rhabdomyosarcoma or
osteosarcoma.
d. Not associated with a smoking history.
e. Tumors are typically very small at the time of presentation.

851.

For each of the following, select the most likely diagnosis from the
diagnostic set (an answer may be used once, more than once, or not at
all): (PIP 2004 A)
Diagnostic set:
a. Adenomyoepithelioma.
b. Fibroadenoma.
c. Fibromatosis.
d. Hamartoma.
e. Leiomyoma.
f. Myofibroblastoma.
g. Nodular fascitis.
A poorly circumscribed spindle cell lesion that infiltrates breast
paranchyma:

Reserved Copyrights - 2013

194

Pathology Questions

MCQs

c. Fibromatosis.
A well circumscribed breast lesion with S-100 positive spindle cells:
a. Adenomyoepithelioma.
A well circumscribed breast lesion that contains breast lobules and a
stromal component of fat, fibrous tissue or smooth muscle:
d. Hamartoma.
852.

Which tumor is most likely to demonstrate defective DNA mismatch


repair? (PIP 2004 A)
a. Colonic adenocarcinoma.
b. Breast carcinoma.
c. Carcinoid.
d. Small cell carcinoma.
e. Squamous carcinoma.

853.

Which tumor is a typical component of Heridetary Nonpolyposis


Colorectal Cancer (Lynch) Syndrome in women?
a. Endometrial carcinoma.
b. Transitional cell carcinoma of bladder.
c. Breast cancer.
d. Carcinoid.
e. Bronchogenic carcinoma.

854.

Which of the following typically occurs following vesical trauma?


a. Myxoid leiomyosarcoma
b. Postoperative spindle cell nodule
c. Pseudosarcomatous reactive myofibroblastic proliferation
d. Sarcomatoid urothelial carcinoma
e. Spindle cell sarcoma

Reserved Copyrights - 2013

195

Pathology Questions
855.

MCQs

Pseudosarcomatous reactive myofibroblastic proliferation may be


histologically indistinguishable from:
a. Postoperative spindle cell nodule
b. Pseudosarcomatous desmoplastic reaction to urothelial carcinoma
c. Both of the above
d. Neither of the above

856.

For each of the following, select the most likely diagnosis from the
diagnostic set (an answer may be used once, more than once, or not at
all): (PIP 2004 A)
Diagnostic set:
a. Chronic (nonspecific) lymphocytic thyroiditis.
b. Diffuse toxic goiter (Graves disease).
c. Diffuse large B-cell lymphoma.
d. Fibrous atrophy.
e. Hashimoto thyroiditis, fibrous variant.
f. Marginal zone B-cell lymphoma (Maltoma).
g. Multifocal fibrosin thyroiditis.
h. Nodular goiter.
i.

Radiation thyroiditis.

j.

Riedel thyroiditis.

k. Subacute thyroiditis (de Quervain thyroiditis).


A thyroid lesion thought to have a viral causation:
k. Subacute thyroiditis (de Quervain thyroiditis).
A fibrous lesion characterized by keloid-like bands that may extend into
cervical soft tissue:
j. Riedel thyroiditis.
A fibrous lesion purportedly associated with an unusual variant of
mucoepidermoid carcinoma:
e. Hashimoto thyroiditis, fibrous variant.
Reserved Copyrights - 2013

196

Pathology Questions

857.

MCQs

For each of the following, select the most likely diagnosis from the
diagnostic set (an answer may be used once, more than once, or not at
all): (PIP 2004 A)
Diagnostic set:
a. Aggressive angiomyxoma.
b. Angiomyofibroblastoma.
c. Cellular angiofibroma.
d. Fibroepithelial stromal polyp.
e. Superficial angiomyxoma.
Which tumor is well circumscribed, nonrecurring and characterized by
epithelioid stromal cells clustered around blood vessels?
b. Angiomyofibroblastoma.
Which myxoid neoplasm is a nonmetastasizing locally infiltrative tumor of
the genital, perineal, and pelvic regions with potential for local
destructive recurrence?
a. Aggressive angiomyxoma.
Which lesion is characterized by the finding of stellate and multinucleated
stromal cells near the epithelial-stromal interface?
d. Fibroepithelial stromal polyp.

858.

For each of the following, select the most likely diagnosis from the
diagnostic set (an answer may be used once, more than once, or not at
all): (PIP 2004 A)
Diagnostic set:
a. Follicular hyperplasia.
b. Follicular lymphoma.
c. Mantle cell lymphoma.
d. Marginal zone B-cell lymphoma.
e. Toxoplasmic lymphadenopathy.

Reserved Copyrights - 2013

197

Pathology Questions

MCQs

Which entity is characterized by follicular hyperplasia with an associated


proliferation of monocytoid B-cells and epithelioid histiocytes?
e. Toxoplasmic lymphadenopathy.
Which entity is often associated with the t(14;18) (q32;q21) cytogenetic
abnormality?
b. Follicular lymphoma.
Immunophenotyping of which entity typically exhibits suface light chain
clonality without expression of CD5 or CD10:
d. Marginal zone B-cell lymphoma.
859.

Spindle-cell lipoma is characterized by all of the following except: (PIP


2004 A)
a. High frequency of recurrence.
b. Male predominance.
c. Median age over 55 years.
d. Predominant location in back, neck and shoulders.
e. S-100 protein-negative spindle cells.

860.

Schwannoma is characterized by all of the following except: (PIP 2004 A)


a. Alternating Antoni A and Antoni B areas.
b. Encapsulating.
c. Frequent axons within the tumor.
d. S-100 positivity in tumor cells.
e. Verocay bodies.

861.

Low-grade fibromyxoid sarcoma most often: (PIP 2004 A)


a. Contains a large number of CD34(+) cells.
b. Has a high mitotic rate.
c. Has a mixture of myxoid and collagenous areas.
d. Occurs in the pelvis.
e. Occurs over 65 years of age.

Reserved Copyrights - 2013

198

Pathology Questions

862.

MCQs

For each of the following, select the most likely diagnosis from the
diagnostic set (an answer may be used once, more than once, or not at
all): (PIP 2004 A)
Diagnostic set:
a. Fibrosarcoma.
b. Hemangiopericytoma.
c. Leiomyosarcoma.
d. Malignant peripheral nerve sheath tumor.
e. Monophasic synovial sarcoma.
f. Sarcomatoid carcinoma.
Aggregates of ovoid tumor cells surrounding prominent endothelial-lined
sinusoidal blood vessels, keratin(-), CD34(+), phenotype:
b. Hemangiopericytoma.
Long sweeping fascicles of spindled cells with a herring-bone pattern and
conspicuous mitotic activity, vimentin(+) only phenotype:
a. Fibrosarcoma.
Short fascicles of plump, spindled cells, prominent mast cell infiltrate,
calcification, keratin(+), CD99(+), bcl2(+) phenotype:
e. Monophasic synovial sarcoma.

863.

For each of the following, select the most likely diagnosis from the
diagnostic set (an answer may be used once, more than once, or not at
all): (PIP 2004 A)
Diagnostic set:
a. Diffuse large B-cell lymphoma.
b. Embryonal carcinoma.
c. Metastaic carcinoma.
d. Seminoma.
e. Seminoma, spermatocytic type.

Reserved Copyrights - 2013

199

Pathology Questions

MCQs

f. Sertoli cell tumor.


g. Yolk sac tumor.
Which is the most comon testicular neoplasm in patients over 60 years of
age?
a. Diffuse large B-cell lymphoma.
Which is the most common pure testicular germ cell neoplasm?
d. Seminoma
Which germ cell tumor is most consistently immunohistochemically
positive for CD30?
b. Embryonal carcinoma.
864.

For each of the following, select the most likely diagnosis from the
diagnostic set (an answer may be used once, more than once, or notat all):
(PIP 2004 A)
Diagnostic set:
a. Chondroblastic osteosarcoma.
b. Chondroblastoma.
c. Chondroid chondroma.
d. Chondrosarcoma.
e. Enchondroma.
f. Osteochondroma.
Which tumor occurs almost exclusively in the spheno-occipital region?
c. Chondroid chondroma.
Which tumor is characterized by a cartilage-capped bony prohection
arising on the external surface of bone, with a marrow cavity continuous
with that of the underlying bone?
f. Osteochondroma.

Reserved Copyrights - 2013

200

Pathology Questions

MCQs

Which tumor usually arises in the epiphysis of young patients and


frequently shows a fine network of pericellular calcification (chicken
wire calcification)?
b. Chondroblastoma.
865.

For each of the following, select the most likely diagnosis from the
diagnostic set (an answer may be used once, more than once, or not at
all): (PIP 2004 B)
Diagnostic set:
a. Abscess resulting from oppotunistic infection.
b. Glioblastoma.
c. Meningeoma.
d. Metastatic poorly differentiated carcinoma.
e. Myeloid sarcoma (chloroma).
f. Oligodendroglioma.
g. Primary CNS lymphoma.
Which is the most likely neoplasm when a patient with AML develops a
(non-hemorrhagic) mass lesion in the brain?
e. Myeloid sarcoma (chloroma)
Specific immunohistochemical conformation of which neoplasm is not
currently feasible?
f. Oligodendroglioma.
Which brain neoplasm is most commonly encountered in patients with
HIV-1 infection?
g. Primary CNS lymphoma.

866.

For each of the following, select the most likely diagnosis from the
diagnostic set (an answer may be used once, more than once, or not at
all): (PIP 2004 B)
Diagnostic set:
a. Cystic nephroma.

Reserved Copyrights - 2013

201

Pathology Questions

MCQs

b. Cystic partially differentiated nephroblastoma.


c. Mesoblastic nephroma.
d. Metanephric adenofibroma.
e. Mixed epithelial and stromal tumor.
f. Sarcomatoid renal cell carcinoma.
Which tumor lacks solid, expansile nodules and is composed entirely of
multiloculated cysts lined by hobnail cells and thin-walled septa
containing fibrous connective tissue?
a. Cystic nephroma.
Which tumor has solid areas as well as clustering of cysts into
multilocular mass and contains estrogen receptor positive spindle cells?
e. Mixed epithelial and stromal tumor.
Which tumor contains vascularized nests of clear cells and an
atypicalmitotically active spindle cell components?
f. Sarcomatoid renal cell carcinoma.
867.

Which is the most common sex cord-stromal tumor of the ovary? (PIP
2004 B)
a. Adult granulosa cell tumor.
b. Fibroma.
c. Leiomyoma.
d. Sclerosing stromal tumor.
e. Thecoma.

868.

Which ovarian sex cord-stromal tumor is most likely to metastasize? (PIP


2004 B)
a. Adult granulosa cell tumor.
b. Juvenile granulosa cell tumor.
c. Sclerosing stromal tumor.
d. Sertoli-leydig cell tumor.

Reserved Copyrights - 2013

202

Pathology Questions

MCQs

e. Thecoma.
869.

Which primary ovarian neoplasm occurs in young adult women? (PIP


2004 B)
a. Thecoma.
b. Sclerosing stromal tumor.
c. Fibroma.
d. Fibromatosis.
e. Krukenberg tumor.

870.

For each of the following, select the most likely diagnosis from the
diagnostic set (an answer may be used once, more than once, or not at
all): (PIP 2004 B)
Diagnostic set:
a. Caroli disease.
b. Ciliated foregut cyst.
c. Hepatobiliary cystadenoma.
d. Metastatic mucinous cystadenocarcinoma.
e. Solitary bile duct cyst.
Which of these lesions is benign, demonstrates ovarian-like stroma and
occurs almost exclusively in females?
c. Hepatobiliary cystadenoma.
Which of these lesions is thought to arise from the embryonic foregut and
shows differntiation towards brinchial structures?
b. Ciliated foregut cyst.
Which of these lesions is commonly associated with inflmmation and
biliary calculi?
a. Caroli disease.

Reserved Copyrights - 2013

203

Pathology Questions
871.

MCQs

For each of the following, select the most likely diagnosis from the
diagnostic set (an answer may be used once, more than once, or not at
all): (PIP 2004 B)
Diagnostic set:
a. Dysgerminoma.
b. Fibrothecoma.
c. Juvenile granulosa cell tumor.
d. Mature cystic teratoma (dermoid cyst).
e. Sclerosing stromal tumor.
f. Yolk sac tumor (endodermal sinus tumor).
Which entity occuring in older women is characterized by predominantly
solid, white to yellow firm mass with dense fibrous bands and occasional
calcification?
b. Fibrothecoma.
Which entity typically exhibits a solid and cystic cut surface, eosiniphilic,
intracellular and extracellular hyaline bodies, Schiller-Duval bodies, and
alpha fetoprotein (AFP) expression?
f. Yolk sac tumor (endodermal sinus tumor)
Which entity is typically solid and cystic with variably-sized follicles lined
by cells with non-grooved, euchromatic nuclei and eosiniphilic
cystoplasm, may have luminal eosiniphilic, mucicarminophilic fluid within
follicles, and is inhibin positive and negative for alpha fetoprotein and
human chorionic gonadotropin?
c. Juvenile granulosa cell tumor.

872.

Which splenic lesion presents grossly as a solitary or multiple nodules,


frequently has PAS positive cytoplasmic globules and is commonly
positive for CD68 and endothelial markers?
a. Hodgkin disease.
b. Littoral cell angioma.

Reserved Copyrights - 2013

204

Pathology Questions

MCQs

c. Granulomatous splenitis.
d. Systemic mastocytosis.
873.

An association with multicentric form of Castleman disease has been


described in all of the following except:
a. Follicular dendritic cell sarcoma.
b. Primary effusion lymphoma.
c. Epstein-Barr virus.
d. Poems

syndrome

(polyneuropathy,

organomegaly,

endocrinopathy, monoclonal gammopathy and skin changes).


874.

Which of the following is true regarding systemic mastocytosis?


a. Skin lesions are common in the aggressive forms of the disease.
b. Indolent systemic mastocytosis usually does not have bone
marrow involvement.
c. A normal serum tryptase level in a pediatric patient with urticaria
pigmentosa essentially rules out the diagnosis of systemic
mastocytosis.
d. Malabsorption in patients with systemic mastocytosis is usually
attributed to mediator release from the mast cells.
e. There is no corelation between the percentage of mast cells inbone
marrow and survival.

875.

For each of the following, select the most likely diagnosis from the
diagnostic set (an answer may be used once, more than once, or not at
all): (PIP 2004 B)
Diagnostic set:
a. Bile duct adenoma.
b. Fibrolamellar hepatocellular carcinoma.
c. Focal nodular hyperplasia.
d. Hepatocellular adenoma.
e. Hepatocellular carcinoma, well differentiated.

Reserved Copyrights - 2013

205

Pathology Questions
876.

MCQs

In which tumor are pale bodies frequently identified within the cytoplasm
of the hepatocytes?
b. Fibrolamellar hepatocellular carcinoma.

877.

Which lesion is most likely to be associated with cavernous hemangiomas


of the liver?
c. Focal nodular hyperplasia.

878.

In which lesion does a solitary central artery with high flow and absent
portal vein account for its characteristic radiologic and pathologic
appearance?
c. Focal nodular hyperplasia.

879.

For each of the following, select the most likely diagnosis from the
diagnostic set( an answer may be used once, more than once, or not at
all): (PIP 2004 B)
Diagnostic set:
a. Adenomyoepithelioma.
b. Angiosarcoma.
c. Fibroadenoma.
d. Fibromatosis.
e. Hamartoma.
f. Metaplastic carcinoma.
g. Phyllodes tumor.
Which fibroepithelial tumor has a small but a definite potential to
metastasize?
g. Phyllodes tumor.
Which entity is a malformation, not a neoplasm?
e. Hamartoma.

Reserved Copyrights - 2013

206

Pathology Questions

MCQs

The spindle cell component of this biphasic tumor commonly expresses


one or more cytokeratins, a useful diagnostic feature?
f. Metaplastic carcinoma.
880.

For each of the following, select the most likely diagnosis from the
diagnostic set (an answer may be used once, more than once, or not at
all): (PIP 2004 B)
Diagnostic set:
a. Fibrosarcoma.
b. Hemangiopericytoma.
c. Leiomyosarcoma.
d. Malignant peripheral nerve sheathh tumor.
e. Solitary fibrous tumor.
f. Spindle cell squamous carcinoma.
g. Synovial sarcoma, monophasic.
Which tumor is usually characterized by the presence of uniform spindleshaped cells, absent mitotic activity, prominent stromal hyalinization
diffuse CD34 immunoreactivity and absence of epithelial markers or S100 protein?
e. Solitary fibrous tumor.
Which tumor is usually characterized by the presence of uniform cells
with low mitotic rate, inconspicuous to dilated and ramifying vascular
spaces, cytokeratin or epithelial membrane antigen immunoreactivity,
demonstration of (X;18) and CD34 negativity?
g. Synovial sarcoma, monophasic.
Which tumor is usually characterized by fascicular growth with fascicles
intersecting at right angles, blunt ended nuclei often with paranuclear
vacuoles, immunoreactivity for smooth muscle actin or desmin and
absence of CD34, bcl-2 and epithelial markers?
c. Leiomyosarcoma.

Reserved Copyrights - 2013

207

Pathology Questions
881.

MCQs

For each of the following, select the most likely diagnosis from the
diagnostic set (an answer may be used once, more than once, or not at
all): (PIP 2004 D)
Diagnostic set:
a. Brain (cerebral) abscess.
b. Herpes simplex encephalitis.
c. Meningitis, suppurative/purulent.
d. Primary angitis of the CNS.
e. Primary CNS (B-cell) lymphoma.
f. Subacute encephalitis of AIDS/HIV-1 infection.
g. West nile virus encephalitis.
Which disorder is characterized by the perivascular accumulation of
multinucleated macrophages in the brain?
f. Subacute encephalitis of AIDS/HIV-1 infection.
Which disorder may cause myelitis with associated flaccid paraparesis?
g. West nile virus encephalitis.
Which disorder is characterized by angiocentric inflammation with
fibrinoid necrosis of vessel walls and thrombi?
d. Primary angitis of the CNS.

882.

For each of the following, select the most likely diagnosis from the
diagnostic set (an answer may be used once, more than once, or not at
all): (PIP 2004 D)
Diagnostic set:
a. Chromophobe renal cell carcinoma.
b. Conventional (clear cell) renal cell carcinoma.
c. Papillary renal cell carcinoma.
d. Renal cell carcinoma, unclassified.
e. Renal oncocytoma.
f. Xp 11 translocation renal cell carcinoma.

Reserved Copyrights - 2013

208

Pathology Questions

MCQs

Which entity has trisomies 7, 17 and 16, as well as loss of the Y


chromosome?
c. Papillary renal cell carcinoma.
Deletions of the VHL gene on chromosome 3p25 are seen in which
entity?
b. Conventional (clear cell) renal cell carcinoma.
Loss of chromosomes 1, 2, 10, 13, 6, 21 and 17 are characteristic of which
entity?
a. Chromophobe renal cell carcinoma.
883.

Which is the most important factor predicting survival in patients with


mucinous borderline tumor of the ovary? (PIP 2004 D)
a. Tumor size.
b. Tumor stage.
c. Architectural complexity.
d. Nuclear grade.
e. Patient age.

884.

Which one of the following statements about primary mucinous tumors in


the ovary is true? (PIP 2004 D)
a. At the time of diagnosis, they are bilateral in 30-40% of cases.
b. Malignant mucinous tumors are more common than borderline
mucinous tumors.
c. Mucinous tumors of the ovary are the most common cause of
pseudomyxoma peritonei.
d. In the ovary, primary mucinous carcinoma is less common than
metastatic mucinous carcinoma
e. Most borderline mucinous tumors are composed of endocervicallike cells.

Reserved Copyrights - 2013

209

Pathology Questions
885.

MCQs

In pathologic staging of borderline and malignant ovarian epithelial


tumors by the TNM/FIGO classification, which of the following is true?
(PIP 2004 D)
a. pT1 tumors involve only one ovary.
b. The presence of malignant cells in ascites or peritonial washings
means that a tumor is at least pT2.
c. The presence of pelvic extension or implants, without more distant
disease, places a tumor in the pT2 category.
d. The presence of plural effusion indicates that the tumor is pM1.
e. Regional lymph node metastasis (pN1) is required to classify a
tumor as Stage III.

886.

For each of the following, select the most likely diagnosis from the
diagnostic set (an answer may be used once, more than once, or not at
all): (PIP 2004 D)
Diagnostic set:
a. Acute cellular rejection.
b. Acute cyclosporine toxicity.
c. Acute renal vein thrombosis.
d. Chronic transplant glomerulopathy.
e. Polyoma (BK virus) nephropathy.
f. Post-transplant lymphoproliferative disorder.
Which entity is characterized by interstitial nephritis, often patchy, with
tubular epithelial cell nuclear inclusions?
e. Polyoma (BK virus) nephropathy.
Which entity can have interstitial nephritis with tubilitis, glomerular
endothelial injury and/or vascular endarteritis or fibrinoid necrosis?
a. Acute cellular rejection.
Which entity has an expansile atypical lymphoplasmacytic proliferation
and is associated with EBV infection?

Reserved Copyrights - 2013

210

Pathology Questions

MCQs

f. Post-transplant lymphoproliferative disorder.


887.

For each of the following, select the most likely diagnosis from the
diagnostic set (an answer may be used once, more than once, or not at
all): (PIP 2004 D)
Diagnostic set:
a. Dysgerminoma.
b. Fibroma.
c. Goblet cell carcinoid tumor.
d. Juvenile granulosa cell tumor.
e. Metastatic gastric carcinoma.
f. Sclerosing stromal tumor.
g. Yolk sac tumor (endodermal sinus tumor).
Which entity occuring in the middle-aged women may be associated with
ascites and pleural effusion (Meig's syndrome)?
b. fibroma
Which typically occurs in children, teens and young adults, is commonly
composed of sheets of cells having rounded, hyperchromatic nuclei,
occasional variably-sized follicles containing eosiniphilic or basophilic
material, numerous luteinized cells, is inhibin(+), alpha-fetoprotein(-),
and human chorionic gonadotropin(-)?
d. Juvenile granulosa cell tumor
Which entity is typically yellow, solid and lobulated, composed of uniform
uninucleate tumor cells with a central, large, round nucleus, containing
one or more prominent nucleoli, brisk mitotic activity, occasionally has
granulomatous inflammation and, rarely, syncytiotrophoblastic cells?
a. Dysgerminoma.

888.

A CK7(-), CK20(+) keratin immunoprofile in a gland-forming cancer


involving the liver favors which of the following diagnoses? (PIP 2004 D)

Reserved Copyrights - 2013

211

Pathology Questions

MCQs

a. Cholangiocarcinoma.
b. Colorectal adenocarcinoma, metastastic.
c. Hepatobiliary cystadenoma.
d. Pancreatic adenocarcinoma, metastatic.
e. Gallbladder adenocarcinoma.
889.

Which of the following is considered to be a risk factor for gallbladder


carcinoma? (PIP 2004 D)
a. Hepatitis C.
b. Hepatitis B.
c. Opisthorchis viverrini infestation.
d. Clonorchis sinesis infestation.
e. Salmonella typhi carrier state.

890.

Which of the following conditions is most strongly associated with


gallbladder adenocarcinoma? (PIP 2004 D)
a. Sclerosing cholangitis.
b. Ulcerative colitis.
c. Porcelain gallbladder.
d. Diabetes mellitus.
e. Aquired immundeficiency syndrome (AIDS).

891.

For each of the following, select the most likely diagnosis from the
diagnostic set (an answer may be used once, more than once, or not at
all): (PIP 2004 D)
Diagnostic set:
a. Differentiating neuroblastoma.
b. Ewing sarcoma-PNET.
c. Lymphoblastic lymphoma.
d. Pleuropulmonary blastoma.
e. Rhabdomyosracoma.
f. Undifferentiated neuroblastoma.

Reserved Copyrights - 2013

212

Pathology Questions

MCQs

Whcih tumor is characterized by mixed blastomatous and sarcomatous


features?
d. Pleuropulmonary blastoma.
Which tumor is likely to express CD99 and demonstrate an EWS type 1
fusion transcription?
b. Ewing sarcoma-PNET.
Which tumor strongly expresses CD99 and TdT?
c. Lymphoblastic lymphoma.
For each of the following, select the most likely diagnosis from the
diagnostic set (an answer may be used once, more than once, or not at
all): (PIP 2004 D)
Diagnostic set:
g. Chondrosarcoma.
h. Fibromatosis.
i.

Inflammatory breast cancer.

j.

Malignant melanoma.

k. Metapalstic carcinoma.
l.

Non-Hodgkin lymphoma.

m. Phyllodes tumor.
A costellation of rapidly evolving clinical manifestations to include
reddened indurated skin and swelling is the sine qua non of which
mammary neoplasm?
c. Inflammatory breast cancer.
The spindle cell component of which tumor commonly expresses one or
more cytokeratins, a useful diagnostic feature?
e. Metapalstic carcinoma.
Which neoplasm frequently has the pathologic finding of dermal
lymphatic tumor emboli?
Reserved Copyrights - 2013

213

Pathology Questions

MCQs

c. Inflammatory breast cancer.


892.

For each of the following, select the most likely diagnosis from the
diagnostic set (an answer may be used once, more than once, or not at
all): (PIP 2004 D)
Diagnostic set:
a. Adenovirus pneumonia.
b. Cytomegalovirus pneumonia.
c. Hard metal lung disease.
d. Herpes simplex pneumonia.
e. Respiratory syncytial virus pneumonia.
Which entity is characterized by enlarged cells with eosiniphilic
intracytoplasmic inclusions, which may be arranged in a paranuclear
distribution and no intranuclear inclusions?
e. Respiratory syncytial virus pneumonia.
Which entity is characterized by cells without significant enlargement
containing basophilic nuclear inclusions which obscure the nuclear
membrane (smudge cells) and no cytoplasmic inclusions?
a. Adenovirus pneumonia.
Which entity is characterized by enlarged, frequently multinucleated cells
with eosiniphilic nuclear inclusions and no cytoplasmic inclusions?
d. Herpes simplex pneumonia.

893.

For each of the following questions choose the most likely diagnosis from
the following diagnostic set (an answer may be used once, more than
once, or not at all). (PIP 2005 A)
Diagnostic set:
a. Adenomatoid nodule.
b. Dysmorphogenetic goiter.
c. Follicular adenoma.

Reserved Copyrights - 2013

214

Pathology Questions

MCQs

d. Follicular carcinoma.
e. Graves disease (diffuse toxic goiter).
f. Lymphocytic thyroiditis.
g. Papillary carcinoma.
Which entity is histologically characterized by diffuse follicular
hyperplasia with prominent papillary architecture, absence of residual
normal thyroid parenchyma, follicular epithelial cells with round and
uniform nuclei, minimal to absent colloid production and the absence of
an associated lymphocytic cell infiltrate?
e. Graves disease (diffuse toxic goiter).
Which entity is histologically characterized by the presence of a discrete
mass showing prominent papillary architecture and a constellation of
cytologic alterations that include enlaged nuclei with irregularities in size
and shapes, disperesed to ground glass appearing nuclear chromatin,
nuclear crowding, nuclear grooves and intranuclear inclusions?
g. Papillary carcinoma.
Which entity is histologically characterized by the presence of an
encapsulated mass with capsular invasion and angioinvasion, and cells
with hyperchromatic round and regular nuclei?
d. Follicular carcinoma.
894.

For each of the following questions choose the most likely diagnosis from
the following diagnostic set (an answer may be used once, more than
once, or not at all). (PIP 2005 A)
Diagnostic set:
a. Choriocarcinoma.
b. Dysgerminoma.
c. Endometrioid carcinoma, secretory pattern.
d. Juvenile granullosa cell tumor.
e. Serous tumor of borderline malignancy (low malignant potential).

Reserved Copyrights - 2013

215

Pathology Questions

MCQs

f. Yolk sac tumor (endodermal sinus tumor).


Which entity typically occurs in middle-aged women, comprises
approximately one-third of ovarian neoplasms, and is characterized by
papillary fronds covered by stratified epithelial cells with scant
cytoplasm, mild to moderate nuclear atypia, inconspicuous cytoplasmic
mucin or mucin localized only to the lips of the cells, and abundant
psammoma bodies?
e. Serous tumor of borderline malignancy (low malignant
potential).
Which entity typically occurs in combination with an ovarian germ cell
tumor, exhibits a hemorrhagic cut surface, is biphasic, and has the
following immunoprofile: cytokeratin(+) and hCG(+)?
a. Choriocarcinoma.
Which entity accounts for approximately 50% of malignant ovarian germ
cell tumors arising in young women?
b. Dysgerminoma.
895.

Which lesion is encapsulated, usually cystic, and contains papillae


oriented toward the center of the lesion and includes follicles in some of
the papillae? (PIP 2005 A)
a. Adenomatous nodule/nodular hyperplasia.
b. Follicular nodule with papillary hyperplasia.
c. Follicular carcinoma, minimally invasive.
d. Papillary carcinoma, follicular variant.
e. Papillary carcinoma, (NOS, usual type.)

896.

Which of the following is not true of papillary carcinoma?


a. Cervical lymph node metastatsis is present in less that 5% at
diagnosis.
b. Relative 10-year survival is 95% or more.

Reserved Copyrights - 2013

216

Pathology Questions

MCQs

c. Most common thyroid cancer.


d. Associated with familial colon polyposis syndromes.
897.

Which of the following is required for the diagnosis of papillary?


a. Psammoma bodies.
b. Sclerotic stroma in at least part of the tumor.
c. Papillae architecture.
d. Typical nuclear features.

898.

For each of the following questions choose the most likely diagnosis from
the following diagnostic set (an answer may be used once, more than
once, or not at all). (PIP 2005 A)
Diagnostic set:
a. Adenocarcinoma, metastatic.
b. Carcinoid, metastatic.
c. Granulosa cell tumor.
d. Sertoli-Leydig cell tumor.
e. Strumal carcinoid.
f. Teratoma, mature.
g. Thyroid follicular carcinoma, metastatic.
Which tumor is characterized by a microfollicular pattern and Call-Exner
bodies?
c. Granulosa cell tumor.
Which tumor is composed predominantly of mature gastrointestinal
epithelium, glial tissue and respiratory epithelium with a microscopic
focus of thyroid tissue?
f. Teratoma, mature.
Which tumor is characterized by a predominantly follicular pattern
containing intraluminal eosinophilic material, a secondary intermixed
trabecular

and

insular

component,

and

immunoreactivity

for

thyroglobulin and synaptophysin?


Reserved Copyrights - 2013

217

Pathology Questions

MCQs

e. Strumal carcinoid.
899.

For each of the following questions choose the most likely diagnosis from
the following diagnostic set (an answer may be used once, more than
once, or not at all). (PIP 2005 A)
Diagnostic set:
a. Angiomyolipoma.
b. Inflammatory myofibroblastic tumor.
c. Leiomyoma.
d. Leiomyosarcoma.
e. Sarcomatoid renal cell carcinoma.
f. Sarcomatoid urothelial carcinoma.
g. Solitary fibrous tumor.
In which tumor can ALK-1 expression be of potential diagnostic value?
b. Inflammatory myofibroblastic tumor.
Which malignant-appearing spindle cell neoplasm has histologic and
immunophenotypic smooth muscle feature is negative for cytokeratin and
HMB-45?
d. Leiomyosarcoma.
For which tumor is the presence in regional lymph nodes, renal vein/vena
cava extension or involvement of perirenal tissue not indicative of
malignant progression?
a. Angiomyolipoma.

900.

Which of the following molecular events has been most specifically


associated with decreased cellular adhesion and diffuse infiltration,
characteristic of signet-ring carcinoma? (PIP 2005 A)
a. hMLH1 inactivation.
b. K-ras mutation.
c. E-cadherin inactivation.

Reserved Copyrights - 2013

218

Pathology Questions

MCQs

d. Microsatellite instability.
e. APC mutation.
901.

Which primary site is favored by the following immunohistochemical


profile exhibited by a metastatic signet ring carcinoma: CDX2(+), MUC1(-)
MUC2(+), MUC5AC(+), CK7(-), CK20(+), TTF(-), PSA(-)?(PIP 2005 A)
a. Lung.
b. Breast.
c. Prostate.
d. Colon.
e. Stomach.

902.

Which of the following statements is true of colorectal signet-ring


carcinoma? (PIP 2005 A)
a. The most common histologic variant of carcinoma involving the
cecum.
b. Occurs in older age group than other histologic variants of
colorectal carcinoma.
c. In advanced disease is more often associated with peritoneal
dissemination than with liver metastasis.
d. Has a better prognosis, adjusted for stage, than non-mucinous
carcinoma of the colorectum.
e. Is typically associated with a diffuse infiltrating scirrhous
appearance.

903.

For each of the following questions choose the most likely diagnosis from
the following diagnostic set (an answer may be used once, more than
once, or not at all). (PIP 2005 A)
Diagnostic set:
a. Chromophobe renal cell carcinoma.
b. Collecting duct carcinoma.
c. Conventional (clear cell) renal cell carcinoma.
d. Papillary renal cell carcinoma.

Reserved Copyrights - 2013

219

Pathology Questions

MCQs

e. Sarcomatoid carcinoma arising in renal cell carcinoma.


f. Urothelial (transitional cell) carcinoma.
g. Xanthogranulomatous pyelonephritis.
Which of the above is composed of neoplastic cells with granular
cytoplasm, papillary and tubular architecture, and may exhibit trisomies
of chromosome 3q, 7 or 17?
d. Papillary renal cell carcinoma.
Which of the above represents less than 2% of kidney malignancies and
shows a distinctive desmoplastic stromal reaction?
b. Collecting duct carcinoma.
Which of the above shows tumor cell differentiation toward interlaced
duct phenotype, and ultrastructurally contains numerous cytoplasmic
microvesicles?
a. Chromophobe renal cell carcinoma.
904.

For each of the following questions choose the most likely diagnosis from
the following diagnostic set (an answer may be used once, more than
once, or not at all). (PIP 2005 A)
Diagnostic set:
a. Adenocarcinoma, goblet cell type.
b. Adenocarcinoma, signet ring type.
c. Bronchioloalveolar carcinoma, mucinous type.
d. Mucinous (colloid) carcinoma.
e. Mucinous cystadenoma.
Which tumor is composed of cytologically bland, tall columnar cells with
apical mucin growing in a purely lepidic pattern without areas of stromal,
pleural or vascular invasion?
c. Bronchioloalveolar carcinoma, mucinous type.

Reserved Copyrights - 2013

220

Pathology Questions

MCQs

Which tumor is a recently described variant of mucinous pulmonary


adenocarcinoma that shows positive staining with CK20 and CDX2 in the
majority of cases?
a. Adenocarcinoma, goblet cell type.
Which tumor is characterized by a fibrous-walled cyst lined by
cytologically bland mucinous epithelium?
e. Mucinous cystadenoma.
905.

For each of the following questions choose the most likely diagnosis from
the following diagnostic set (an answer may be used once, more than
once, or not at all). (PIP 2005 A)
Diagnostic set:
a. Desmoplastic small round cell tumor.
b. Embryonal rhabdomyosarcoma.
c. Lymphoblastic lymphoma.
d. Neuroblastoma.
e. Peripheral primitive neuroectodermal tumor/Ewing sarcoma.
Which

tumor

is

characterized

by

unique

phenotypic

immunohistochemical profile?
a. Desmoplastic small round cell tumor.
Which neoplasm is typically and uniformly CD99 (MIC2 gene product)
negative?
d. Neuroblastoma.
Which is the most common malignant soft tissue tumor in the first two
decades of life?
b. Embryonal rhabdomyosarcoma.
906.

For each of the following questions choose the most likely diagnosis from
the following diagnostic set (an answer may be used once, more than
once, or not at all). (PIP 2005 A)

Reserved Copyrights - 2013

221

Pathology Questions

MCQs

Diagnostic set:
a. Atopic dermatitis.
b. Bullous pemphigoid of childhood.
c. Erythema toxicum neonatorum.
d. Herpes gestationis.
e. Hypomelanosis of Ito.
f. Incontinentia pigmenti.
g. Inflammatory linear verrucous epidermal nevus.
h. Pemphigus vulgaris.
Which vesiculobullous disease characterized by intercellular deposition of
IgG and CD3?
h. Pemphigus vulgaris.
Which disease usually results in death in utero in males?
f. Incontinentia pigmenti.
Which entity is associated with CHILD syndrome?
g. Inflammatory linear verrucous epidermal nevus.
907.

Strong diffuse immunostaining with which one of the following antibodies


is most useful in separating choroid plexus carcinoma from atypical
teratoid/rhabddoid tumor? (PIP 2005 B)
a. Cytokeratin.
b. Epithelial membrane antigen.
c. Glial fibrillary acidic protein.
d. Synaptophysin.
e. Vimentin.

908.

The BAF-47 antibody recognizes the protein encoded by which one of the
following genes? (PIP 2005 B)
a. INI 1
b. NF 1

Reserved Copyrights - 2013

222

Pathology Questions

MCQs

c. NF 2
d. P53
e. PTEN
909.

CNS atypical teratoid/rhabdoid tumors occur predominantly in which of


the following locations? (PIP 2005 B)
a. Cerebrum.
b. Lateral ventricle.
c. Third ventricle.
d. Posterior fossa.
e. Spinal cord.

910.

For each of the following questions choose the most likely diagnosis from
the following diagnostic set (an answer may be used once, more than
once, or not at all). (PIP 2005 B)
Diagnostic set:
a. Adenomatoid tumor.
b. Endometrial stromal tumor with sex-cord like elements.
c. Epithelioid leiomyoma.
d. Leiomyosarcoma.
e. Malignant mixed mullerian tumor.
f. Metastatic carcinoma.
g. Metastatic granulosa cell tumor.
h. Perivascular epithelioid cell tumor (PEComa).
Which tumor is usually strongly and uniformly immunopositive for
HMB45?
h. Perivascular epithelioid cell tumor (PEComa).
Which tumor most commonly occurs in postmenopausal women, has a
poor

prognosis

and

demonstrates

epithelial

and

mesenchymal

differentiation?
e. Malignant mixed mullerian tumor.
Reserved Copyrights - 2013

223

Pathology Questions

MCQs

Which uterine tumor is typically located subserosally, most frequently is


circumscribed, usually occurs in women of reproductive age and is
calretinin and thrombomodulin positive?
a. Adenomatoid tumor.
911.

For each of the following questions choose the most likely diagnosis from
the following diagnostic set (an answer may be used once, more than
once, or not at all). (PIP 2005 B)
Diagnostic set:
a. Anaplastic large cell tumor.
b. Burkitt lymphoma.
c. Carcinoid tumor.
d. Endometrial stromal sarcoma.
e. Follicular lymphoma.
f. Gastrointestinal stromal tumor (GIST).
g. Large cell neuroendocrine carcinoma.
h. Myeloid sarcoma (Extramedullary myeloid tumor).
i.

Precursor of B-cell acute lymphoblastic leukemia/lymphoma.

Which tumor typically involves distal ileum, occurs with equal frequency
in both sexes, with a peak in the sixth and seventh decades, and classically
has an insular pattern of growth?
c. Carcinoid tumor.
Which tumor characteristically has a translocation involving the
immunoglobulin heavy chain (IgH) and BCL-2 genes?
e. Follicular lymphoma.
Which tumor can present in children and adults and when it exhibits
t(9;22) translocation (philladelphia chromosome) portends a poor
prognosis?
i. Precursor of B-cell acute lymphoblastic leukemia/lymphoma.
Reserved Copyrights - 2013

224

Pathology Questions

912.

MCQs

For each of the following questions choose the most likely diagnosis from
the following diagnostic set (an answer may be used once, more than
once, or not at all). (PIP 2005 B)
Diagnostic set:
a. Adrenal cortical adenoma.
b. Adrenal medullary hyperplasia.
c. Adrenal pseudocyst.
d. Metastatic carcinoma.
e. Pheochromocytoma.
Which condition may be accompaned by signs of hypercoricalism?
a. Adrenal cortical adenoma.
Which neoplasm shows positive immunohistochemical positivity for
chromogranin and synaptophysin?
Which is the most common malignant neoplasm occuring in the adrenal
gland?
d. Metastatic carcinoma.

913.

For each of the following questions choose the most likely diagnosisfrom
the following diagnostic set (an answer may be used once, more than
once, or not at all). (PIP 2005 B)
Diagnostic set:
a. Angiomyolipoma.
b. Metastatic melanoma.
c. Sarcomatoid renal cell carcinoma.
d. Solitary fibrous tumor.
e. Well-differentiated liposarcoma.
Which tumor is characterized by perivascular epithelioid and spindle cells
immunoreactive for muscle specific actin and HMB-45?

Reserved Copyrights - 2013

225

Pathology Questions

MCQs

a. Angiomyolipoma.
Which tumor is composed of fat cells with hyperchromatic nuclei anD
spindle cells within broad fibrous bands, and is immunoreactive forS100
protein and negative for HMB-45?
e. Well-differentiated liposarcoma.
Which tumor characteristically contains pleomorphic epithelioid cells and
spindle cells immunoreactive for keratin and epithelial membrane
antigen?
c. Sarcomatoid renal cell carcinoma.
914.

For each of the following questions choose the most likely diagnosis from
the following diagnostic set (an answer may be used once, more than
once, or not at all). (PIP 2005 B)
Diagnostic set:
a. Embryonal undifferentiated hepatic sarcoma.
b. Hepatoblastoma.
c. Mesenchymal hamartoma.
d. Rhabdomyosarcoma.
e. Teratoma.
In which primary malignant hepatic tumor may mitoses and histology be
prognostic determinants?
b. Hepatoblastoma.
Which hepatic mass may be the cause of heart failure?
c. Mesenchymal hamartoma.
Which hepatic tumor arising in the biliary tree most typically causes
obstruction of bile?
d. Rhabdomyosarcoma.

Reserved Copyrights - 2013

226

Pathology Questions
915.

MCQs

Which of the following has the most striking female predisposition? (PIP
2005 B)
a. Collagenous colitis.
b. Lymphocytic colitis.
c. Ischemic colitis.
d. Ulcerative colitis.
e. Neutrophilic enterocolitis.

916.

Pseudomembrane formation may be evident in which of the following


diseases? (PIP 2005 B)
a. Ischemic colitis.
b. Clostredium difficile associated colitis.
c. Amebic colitis.
d. All of the above.

917.

For each of the following questions choose the most likely diagnosis from
the following diagnostic set (an answer may be used once, more than
once, or not at all). (PIP 2005 B)
Diagnostic set:
a. Angiolipoma.
b. Atypical lipomatous tumor.
c. Desmoid type fibromatosis.
d. Elastofibroma.
e. Low grade fibromyxoid sarcoma.
f. Myolipoma.
g. Neurofibroma.
h. Solitary fibrous tumor.
i.

Spindle cell lipoma.

Which benign tumor exhibits a marked predilection for the neck and
shoulders in men over 50 years of age?
i. Spindle cell lipoma.

Reserved Copyrights - 2013

227

Pathology Questions

MCQs

Which benign tumor exhibits a marked predilection for the soft tissue
between the scapula and the chest wall in elderly women?
d. Elastofibroma.
Which tumor is characterized by a mixture of heavily collagenized and
myxoid zones, bland spindle cells and arcades of blood vessels?
e. Low grade fibromyxoid sarcoma.
918.

For each of the following questions choose the most likely diagnosis from
the following diagnostic set (an answer may be used once, more than
once, or not at all). (PIP 2005 B)
Diagnostic set:
a. Anaplastic large cell lymphoma.
b. Classical Hodgkin lymohoma.
c. Diffuse large B-cell lymphoma.
d. Extramedullary hematopoiesis.
e. Hepatosplenic T-cell lymphoma.
Which entity is a CD45 negative B-cell malignancy?
b. Classical Hodgkin lymohoma.
Which tumor is immunohistochemically positive for CD30 and CD45, and
negative for CD20?
a. Anaplastic large cell lymphoma.
Which entity is most commonly associated with the t(2;5) translocation?
a. Anaplastic large cell lymphoma.

919.

For each of the following questions choose the most likely diagnosis from
the following diagnostic set (an answer may be used once, more than
once, or not at all). (PIP 2005 B)
Diagnostic set:
a. Castleman disease.

Reserved Copyrights - 2013

228

Pathology Questions

MCQs

b. Epithelioid hemangioma.
c. Hodgkin lymphoma.
d. Kimura disease.
e. Langerhans cell histiocytosis.
Which entity is characterized by a recurrent chronic inflammatory
process in Oriental patients with peripheral eosinophilia and raised seum
IgE?
d. Kimura disease.
Which lesion typically has proliferating blood vessels lined by endothelial
cells that have been described as histiocytoid in appearance?
b. Epithelioid hemangioma.
Which disorder harbors large cells that have elongated, twisted, grooved
nuclei and are CD1a(+) and S-100(+)?
e. Langerhans cell histiocytosis.
920.

For each of the following questions choose the most likely diagnosis from
the following diagnostic set (an answer may be used once, more than
once, or not at all). (PIP 2005 C)
Diagnostic set:
a. Papillary urothelial carcinoma, high grade.
b. Papillary urothelial carcinoma, low grade.
c. Papillary urothelial hyperplasia.
d. Papillary urothelial neoplasm of low malignant potential.
e. Urothelial carcinoma with inverted papilloma-like pattern.
f. Urothelial carcinoma with micropapillary component.
g. Urothelial papilloma.
Which papillary lesion can be diagnosed cytologically in urine specimens
by the presence of 3-dimensional clusters of dysplastic urothelial cells
with altered polarity?

Reserved Copyrights - 2013

229

Pathology Questions

MCQs

b. Papillary urothelial carcinoma, low grade.


Which lesion is not biologically aggressive but is associated with
recurrences and carries a risk for progression i.e. invasion?
d. Papillary urothelial neoplasm of low malignant potential.
Which neoplasm is characterized by slender delicate filiform process on
the mucosal surface and tight cell clusters in the subepithelial connective
tissue with prominent retraction artifact resembling angiolymphatic
invasion?
f. Urothelial carcinoma with micropapillary component.
921.

For each of the following questions choose the most likely diagnosis from
the following diagnostic set (an answer may be used once, more than
once, or not at all). (PIP 2005 C)
Diagnostic set:
a. Angiosarcoma.
b. Dendritic/reticulum cell tumor.
c. Hemangioma.
d. Inflammatory myofibroblastic tumor.
e. Littoral cell angioma.
f. Metastatic melanoma.
g. Mycobacterial spindle cell pseudotumor.
Which is the most common primary tumor of the spleen?
c. Hemangioma.
Which tumor is characterized by a CD21(+), CD31(+), CD68(+)
phenotype?
e. Littoral cell angioma.
Which tumor is characterized by a zonal histologic pattern?
d. Inflammatory myofibroblastic tumor.

Reserved Copyrights - 2013

230

Pathology Questions

922.

MCQs

For each of the following questions choose the most likely diagnosis from
the following diagnostic set (an answer may be used once, more than
once, or not at all). (PIP 2005 C)
Diagnostic set:
a. Bile duct adenoma.
b. Caroli disease.
c. Congenital cystic (polycystic) disease of the liver.
d. Congenital hepatic fibrosis.
e. Cystic fibrosis involving the liver.
f. Focal nodular hyperplasia.
g. Von Meyenburg complexes.
Which condition is most stongly associated with the development of
cholangiocarcinoma?
b. Caroli disease.
Which condition is most likely to present as bleeding from esophegeal
varices?
d. Congenital hepatic fibrosis.
Which condition may be associated with small scars that resemble
corpora albicantia?
c. Congenital cystic (polycystic) disease of the liver.

923.

For each of the following questions choose the most likely diagnosis from
the following diagnostic set (an answer may be used once, more than
once, or not at all)
Diagnostic set:
a. Deep benign histiocytoma.
b. Malignant fibrous histiocytoma.
c. Neurilemmoma.
d. Nodular fasciitis.

Reserved Copyrights - 2013

231

Pathology Questions

MCQs

e. Pleomorphic hyalinizing angiectatic tumor.


f. Psammomatous melanotic schwannoma.
g. Spindle cell lipoma.
Which tumor is frequently associated with Carney syndrome?
f. Psammomatous melanotic schwannoma.
Which tumor exhibits a local recurrence rate of up to 50%, lacks a high
mitotic rate, and shows perivascular hyalinization?
e. Pleomorphic hyalinizing angiectatic tumor.
Which tumor has a strong predilection for men?
g. Spindle cell lipoma.
924.

For each of the following questions choose the most likely diagnosis from
the following diagnostic set (an answer may be used once, more than
once, or not at all). (PIP 2005 C)
Diagnostic set:
a. Endometrioid carcinoma.
b. Metastatic carcinoma of the breast.
c. Papillary urothelial carcinoma, metastatic.
d. Serous carcinoma, high grade.
e. Serous carcinoma, low grade.
f. Transitional cell carcinoma of the ovary.
Which tumor is thought to arise from surface epithelial inclusions of the
ovary by mutations of the p53 tumor suppressor gene and functional
alterations in BRCA 1 and BRCA 2?
d. Serous carcinoma, high grade.
Which tumor is characterized by a predominantly tubuloglandular
architecture, strong expression of estrogen and progesterone receptors,

Reserved Copyrights - 2013

232

Pathology Questions

MCQs

and mutations of the PTEN tumor suppressor gene, with loss of PTEN
protein expression?
a. Endometrioid carcinoma.
Which tumor is thought to arise by progression from a benign neoplasm
by mutations in KRAS and BRAF?
e. Serous carcinoma, low grade.
925.

For each of the following questions choose the most likely diagnosis from
the following diagnostic set (an answer may be used once, more than
once, or not at all). (PIP 2005 C)
Diagnostic set:
a. Acinar cell carcinoma.
b. Ductal adenocarcinoma.
c. Intraductal papillary mucinous neoplasm.
d. Pancreatic endocrine tumor.
e. Pancreatoblastoma.
f. Solid-pseudopapillary tumor.
Which tumor is most likely to be negative for cytokeratin?
f. Solid pseudopapillary tumor.
Which tumor is considered primarily a tumor of childhood?
e. Pancreatoblastoma.
Which cystic tumor commonly extends along ductal structures within the
pancreas?
c. Intraductal papillary mucinous neoplasm.

926.

For each of the following questions choose the most likely diagnosis from
the following diagnostic set (an answer may be used once, more than
once, or not at all). (PIP 2005 C)
Diagnostic set:

Reserved Copyrights - 2013

233

Pathology Questions

MCQs

a. Chondroblastic osteosarcoma.
b. Chondrosarcoma.
c. Enchondroma.
d. Juxta-articular chondroma.
e. Osteocartilaginous loose bodies.
f. Pigmented villondular synovitis.
g. Rice bodies.
h. Synovial chondromatosis.
Which of the above may be polyarticular, and represents a secondary
metaplasia of synovium, often due to chronic osteoarthritis?
e. Osteocartilaginous loose bodies.
Which of the above is most often monoarticular, and represents a primary
de novo metaplasia or possible neoplasia of synovium?
h. Synovial chondromatosis.
Which intraarticular process listed above is composed of fibrin?
g. Rice bodies.
927.

For each of the following questions choose the most likely diagnosis from
the following diagnostic set (an answer may be used once, more than
once, or not at all). (PIP 2005 C)
Diagnostic set:
a. Intrathoracic desmoid tumor.
b. Malignant mesothelioma, sarcomatoid type.
c. Sarcomatoid carcinoma.
d. Solitary fibrous tumor.
e. Synovial sarcoma.
Which tumor may present either as a localized mass or diffuse pleural
thickening and is composed of spindled cells which show strong positive

Reserved Copyrights - 2013

234

Pathology Questions

MCQs

staining with bcl-2, variable/patchy staining for cytokeratin and


calretinin, and negative staining for CD34?
e. Synovial sarcoma.
Which tumor presents as a large pleural-based mass and is composed of
spindled cells in a collagenous background which show positive staining
for CD34 nad bcl-2 and lack staining with cytokeratin and calretinin?
d. Solitary fibrous tumor.
Which tumor presents as multiple pleural nodules or diffuse pleural
thickening and is composed of spindled cells which are diffusely and
strongly positive with cytokeratin, variably positive for calretinin, and
negative for bcl-2 and CD34?
b. Malignant mesothelioma, sarcomatoid type.
928.

Which diagnosis implies the most aggressive natural history: (PIP 2005 C)
a. Carcinoid.
b. Goblet cell carcinoid.
c. Mixed carcinoid-adenocarcinoma.
d. Tubular carcinoid.

929.

Which is most reliable feature for the light microscopic distinction


between tubular carcinoid and mixed endocrine-adenocarcinoma? (PIP
2005 C)
a. Chromogranin expression.
b. Cytologic, especially nuclear, atypia.
c. Cytoplasmic.
d. Desmoplastic response.

930.

Which anatomic location is associated with the most favorable prognosis


for carcinoid and related tumors? (PIP 2005 C)
a. Appendix.
b. Colon.
c. Small bowel.

Reserved Copyrights - 2013

235

Pathology Questions

MCQs

d. There is no difference.
931.

For each of the following questions choose the most likely diagnosis from
the following diagnostic set (an answer may be used once, more than
once, or not at all). (PIP 2005 C)
Diagnostic set:
a. Acute chorioamnionitis.
b. Acute villitis.
c. Chorangiosis.
d. Hemorrhagic endovasculitis.
e. Meconium staining.
f. Villitis of unkown etiology.
Which entity is characterized by chronic inflammation, including plasma
cells in the villi, and may recur in subsequent pregnancies?
f. Villitis of unkown etiology.
Cytomegalovirus, Rubella, Herpes simplex virus, Parvovirus B19,
Treponema pallidum, Listeria monocytogenes, and Escherichia coli may
be responsible for which entity?
b. Acute villitis.
Which entity is the most common form of villitis?
f. Villitis of unkown etiology.

932.

Patients with intravascular lymphoma present clinically with neurological


signs due to which of the following? (PIP 2005 D)
a. Hemorrhages.
b. Infarcts.
c. Intraparenchymal infiltration.
d. Secondary (opportunistic) infection.
e. Vasculitis.

Reserved Copyrights - 2013

236

Pathology Questions
933.

MCQs

Which of the following is the most important histologic finding in


intravascular lymphoma? (PIP 2005 D)
a. Distended vessels.
b. Fibrin and platelet thrombi.
c. Immunoreactivity of neoplastic cells to factor VIII.
d. Intravascular proliferation of large B-cells.
e. Perivascular inflammation.

934.

Which of the following is a characteristic feature of intravascular


lymphoma? (PIP 2005 D)
a. Association with HIV positivity.
b. Peak occurrence in childhood.
c. Predominantly cerebral and cutaneous involvement.
d. Presentation with mass effect and enhancement on imaging.
e. Relatively favorable prognosis.

935.

For each of the following questions choose the most likely diagnosis from
the following diagnostic set (an answer may be used once, more than
once, or not at all). (PIP 2005 D)
Diagnostic set:
a. Alveolar soft part sarcoma.
b. Gastrointestinal stromal tumor (GIST).
c. Hemangiopericytoma/Solitary fibrous tumor of soft tissue.
d. Leiomyosarcoma.
e. Perivascular epithelioid cell tumor (PEComa).
Which tumor is has the characteristic cytogenetic abnormality
der(17)t(X;17)?
a. Alveolar soft part sarcoma.
Which tumor is typically positive for CD34 and CD117 (c-kit)?
b. Gastrointestinal stromal tumor (GIST).

Reserved Copyrights - 2013

237

Pathology Questions

MCQs

Which tumor expresses smooth muscle actin and melanocytic markers


including HMB-45?
e. Perivascular epithelioid cell tumor (PEComa).
936.

For each of the following questions choose the most likely diagnosis from
the following diagnostic set (an answer may be used once, more than
once, or not at all). (PIP 2005 D)
Diagnostic set:
a. Carcinoma with thymic-like elements (CASTLE).
b. Germ cell tumor.
c. Hodgkin lymphoma.
d. Metastatic carcinoma.
e. Non-Hodgkin lymphoma.
f. Solitary fibrous tumor.
g. Thymic carcinoma.
h. Thymoma.
Which entity often arises from pleura and is richly collagenized?
f. Solitary fibrous tumor.
Which entity occurs in multiple microscopic variants and may be
associated with autoimmune disorders?
h. Thymoma.
Which lesion is usually anatomically associated with thyroid disease?
a. Carcinoma with thymic-like elements (CASTLE).

937.

For each of the following questions choose the most likely diagnosis from
the following diagnostic set (an answer may be used once more than once,
or not at all). (PIP 2005 D)
Diagnostic set:
a. Desmoplastic small round cell tumor (DSRCT).
b. Dysgerminoma.

Reserved Copyrights - 2013

238

Pathology Questions

MCQs

c. Juvenile granulosa cell tumor.


d. Malignant melanoma.
e. Non-Hodgkin lymphoma.
f. Ovarian small cell carcinoma, hypercalcemic type (OSCCHT).
g. Ovarian small cell carcinoma, pulmonary type.
Which neoplasm is typically OCT3/4 positive?
b. Dysgerminoma.
Which neoplasm occurs in teens and 20s and is immunopositive for
inhibin?
c. Juvenile granulosa cell tumor.
Which neoplasm is commonly bilateral, occurs in postmenopausal women
and may be associated with a paraneoplastic endocrine hormone
syndrome?
g. Ovarian small cell carcinoma, pulmonary type.
938.

For each of the following questions choose the most likely diagnosis from
the following diagnostic set (an answer may be used once, more than
once, or not at all). (PIP 2005 D)
Diagnostic set:
a. Bile duct adenoma.
b. Focal nodular hyperplasia.
c. Hepatic adenoma.
d. Hepatocellular carcinoma, fibrolammelar type.
e. Nodular regenerative hyperplasia.
f. Primary biliart cirrhosis.
Which lesion is a diffuse nodular transformation of the entire liver due to
heterogenous blood flow?
e. Nodular regenerative hyperplasia.

Reserved Copyrights - 2013

239

Pathology Questions

MCQs

Which lesion most commonly consists of a solitary tumor due to aberrant


blood flow?
b. Focal nodular hyperplasia.
Which hepatocellular neoplasm shows no gender bias?
d. Hepatocellular carcinoma, fibrolammelar type.
939.

For each of the following questions choose the most likely diagnosis from
the following diagnostic set (an answer may be used once, more than
once, or not at all). (PIP 2005 D)
Diagnostic set:
a. Diffuse large B-cell lymphoma.
b. Extranodular marginal zone B-cell lymphoma of mucosaassociated lymphoid tissue (MALT lymphoma).
c. Follicular lymphoma, grade 2.
d. Mantle cell lymphoma.
e. Small lymphocytic lymphoma.
Which typically CD5 negative lymphoma may have reactivemlymphoid
follicles, plasmacytic differentiation and lymphoepithelial lesions?
b. Extranodular marginal zone B-cell lymphoma of mucosaassociated lymphoid tissue (MALT lymphoma).
Which lymphoma typically demonstrates CD5 and cyclin-D1 positivity?
d. Mantle cell lymphoma.
Which lymphoma would be expected to express both CD10 and Bcl-6?
c. Follicular lymphoma, grade 2.

940.

Which of the following favors the diagnosis of meningioma over


meningeal hemangiopericytoma? (PIP 2005 D)
a. Young age at presentation.
b. Lytic bony involvement.

Reserved Copyrights - 2013

240

Pathology Questions

MCQs

c. Lack of nuclear pseudoinclusions.


d. Expression of CD34 and absence of epithelial membrane antigen
by immunohistochemistry.
e. Allelic losses on 22q in the region of NF2 gene.
941.

Which of the following favors a diagnosis of angiosarcoma over


epithelioid hemangioendothelioma of the liver? (PIP 2005 D)
a. Grossly appears as a hemorrhagic tumor with solid and cystic
areas.
b. Tumor calcification.
c. Central area of the tumor is often fibrotic and paucicellular.
d. Marked predilection for invading vascular structures such as
portal and central veins, often mimicking vascular thrombosis.
e. Immunohistochemical expression of endothelial markers like
CD31 and factor VIII.

942.

Which of the following is not a characteristic feature of solitary fibrous


tumor? (PIP 2005 D)
a. Pleural location.
b. Alternating hypercellular and paucicellular areas.
c. Bland spindle cells with broad zones of hyalinized collagen.
d. CD34 expression by immunohistochemistry.
e. Aggressive clinical behavior.

943.

For each of the following questions choose the most likely diagnosis from
the following diagnostic set (an answer may be used once, more than
once, or not at all). (PIP 2005 D)
Diagnostic set:
a. Hydrops placentalis.
b. Mesenchymal dysplasia.
c. Metabolic storage disorder.
d. Partial hydatidiform mole.

Reserved Copyrights - 2013

241

Pathology Questions

MCQs

Which entity is characterized by massive enlargement of the placenta,


diffuse villous edema and fetal hydrops?
a. Hydrops placentalis.
944.

Which entity is characterized by marked placental enlargement, chorionic


vascular abnormalities and cystic villous edema with approximately 50%
of cases also having associated findings suggestive of BeckwithWiedermann syndrome?
b. Mesenchymal dysplasia.

945.

Which entity may be caused by Parvovirus B19 infection?


a. Hydrops placentalis.

946.

For each of the following questions choose the most likely diagnosis from
the following diagnostic set (an answer may be used once, more than
once, or not at all). (PIP 2005 D)
Diagnostic set:
a. Infantile fibromatosis, cellular type.
b. Infantile fibrosarcoma.
c. Infantile hemangiopericytoma.
d. Infantile myofibromatosis.
e. Inflammatory myofibroblastic tumor.
f. Spindle cell rhabdomyosarcoma (embryonal rhabdomyosarcoma,
spindle cell type).
Which tumor in a 6-month-old child has a pattern of uniform spindle cells
in tightly packed fascicles with focal areas of a complex network of thin
walled sinusoidal blood vessels and shows a t(12;15)(p13;q25) and/or
fusion gene ETV6-NTRK3 by RT-PCR?
b. Infantile fibrosarcoma.

Reserved Copyrights - 2013

242

Pathology Questions
947.

MCQs

Which spindle cell tumor in an 18-month-old child may have a prominent


background of mostly lymphocytes and plasma cells and is positive for
ALK1 and/or p80?
e. Inflammatory myofibroblastic tumor.

948.

Which tumor has uniform spindle cells with eosinophilic cytoplasm,


variable amounts of collagen and is vimentin (+), cytokeratin(-), EMA(-),
desmin(+) and myogenin(+)?
f.Spindle cell rhabdomyosarcoma (embryonal
rhabdomyosarcoma,spindle cell type).

949.

For each of the following questions choose the most likely diagnosis from
the following diagnostic set (an answer may be used once, more than
once, or not at all). (PIP 2005 D)
Diagnostic set:
a. Acute myeloid leukemia.
b. Acute septic splenitis.
c. Chronic myelogenous leukemia.
d. Extramedullary hematopoiesis.
e. Extramedullary myeloid tumor/ granulocytic sarcoma.
f. Hairy cell leukemia.
g. Hodgkin lymphoma.
h. Splenic marginal zone B-cell lymphoma.
Which of the above is a cause of massive splenomegaly and is associated
with t(9;22)?
c. Chronic myelogenous leukemia.
Which of the above is a cause of massive splenomegaly and shows marked
red pulp expansion by a CD11c(+), CD20(+), CD25(+) population?
f. Hairy cell leukemia.

Reserved Copyrights - 2013

243

Pathology Questions

MCQs

Which of the above involves the spleen in a nodular fashion and shows
rare CD30(+), CD15(+) cells in a polymorhic inflammatory background?
g. Hodgkin lymphoma.
950.

Which of the following is considered noninvasive? (PIP 2006 B)


a. Adenocarcinoma, primary pulmonary.
b. Bronchioloalveolar carcinoma, nonmucinous type.
c. Metastatic renal cell carcinoma.
d. Mucinous (colloid) adenocarcinoma, primary pulmonary.
e. All of the above are invasive.

951.

Which of the following is typically negative for BOTH TTF-1 and CDX2?
(PIP 2006 B)
a. Adenocarcinoma, primary pulmonary.
b. Bronchioloalveolar carcinoma, mucinous type.
c. Bronchioloalveolar carcinoma, nonmucinous type.
d. Mucinous (colloid) adenocarcinoma, primary pulmonary.
e. Metastatic colonic adenocarcinoma.

952.

Which entity is most likely to be TTF-1 negative, cytokeratin 7 negative


and CDX2 positive? (PIP 2006 B)
a. Adenocarcinoma, primary pulmonary.
b. Bronchioloalveolar carcinoma,mucinous type.
c. Bronchioloalveolar carcinoma, nonmucinous type.
d. Mucinous (colloid) adenocarcinoma, primary pulmonary.
e. Metastatic colon adenocarcinoma.

953.

Which of the following meningioma subtype is considered to be WHO


Grade I? (PIP 2006 B)
a. Chordoid meningioma.
b. Clear cell meningioma.
c. Microcytic meningioma.
d. Papillary meningioma.

Reserved Copyrights - 2013

244

Pathology Questions

MCQs

e. Rhabdoid meningioma.
954.

The estimated 5-year recurrence rate for totally resected WHO Grade II
(atypical) meningioma is: (PIP 2006 B)
a. 20%.
b. 40%.
c. 60%.
d. 80%.
e. 100%.

955.

Multiple meningiomas commonly accompany which of the following


syndromes? (PIP 2006 B)
a. Li-Fraumeni.
b. Neurofibromatosis, type 1.
c. Neurofibromatosis, type 2.
d. Tuberous sclerosis.
e. Von-Hippel Lindau.

956.

Which are the most important prognostic features of Wilms tumor? (PIP
2006 B)
a. Stage.
b. Histology.
c. Age and MYC-N expression.
d. A & B.
e. A, B and C.

957.

Which of the following antigens may react in Wilms tumor? (PIP 2006 B)
a. Vimentin and perhaps cytokeratin.
b. Vimentin, cytokeratin and EMA.
c. NSE and synaptophysin.
d. CD99.
e. None of the above.

Reserved Copyrights - 2013

245

Pathology Questions
958.

MCQs

Which of the following tumors are likely to be considered in the


differential diagnosis of blatesmal-rich Wilms tumor? (PIP 2006 B)
a. Primary

renal

neuroblastoma

and

peripheral

primitive

neuroectodermal tumor (PNET).


b. Malignant rhabdoid tumor of kidney.
c. Clear cell sarcoma of kidney.
d. Congenital mesoblastic nephroma.
e. B, C and D.
959.

For each of the following, select the most likely diagnosis from the
diagnostic set (an answer may be used once, more than once, or not at
all): (PIP 2006 B)
Diagnostic set:
a. Elastofibroma.
b. Fibrolipoma.
c. Neurofibroma.
d. Nuchal-type fibroma.
e. Scar.
f. Spindle cell lipoma.
Which lesion is occasionally associated with Gardner syndrome?
d. Nuchal-type fibroma.
Which lesion favors the shoulder and posterior neck of middle-aged men?
f. Spindle cell lipoma.
Which lesion contains increased and abnormal elastic fibers?
a. Elastofibroma.

960.

For each of the following, select the most likely diagnosis from the
diagnostic set (an answer may be used once, more than once, or not at
all): (PIP 2006 B)
Diagnostic set:

Reserved Copyrights - 2013

246

Pathology Questions

MCQs

a. Adrenal medullary hyperplasia.


b. Adrenal pseudocyst.
c. Ganglioneuroma.
d. Metastatic carcinoma.
e. Neuroblastoma.
f. Pheochromocytoma.
Which condition may arise as a consequence of maturation or
differentiation of another entity in the diagnostic set?
c.Ganglioneuroma.
Which entity is among the most common neoplasms?
e.Neuroblastoma.
Which is the most common malignant neoplasm occuring in the adult
adrenal gland?
d. Metastatic carcinoma.
961.

For each of the following, select the most likely diagnosis from the
diagnostic set (an answer may be used once, more than once, or not at
all): (PIP 2006 B)
Diagnostic set:
a. Adenosarcoma.
b. Carcinosarcoma (malignant mixed mllerian tumor).
c. Endometrial stromal sarcoma, low grade.
d. Endometrioid adenocarcinoma.
e. Leiomyosarcoma.
f. Serous carcinoma.
Which tumor most commonly contains heterologous elements?
b. Carcinosarcoma (malignant mixed mllerian tumor).

Reserved Copyrights - 2013

247

Pathology Questions

MCQs

In which tumor is benign appearing squamous metaplasia most


commonly obsereved?
d. Endometrioid adenocarcinoma.
Which tumor characteristically exhibits a histological growth pattern
similar to that of phyllodes tumor of the breast?
a.Adenosarcoma.
962.

For each of the following, select the most likely diagnosis from the
diagnostic set (an answer may be used once, more than once, or not at
all): (PIP 2006 B)
Diagnostic set:
a. Hibernoma.
b. Lipoblastoma.
c. Lipofibromatosis (Infantile Fibromatosis).
d. Lipoma.
e. Myxoid liposarcoma.
The cytogenetic finding of t(12;16) is associated with which entity?
e. Myxoid liposarcoma.
Which entity may show progressive maturation with each recurrence?
b. Lipoblastoma.
Which entity is subclassified based on the tissue or site involved by the
tumor?
b. Lipoblastoma.

963.

Xanthogranulomatous

pyelonephritis

is

probably

caused

by

combination of urinary tract obstruction and infection. Which


organism(s) are isolated most frequently? (PIP 2006 B)
a. E.coli or Proteus mirabilis.
b. Staphylococcus aureus.
Reserved Copyrights - 2013

248

Pathology Questions

MCQs

c. Klebsiella species.
d. Pseudomonas species.
964.

Features that will favor a diagnosis of XPN over clear cell RCC include:
(PIP 2006 B)
a. Numerous

macrophages

with

target-like

intracytoplasmic

inclusions or Michaelis-Gutmann bodies.


b. A C.T Scan that shows an 8 cm. renal mass.
c. A zonal pattern of inflammation with central acute inflammation,
clusters of foamy histiocytes, fibrosis and giant cells.
d. Positive immunohistochemichal stains for cytokeratin and EMA.
965.

For each of the following, select the most likely diagnosis from the
diagnostic set (an answer may be used once, more than once, or not at
all):
Diagnostic set:
a. Adult granulosa cell tumor.
b. Endometrioid adenocarcinoma.
c. Sertoli-Leydig cell tumor.
Which entity most commonly occurs in menopausal and postmenopausal
women and is the most likely tumor to be associated with estrogenic
manifestations?
a. Adult granulosa cell tumor.
Which entity most commonly occurs in younger women and may be
associated with virilization?
c. Sertoli-Leydig cell tumor.
Which entity typically positive for epithelial membrane antigen?
b. Endometrioid adenocarcinoma.

Reserved Copyrights - 2013

249

Pathology Questions
966.

MCQs

For each of the following, select the most likely diagnosis from the
diagnostic set( an answer may be used once, more than once, or not at
all): (PIP 2006 B)
Diagnostic set:
a. Caroli disease.
b. Caroli syndrome.
c. Choledochal cyst.
d. Congenital hepatic fibrosis.
e. Polycystic liver disease.
Which of the above is inherited in an autosomal recessive manner and is
not associated with grossly visible hepatic cysts?
d. Congenital hepatic fibrosis.
Which of the above is almost invariably associated with autosomal
dominant polycystic kidney disease?
e. Polycystic liver disease.
Which of the above shows cystic dilation of interlobular and segmental
bile ducts, often presents with recurrent colangitis, and microscopically
does not show evidence of congenital hepaticfibrosis?
a. Caroli disease.

967.

For each of the following, select the most likely diagnosis from the
diagnostic set (an answer may be used once, more than once, or not at
all): (PIP 2006 B)
Diagnostic set:
a. Gaucher disease.
b. Histiocytic sarcoma.
c. Hodgkin lymphoma.
d. Interdigitating dindritic cell sarcoma.
e. Langerhans cell histiocytes.
f. Sinus histiotitis with massive lymphadenopathy (Rosai Dorfman
disease).

Reserved Copyrights - 2013

250

Pathology Questions

MCQs

Which tumor shows birbeck granules by electron microscopy?


e. Langerhans cell histiocytes.
Which tumor is characterized by an S-100 positive, fascin positive,
vimentin positive, CD68 positive phenotype:
d. Interdigitating dindritic cell sarcoma.
Which tumor is characterized by cells that have fibrillary cytoplasm and
stain faintly with PAS?
a. Gaucher disease.
968.

For each of the following, select the most likely diagnosis from the
diagnostic set (an answer may be used once, more than once, or not at
all): (PIP 2006 C)
Diagnostic set:
a. Cirrhosis due to alpha1 antitrypsin deficiency.
b. Cirrhosis due to amiodarone toxicity.
c. Cirrhosis due to autoimmune hepatitis.
d. Cirrhosis due to steatohepatitis.
e. Cirrhosis due to Wilson disease.
Which is least likely to be associated with mallory hyaline?
c. Cirrhosis due to autoimmune hepatitis.
Which can be regarded as a model of protein conformational disease?
a. Cirrhosis due to alpha1 antitrypsin deficiency.
Which is related to mutations of the SERPINA gene on chromosome 15?
a. Cirrhosis due to alpha1 antitrypsin deficiency.

Reserved Copyrights - 2013

251

Pathology Questions
969.

MCQs

For each of the following, select the most likely diagnosis from the
diagnostic set (an answer may be used once, more than once, or not at
all): (PIP 2006 C)
Diagnostic set:
a. Paraganglioma.
b. Small cell carcinoma.
c. Thymic carcinoid.
d. Thymic carcinoma.
e. Thymoma.
Which tumor occurs more commonly in the middle or posterior
mediastinal, is positive for neuroendocrine markers but is typically
negative for cytokeratin?
a.Paraganglioma.
Which tumor occurs in the anterior mediatinum, is positive for
neuroendocrine markers and cytokeratin, has a low mitotic rate and may
be associated with cushing's syndrome?
c. Thymic carcinoid.

970.

Which tumor occurs as an anterior mediastinal mass, shows lobular


growth with admixed lymphocyles, and is positive for keratin while being
negative for chromogranin and synaptophysin?
e. Thymoma.

971.

For each of the following, select the most likely diagnosis from the
diagnostic set (an answer may be used once, more than once, or not at
all):
Diagnostic set:
a. Angiomyolipoma.
b. Collecting duct cracinoma.
c. Juxtaglomerular cell tumor.
d. Metanephric adenoma.

Reserved Copyrights - 2013

252

Pathology Questions

MCQs

e. Mucinous tubular and spindle cell carcinoma of kidney(MTSCC).


f. Renal cell carcinoma with sarcomatoid differentiation.
Which

tumor

possesses

rhamboid-shaped

renin

granules

ultrastructurally?
c. Juxtaglomerular cell tumor.
Which tumor typically expresses HMB-45 in neoplastic cells?
a.Angiomyolipoma.
Which tumor frequently has psammoma bodies in the stroma?
d. Metanephric adenoma.
972.

For each of the following, select the most likely diagnosis from the
diagnostic set (an answer may be used once, more than once, or not at
all): (PIP 2006 C)
Diagnostic set:
a. Amyloidosis.
b. Cirrhosis due to steatohepatitis.
c. Epithelioid hemangioendothelioma.
d. Venous outflow obstruction with extensive fibrosis (cardiac
cirrhosis).
Which condition does not feature perisinusoidal connective tissue
deposition?
c. Epithelioid hemangioendothelioma.
Which non-neoplastic entity typically involves the entire hepatic acinus
(i.e: nonzonal)?
a. Amyloidosis.
Which entity is not associated with portal hypertension?
c. Epithelioid hemangioendothelioma.

Reserved Copyrights - 2013

253

Pathology Questions

973.

MCQs

For each of the following, select the most likely diagnosis from the
diagnostic set (an answer may be used once, more than once, or not at
all): (PIP 2006 C)
Diagnostic set:
a. Cellular schwannoma.
b. Fibrosarcoma.
c. Hemangiopericytoma.
d. Leiomyosarcoma.
e. Malignant peripheral nerve sheath tumor.
f. Poorly differentiated synovial sarcoma.
Which tumor is most commonly associated with NF 1 deletions?
e. Malignant peripheral nerve sheath tumor.
Which tumor is an (X;18) fusion protein characteristic?
f. Poorly differentiated synovial sarcoma.
Which malignant tumor often arises in the retroperitoneum in adults?
d. Leiomyosarcoma.

974.

Which of the following is not true about herpes simplex virus hepatitis?
a. Can affect immunocompetent adults.
b. Often follows a rapidly progressive course.
c. Histological findings are often indistinguishable from adenoviral
hepatitis.
d. Marked lymphoplasmacytic inflammation is common.
e. Necrosis is often extensive and non-zonal.

975.

Which of the following statements about acetaminophen toxicity is not


correct?
a. It is the most common cause of fulminant hepatic failure in the US.
b. Chronic alcohol use can enhance hepatotoxicity of acetaminophen.

Reserved Copyrights - 2013

254

Pathology Questions

MCQs

c. Necrosis most often occurs in a non-zonal distribution.


d. Histological distinction from ischemic necrosis can be difficult.
e. Sinusoidal dilatation and congestion can be seen in the centrizonal
areas.
976.

All the following are true about Wilson disease except:


a. Presentation after 50 years of age is uncommon.
b. 24-hour urinary copper is markedly elevated in fulminant disease.
c. Normal ceruloplasmin levels can be seen in around 10% of cases.
d. Absence of copper on histochemical stains rules out Wilson
disease.
e. Quantitative copper levels from the paraffin block is the most
accurate method of assessing hepatic copper.

977.

For each of the following, select the most likely diagnosis from the
diagnostic set (an answer may be used once, more than once, or not at
all): (PIP 2006 C)
Diagnostic set:
a. Brenner tumor.
b. Endometrioid adenofibroma.
c. Granulosa cell tumor.
d. Mucinous cystadenoma.
e. Metastatic carcinoma.
Which entity is characterized by bland, stratified, typically non-mucin
containing glands in a fibromatous stroma that may be associated with
squamous morules?
b. Endometrioid adenofibroma.
Which entity may have solid nests of epithelial cells that stain positively
for uroplakin III, cystic cavities containing eosinophilic secretions and a
fibromatous stroma?
a. Brenner tumor.

Reserved Copyrights - 2013

255

Pathology Questions

MCQs

Which entity is frequently found in association with a Brenner tumor?


d. Mucinous cystadenoma.
978.

For each of the following, select the most likely diagnosis from the
diagnostic set( an answer may be used once, more than once, or not at
all): (PIP 2006 C)
Diagnostic set:
a. Angiosarcoma.
b. Bacillary angiomatosis.
c. Epithelioid hemangioma.
d. Granulation tissue.
e. Kaposi sarcoma.
f. Lobular capillary hemangioma.
Which lesion contains hyaline globules and extravasated red cells?
e. Kaposi sarcoma.
Which lesion is non-neoplastic, caused by Bartonella species, and
responsive to antibiotics?
b. Bacillary angiomatosis.
Which lesion is malignant and not associated with HIV/AIDS?
a. Angiosarcoma.

979.

Which of the following is not significant in the diagnosis of glioblastoma?


(PIP 2006 D)
a. High cell density.
b. Mitotic activity.
c. Necrosis.
d. Vascular proliferation.

Reserved Copyrights - 2013

256

Pathology Questions
980.

MCQs

Which molecular marker would most likely show an abnormality in


recurrent high-grade astrocytoma with gemistocytic features? (PIP 2006
D)
a. EWS-Fli 1 gene fusion PCR.
b. Loss of heterozygosity of PTEN locus (10q.23).
c. MYCC amplification analysis.
d. OTX2 amolification.

981.

What is the patient's expected 5-year survival in recurrent high-grade


astrocytoma with gemistocytic features (WHO grade III-IV)? (PIP 2006 D)
a. 2%.
b. 25%.
c. 50%.
d. 75%.

982.

For each of the following, select the most likely diagnosis from the
diagnostic set (an answer may be used once, more than once, or not at
all): (PIP 2006 D)
Diagnostic set:
a. Acute fibrinous and organizing pneumonia.
b. Acute lobar pneumonia.
c. Diffuse alveolar damage.
d. Diffuse alveolar hemorrhage with capillaritis.
e. Eosiniphilic pneumonia.
Which entity is characterized by prominent eosiniphils, intra-alveolar
fibrin and macrophages?
e. Eosiniphilic pneumonia.
Which entity is characterized by intra-alveolar hemosiderin-laden
macrophages and neutrophilic infiltrates located predominantly in the
alveolar septa?
d. Diffuse alveolar hemorrhage with capillaritis.

Reserved Copyrights - 2013

257

Pathology Questions

MCQs

Which entity is characterized by hyaline membrane formation and


sparse/absent inflammatory cell infiltrates?
c. Diffuse alveolar damage.
983.

Which of the following is true regarding spermatocytic seminoma? (PIP


2006 D)
a. One of the most common germ cell tumors in adolescence.
b. Commonly associated with IGCNU.
c. One of the most common germ cell tumors in cryptorchid testes.
d. Generally has an excellent prognosis except in the rare
circumstance when it is associated with a sarcomatous component.

984.

Seminoma, classic type may be differentiated from spermatocytic


seminoma by all of the following criteria except: (PIP 2006 D)
a. Seminoma, classic type is commonly associated with IGCNU and
other germ cell types.
b. The neoplastic cells in seminoma, classic type, commonly have
abundant cytoplasmic glycogen and are usually diffusely positive
for OCT .
c. The neoplastic cells of seminoma, classic type are characteristically
a polymorphous population of small, medium and large cells.
d. Seminoma, classic type commonly has a prominent lymphocytic
infiltrate and occasional granulomatous stromal reaction.

985.

Which of the following tumors shows an alveolar growth pattern with


strong nuclear staining for Myo-D1 and cytoplasmic staining for desmin?
(PIP 2006 D)
a. Granular cell tumor.
b. Alveolar rhabdomyosarcoma.
c. Alveolar soft part sarcoma.
d. Leiomyosarcoma.

Reserved Copyrights - 2013

258

Pathology Questions
986.

MCQs

Which of the following tumors shows rhomboid crystal formation on


electron microscopy? (PIP 2006 D)
a. Granular cell tumor.
b. Alveolar soft part sarcoma.
c. Alveolar rhabdomyosarcoma.
d. Paraganglioma.

987.

Which of the following immunohistochemical stains is most specific for


alveolar soft part sarcoma?
a. S-100 protein.
b. Smooth muscle actin.
c. Cytokeratin AE1/3.
d. TFE-3.

988.

For each of the following, select the most likely diagnosis from the
diagnostic set (an answer may be used once, more than once, or not at
all): (PIP 2006 D)
Diagnostic set:
a. Extraskeletal myxoid chondrosarcoma.
b. Intramuscular myxoma.
c. Low-grade fibromyxoid sarcoma.
d. Myxofibrosarcoma.
e. Myxoid liposarcoma.
f. Myxoid neurothekeoma (nerve sheath myxoma).
Which tumor is characterized by myxoid stroma that is chonroitin
sulfate-rich instead of hyaluronic acid-rich and shows chromosomal
rearrangements involving 9q22 resulting in NR4A3 fusion gene?
a. Extraskeletal myxoid chondrosarcoma.
Which lesion prefers the cervicofascial areas of young women and is A100 protein immunoreactive?
f. Myxoid neurothekeoma (nerve sheath myxoma).

Reserved Copyrights - 2013

259

Pathology Questions

MCQs

Which lesion may be associated with precocious puberty and fibrous


dysplasia?
b. Intramuscular myxoma. (often as part of McCune-Albright
syndrome).
989.

For each of the following, select the most likely diagnosis from the
diagnostic set( an answer may be used once, more than once, or not at
all): (PIP 2006 D)
Diagnostic set:
a. Adult granulosa cell tumor.
b. Endometrioid adenocarcinoma.
c. Sex-cord stromal tumor with annular tubules.
d. Small cell carcinoma, hypercalcemic type.
Which

tumor

is

most

commonly

associated

with

estrogenic

manifestations?
a. Adult granulosa cell tumor.
Which entity most often has spread beyond the ovary at the time of
presentation and typically occurs in young women?
d. Small cell carcinoma, hypercalcemic type.
Which entity may be associated with Peutz-Jeghers syndrome?
c. Sex-cord stromal tumor with annular tubules.
990.

For each of the following, select the most likely diagnosis from the
diagnostic set (an answer may be used once, more than once, or not at
all): (PIP 2006 D)
Diagnostic set:
a. Eighth nerve (acoustic) schwannoma.
b. Neurothekeoma (nervev sheath myxoma).
c. Plexiform fibrohistiocytic tumor.

Reserved Copyrights - 2013

260

Pathology Questions

MCQs

d. Plexiform neurofibroma.
e. Plexiform schwannoma.
Which lesion demonstrates expression of CD68 and smooth muscle actin?
c. Plexiform fibrohistiocytic tumor.
Which lesion is the most likely to occur in association with a genetic
disorder involving chromosome 22?
a. Eighth nerve (acoustic) schwannoma.
Which lesion is most strongly associated with neurofibromatosis type 1
(NF1)?
d. Plexiform neurofibroma.
991.

For each of the following, select the most likely diagnosis from the
diagnostic set(an answer may be used once, more than once, or not at all):
(PIP 2006 D)
Diagnostic set:
a. Atypical leiomyoma.
b. Leiomyosarcoma.
c. Lipoleiomyoma.
d. Lipoma.
Which tumor is characterized by the admixture of adipocytes and smooth
muscle?
c. Lipoleiomyoma.
Which tumor is characterized by cytologically atypical cells but
infrequent mitoses?
a. Atypical leiomyoma.
Which tumor typically exhibits the presence of coagulative necrosis of
neoplastic cells and mitotic figures?

Reserved Copyrights - 2013

261

Pathology Questions

MCQs

b. Leiomyosarcoma.
992.

All of the following are true of paraganglioma of the cauda equina, except:
(PIP 2007 B)
a. Cytoplasm of tumor cells is GFAP positive.
b. May be cytokeratin positive.
c. May show ganglionic differentiation.
d. Sustentacular cells are S100 positive.
e. Ultrastructural features show neurosecretory granules.

993.

Which of the following characteristics are unique to paragangliomas of


the cauda equina? (PIP 2007 B)
a. EM shows filament skeins, intercellular microvilli, and numerous
zonula adherens junctional complexes.
b. May show ganglionic differentiation and cytokeratin positivity.
c. NSE is positive in a cytoplasmic distribution.
d. May secrete neuropeptides.

994.

Which of the following is true in distinguishing paraganglioma from


myxopapillary ependymoma? (PIP 2007 B)
a. Although both tumors may express GFAP, the stain shows
predominance in the sustentacular cells of paragangliomas and the
tumor cells of ependymomas.
b. Electron microscopy of myxopapillary ependymoma shows
neurosecretory granules.
c. Ependymomas show a characteristic zellballen configuration.
d. Myxopapillary

ependymomas

characteristically

express

synaptophysin and chromogranin.


995.

For each of the following questions choose the most likely diagnosis
from the following diagnostic set (an answer may be used once, more
than once, or not at all). (PIP 2007 B)
Diagnostic set:

Reserved Copyrights - 2013

262

Pathology Questions

MCQs

a. Dermatofibrosarcoma protuberans.
b. Myxoid liposarcoma.
c. Neurofibroma.
d. Nodular fasciitis.
e. Spindle cell lipoma.
Which entity is typically a rapidly growing process characterized by cells
with a tissue culture appearance and extravasated red blood cells?
d. Nodular fasciitis.
Which entity is a slowly growing well circumscribed tumor that is
composed of a mixture of mature adipocytes and spindle cells and is
typically positive for CD34?
e. Spindle cell lipoma.
Which tumor is characterized by mitotically active CD34 positive spindle
cells arranged in a storiform pattern with an infiltrative border?
a. Dermatofibrosarcoma protuberans.
996.

For each of the following questions choose the most likely diagnosis from
the following diagnostic set (an answer may be used once, more than
once, or not at all). (PIP 2007 B)
Diagnostic set:
a. Adenosarcoma.
b. Cellular leiomyoma.
c. Endometrial stromal sarcoma, low grade.
d. Intravenous leiomyomatosis.
e. Leiomyosarcoma.
f. Undifferentiated uterine sarcoma.
g. Uterine tumor resembling ovarian sex cord tumor.
Which tumor exhibits periglandular cuffing by malignant stromal cells?
a. Adenosarcoma.

Reserved Copyrights - 2013

263

Pathology Questions

MCQs

Which tumor is composed of monotonus cells with round-to-oval nuclei,


contains blood vessels resembling spiral arterioles and typically exhibits a
CD10(+), h-caldesmon (-), desmin(-) immunophenotype?
c. Endometrial stromal sarcoma, low grade.
Which tumor is most likely to exhibit prominent staining for alphainhibin, CD99, and or calretinin?
g. Uterine tumor resembling ovarian sex cord tumor.
997.

For each of the following questions choose the most likely diagnosis
from the following diagnostic set (an answer may be used once, more
than once, or not at all). (PIP 2007 B)
Diagnostic set:
a. Alveolar rhabdomyosarcoma.
b. Desmoplastic small round cell tumor.
c. Ewing's sarcoma/primitive neuroectodermal tumor.
d. Lymphoblastic lymphoma.
e. Metastatic small cell carcinoma.
f. Neuroblastoma.
Which tumor is typically characterized by an immunohistochemical
profile of CD99(+), CD43(+), CD34(+), CD10(+), CD79a(+)?
d. Lymphoblastic lymphoma.
Which tumor frequently shows the translocation t(2'13)(q35;q14)?
a. Alveolar rhabdomyosarcoma.
Which tumor typically arises within the peritoneal cavity of adolescent
males?
b. Desmoplastic small round cell tumor.

Reserved Copyrights - 2013

264

Pathology Questions
998.

MCQs

For each of the following questions choose the most likely diagnosis
from the following diagnostic set (an answer may be used once, more
than once, or not at all). (PIP 2007 B)
Diagnostic set:
a. Choriocarcinoma.
b. Embryonal carcinoma.
c. Seminoma.
d. Sertoli cell tumor.
e. Yolk sac tumor.
Which adult testicular germ cell tumor characteristically has the following
immunohistochemical profile: PLAP(+), CD117(+), CD30(-), AFP(-) and
inhibin(-)?
c. Seminoma.
Which adult testicular germ cell tumor characteristically has the following
immunohistochemical

profile:

AFP(+),

AE1/AE3(+),

CD30(-)

and

OCT3/4(-)?
e. Yolk sac tumor.
Which adult testicular germ cell tumor characteristically has the following
immunohistochemical

profile:

PLAP(+),

OCT3/4(+),

CD30(+),

AE1/AE3(+), inhibin(-)?
b. Embryonal carcinoma.
999.

For each of the following questions choose the most likely diagnosis from
the following diagnostic set (an answer may be used once, more than
once, or not at all). (PIP 2007 B)
Diagnostic set:
a. Atypical leiomyoma.
b. Endometrial stromal nodule.
c. Epithelioid leiomyoma.
d. Epithelioid leiomyosarcoma.

Reserved Copyrights - 2013

265

Pathology Questions

MCQs

An epithelioid, desmin(+), caldesmon(+), 3.0 cm well-circumscribed


uterine tumor with no necrosis, no cytologic atypia and a mitotic count<3
mitoses per 10 high power fields is best classified as what type of tumor?
c. Epithelioid leiomyoma.
An epithelioid, desmin(+), caldesmon(+), 10.0cm well-circumscribe
uterine tumor with multifocal cytologic atypia duscernible at low power
and mitotic count of 5 mitoses per 10 high power fields is best classified
as what type of tumor?
d. Epithelioid leiomyosarcoma.
An epithelioid, desmin(+), caldesmon(+), infiltrative uterine tumor with
bland cytomorphology and mitotic count of 5 mitoses per 10 high power
fields is best classified as what type of tumor?
d. Epithelioid leiomyosarcoma.
1000. For each of the following questions choose the most likely diagnosis from
the following diagnostic set (an answer may be used once, more than
once, or not at all). (PIP 2007 B)
Diagnostic set:
a. Acute cellular allograft rejection.
b. Acute tubular necrosis.
c. Chronic allograft nephropathy.
d. Cyclosporine/tacrolimus toxicity.
e. Post-transplant lymphoproliferative disorder.
Which condition shows non-specific changes of interstitial fibrosis and
tubular atrophy?
c. Chronic allograft nephropathy.
Which condition is graded according to severity of tubulitis or arteritis?
a. Acute cellular allograft rejection.
Reserved Copyrights - 2013

266

Pathology Questions

MCQs

Which condition shows a predominantly B-cell infiltrate?


e. Post-transplant lymphoproliferative disorder.
1001. For each of the following questions choose the most likely diagnosis from
the following diagnostic set (an answer may be used once, more than
once, or not at all). (PIP 2007 B)
Diagnostic set:
a. Cardiac myxoma.
b. Hemangioma.
c. Organizing mural thrombus.
d. Papillary fibroelastoma.
The findings of benign mucin-producing cytokeratin 6 positive glands and
extramedullary hematopoiesis have been described in this entity:
a. Cardiac myxoma.
Which entity most often involves the cardiac valves?
d. Papillary fibroelastoma.
Which entity may present as an intracavitary mass which usually involves
the ventricles?
b. Hemangioma.
1002. For each of the following questions choose the most likely diagnosis from
the following diagnostic set (an answer may be used once, more than
once, or not at all). (PIP 2007 B)
Diagnostic set:
a. Fibrous dysplasia.
b. Malignant fibrous histiocytoma of bone.
c. Mesenchymal chondrosarcoma.
d. Osteoid osteoma.
e. Osteosarcoma.
Reserved Copyrights - 2013

267

Pathology Questions

MCQs

f. Reactive changes secondary to infection.


Which entity is a well circumscribed lesion composed of various
proportions of fibrous and osseous elements. The osseous component is
represented by irregular, curvilinear, discontinuous trabeculae of woven
(or rarely lamellar) bone with no osteoblastic rimming?
a. Fibrous dysplasia.
Which entity frequently complicates preexisting osseous lesions (i.e:
radiation damage, Paget disease, cartilaginous neoplasms, and rarely
bone infarcts) and is microscopically composed of pleomorphicspindle
cells arranged in a storiform pattern and not associated with tumoral
osteoid formation?
b. Malignant fibrous histiocytoma of bone.
Which entity is most often associated in later life with an underlying
primary pathologic cindition of bone such as fibrous dysplasia or Paget
disease?
e. Osteosarcoma.
1003. For each of the following questions choose the most likely diagnosis
from the following diagnostic set (an answer may be used once, more
than once, or not at all). (PIP 2007 C)
Diagnostic set:
a. Aggressive angiomyxoma.
b. Angimyofibroblastoma.
c. Myxoid leiomyoma.
d. Myxoid neurofibroma.
Which myxoid tumor may be locally aggressive and is characterized by
bland

stellate-shaped,

muscle

specific

actin

immunoreactive

mesenchymal cells in an abundant myxoid stroma containing thick-walled


blood vessels?
Reserved Copyrights - 2013

268

Pathology Questions

MCQs

a. Aggressive angiomyxoma.
Which tumor is well circumscribed, usually does not recur after excision,
and is characterized by epithelioid cells clustered around capillary sized
blood cells?
b. Angimyofibroblastoma.
Which tumor shows interlacing fascicles of wavy buckled elongated
cells in asoociation with strands of collagen (shredded carrots) within a
myxomatous background with no mitoses and can be associated with
mutations involving the long arm of chromosome 17?
c. Myxoid leiomyoma.
1004. Which of the following is true of immunophenotype of renal cell
carcinoma? (PIP 2007 C)
a. Both CK7 and CK20 are usually positive in clear cell renal cell
carcinoma.
b. RCC antigen I is expressed in nearly 100% of clear cell renal cell
carcinoma.
c. Vimentin is negative in most cases of chromophobe renal cell
carcinoma.
d. CD10 is negative in most cases of papillary renal cell carcinoma.
1005. Which of the following tumors typically lacks expression of the
transcription factor CDX-2 by immunohistochemistry? (PIP 2007 C)
a. Preampullary adenocarcinoma.
b. Colorectal adenocarcinoma.
c. Cholangiocarcinoma.
d. Gastric adenocarcinoma.
1006. Which of the following is true about immunohistochemical distinction of
clear cell renal cell carcinoma and adrenocortical carcinoma?
a. Expression of vimentin favors clear cell renal cell carcinoma.
Reserved Copyrights - 2013

269

Pathology Questions

MCQs

b. Chromogranin and synaptophysin are generally positive in


adrenocortical carcinoma.
c. Cytokeratin is expressed in clear cell renal cell carcinoma but is
usually not expressed in adrenocortical carcinoma.
d. Expression of melan A and inhibin is often seen in both tumors.
1007. For each of the following questions choose the most likely diagnosis from
the following diagnostic set (an answer may be used once, more than
once, or not at all). (PIP 2007 C)
Diagnostic set:
a. Clear cell tumor of the lung.
b. Inflammatory myofibroblastic tumor.
c. Malakoplakia.
d. Metastatic renal cell carcinoma.
e. Mycobacterial pseudotumor.
f. Pulmonary cryptococcosis.
Which entity is typically encountered in immunocompromised individuals
and is associated with Rhodococcus equii infection?
c. Malakoplakia.
Which entity is composed of clear cells which are positive for HMB-45?
a. Clear cell tumor of the lung.
Which entity may produce a wide spectrum of histologic change including
granulomas, fibrohistiocytic masses, histiocytic infiltrates and mucoid
pneumonia?
e. Mycobacterial pseudotumor.
1008. For each of the following questions choose the most likely diagnosis from
the following diagnostic set (an answer may be used once, more than
once, or not at all). (PIP 2007 C)
Diagnostic set:
Reserved Copyrights - 2013

270

Pathology Questions

MCQs

a. Cystic islet cell neoplasm.


b. Intraductal papillary mucinous neoplasm.
c. Mucinous cystic neoplasm (mucinous cystadenoma).
d. Pancreatic pseudocyst.
e. Serous cystadenoma.
f. Solid pseudopapillary neoplasm of the pancreas.
Which of the above entities is not a neoplasm?
d. Pancreatic pseudocyst.
Which entity features ovarian-type spindle cell stroma, which is often
positive for estrogen and progesterone receptors?
c. Mucinous cystic neoplasm (mucinous cystadenoma).
Frozen section analysis of the pancreatic duct margin is most important
for which entity?
b. Intraductal papillary mucinous neoplasm.
1009. For each of the following questions choose the most likely diagnosis from
the following diagnostic set (an answer may be used once, more than
once, or not at all). (PIP 2007 C)
Diagnostic set:
a. Adenosarcoma.
b. Endometrial stromal sarcoma with sex cord-like elements.
c. Endometrioid adenocarcinoma with prominent spindle cell
component.
d. Immature teratoma.
e. Malignant mixed mullerian tumor.
f. Mucinous tumor of ovary with mural nodule.
g. Sertoli-Leydig cell tumor with heterologous elements.
Which tumor can have an epithelial component, but also experesses
alpha-inhibin and calretinin?
g. Sertoli-Leydig cell tumor with heterologous elements.
Reserved Copyrights - 2013

271

Pathology Questions

Which

of

the

MCQs

above

periglandularstromal

is

cuffs

biphasic

and

polypoid

tumor

that

may

intraglandular

have

stromal

projections?
a. Adenosarcoma.
Which tumor occurs predominantly in young patients and is
characterized by tubule-like structures that stain positively for GFAP, NSE
and neurofilament protein?
d. Immature teratoma.
1010. For each of the following questions choose the most likely diagnosis from
the following diagnostic set (an answer may be used once, more than
once, or not at all). (PIP 2007 C)
Diagnostic set:
a. Carcinoid tumor.
b. Metanephric adenofibroma.
c. Metanephric adenoma.
d. Papillary renal cell carcinoma.
e. Wilm's tumor.
Which tumor most likely demonstrates gains of chromosomes 7 and 17?
d. Papillary renal cell carcinoma.
Which tumor is most likely to show abnormalities of chromosome 1?
e. Wilm's tumor.
Which tumor would typically show an immunophenotype of EMA(+),
cytokeratin 7(+), AMACR(+) and WTI(-)?
c. Metanephric adenoma.
1011. For each of the following questions choose the most likely diagnosis from
the following diagnostic set (an answer may be used once, more than
once, or not at all). (PIP 2007 C)
Reserved Copyrights - 2013

272

Pathology Questions

MCQs

Diagnostic set:
a. Alveolar rhabdomyosarcoma.
b. Alveolar soft part sarcoma.
c. Granular cell tumor.
d. Metastatic melanoma.
e. Metastatic renal cell carcinoma.
f. Paraganglioma.
Which entity is characterized by strong immunoreactivity for epithelial
membrane antigen and cytokeratins?
e. Metastatic renal cell carcinoma.
Which entity is characterized by membrane-bound, rhomboid crystals
that are periodic acid-Schiff (PAS) positive-diastase resistant?
b. Alveolar soft part sarcoma.
Which entity is associated with a t(2;13)(q35;q14) translocation involving
the PAX3 gene?
a. Alveolar rhabdomyosarcoma.
1012. For each of the following questions choose the most likely diagnosis from
the following diagnostic set (an answer may be used once, more than
once, or not at all). (PIP 2007 C)
Diagnostic set:
a. Clear cell carcinoma of the ovary.
b. Dysgerminoma.
c. Endometrioid carcinoma, secretory variant.
d. Metastatic renal cell carcinoma.
e. Yolk sac tumor.
Which of the above entities is most frequently associated with
endometriosis?
a. Clear cell carcinoma of the ovary.
Reserved Copyrights - 2013

273

Pathology Questions

MCQs

Elevated alpha-fetoprotein levels and positive stainingfor AFP within


tumor cells characterize which of the above tumors?
e. Yolk sac tumor.
Which tumor is characterized by a papillary growth pattern with
prominent hyalinized stromal cores?
a. Clear cell carcinoma of the ovary.
1013. For each of the following questions choose the most likely diagnosis from
the following diagnostic set (an answer may be used once, more than
once, or not at all). (PIP 2007 C)
Diagnostic set:
a. Angiosarcoma.
b. Malignant phyllodes tumor.
c. Melanoma.
d. Metaplastic carcinoma.
e. Undifferentiated pleomorphic sarcoma.
The presence of small foci of convevtional infiltrating ductal carcinoma of
ductal carcinoma in situ best supports which diagnosis?
d. Metaplastic carcinoma.
Which tumor displays a leaf-like architecture and a biphasic growth
pattern?
b. Malignant phyllodes tumor.
Which tumor occurs in a chronically lymhadematous arm, following
axillary node dissection?
a. Angiosarcoma.
1014. Which of the following features is required for the diagnosis of anplastic
meningioma? (PIP 2007 D)
Reserved Copyrights - 2013

274

Pathology Questions

MCQs

a. Brain invasion.
b. Geographic necrosis.
c. Greater than or equal to 20 mitotic figures per 10 high-power
fields.
d. Prominent nucleoli.
1015. Which of these meningioma varients is a WHO grade III tumor? (PIP 2007
D)
a. Clear cell.
b. Choroid.
c. Papillary.
d. Secretary.
1016. Which of the following is typically seen in meningiomas? (PIP 2007 D)
a. CD34 positivity.
b. EMA positivity.
c. Pericellular reticulin.
d. Strong GFAP reactivity.
1017. For each of the following questions choose the most likely diagnosis
from the following diagnostic set (an answer may be used once, more
than once, or not at all).
Diagnostic set:
a. Fibrolamellar hepatocellular carcinoma.
b. Focal nodular hyperplasia.
c. Hepatocellular carcinoma, well differentiated.
d. Oncocytic carcinoid tumor.
e. Scirrhous hepatocellular carcinoma.
Hepatocellular lesion in which cytoplasmic pale bodies are most
frequently identified?
a. Fibrolamellar hepatocellular carcinoma.

Reserved Copyrights - 2013

275

Pathology Questions

MCQs

Hepatocellular lesion that is most frequently associated with a central


scar containing large vessels?
b. Focal nodular hyperplasia.
Hepatocellular lesion that lacks a fibrous stromal component?
c. Hepatocellular carcinoma, well differentiated.
1018. For each of the following questions choose the most likely diagnosis from
the following diagnostic set (an answer may be used once, more than
once, or not at all). (PIP 2007 D)
Diagnostic set:
a. Follicular dendritic cell tumor/sarcoma.
b. Interdigitating dendritic cell sarcoma.
c. Large B cell lymphoma.
d. Large cell neurodendritic carcinoma.
e. Malignant melanoma.
f. Thymoma.
Which neoplasm exhibits a paracortical pattern of growth in lymph nodes,
frank

cytologic

atypia,

keratin(-),

S100(+),

HMB-45(-)

immunophenotype, and ultrastrucutrally prominent cell processes


lacking chromosomes?
b. Interdigitating dendritic cell sarcoma.
Which tumor exhibts syncitial aggregates of benign appearingepithelioid
cells admixed with lymphocytes, prominent perivascular spaces, and a
keratin(+), S100(-), synaptophysin(-) immunophenotype?
f. Thymoma.
Which neoplasm typically shows tumor cells in whorls and fascicles
admixed with lymphocytes, expresses CD21, CD35 and clustering (keratin
and

HMB-45

negative)

and

ultrastrucutrally

shows

prominent

desmosome-rich cell processes?


Reserved Copyrights - 2013

276

Pathology Questions

MCQs

a. Follicular dendritic cell tumor/sarcoma.


1019. For each of the following questions choose the most likely diagnosis from
the following diagnostic set (an answer may be used once, more than
once, or not at all). (PIP 2007 D)
Diagnostic set:
a. Follicular lymphoma.
b. Hairy cell leukemia.
c. Mantle cell lymphoma.
d. Small lymphocytic lymphoma.
e. Splenic marginal zone lymphoma.
Which entity typically is CD20(+), CD5(+), CD23(-) and cyclin D1(+)?
c. Mantle cell lymphoma.
Which entity typically forms splenic lakes, is CD103(+) and DBA44(+) and
often results in a dry tap bone marrow?
b. Hairy cell leukemia.
Which entity is typically CD5(-), CD10(-), CD23(-), CD43(-), CD103(-) and
DBA44(-), Bcl2(+) and cyclin D1(-)?
e. Splenic marginal zone lymphoma.
1020. For each of the following questions choose the most likely diagnosis
from the following diagnostic set (an answer may be used once, more
than once, or not at all). (PIP 2007 D)
Diagnostic set:
a. Desmoplastic fibroma.
b. Fibromatosis.
c. Fibrosarcoma.
d. Post-radiation osteosarcoma, fibroblastic type.
e. Reactive/radiation changes.
f. Solitary fibrous tumor.
Reserved Copyrights - 2013

277

Pathology Questions

MCQs

Which vanishingly rare primary tumor of bone or soft tissue classically


displays a herringbone pattern?
c. Fibrosarcoma.
Which tumor is known for its infiltrative borders and high rate of local
reccurence but is without metastatic potential?
b. Fibromatosis.
Which tumor can occur years after radiation therapy and produces
osteoid by its malignant cells?
d. Post-radiation osteosarcoma, fibroblastic type.
1021. For each of the following questions choose the most likely diagnosis from
the following diagnostic set (an answer may be used once, more than
once, or not at all). (PIP 2007 D)
Diagnostic set:
a. Clear cell carcinoma, oxyphilic variant.
b. Hepatoid carcinoma.
c. Hepatoid yolk sac tumor.
d. Steroid cell tumor.
Which ovarian neoplasm may show histologic evidence of bile production
and is immunohistochemically positive for AFP?
b. Hepatoid carcinoma.
Which ovarian neoplasm typically occurs in young women and is
immunohistochemically positive for AFP?
c. Hepatoid yolk sac tumor.
Which ovarian neoplasm is typically positive for inhibin and negative for
AFP?
d. Steroid cell tumor.
Reserved Copyrights - 2013

278

Pathology Questions

MCQs

1022. For each of the following questions choose the most likely diagnosis from
the following diagnostic set (an answer may be used once, more than
once, or not at all). (PIP 2007 D)
Diagnostic set:
a. Embryonal rhabdomyosarcoma.
b. Leiomyosarcoma.
c. Malignant mixed cell Mllerian tumor.
d. Nodular fasciitis.
e. Pleomorphic rhabdomyosarcoma.
Which tumor is the most common subtype of rhabdomyosarcoma in
children and in audlts?
a. Embryonal rhabdomyosarcoma.
Which variant of rhabdomyosarcoma occurs essentially only in adults?
e. Pleomorphic rhabdomyosarcoma.
Which tumor most commn presents in older women as a polypoid intrauterine mass?
c. Malignant mixed cell Mllerian tumor.
1023. For each of the following questions choose the most likely diagnosis from
the following diagnostic set (an answer may be used once, morethan once,
or not at all). (PIP 2007 D)
Diagnostic set:
a. Lymphoma, diffuse large B-cell.
b. Malignant melanoma, metastatic.
c. Multiple myeloma.
d. Plasmacytoma.
e. Rhabdomyosarcoma.

Reserved Copyrights - 2013

279

Pathology Questions

MCQs

This malignant neoplasm can display complex karyotypes but is best


known for translocations involving chromosomes 1, 2, and 13 (t(1;13)
or t(2;13)):
e. Rhabdomyosarcoma.
Patients with this malignant neoplasm can develop additional disorders
due to deposition of abnormal immunoglobulin chain (amyloid) in various
tissues:
c. Multiple myeloma.
Which tumor is the most common neoplasm of bone?
d. Plasmacytoma.
1024. Which lesion is thought to arise from the pancreatic duct system and is
often multifocal? (PIP 2007 D)
a. Intraductal papillary mucinous tumor.
b. Mucinous cystadenoma.
c. Pancreatic acinar cell carcinoma.
d. Pancreatic ductal adenocarcinoma.
e. Pancreatic endocrine carcinoma.
f. Pancreatoblastoma.
g. Solid-pseudopapillary tumor.
1025. Which lesion occurs in young women and may have a pseudo cystic
appearance on gross examination? (PIP 2007 D)
a. Intraductal papillary mucinous tumor.
b. Mucinous cystadenoma.
c. Pancreatic acinar cell carcinoma.
d. Pancreatic ductal adenocarcinoma.
e. Pancreatic endocrine carcinoma.
f. Pancreatoblastoma.
g. Solid-pseudopapillary tumor.

Reserved Copyrights - 2013

280

Pathology Questions

MCQs

1026. For each of the following questions choose the most likely diagnosis from
the following diagnostic set (an answer may be used once, more than
once, or not at all). (PIP 2008 A)
Diagnostic set:
a. Adenovirus pneumonia.
b. Cytomegalovirus pneumonia.
c. Cytomegalovirus pneumonia and Pnemocystis pneumonia.
d. Pnemocystis pneumonia.
e. Pulmonary alveolar proteinosis.
f. Pulmonary edema.
Which entity is characterized by coarse granular intra-alveolar exudates
which are typically PAS positive and may contain observable cholesterol
clefts?
e. Pulmonary alveolar proteinosis.
Which entity has a classic histologic pattern of frothy, vacuolated intraalveolar exudates but may also be associated with diffusealveolar damage
(DAD), interstitial exudates or minimal histologic changes?
d. Pnemocystis pneumonia.
Which entity is characterized by enlarged cells with intranuclear
eosinophilic inclusions and smaller intracytoplasmic inclusions?
b. Cytomegalovirus pneumonia.
1027. Which of the following is most likely to to show false negative staining for
mast cells? (PIP 2008 A)
a. Alcain Blue staining, performed on an air-dried touch prep.
b. Giemsa staining, performed on formalin-fixed paraffin embedded
tissue.
c. Immunohistochemical staining for human tryptase, performed on
anair-dried touch prep.
d. Toluidine Blue staining, performed on an alcohol-fixed touch prep.
Reserved Copyrights - 2013

281

Pathology Questions

MCQs

1028. Which of the following is not considered a diagnostic criterion of systemic


mastocytosis? (PIP 2008 A)
a. Anatomical distribution of mast cell aggregates.
b. Immunohistochemical staining of mast cells.
c. Morphologic features of mast cells.
d. Presence of additional clonal hematopoietic disorder/malignancy.
e. Serum total tryptase level.
1029. Which is the cause of non-cirrhotic portal hypertension?
a. Hepatic veno-occlusive disease.
b. Hereditary hemochromatosis.
c. Sarcoidosis.
d. Schistosomiasis.
e. Systemic mastocytosis.
1030. For each of the following questions choose the most likely diagnosis from
the following diagnostic set (an answer may be used once, more than
once, or not at all). (PIP 2008 A)
Diagnostic set:
a. Angiofollicular hyperplasia (Castleman disease).
b. Follicular lymphoma.
c. Lymphocyte-predominant thymoma.
d. Plasmacytoma.
e. Reactive follicular hyperplasia.
f. Regressing follicular hyperplasia.
Which entity is characterized by positive staining for CD20 and bcl?
b. Follicular lymphoma.
Which entity has a preponderance of CD5 positive lymphoid cells
suggesting an abnormal proliferation of CD5 positive lymphocytes
stimulated by specific lymphokines as its possible histiogenesis?
a. Angiofollicular hyperplasia (Castleman disease).
Reserved Copyrights - 2013

282

Pathology Questions

MCQs

Which entity may have Hassall's corpuscles and large cells positive for
cytokeratins?
c. Lymphocyte-predominant thymoma.
1031. For each of the following questions choose the most likely diagnosis from
the following diagnostic set (an answer may be used once, more than
once, or not at all). (PIP 2008 A)
Diagnostic set:
a. Epithelioid sarcoma.
b. Hodgkin lymphoma.
c. Lymphoma, diffuse large B-cell type.
d. Melanoma.
e. Metastatic renal cell carcinoma.
Which

tumor

characteristically

shows

the

following

immunohistochemical profile: cytokeratin(+), vimentin(+), CD34(+), S100(-), CD45(-)?


a. Epithelioid sarcoma.
Which

tumor

characteristically

shows

the

following

immunohistochemical profile: cytokeratin(+), vimentin(+), S-100(-),


CD45(-), EMA(+), CD34(-)?
e. Metastatic renal cell carcinoma.
Which

tumor

characteristically

shows

the

following

immunohistochemical profile: cytokeratin(-), vimentin(+), S-100(-),


CD45(-), CD15(+), CD30(+)?
b. Hodgkin lymphoma.
1032. For each of the following questions choose the most likely diagnosis from
the following diagnostic set (an answer may be used once, more than
once, or not at all). (PIP 2008 A)
Reserved Copyrights - 2013

283

Pathology Questions

MCQs

Diagnostic set:
a. Atypical carcinoid.
b. Large cell neuroendocrine carcinoma.
c. Small cell carcinoma.
d. Squamous cell carcinoma, poorly differentiated.
e. Typical carcinoid.
Which tumor consistently shows nuclear expression of p63 and is
negative for TTF-1?
d. Squamous cell carcinoma, poorly differentiated.
Which neuroendocrine tumor is defined in the WHO classification as
having 2-10 mitoses/10HPF or necrosis?
a. Atypical carcinoid.
Which tumor is characterized by large cells with prominent nucleoli,
extensive

infarct-like

necrosis,

chromogranin

expression

and

mitoticcounts exceeding 10 per 10 high power fields?


b. Large cell neuroendocrine carcinoma.
1033. For each of the following questions choose the most likely diagnosis from
the following diagnostic set (an answer may be used once, more than
once, or not at all). (PIP 2008 A)
Diagnostic set:
a. Lymphangioma.
b. Metastatic renal cell carcinoma.
c. Microcystic serous cystadenoma.
d. Mucinous cystic neoplasm.
e. Serous cystadenocarcinoma.
Which multicystic tumor is positive for CD31?
a. Lymphangioma.

Reserved Copyrights - 2013

284

Pathology Questions

MCQs

Which tumor is associated with von Hipple-Lindau syndrome, can have


VHL gene mutations, and is a primary pancreatic neoplasm?
c. Microcystic serous cystadenoma.
Which multicystic tumor contains ovarian-type stroma in the cyst walls?
d. Mucinous cystic neoplasm.
1034. For each of the following questions choose the most likely diagnosis from
the following diagnostic set (an answer may be used once, more than
once, or not at all). (PIP 2008 A)
Diagnostic set:
a. Angiosarcoma.
b. Malignant lymphoma.
c. Papillary carcinoma.
d. Riedel thyroiditis.
e. Undifferentiated (anaplastic) carcinoma.
The finding of a sclerotic process involving the thyroid with bland nuclear
features and lacking vascular occlusion by spindle cells is most
compatible with this entity?
d. Riedel thyroiditis.
Which entity shares overlap of immunohistochemical markers and clinical
course with undifferentiated carcinoma and frequently is composed of
epithelioid cells?
a. Angiosarcoma.
Which of the above tumors is characterized by extensive vascular
plugging with malignant spindled cells that shows the majority of which
70% positive staining with p53?
e. Undifferentiated (anaplastic) carcinoma.

Reserved Copyrights - 2013

285

Pathology Questions

MCQs

1035. For each of the following questions choose the most likely diagnosis from
the following diagnostic set (an answer may be used once, more than
once, or not at all). (PIP 2008 A)
Diagnostic set:
a. Atypical adenomatous hyperplasia.
b. Metastatic adenocarcinoma.
c. Pulmonary adenocarcinoma, bronchioalveolar type.
d. Pulmonary adenocarcinoma, fetal type.
e. Pulmonary adenocarcinoma, mixed subtype.
f. Pulmonary adenocarcinoma, mucinous type.
Which malignant lesion is, by definition, non-invasive?
c. Pulmonary adenocarcinoma, bronchioalveolar type.
Which lesion above typically shows glycogen rich glands with
supranuclear and subnuclear vacuoles imparting an endometrioid
appearance?
d. Pulmonary adenocarcinoma, fetal type.
Which histologic pattern of primary adenocarcinoma of the lung is most
common?
e. Pulmonary adenocarcinoma, mixed subtype.
1036. Which of the following is true regarding radiation-induced colitis? (PIP
2008 A)
a. Granulomas may be present.
b. Perivascular hyalinization helps distinguish it from crohn's
disease.
c. Radiation-induced colitis is usally an acute event following
radiation.
d. Stricture formation and fistulae are not characteristic.

Reserved Copyrights - 2013

286

Pathology Questions

MCQs

1037. Which of the following statements regarding Crohn disease is FALSE? (PIP
2008 A)
a. Deep fissuring ulcers and fistula formation are seen in crohn
disease and are not seen in ulcerative colitis.
b. Non-caseating granulomas are seen in 60-90% of mucosal biopsy
samples.
c. The presence of small bowel involvement typically helps
distinguish crohn disease from ulcerative colitis.
d. Transmural lymphoid inflammation is typical of this entity.
1038. Which site represents the commonest location of luminal gastrointestinal
involvement by sarcoidosis? (PIP 2008 A)
a. Esophagus.
b. Large bowel.
c. Small bowel.
d. Stomach.
1039. Which is most typical for null cell adenomas of the pituitary gland? (PIP
2008 B)
a. Brisk mitotic activity.
b. Frequent recurrence.
c. Markedly elevated MIB-1 labeling.
d. Patchy immunoreactivity for leutenizing and follicle stimulating
hormones.
e. Strong diffuse immunostaining for TP53.
1040. The diagnosis of pituitary cracinoma requires: (PIP 2008 B)
a. Cavernous sinus invasion.
b. Elevated MB-1 cell cycle labeling index.
c. Elevated mitotic rate.
d. Metastasis.
e. Microscopic dural invasion.

Reserved Copyrights - 2013

287

Pathology Questions

MCQs

1041. For each of the following questions choose the most likely diagnosis from
the following diagnostic set (an answer may be used once, more than
once, or not at all). (PIP 2008 B)
Diagnostic set:
a. Extranodal marginal zone lymphoma, MALT type.
b. Lymphoid interstitial pneumonia.
c. Mantle cell lymphoma.
d. Nodular lymphoid hyperplasia.
e. Small lymphocytic lmphoma.
Which entity is characterized by a nodular lymphoid infiltrate centered on
bronchioles and sharply demarcated from normal lung, showing a
predominance of interfollicular T cells, polytypic B cells and bcl-2
negative germinal centers?
d. Nodular lymphoid hyperplasia.
Which lymphoid proliferation typically involves the interstitium of the
lung preferentially, and is composed of T lymphocytes withrelatively few
B cells and polytypic plasma cells?
b. Lymphoid interstitial pneumonia.
Which lesion exhibits spread along bronchovascular bundles with
lymphoepithelial lesions, infiltrates interlobular septa and pleura and is
characterized by cytologically by an admixture of small lymphocytes with
few admixed large cells, monocytoid and plasma cells with monotypic
light chain expression, and plasmacytoid cells with Dutcher bodies?
a. Extranodal marginal zone lymphoma, MALT type.
1042. For each of the following questions choose the most likely diagnosis from
the following diagnostic set (an answer may be used once, more than
once, or not at all). (PIP 2008 B)
Diagnostic set:
a. Choroid glioma.
b. Choroid meningioma.
Reserved Copyrights - 2013

288

Pathology Questions

MCQs

c. Chordoma.
d. Mucinous adenocarcinoma, metastatic.
e. Myxoid chondrosarcoma.
Which entity is characterized by a GFAP(-), EMA(+), vimentin(+),
keratin(-), S-100(-) phenotype?
b. Choroid meningioma.
Which entity is characterized by a GFAP(+), EMA(-), keratin(-), S-100(-)
phenotype?
a. Choroid glioma.
Which entity is characterized by abundant mucoid matrix with variable
cellularity and scattered cells showing abundant vacuolated cytoplasm
(physaliferous cells)?
c. Chordoma.
1043. For each of the following questions choose the most likely diagnosisfrom
the following diagnostic set (an answer may be used once, more than
once, or not at all). (PIP 2008 B)
Diagnostic set:
a. Adult granulosa cell tumor.
b. Clear cell carcinoma.
c. Juvenile granulosa cell tumor.
d. Small cell carcinoma, hypercalcemic type.
e. Yolk sac tumor.
Which entity typically occurs in women >30 years of age and is the most
common ovarian tumor associated with estrogenic manifstations?
a. Adult granulosa cell tumor.

Reserved Copyrights - 2013

289

Pathology Questions

MCQs

Which entity typically occurs in young women, and is characterized


histologically by follicle-like spaces and cells with scanty cytoplasm and
has often spread beyond the ovary at the time of presentation?
d. Small cell carcinoma, hypercalcemic type.
Which entity commonly occurs in women <30 years of age, may be
associated with estrogenic manifstations and is positive for inhibin?
c. Juvenile granulosa cell tumor.
1044. For each of the following questions choose the most likely diagnosis from
the following diagnostic set (an answer may be used once, more than
once, or not at all). (PIP 2008 B)
Diagnostic set:
a. Aggressive angiomyxoma.
b. Angiomyofibroblastoma-like tumor of the genital tract (cellular
angiofibroma).
c. Solitary fibrous tumor.
d. Spindle cell lipoma.
Which tumor typically shows a proliferation of bland spindle-shaped cells
with a noticable amount of adipose tissue and thick bundles of brightly
eosinophilic collagen?
d. Spindle cell lipoma.
Which tumor shows a proliferation of bland spindle-shaped cells and
numerous small and medium sized vessels within anedematous to fibrous
stroma?
b. Angiomyofibroblastoma-like tumor of the genital tract (cellular
angiofibroma).
1045. Which tumor shows a proliferation of bland spindle-shaped cells
dispersed

among

keloid-type

collagen

bundles

and

hemangiopericytoma-like vascular pattern?


Reserved Copyrights - 2013

290

Pathology Questions

MCQs

c. Solitary fibrous tumor.


1046. For each of the following questions choose the most likely diagnosis from
the following diagnostic set (an answer may be used once, more than
once, or not at all). (PIP 2008 B)
Diagnostic set:
a. Gastrointestinal stromal tumor.
b. Malignant mesothelioma.
c. Mesothelial hyperplasia.
d. Metastatic colorectal adenocarcinoma.
e. Metastatic pulmonary adenocarcinoma.
Which tumor exhibits CD-117 and CD-34 immunoreactivity and can have
an epithelioid morphology?
a. Gastrointestinal stromal tumor.
Which is a common incidental finding in the peritoneum of women with
chronic salpingitis and endometriosis?
c. Mesothelial hyperplasia.
Nuclear staining for TIF-1 is a common pattern of immunoreactivity for
which tumor?
e. Metastatic pulmonary adenocarcinoma.
1047. Which of the following lesins is LEAST likely encountered in the anterior
mediastinum? (PIP 2008 B)
a. Germ cell tumor.
b. Mediastinal(thymic) large B-cell lymphoma.
c. Schwannoma.
d. Thymic cyst.
e. Thymoma.
1048. Which of the following associations is most correct?
Reserved Copyrights - 2013

291

Pathology Questions

MCQs

a. Myasthenia gravis and thymic follicular hyperplasia.


b. Nodular sclerosis variant of Hodgkin's lymphoma and CD45
expression.
c. Seminomas and elevated serum AFP.
d. Spindle cell thymoma and cytologic atypia.
e. Thymomas and immunophenotypically aberrant T-cells.
1049. For each of the following questions choose the most likely diagnosis from
the following diagnostic set (an answer may be used once, more than
once, or not at all). (PIP 2008 B)
Diagnostic set:
a. Focal nodular hyperplasia.
b. Hepatic adenoma.
c. Hepatocellular carcinoma, fibrolamellar type.
d. Macroregenerative nodule.
e. Nodular regenerative hyperplasia.
Which

non-cirrhotic

condition

may

be

associated

with

portal

hypertension and is often seen in patients with hematologic and


immunologic disorders?
e. Nodular regenerative hyperplasia.
Which tumor is usually seen in young non-cirrhotic patients, contains
cytoplasmic pale bodies and has aggressive behavior but carries a more
favorable prognosis than its related variants occurring in cirrhotic lives?
c. Hepatocellular carcinoma, fibrolamellar type.
Which lesion most often occurs in young women taking oral contraceptive
agents,

and

when

large,

carries

risk

of

life-threatening

hemoperitoneum?
b. Hepatic adenoma.

Reserved Copyrights - 2013

292

Pathology Questions

MCQs

1050. For each of the following questions choose the most likely diagnosis from
the following diagnostic set (an answer may be used once, more than
once, or not at all). (PIP 2008 B)
Diagnostic set:
a. Dermoid-type fibromatosis.
b. Gastrointestinal stromal tumor.
c. Sclerosing mesenteritis.
d. Sclerosing well-differentiated liposarcoma.
e. Solitary fibrous tumor.
Which tumor typically shows nuclear accumulation of -catenin protein
as the result of either APC or -catenin gene mutations?
a. Dermoid-type fibromatosis.
The presence of extensive fat necrosis is most characteristic ofwhich
entity?
c. Sclerosing mesenteritis.
Which tumor contains enlarged hyperchromatic cells within fibroblastic
septa and abnormally configured thich walled blood vessels containing
similar cells?
d. Sclerosing well-differentiated liposarcoma.
1051. Which splenic tumor is found in a subcapsular location and consists of
vascular-type spaces filled with lymphocytes, proteinaceous material, and
sometimes erytherocytes, and generally stains positively for CD31, CD34
and VEGFR3? (PIP 2008 C)
a. Angiosarcoma.
b. Hemangioendothelioma.
c. Hemangioma.
d. Littoral cell angioma.
e. Littoral cell angiosarcoma.
f. Lymphangioma.
Reserved Copyrights - 2013

293

Pathology Questions

MCQs

1052. Which splenic tumor is grossly deep red-purple with random distribution
throughout the spleen, and microscopically reveals small to large spaces
filled largely with erytherocytes, seperated by delicate fibrous septae or
splenic pulp, and lined by bland endothelial cells which generally stain
negatively for VEGFR3 and D2-40? (PIP 2008 C)
a. Angiosarcoma.
b. Hemangioendothelioma.
c. Hemangioma.
d. Littoral cell angioma.
e. Littoral cell angiosarcoma.
f. Lymphangioma.
1053. Which splenic tumor is sponge-like, partially solid mass microscopically
cosisting of complex anastomotic spaces filled with red blood cells and
lined by plump, bizzare endothelial cells with hyaline globules? (PIP 2008
C)
a. Angiosarcoma.
b. Hemangioendothelioma.
c. Hemangioma.
d. Littoral cell angioma.
e. Littoral cell angiosarcoma.
f. Lymphangioma.
1054. Which of the following tumors occurs almost exclusively in women and
is notable for a peri-cystic stromal proliferation termed ovarian-type
stroma? (PIP 2008 C)
a. Intraductal papillary mucinous carcinoma, non-invasive.
b. Mucinous carcinoma arising in an intraductal papillary mucinous
neoplasm.
c. Mucinous cystic neoplasm.
d. Pancreatic ductal adenocarcinoma, usual type.
e. Serous microcystic adenoma.
Reserved Copyrights - 2013

294

Pathology Questions

MCQs

1055. This tumor is associated with mutations in KRAS, p16, TP53 and loss of
DCP4? (PIP 2008 C)
a. Intraductal papillary mucinous carcinoma, non-invasive.
b. Mucinous carcinoma arising in an intraductal papillary mucinous
neoplasm.
c. Mucinous cystic neoplasm.
d. Pancreatic ductal adenocarcinoma, usual type.
e. Serous microcystic adenoma.
1056. Which of the following lesions show nuclear expression of -catenin,
negative staining for S100 and variable positivity for KIT, smooth muscle
actin and desmmin? (PIP 2008 C)
a. Gastrointestinal stromal tumor.
b. Leiomyoma.
c. Mesenteric fibromatosis.
d. Schwannoma.
1057. All of the following typically show KIT positive staining on
immunohistochemistry EXCEPT: (PIP 2008 C)
a. Malignant melanoma.
b. Mastocytosis.
c. Schwannoma.
d. Seminoma.
1058. Which of the following is correct about GISTs? (PIP 2008 C)
a. Epithelioid GIST are more common in the small intestine.
b. Imatinib (Gleevec)therapy is not effective in tumors that are KIT
negative by immunohistochemistry.
c. Immunohistochemical expession of desmin is common.
d. Small intestinal GISTs are more likely to be malignant than gastric
GISTs.

Reserved Copyrights - 2013

295

Pathology Questions

MCQs

1059. Which tumor, after mucoepidermoid carcinoma, is the second most


common malignant salivary gland tumor in children? (PIP 2008 C)
a. Acinic cell carcinoma.
b. Adenocarcinoma, not otherwise specified (NOS).
c. Glandular cell tumor.
d. Metastatic thyroid carcinoma.
e. Normal salivary gland.
1060. Which which tumor will show strong, diffuse S-100 and CD68 reactivity?
(PIP 2008 C)
a. Acinic cell carcinoma.
b. Adenocarcinoma, not otherwise specified (NOS).
c. Grandular cell tumor.
d. Metastatic thyroid carcinoma.
e. Normal salivary gland.
1061. Which of the following structures accounts for the granularity of the
cytoplasm in renal oncocytoma? (PIP 2008 C)
a. Abundant microvilli.
b. Iron pigment.
c. Lysosomes.
d. Mitochondria.
e. Rhomboid crystals.
1062. Which of the following features of chromophobe renal cell carcinoma is
most useful in distinguishing it from oncocytoma? (PIP 2008 C)
a. Absence of vimentin expression.
b. Collecting duct immunophenotype.
c. Diffuse strong cytoplasmic staining pattern with Hale's colloidal
iron.
d. Finely granular cytoplasm.
e. Presence of mitochondrial DNA abnormalities.

Reserved Copyrights - 2013

296

Pathology Questions

MCQs

1063. Which of the following is a recurrent cytogenetic abnormality in


oncocytoma? (PIP 2008 C)
a. Loss of chromosome 3p.
b. Rearrangement of chromosome 11q12-13.
c. Trisomy 12.
d. Trisomy 16.
e. Trisomy 20.
1064. Which immunostains would be most helpful in the differential diagnosis
of lymphoepithelial cysts of the pancreas and pancreatic lymphangioma?
(PIP 2008 C)
a. CD3, CD20, and factor VIII.
b. CD45, CD68, and factor VIII.
c. Factor VIII and HMB-45.
d. Factor VIII and CD3.
e. Factor VIII, CD31 and cytokeratins.
1065. Which of the following statements regarding pancreatic cysts is correct?
(PIP 2008 C)
a. The epithelium in dermoid cysts of the pancreas never shows
evidence of mucinous or sebaceous differentiation.
b. The epithelium in mucinous cystic neoplsm and intraductal
pancreatic mucinous tumor are typically of the transitional type.
c. Like lymphoepithelial cysts of the slivary gland, lymphoepithelial
cyst of the pancreas is associated with HIV disease.
d. Pancreatic pseudocyst is devoid of an epithelial lining.
1066. In lymphoepithelial cyst of the pancreas, all of the following may be seen
EXCEPT: (PIP 2008 C)
a. Clusters of foamy macrophages.
b. Extensive sebaceous differentiation.
c. Foci of aft necrosis.
d. Keratin granulomas.
Reserved Copyrights - 2013

297

Pathology Questions

MCQs

e. Lymphoepithelial nests in the lymphoid tissue.


1067. Which of the following statements regarding paragangliomas is incorrect?
(PIP 2008 C)
a. Mitotic activity, nuclear pleomorphism, and the presence of
necrosis are poorly correlated with clinical behavior.
b. Paragangliomas can be differentiated from most epithelial tumors
based upon their negativity for cytokeratin and EMA, and
positivity for neuroendocrine markers.
c. Paraverteberal paragangliomas are more likely to be functional
than aortic body paragangliomas.
d. The presence of multiple tumors along the vertebrae in the thorax
and abdomen is indicative of metastasis.
1068. Which of the following associations with paraganglioma is incorrect? (PIP
2008 C)
a. Carney syndrome.
b. Carney triad.
c. MEN II.
d. Neurofibromatosis type I (Von Recklinghausen disease.)
e. Von-Hipple Lindau syndrome.
1069. Which primary tumor is characterized by production of both high-grade
neoplastic crtilage and osteoid?
a. Chondroblastic osteosracoma.
b. Chondrosarcoma.
c. Differentiated chondrosarcoma.
d. Echondroma.
e. Synovial chondromatosis.
1070. Which neoplasm is characterized by production of extraosseous lobules of
bland hyaline cartilage with mildly increased cellularity, clonal

Reserved Copyrights - 2013

298

Pathology Questions

MCQs

arrangements of chondrocytes and nesting within a thin connective tissue


membrane? (PIP 2008 C)
a. Chondroblastic osteosracoma.
b. Chondrosarcoma.
c. Differentiated chondrosarcoma.
d. Echondroma.
e. Synovial chondromatosis.
1071. Which bone neoplasm is characterized by juxtaposition of low-grade
neoplastic hyaline cartilage and high-grade pleomorphic sarcoma? (PIP
2008 C)
a. Chondroblastic osteosracoma.
b. Chondrosarcoma.
c. Differentiated chondrosarcoma.
d. Echondroma.
e. Synovial chondromatosis.
1072. Which are the most common systemic, extracranial sites of metastasis in
central nervous system hemangiopericytoma? (PIP 2008 D)
a. Bones, lungs, liver.
b. Heart, lungs.
c. Kidney, lungs.
d. Skin, lungs, adrenals.
e. Spleen, lungs, adrenals.
Which marker shows immunoreactivity in tumor cells in both solitary
fibrous tumor and hemangiopericytoma? (PIP 2008 D)
a. CD31.
b. CD34.
c. Glial fibrillary acidic protein.
d. Keratin 7 (CK7).
e. Inhibin.
Which is true regarding recurrence for this tumor?
Reserved Copyrights - 2013

299

Pathology Questions

MCQs

a. Tumor almost never recurs locally after surgical excision.


b. Recurrences tend to occur within months after surgery.
c. Recurrent tumor(s) are associated with hyperostotic bone lesions.
d. Greater than 80% of tumors occur on long long-term followup.
e. Recurrent tumor(s) are typically heavily calcified.
1073. For each of the following questions choose the most likely diagnosis from
the following diagnostic set (an answer may be used once, more than
once, or not at all). (PIP 2008 D)
Diagnostic set:
a. Chronic myeloid leukemia.
b. Hairy cell leukemia.
c. Large B-cell lymphoma.
d. Mantle cell lymphoma.
e. Small lymphocytic lmphoma/chronic lymphocytic leukemia.
f. Splenic marginal zone lymphoma.
Which of these lymphoma/leukemias characteristically shows a CD20(+),
CD11c(+), CD25(+), CD103(+) immunophenotype?
b. Hairy cell leukemia.
Which hematolymphoid proliferation most commonly shows strong
nuclear expression of cyclin D1?
d. Mantle cell lymphoma.
Which

of

these

lymphoma/leukemias

most

commonly

preferentiallyinvolves the red pulp of the spleen with relative sparing of


the white pulp?
b. Hairy cell leukemia.
1074. For each of the following questions choose the most likely diagnosis from
the following diagnostic set (an answer may be used once, morethan once,
or not at all). (PIP 2008 D)
Reserved Copyrights - 2013

300

Pathology Questions

MCQs

Diagnostic set:
a. Angiomyofibroblastoma.
b. Cellular angiofibroma.
c. Deep (aggressive) angiomyxoma.
d. Mammary-type myofibroblastoma.
e. Spindle cell lipoma/pleomorphic lipoma.
f. Well differentiated liposarcoma.
Which lesion is circumscribed, most often occurs in vulva, is highlighted
by perivascular cuffs of epithelioid desmin-positive cells and is normally
cured with simple excision?
a. Angiomyofibroblastoma.
Which lesion is probably related to spindle cell lipoma includingsharing of
13q and 16q chromosomal abnormalities?
d. Mammary-type myofibroblastoma.
Which poorly circumscribed myxoid lesion is typically large and apt to
recur with incomplete excision?
c. Deep (aggressive) angiomyxoma.
1075. For each of the following questions choose the most likely diagnosis from
the following diagnostic set (an answer may be used once, more than
once, or not at all). (PIP 2008 D)
Diagnostic set:
a. Angiosarcoma.
b. Epithelioid sarcoma.
c. Kaposi sarcoma.
d. Pseudovascular squamous cell carcinoma.
e. Spindle cell hemangioma.
Which tumor is associated with both prior irradiation and long-standing
lymphedema?
a. Angiosarcoma.
Reserved Copyrights - 2013

301

Pathology Questions

MCQs

Loss of INI-1 expression is most characteristic of which lesion?


b. Epithelioid sarcoma.
Which tumor contains an admixture of thick-walled vessels, cavernous
hemangioma-like zones, and spindled zones containing vacuolated
endothelial cells?
e. Spindle cell hemangioma.
1076. For each of the following questions choose the most likely diagnosis from
the following diagnostic set (an answer may be used once, more than
once, or not at all). (PIP 2008 D)
Diagnostic set:
a. Chromophobe renal cell carcinoma.
b. Conventional (clear cell) renal cell carcinoma.
c. Epithelioid angiomyolipoma.
d. Metastatic hepatocellular carcinoma.
e. Renal oncocytoma.
Which entity is characterized by HMB-45(+), MiTF(+), Keratin(-), EMA(-)
phenotype?
c. Epithelioid angiomyolipoma.
Which entity is characterized ultrastructurally by cytoplasmic membrane
bound vesicles and show diffuse granular cytoplasmic staining with Hale's
colloidal iron stain?
a. Chromophobe renal cell carcinoma.
Which entity is characterized by a central scar with a nested growth
pattern and round nuclear contours?
e. Renal oncocytoma.

Reserved Copyrights - 2013

302

Pathology Questions

MCQs

1077. For each of the following questions choose the most likely diagnosis from
the following diagnostic set (an answer may be used once, more than
once, or not at all). (PIP 2008 D)
Diagnostic set:
a. Chronic villitis of unkown etiology.
b. Chronic villitis secondary to cytomegalovirus infection.
c. Chronic villitis secondary to herpes simplex infection.
d. Chronic villitis secondary to syphillis.
e. Chronic villitis secondary to Toxoplasma gondii infection.
Which entity reprsents the most common type of chronic villitis and is
associated with a recurrence risk of up to 20%?
a. Chronic villitis of unkown etiology.
Which entity is characterized by a triad of hypercellular villi, proliferative
fetal vascular changes with vascular oblitreration and onion skinning of
vessels and chronic villitis?
d. Chronic villitis secondary to syphillis.
The prsence of chronic villitis is associated with plasma cells, villous
necrosis and calcifications is most commonly associated with which
entity?
b. Chronic villitis secondary to cytomegalovirus infection.
1078. For each of the following questions choose the most likely diagnosis
from the following diagnostic set (an answer may be used once, more
than once, or not at all). (PIP 2008 D)
Diagnostic set:
a. Angiosarcoma.
b. Hemangioma.
c. Littoral cell angioma.
d. Splenic hamartoma.

Reserved Copyrights - 2013

303

Pathology Questions

MCQs

Which tumor has nuclear atypia, increased mitotic activity and necrosis?
a. Angiosarcoma.
Which tumor is characterized by positivity for CD68, von Willbrand
factor, CD31 and CD21?
c. Littoral cell angioma.
Which tumor is composed of disorganized stroma and vascular channels
of varying width lined by CD8 positive cells?
d. Splenic hamartoma.
1079. For each of the following questions choose the most likely diagnosis from
the following diagnostic set (an answer may be used once, more than
once, or not at all). (PIP 2008 D)
Diagnostic set:
a. Angiosarcoma.
b. Cavernous hemangioma.
c. Epithelioid hemangioendothelioma.
d. Focal nodular hyperplasia.
e. Peliosis hepatis.
Which lesion is the most common benign tumor of the liver?
b. Cavernous hemangioma.
Which lesion is characterized by tumor cells with an epithelioid
appearance and cytoplasmic vacuoles that may contain red blood cells?
c. Epithelioid hemangioendothelioma.
Which lesion is characterized by the presence of a central scar in which
malformed arteries are found?
d. Focal nodular hyperplasia.

Reserved Copyrights - 2013

304

Pathology Questions

MCQs

1080. For each of the following questions choose the most likely diagnosis from
the following diagnostic set (an answer may be used once, more than
once, or not at all). (PIP 2008 D)
Diagnostic set:
a. Extramedullary hematopoiesis.
b. Extramedullary myeloid tumor.
c. Lipoma.
d. Liposarcoma.
e. Myelolipoma.
f. Neuroblastoma.
In which tumor is numerous blasts and myeloid precursors identified?
b. Extramedullary myeloid tumor.
Which tumor is composed of mature adipose tissue admixed with benign
hematopoietic elelments?
e. Myelolipoma.
Which tumor is most often associated with N-myc amplification?
f. Neuroblastoma.
991.

For each of the following questions choose the most likely diagnosis from
the following diagnostic set (an answer may be used once, more than
once, or not at all). (PIP 2009 A)
Diagnostic set:
a. Atypical fibroxanthoma.
b. Merkel cell carcinoma.
c. Metastatic breast carcinoma.
d. Nodular melanoma.
Which tumor is typically composed of highly pleomorphic spindle cells
and large, sometimes multinucleated lipid-laden histocytic cells that

Reserved Copyrights - 2013

305

Pathology Questions
exhibit

MCQs
viment(+),

HMB-45(-),

and

MART-1/Melan-A(-)

immunophenotype?
a. Atypical fibroxanthoma.
992.

Which tumor shows peri-nuclear dot-like positivity for CK20 and is CD56
positive?
b. Merkel cell carcinoma.
Which tumor occasionally occurs in association with basal cell carcinoma
and squamous cell carcinoma of the skin?
b. Merkel cell carcinoma.

993.

Which of the following is not true about acute liver failure due to herpex
simplex virus hepatitis?
a. Can occur in immunocompetent adults.
b. Rapidly progressive course.
c. Marked lymphoplasmacytic inflammation.
d. Extensive necrosis.

994.

Which of the following statements about acetaminophen toxicity is not


correct?
a. It is the most common cause of fulminant hepatic failure in the US.
b. Chronic alcohol use can enhance hepatotoxicity of acetaminophen.
c. Necrosis most often occurs in non-zonal distribution.
d. Histological distinction from ischemic necrosis can be difficult.

995.

All the following are true about Wilson disease except: (PIP 2009 A)
a. Presentation after 50 years of age is uncommon.
b. 24-hour urinary copper is markedly elevated in fulminant disease.
c. Normal ceruloplasmin level rules out the diagnosis.
d. Quantitative copper level from the paraffin block is the most
accurate method of assessing hepatic copper.

Reserved Copyrights - 2013

306

Pathology Questions
996.

MCQs

For each of the following questions choose the most likely diagnosis from
the following diagnostic set (an answer may be used once, more than
once, or not at all). (PIP 2009 A)
Diagnostic set:
a. High grade serous carcinoma.
b. Low grade serous cracinoma.
c. Retiform Sertoli-leydig cell tumor.
d. Serous neoplasm of low malignant proliferation.
Which entity has uniform nuclei with mild to moderate atypia, up to 12
mitoses per 10 high power fields, and destructive stromal invasion?
b. Low-grade serous cracinoma.
Which entity expresses WT-1 and inhibin?
c. Retiform Sertoli-leydig cell tumor.
The prognosis of which entity depends upon the stage and type of
implant?
d. Serous neoplasm of low malignant proliferation.

997.

For each of the following questions choose the most likely diagnosis from
the following diagnostic set (an answer may be used once, more than
once, or not at all). (PIP 2009 A)
Diagnostic set:
a. Clear cell carcinoma.
b. High grade endometrioid carcinoma.
c. Low grade endometrioid carcinoma.
d. Serous carcinoma.
e. Type 2 endometrial carcinoma.
f. Type 1 endometrial carcinoma.
Which endometrial carcinoma is frequently associated with PTEN
suppressor gene mutation and histologically is characterized by a

Reserved Copyrights - 2013

307

Pathology Questions

MCQs

predominance of glandular architecture resembling nomal proliferative


endometrium?
c. Low grade endometrioid carcinoma.
Although type 1 carcinoma is infrequent in patients with HNPCC, which
type 1 tumor has been noted to occur in this subset of patients?
b. High grade endometrioid carcinoma.
Which type of endometrial carcinoma is frequently associated with
HNPCC?
e. Type 2 endometrial carcinoma.
998.

Neuroblastic tumors are most likely to arise from which of the following
anatomic sites? (PIP 2009 A)
a. Adrenal gland.
b. Anterior mediastinum.
c. Optic nerve.
d. Organ of Zuckerkandi.
e. Spinal cord.

999.

Neuroblastic tumors are most often associated with which of the


following genetic abnormalities or syndromes? (PIP 2009 A)
a. Multiple endocrine neoplasia syndrome.
b. MYCN gene amplification.
c. Neurofibromatosis type I.
d. Neurofibromatosis type II.
e. RET oncogene mutation.

1000. Which

morphologic

feature

distinguishes

intermixed

ganglioneuroblastoma from ganglioneuroma? (PIP 2009 A)


a. High mitotic-karyorrhetic index (MKI).
b. Nodular gross morphology.
c. Presesnce of islands of neuropil and differentiating cells.
Reserved Copyrights - 2013

308

Pathology Questions

MCQs

d. Presence of mature ganglion cells.


e. Schwannian stroma >50% of tumor area.
1001. Which of the following lesions is MOST commonly encountered in the
anterior mediastinum? (PIP 2009 A)
a. Neurofibroma.
b. Paraganglioma.
c. Thymoma.
d. Schwannoma.
e. Liposracoma.
1002. Which of the following lesions is not immunoreactive to epithelial
antigens? (PIP 2009 A)
a. Fibrosarcoma.
b. Leiomyosarcoma.
c. Malignant peripheral nerve sheath tumor.
d. Synovial sarcoma.
1003. Which

of

the

following

is

most

typically

c-kit

positive

on

immunohistochemistry? (PIP 2009 A)


a. GIST.
b. Leiomyosracoma.
c. Malignant peripheral nerve sheath tumor.
d. Synovial sarcoma.
1004. The translocation t(X;18) is specific for which tumor?
a. Fibrosarcoma.
b. GIST.
c. Leiomyosarcoma.
d. Synovial sarcoma.

Reserved Copyrights - 2013

309

Pathology Questions

MCQs

1005. For each of the following questions choose the most likely diagnosis from
the following diagnostic set (an answer may be used once, more than
once, or not at all). (PIP 2009 A)
Diagnostic set:
a. Chromophobe renal cell carcinoma.
b. Collecting duct carcinoma.
c. Convential (clear cell) renal cell carcinoma.
d. Papillary renal cell carcinoma.
e. Papillary urothelial carcinoma.
Which entity is peanut lectin(+), Ulex europaeus (+), RCC(-) and CD10(-)?
b. Collecting duct carcinoma.
1006. Which entity characteristically shows karyotype abnormalities trisomy 7,
trisomy 17 and loss of chromosome Y?
d. Papillary renal cell carcinoma.
1007. The most commonly involved anatomic location for gliomatosis cerebri is:
(PIP 2009 B)
a. Basal ganglia.
b. Cerebellar vermis.
c. Cerebral white matter.
d. Hippocampus.
e. Optic nerves.
1008. How is gliomatosis currntly classified?
a. Extensively infiltrative glial neoplasm.
b. Highly reactive inflammatory condition.
c. Low-grade glioma.
d. Neurodegenerative disorder.
e. Uniformly fatal infectious condition.

Reserved Copyrights - 2013

310

Pathology Questions

MCQs

1009. The single key histologic feature that allows accurate diagnosis is which of
the following? (PIP 2009 B)
a. Cytologic atypia.
b. Demyelination.
c. Microglial clusters.
d. Spongiform change.
e. Viral inclusions.
1010. All of the following have been correlated with adverse outcome in myxoid
liposarcomas except: (PIP 2009 B)
a. FUS-CHOP fusion type.
b. Large size.
c. Necrosis.
d. Proliferation index.
e. Round cell differentiation.
1011. Which of the following classic syndromes may be associated with multiple
myxomas? (PIP 2009 B)
a. Angelman syndrome.
b. Cotard's syndrome.
c. Dandy-Walker syndrome.
d. Ellis van-Creveld syndrome.
e. McCune-Albright syndrome.
1012. For each of the following questions choose the most likely diagnosis from
the following diagnostic set (an answer may be used once, more than
once, or not at all). (PIP 2009 B)
Diagnostic set:
a. Angiomyolipoma.
b. Cystic partially differentiated nephroblastoma.
c. Metanephric adenofibroma.
d. Mixed epithelial and stromal tumor.
e. Multilocular cystic renal cell carcinoma.
f. Solitary fibrous tumor.
Reserved Copyrights - 2013

311

Pathology Questions

MCQs

Which tumor shows significant clinical and histologic overlap with cystic
nephroma?
d. Mixed epithelial and stromal tumor.
Which tumor is seen predominantly in children under the age of 2?
b. Cystic partially differentiated nephroblastoma.
Which tumor is characterized by cysts lined by flattened, cuboidal or
hobnail epithelium and septa containing ovarian-like stroma?
d. Mixed epithelial and stromal tumor.
1013. For each of the following questions choose the most likely diagnosis from
the following diagnostic set (an answer may be used once, more than
once, or not at all). (PIP 2009 B)
Diagnostic set:
a. Gastrointestinal stromal tumor.
b. Hodgkin lymphoma.
c. Liposarcoma.
d. Mesenteric fibromatosis.
e. Sclerosing mesenteritis.
The tumor that would consistentlay demonstrate CD117 expression is:
a. Gastrointestinal stromal tumor.
The entity characterized by inflammation and fat necrosis in early phases,
and a predominance of fibrosis as the process matures is:
e. Sclerosing mesenteritis.
A bland fibroblastic proliferation in the mesentery with involvement of
the intestinal muscularis propria would be typical of:
d. Mesenteric fibromatosis.

Reserved Copyrights - 2013

312

Pathology Questions

MCQs

1014. For each of the following questions choose the most likely diagnosis from
the following diagnostic set (an answer may be used once, more than
once, or not at all). (PIP 2009 B)
Diagnostic set:
a. Metastatic mucinous adenocarcinoma (mucinous carcinoma
peritonei), high grade.
b. Metastatic

mucinous

adenocarcinoma

(mucinous

carcinomaperitonei), low grade.


c. Peritoneal mesothelioma.
d. Primary peritoneal serous carcinoma.
Which entity is characterized by diffuse peritoneal involvement by a
tumor comprised of abundant pools of extracellular mucincontaining
identfiable neoplastic epithelium with prominent solid or cribriform
growth with obvious cytologic atypia or the presence of signet ring cells?
a.Metastatic mucinous adenocarcinoma (mucinous carcinoma
peritonei), high grade.
Which tumor may rarely produce abundant hyaluronic acid mimicking
extracellular mucin, but the tumor cells are positive for calretinin, WT-1
and CK5/6?
c. Peritoneal mesothelioma.
Which entity is characterized by diffuse peritoneal involvement by tumor
comprised of abundant pools of extracellular mucincontaining identfiable
neoplastic epithelium with bland cytologicfeatures and cells arranged in
strips or simple glands without a signet ring cell component?
b.Metastatic mucinous adenocarcinoma (mucinous carcinoma
peritonei), low grade.
1015. For each of the following questions choose the most likely diagnosis from
the following diagnostic set (an answer may be used once, more than
once, or not at all). (PIP 2009 B)
Reserved Copyrights - 2013

313

Pathology Questions

MCQs

Diagnostic set:
a. Angiomyolipoma.
b. Benign multicystic peritoneal mesothelioma.
c. Lymphangioma (Lymphatic malformation).
d. Mesenteric cyst.
e. Solid pseudopapillary tumor of pancreas.
Which entity is characterized by a HMB45 (+), EMA (-), keratin (-), S-100
(-) phenotype?
a. Angiomyolipoma.
Whcih potentially malignant tumor is characterized by a grossly cystic
and solid appearance, and positivity for CD10 and beta-catenin?
e. Solid pseudopapillary tumor of pancreas.
Which entity is characterized by collagenous stroma and cyst lined by
keratin (+), calretinin (+) cells?
b. Benign multicystic peritoneal mesothelioma.
1016. Which one of the following is not a helpful feature in distinguishing
benign phyllodes tumor from malignant phyllodes tumor? (PIP 2009 B)
a. High mitotic index.
b. Infiltrative margins.
c. Sarcomatous growth.
d. Squamous differentiation.
e. Stromal overgrowth.
1017. Metaplastic carcinoma can be reliably distinguished from phyllodes
tumor by which one of the following features? (PIP 2009 B)
a. Atypical spindled cells.
b. Cytokeratin positive spindled cells.
c. Epithelial ducts interspersed within spindled cells.
d. High mitotic index.
Reserved Copyrights - 2013

314

Pathology Questions

MCQs

e. Infiltrative margins.
1018. Cartilaginous elements are present within a vulvar mass, with prominent
stromal elements and a leaf-like architecture. Which of the following is
correct?
a. Phyllodes tumors can be present anywhere in the milk-line and
can have heterologous elements such as cartilage.
b. This cannot be phyllodes tumor since these tumors are restricted
to mammary/breast paranchyma only.
c. This is a vulvar teratoma.
d. This may represent a chondroma or chondrosarcoma.
1019. For each of the following questions choose the most likely diagnosis from
the following diagnostic set (an answer may be used once, more than
once, or not at all). (PIP 2009 B)
Diagnostic set:
a. Endometrial stromal nodule.
b. Endometrial stromal sarcoma, low grade.
c. Leiomyosracoma.
d. Undifferentiated uterine sarcoma.
Which tumor is composed of cells which morphologically resemble
proliferative phase endometrial stroma and shows proliferative
infiltration of the myometrium?
b. Endometrial stromal sarcoma, low grade.
Which tumor is composed of cells with marked cellular atypia and
numerous mitoses (including atypical forms) without evidence of
differentiation towards a recognizable uterine component?
d. Undifferentiated uterine sarcoma.
Which tumor is well circumscribed and composed of cells which
morphologically resemble proliferative phase endometrial stroma?
Reserved Copyrights - 2013

315

Pathology Questions

MCQs

a. Endometrial stromal nodule.


1020. For each of the following questions choose the most likely diagnosis from
the following diagnostic set (an answer may be used once, more than
once, or not at all). (PIP 2009 B)
Diagnostic set:
a. Clear cell papillary renal cell carcinoma.
b. Clear cell renal cell carcinoma.
c. Multilocular renal cell carcinoma.
d. Papillary renal cell carcinoma.
e. Renal carcinoma associated with Xp11.2 translocations.
Which tumor is characterized by positive AMACR and cytokeratin 7
immunostainings and trisomy 7 and 17?
d. Papillary renal cell carcinoma.
Which tumor is positive for nuclear TFE3 immunostaining?
e. Renal carcinoma associated with Xp11.2 translocations.
Which tumor is most often associated with VHL gene alteration?
b. Clear cell renal cell carcinoma.
1021. Which of the following is true about the use of HEP PAR 1 in the diagnosis
of hepatocellular carcinoma (HCC)? (PIP 2009 C)
a. Not useful in distinction from renal cell carcinoma.
b. Sensitivity is high (>80%) in poorly differentiated HCC.
c. Sensitivity is low (<50%) in clear cell HCC.
d. Strong expression can occur in metastatic adenocarcinomas from
lung and stomach.
1022. Which of the following immunohistochemical results favors renal cell
carcinoma metastatic to liver over hepatocellular carcinoma? (PIP 2009
C)
Reserved Copyrights - 2013

316

Pathology Questions

MCQs

a. Cytokeratin 7 and 20 negative.


b. Hep par 1 positive.
c. MOC-31 negative.
d. PAX-2 positive.
1023. Which of the following is TRUE about making an immunohistochemical
diagniosis of HCC? (PIP 2009 C)
a. AFP has high sensitivity, but low specifity.
b. Cytoplasmic reactivity with polyclonal CEA is specific for
hepatocellular differentiation.
c. Glypican-3 has lower sensitivity for poorly differentiated HCC than
Hep Par 1.
d. MOC-31 expression is seen in a small minority of HCC.
1024. For each of the following questions choose the most likely diagnosis from
the following diagnostic set (an answer may be used once, morethan once,
or not at all). (PIP 2009 C)
Diagnostic set:
a. Acinar cell carcinoma.
b. Pancreatic endocrine tumor.
c. Pancreatoblastoma.
d. Solid pseudopapillary neoplasm of the pancreas.
e. Well-differentiated ductal adenocarcinoma.
Which of the above entities is most commonly seen in children?
c. Pancreatoblastoma.
Which entity is more common in women?
d. Solid pseudopapillary neoplasm of the pancreas.
Which adult tumor is immunohistochemically positive for pancreatic
enzymes trypsin and chymotrypsin?
a. Acinar cell carcinoma.
Reserved Copyrights - 2013

317

Pathology Questions

MCQs

1025. For each of the following questions choose the most likely diagnosis from
the following diagnostic set (an answer may be used once, more than
once, or not at all). (PIP 2009 C)
Diagnostic set:
a. Endometrioid adenocarcinoma.
b. Endometrioid borderline tumor.
c. Metastatic colorectal adenocarcinoma.
d. Sertoli-Leydig cell tumor.
The median age of patients with this tumor is less than 50 years.
d. Sertoli-Leydig cell tumor.
Which of the tumors is often associated with endometriosis or an
adenofibroma and has either a confluent pattern af at least 5 mm or
infiltration?
a. Endometrioid adenocarcinoma.
Which of the tumors is generally negative for EMA?
d. Sertoli-Leydig cell tumor.
1026. For each of the following questions choose the most likely diagnosis from
the following diagnostic set (an answer may be used once, more than
once, or not at all). (PIP 2009 C)
Diagnostic set:
a. Follicular adenoma.
b. Follicular carcinoma.
c. Papillary thyroid carcinoma, classical type.
d. Papillary thyroid carcinoma, columnar cell type.
e. Papillary thyroid carcinoma, cribriform-morula type.
Which entity has a propensity for hematogenous spread?
b. Follicular carcinoma.
Reserved Copyrights - 2013

318

Pathology Questions

MCQs

Which entity is associated with familial adenomatous polyposis?


e. Papillary thyroid carcinoma, cribriform-morula type.
The most aggressive clinical course is seen with which variant of papillary
thyroid carcinoma?
d. Papillary thyroid carcinoma, columnar cell type.
1027. Which of the following is the most common form of hereditary liver
disease in adults? (PIP 2009 C)
a. Alpha-1 antitrypsin deficiency.
b. Glycogen storage disease type IV.
c. Hereditary hemochromatosis.
d. Hereditary hyperfibroginemia.
e. Wilson disease.
1028. Which of the following ultrastructural abnormalities is characteristic of
alpha-1-antitrypsin deficiency? (PIP 2009 C)
a. Dilated endoplasmic reticulum containing amorphous material.
b. Electron-dense cytoplasmic material with a starburst pattern.
c. Excess cytoplasmic glycogen.
d. Filamentous viral particles.
e. Lysosomal storage material.
1029. Which of the following therapeutic options is most commonly used for
treating liver disease associated with alpha-1-antitrypsin deficiency? (PIP
2009 C)
a. Anti-viral medication.
b. Copper supplementation.
c. Enzyme repalcement therapy.
d. Iron chelation.
e. Liver transplantation.

Reserved Copyrights - 2013

319

Pathology Questions

MCQs

1030. Which of the following B cell lymphomas most commonly involves


predominantly the red pulp of the spleen? (PIP 2009 C)
a. CLL/SLL.
b. DLBCL.
c. Hairy cell leukemia.
d. Hodgkin lymphoma.
e. Mantle cell lymphoma.
f. SMZL.
1031. Which of the following associations is most correct?
a. CD5 expression and SMZL.
b. CLL and nuclear irregularities.
c. CLL/SLL and cyclin D1 overexpression.
d. DLBCL and red blood cell lakes.
e. Mantle cell lymphoma and t(11;14).
1032. For each of the following questions choose the most likely diagnosis from
the following diagnostic set (an answer may be used once, more than
once, or not at all). (PIP 2009 C)
Diagnostic set:
a. Acinar cell carcinoma.
b. Pancreatic endocrine neoplasm.
c. Pancreatoblastoma.
d. Poorly differentiated pancreatic adenocarcinoma.
e. Solid pseudopapillary neoplasm.
This asymptomatic tumor frequently occurs as a pancreatic tail mass in
young women:
e. Solid pseudopapillary neoplasm.
CD56 is often considered a neuroendocrine marker. In addition to
pancreatic endocrine tumor, this tumor frequently shows strong CD56
immunoreactivity:
Reserved Copyrights - 2013

320

Pathology Questions

MCQs

e. Solid pseudopapillary neoplasm.


The squamoid corpuscle is a characteristic feature of this tumor:
c. Pancreatoblastoma.
Which

of

the

following

is

true

regarding

the

diagnosis

of

angiomyolipoma? (PIP 2009 C)


a. A non-infiltrative pattern of growth helps distinguish AML from
renal cell carcinoma.
b. Splenic and lymph node deposits of AML should always be
considered metastasis from either renal or hepatic primary
tumors.
c. The polygonal-shaped smooth muscle cells in hepatic AML can be
mistaken for hepatocellular carcinoma.
d. The presence of cells resembling lipoblasts help distinguish AML
from liposarcoma.
1033. Which of the following statements regarding angiomyolipoma is FALSE?
(PIP 2009 C)
a. Approximately half of all cases aof AML are associated with
tuberous sclerosis.
b. Atypia in epithelioid variants of AML is often confused with
leiomyosarcoma.
c. Most cases of AML in the liver are associated with tuberous
sclerosis.
d. The presence of bilateral AMLs is highly suggestive of tuberous
sclerosis.
1034. Which pair of immunostains is MOST likely to help differentiate a fatpredominant

angiomyolipoma

from

lipoma

or

well-

differentiatedliposracoma? (PIP 2009 C)


a. C-kit and Melan-A.
b. Desmin and S-100.
Reserved Copyrights - 2013

321

Pathology Questions

MCQs

c. EMA and MITF.


d. S-100 and HMB-45.
1035. Which of the following features favors radiation colitis over ulcerative
colitis? (PIP 2009 C)
a. Cryptitis.
b. Crypt architectural distortion.
c. Lamina propria fibrosis.
d. Perivascular hyalinization.
1036. Which of the following is a feature common to both active ulcerative
colitis and crohn disease? (PIP 2009 C)
a. Cryptitis and crypt abscess.
b. Discontinuous involvement of the colon.
c. Transmural inflammation.
d. Well-formed epithelioid granulomas.
1037. Creeping fat is a feature of which of the following? (PIP 2009 C)
a. Crohn disease.
b. Infectious colitis.
c. Radiation colitis.
d. Ulcerative colitis.
1038. For each of the following questions choose the most likely diagnosis from
the following diagnostic set (an answer may be used once, more than
once, or not at all). (PIP 2009 C)
Diagnostic set:
a. Desmoid tumor.
b. Low grade fibromyxoid sarcoma.
c. Malignant peripheral nerve sheath tumor.
d. Myxofibrosarcoma.
e. Myxoid liposarcoma.
f. Schwannoma.
Reserved Copyrights - 2013

322

Pathology Questions

MCQs

g. Solitary fibrous tumor.


Whcih tumor exhibits consistent nuclear positivity for beta catenin?
a. Desmoid tumor.
Which tumor lacks the alternating areas of myxoid and fibrous areas but
is consistently positive for CD34?
g. Solitary fibrous tumor.
Which tumor is characterized by alternating areas of fibrous and myxoid
zones, curvilinear blood vessels, alterations in FUS gene and absence of
lipoblasts?
b. Low grade fibromyxoid sarcoma.
1039. For each of the following questions choose the most likely diagnosis from
the following diagnostic set (an answer may be used once, more than
once, or not at all). (PIP 2010 A)
Diagnostic set:
a. Kaposi sarcoma.
b. Leiomyosarcoma.
c. Lymphangioleiomyomatosis.
d. Peritoneal leiomyomatosis.
e. Vascular transformation of sinuses (lymph node).
Which lesion is characterized by positive staining with smooth muscle
markers and HMB-45?
c. Lymphangioleiomyomatosis.
Which lesion is characterized by mutations of the tuberous sclerosis
genes TSC-1 or TSC-2?
c. Lymphangioleiomyomatosis.

Reserved Copyrights - 2013

323

Pathology Questions

MCQs

Which lesion may involve the lymph nodes or viscera, is associated with
HHV-8, is positive for endothelial markers and is negative for HMB-45?
a. Kaposi sarcoma.
1040. For each of the following questions choose the most likely diagnosis from
the following diagnostic set (an answer may be used once, more than
once, or not at all). (PIP 2010 A)
Diagnostic set:
a. Crohn disease.
b. Enteritis cystica profunda.
c. Intestinal lymphangiectasia.
d. Pneumatosis (cystoides) intestinalis related to ischemic bowel
disease.
e. Pneumatosis (cystoides) intestinalis related to progressive
systemic sclerosis.
Which entity is characterized by multiple cysts in the bowel wall that are
often lined by histiocytes and giant cells with associated acute mucosal
necrosis?
d.Pneumatosis (cystoides) intestinalis related to ischemic bowel
disease.
Which entity is characterized by cyst-like spaces that contain foamy
macrophages and are lined by endothelial cells?
c. Intestinal lymphangiectasia.
Which entity is associated with mucosal polyps and submucosal cysts that
are lined by benign intestinal epithelium?
b. Enteritis cystica profunda.
1041. Which of the following is true regarding myxoid round cell liposarcoma?
(PIP 2010 A)
a. It is associated with mutations in the KIT gene on chromosome 4.
Reserved Copyrights - 2013

324

Pathology Questions

MCQs

b. It shares the same chromosomal translocation as Ewing


sarcoma,t(11;22).
c. The gastrointestinal tract is a typical site for this tumor.
d. The proportion of round cell component has prognostic
significance.
e. It has a typical curvilinear vascular pattern with pericystic
investment.
1042. Which of the following is/are typical histologic features of myxoid/round
cell liposarcoma? (PIP 2010 A)
a. Branching thin blood vessels ('chicken wire' pattern).
b. Primitive non-lipogenic mesenchymal cells.
c. Prominent myxoid sarcoma.
d. Signet ring lipoblasts.
e. All of the above.
1043. Which of the following is NOT a typical site for metastasis from
myxoid/round cell liposarcoma?
a. Brain.
b. Liver.
c. Lung.
d. Lymph nodes.
e. Soft tissue.
1044. Which of the following features strongly favors crohn's colitis over
ulcerative colitis?
a. Chronic inflammation in biopsies from upper gastrointestinal
tract.
b. Diffuse colonic involvement.
c. Perianal disease.
d. Presence of primary sclerosing colangitis.

Reserved Copyrights - 2013

325

Pathology Questions

MCQs

1045. Which of the following infections can closely mimic crohn's disease by
showing ileocolonic involvement, fissuring ulcers, granulomas and
transmural inflammation?
a. Campylobacter.
b. Cytomegalovirus.
c. Entamoeba.
d. Spirochetosis.
e. Yersinia.
1046. Which

of

the

following

features

favors

crohn's

disease

over

Mycobacterium tuberculosis infection?


a. Granulomas in draining lymph nodes.
b. Ileocolonic involvement.
c. Large andd confluent granulomas.
d. Longitudinal ulcers and cobblestoning.
e. Prominent necrosis.
1047. For each of the following questions choose the most likely diagnosis from
the following diagnostic set (an answer may be used once, more than
once, or not at all). (PIP 2010 A)
Diagnostic set:
a. Aquired cystic disease-associated renal cell carcinoma.
b. Chromophobe renal cell carcinoma.
c. Clear cell renal cell carcinoma.
d. Oncocytoma.
e. Renal carcinoma associated with Xp 11.2 translocation.
Which tumor is characterized by diffuse positivity for cytokeratin 7 and
Hale's colloidal iron stain?
b. Chromophobe renal cell carcinoma.
Which tumor is positive for nuclear TEF3 immunostaining?
e. Renal carcinoma associated with Xp 11.2 translocation.
Reserved Copyrights - 2013

326

Pathology Questions

MCQs

Which tumor is characterized by the presence of calcium oxalate crystal


deposition?
a. Aquired cystic disease-associated renal cell carcinoma.
1048. Which of the following is correct about well-differentiated pancreatic
endocrine tumors? (PIP 2010 A)
a. Most insulinomas are malignant.
b. The distiction between low- and high-grade neuroendocrine
carcinoma is based on vascular and perinueral invasion.
c. Tumors lacking nuclear atypia can show lymph node metastasis.
d. Tumors larger than 2.0 cm are cosidered malignant.
1049. Which of the following immunohistochemichal staining results is
characteristic of solid pseudopapillary tumor? (PIP 2010 A)
a. Cytoplasmic staining with chromogranin.
b. Membrane staining with beta-ctenin.
c. Membrane staining with E-cadherin.
d. Nuclear staining with progesterone receptor.
1050. Which of the following immunohistochemichal staining results is correct
about the diagnosis of pancreatic tumor? (PIP 2010 A)
a. Chromogranin expression is characteristic of solid pseudopapillary
tumors.
b. Loss of membrane E-cadherin staining is characteristic of acinar
cell carcinoma.
c. Nuclear accumulation of beta-catenin is characteristic of
neuroendocrine tumors.
d. Trypsin expression is characteristic of pancreatoblastoma.
1051. For each of the following questions choose the most likely diagnosis from
the following diagnostic set (an answer may be used once, more than
once, or not at all). (PIP 2010 A)
Reserved Copyrights - 2013

327

Pathology Questions

MCQs

Diagnostic set:
a. Clear cell carcinoma of the ovary.
b. Leydig cell tumor.
c. Metastatic renal cell carcinoma.
d. Sertoli cell tumor.
e. Steroid cell tumor, not otherwise specified.
Which ovarian neoplasm characteristically contains Reinke crystals?
b. Leydig cell tumor.
Which ovarian neoplasm is typically composed of well formed hollow or
solid tubules lined by cuboidal to columnar cells with moderate to
abundant eosinophilic or vacuolated cytoplasmic and round to oval
regular nuclei with nuclear grooves?
d. Sertoli cell tumor.
Which ovarian neoplasm is typically positive for cytokeratin 7 but
negative for inhibin?
a. Clear cell carcinoma of the ovary.
1052. For each of the following questions choose the most likely diagnosis from
the following diagnostic set (an answer may be used once, more than
once, or not at all). (PIP 2010 A)
Diagnostic set:
a. Angiomyolipoma.
b. Leiomyosarcoma.
c. Renal cell carcinoma with sarcomatoid component.
d. Renal cell carcinoma, unclassified.
e. Sarcomatoid urothelial carcinoma.
A malignant spindle cell tumor of the kidney that contains an in situ
epithelial component in the renal pelvis is best classified as?
e. Sarcomatoid urothelial carcinoma.
Reserved Copyrights - 2013

328

Pathology Questions

MCQs

Which of the above is associated wiyh malformed vascular channels and is


positive for HMB-45?
a. Angiomyolipoma.
1053. A malignant spindle cell neoplasm with no associated epithelial
component and a CD10-/ smooth muscle actin + immunoprofile most
likely represents which entity in the diagnostic set?
b. Leiomyosarcoma.
1054. For each of the following questions choose the most likely diagnosis from
the following diagnostic set (an answer may be used once, more than
once, or not at all). (PIP 2010 B)
Diagnostic set:
a. Acute emphyseatous cholecystitis.
b. Cholesterelosis.
c. Gaucher disease.
d. Signet ring cell carcinoma.
e. Xanthogranulomatous cholecystitis.
f. Xanthoma.
Which entity is characterized by isolated increased lipid uptake/storage
in macrophages in the lamina propria of the gallbladder?
b. Cholesterelosis.
Which entity is characterized by the accumulation of glucocerebroside in
cells including tissue macrophages, secondary to deficiency of beta
glucosidase?
c. Gaucher disease.
Which condition is characterized by accumulation of lipofuscin (ceroid)
granules within macrophages?
e. Xanthogranulomatous cholecystitis.

Reserved Copyrights - 2013

329

Pathology Questions

MCQs

1055. Which of the following features is required for the diagnosis of anaplasia
in Wilm's tumor? (PIP 2010 B)
a. Abnormal multipolar mitotic figures.
b. Marked nuclear enlargement (nuclear size 3x that of surrounding
tumor cells).
c. Nuclear hyperchromasia.
d. Any one of the above features.
e. All of the above.
1056. Which of the following renal tumor predisposition syndromes
ischaracterized by gonadal dysgenesis and nephropathy due to mesangial
sclerosis? (PIP 2010 B)
a. Beckwith-Widermann syndrome.
b. Hippel-Lindau syndrome.
c. Denys-Drash syndrome.
d. Simpson-Golabi-Behmel syndrome.
e. WAGR syndrome.
1057. This pediatric renal tumor showed pre-operative rupture of the renal
capsule and extension of tumor into perirenal fat. Imaging studies showed
no evidence of the tumor in the contralateral kidney and no evidence of
metastatic tumor outside of the abdomen and pelvis. What is the
pathologic stage of this tumor?
a. Stage I.
b. Stage II.
c. Stage III.
d. Stage IV.
e. Stage V.
1058. Which tumor is typically characterized by comedo-type necrosis?
a. Adenoid cystic carcinoma.
b. Adenosquamous carcinoma.
c. Basaloid squamous cell carcinoma.
Reserved Copyrights - 2013

330

Pathology Questions

MCQs

d. Neuroendocrine carcinoma.
1059. Which tumor is characterized by the presence of myoepithelial
component?
a. Adenoid cystic carcinoma.
b. Adenosquamous carcinoma.
c. Basaloid squamous cell carcinoma.
d. Neuroendocrine carcinoma.
1060. For each of the following questions choose the most likelydiagnosis from
the following diagnostic set (an answer may be used once, more than
once, or not at all)
Diagnostic set:
a. Angiosarcoma.
b. Fibroma.
c. Metastatic neoplasm.
d. Myxoma.
e. Papillary fibroelastoma.
The most common cardiac tumor is:
c. Metastatic neoplasm.
The most common benign primary cardiac tumor is:
d. Myxoma.
A primary malignant cardiac tumor that usually occurs on the right side of
the heart is:
a. Angiosarcoma.
1061. For each of the following questions choose the most likely diagnosis from
the following diagnostic set (an answer may be used once, more than
once, or not at all). (PIP 2010 B)
Diagnostic set:
Reserved Copyrights - 2013

331

Pathology Questions

MCQs

a. Extramedullary hematopoiesis.
b. Gaucher disease.
c. Hairy cell leukemia.
d. Large B-cell lymphoma.
e. Splenic marginal zone lymphoma.
Which of the above coditions characteristically involves the red pulp?
c. Hairy cell leukemia.
Which of these disorders will show tartrate-resistant acid phosphatase
(TRAP) positivity within peripheral blood cells?
c. Hairy cell leukemia.
Which of these disorders characteristically shows bi-polar prohections in
peripheral blood?
e. Splenic marginal zone lymphoma.
1062. Which of the following is true regarding the diagnosis of AGCT? (PIP 2010
B)
a. Nuclear grooves are a specific feature limited to AGCT and never
found in cellular fibromas.
b. Reticulin staining demonstrating distinct fibers surrounding large
groups of cells helps distinguish AGCT from cellular fibroma.
c. There is often a fibromtous background in AGCT, and at times the
granulosa cell can be a minor component, making up less then
10% of the tumor.
d. A positive pankeratin immunohistochemichal stain always
distinguishes undefferintiated carcinoma from adult granulosa cell
tumor.
1063. Which immunohistochemichal staining problem is compatible with an
AGCT? (PIP 2010 B)
a. Calretinin(+), WT1(-), Cytokeratin 7(+), CD99 (+).
Reserved Copyrights - 2013

332

Pathology Questions

MCQs

b. Calretinin(+), WT1(+), Cytokeratin 7(+), CD99 (+).


c. Pankeratin(+), EMA(-), Calretinin(+), WT1(+).
d. Pankeratin(+), EMA(+), Calretinin(+), WT1(+).
1064. Which of the following features is an important prognostic factor for
AGCT? (PIP 2010 B)
a. Absence of Call-Exner bodies.
b. Bizzare nuclei.
c. Luetinized cells.
d. Stage.
1065. Pathologic stage of mammary carcinoma is based upon: (PIP 2010 B)
a. Intensity of estrogen receptor expression.
b. Maximum dimension of the entire tumor including the in-situ and
invasive components.
c. Maximum dimension of the invasive component only.
d. Presence or absence of lymphvascular invasion.
e. Presence or absence of 'positive' linked margins.
1066. In staging multifocal invasive mammary carcinoma: (PIP 2010 B)
a. Multifocal carcinomas are not pathologically staged by convention.
b. Multifocal carcinomas are not staged if DCIS is more than 50% of
the tumor.
c. Only ER positive portions of multifocal carcinomas are measured.
d. Stage is based upon the largest linear dimension of the largest
discrete focus of carcinoma.
e. The largest linear dimensions of each focus are added.
1067. A poorly differentiated breast mass is CK7(-), AE/AE3(-), S-100(+),
HMB45(+), and Melan A (+). A likely diagnosis is: (PIP 2010 B)
a. Duct carcinoma in situ.
b. Infiltrating duct carcinoma.
c. Melanoma.
Reserved Copyrights - 2013

333

Pathology Questions

MCQs

d. Metastatic ovarian carcinoma.


e. Microglandular hyperplasia of the breast.
1068. Which of the following immunohistochemichal stains is most specificfor
Kaposi sarcoma?
a. CD31.
b. Factor VII-related antigen.
c. FLI-1.
d. LANA-1.
e. VEGFR-3.
1069. Which one of the following markers is expressed by cystic tumor of the
arterioventricular region?
a. B72.3 (tumor-associated glycoprotein 72).
b. Calretinin.
c. factor VIII-related antigen.
d. LANA-1.
e. Thrombodulin.
1070. Which of the following antigens is most specific for lymphatic endothelial
cells? (PIP 2010 B)
a. CD31.
b. D2-40.
c. FLI-1.
d. LANA-1.
e. Ulex-lectin.
1071. Which tumor occurs most commonly in the long bones of children or
young adults, and is characterized by a spindle cell sarcomatous
component with osteoid formaation intimately admixed with a
cartiliginous component of equivelant grade? (PIP 2010 C)
a. Chondromyxoid fibroma.
b. Differentiated chondrosarcoma.
Reserved Copyrights - 2013

334

Pathology Questions

MCQs

c. Meningioma.
d. Mesenchymal chondrosarcoma.
e. Osteosarcoma, chondroblastic type.
1072. Which tumor occurs most commonly in the pelvis or femur of older
individuals and consists of a low grade cartiliginous component which
abruptly transitions to a high-grade spindle cell sarcoma that may contain
osteoid? (PIP 2010 C)
a. Chondromyxoid fibroma.
b. Differentiated chondrosarcoma.
c. Meningioma.
d. Mesenchymal chondrosarcoma.
e. Osteosarcoma, chondroblastic type.
1073. Which tumor occurs primarily in the second or third decade, most
commonly involves the jaw but may arise in the soft tissue or meninges,
and consists of lobules of well-differentiated cartilage in a background of
undifferentiated small blue cells? (PIP 2010 C)
a. Chondromyxoid fibroma.
b. Differentiated chondrosarcoma.
c. Meningioma.
d. Mesenchymal chondrosarcoma.
e. Osteosarcoma, chondroblastic type.
1074. The invasive lobular carcinoma variant with poor prognosis: (PIP 2010 C)
a. Alveolar.
b. Classical.
c. Histiocytoid.
d. Pleomorphic.
e. Solid.
1075. The most typical immunohistochemichal profile for classicalinvasive
lobular carcinoma is: (PIP 2010 C)
Reserved Copyrights - 2013

335

Pathology Questions

MCQs

a. ER(+), PR(+), HER2(+), E-cadherin(-).


b. ER(+), PR(+), HER2(-), E-cadherin(-).
c. ER(+), PR(+), HER2(+), E-cadherin(-), GCDF-15(+).
d. ER(+), PR(-), HER2(+), E-cadherin(+).
e. ER(-), PR(-), HER2(-), E-cadherin(-), GCDF-15(-).
1076. Which of the following statements about the prognosis of invasive lobular
carcinoma is true? (PIP 2010 C)
a. All of the invasive lobular variants have essentially the same
prognosis.
b. E-cadherin-negative lobular variants have a worse prognosis than
E-cadherin-positive tumors.
c. In lobular tumors, only the nuclear grade has prognostic
significance.
d. The Nottingham combined histologic grade has prognostic
significance in invasive lobular tumors.
e. With long-term follow-up (10 years or more), invasive lobular
tumors show a better overall survival than invasive ductal tumors.
1077. Which adenocarcinoma within an ovary is most commonly composed of
confluent necrosis and has an immunoprofile of CK7(-), CK20(+), ER(-),
PR(-)? (PIP 2010 C)
a. Carcinoid tumor.
b. Endometrioid adenocarcinoma.
c. Endometrioid adenofibroma.
d. Metastatic colorectal adenocarcinoma.
e. Sertoli-Leydig cell tumor.
1078. Which tumor generally stains positive for calretinin? (PIP 2010 C)
a. Carcinoid tumor.
b. Endometrioid adenocarcinoma.
c. Endometrioid adenofibroma.
d. Metastatic colorectal adenocarcinoma.
Reserved Copyrights - 2013

336

Pathology Questions

MCQs

e. Sertoli-Leydig cell tumor.


1079. Which tumor exhibits similar molecular abnormalities as those seen in
the endometrial cavity (ie: PTEN, beta ctaerin and KRAS mutations, and
microsatellite instability)? (PIP 2010 C)
a. Carcinoid tumor.
b. Endometrioid adenocarcinoma.
c. Endometrioid adenofibroma.
d. Metastatic colorectal adenocarcinoma.
e. Sertoli-Leydig cell tumor.
1080. Which entity represents the most common type of chronic villitis
encountered in daily practice? (PIP 2010 C)
a. Chronic villitis of unkown etiology.
b. Chronic villitis secondary to cytomegalovirus infection.
c. Chronic villitis secondary to herpes simplex infection.
d. Chronic villitis secondary to syphilis.
e. Chronic villitis secondary to Toxoplasma gondii infection.
1081. Chronic villitis associated with plama cells, villous necrosis and
calcifications is most commonly associated with which entity? (PIP 2010
C)
a. Chronic villitis of unkown etiology.
b. Chronic villitis secondary to cytomegalovirus infection.
c. Chronic villitis secondary to herpes simplex infection.
d. Chronic villitis secondary to syphilis.
e. Chronic villitis secondary to Toxoplasma gondii infection.
1082. Which type of chronic villitis includes hypercellular villi, proliferative
fetal vascular changes with vascular obliteration and onion skinning' of
vessels? (PIP 2010 C)
a. Chronic villitis of unkown etiology.
b. Chronic villitis secondary to cytomegalovirus infection.
Reserved Copyrights - 2013

337

Pathology Questions

MCQs

c. Chronic villitis secondary to herpes simplex infection.


d. Chronic villitis secondary to syphilis.
e. Chronic villitis secondary to Toxoplasma gondii infection.
1083. Germline mutations in the STK11/LKB1 gene have been recognized as a
cause of: (PIP 2010 C)
a. Bannayan-Riley-Ruvalcaba syndrome.
b. Cowden syndrome.
c. Familial adenomatous polyposis.
d. Juvenile polyposis.
e. Peutz-Jeghers syndrome.
1084. The association of adenomatous polyposis with osteomas and soft tissue
fibromas is called: (PIP 2010 C)
a. Cowden syndrome.
b. Gardner syndrome.
c. Maffucci syndrome.
d. Ollier syndrome.
e. Turcot syndrome.
1085. Classically, the lifetime risk of colorectal carcinoma in patients with APC
gene mutation-associated polyposis is: (PIP 2010 C)
a. 15%.
b. 30%.
c. 50%.
d. 85%.
e. 100%.
1086. Which of the following features is more characteristic of fibrolamellar
carcinoma compared to conventional hepatocellular carcinoma? (PIP
2010 C)
a. Prominent nucleoli.
b. Mallory hyaline.
c. Pale bodies.
Reserved Copyrights - 2013

338

Pathology Questions

MCQs

d. Cytoplasmic fat.
1087. Which of the following correctly describes the clinical impact of
distinguishing fibrolamellar carcinoma and conventionalhepatocellular
carcinoma? (PIP 2010 C)
a. Less aggressive surgical approach in fibrolamellar carcinoma.
b. More aggressive approach in lymph node dissection in
fibrolamellar cracinoma.
c. Highly effective radiation therapy in fibrolamellar carcinoma.
d. Highly effective chemotherapy in fibrolamellar carcinoma.
1088. Which of the following argues against a diagnosis of fibrolamellar
carcinoma? (PIP 2010 C)
a. Presence of mucin.
b. Diffuse CK7 expression.
c. Diffuse glypican-3 expression.
d. Diffuse chromogranin expression.
1089. Which gene is most often hypermethylated in colonic medullary
carcinomas? (PIP 2010 C)
a. MLH1.
b. MSH2.
c. MSH6.
d. PMS2.
1090. Which of the following is required for a diagnosis of colonic high-grade
large cell neuroendocrine carcinoma? (PIP 2010 C)
a. Demonstration

of

cytokeratin

expression

by

differentiation

by

immunohistochemistry.
b. Demonstration

of

neuroendocrine

immunohistochemistry.
c. Focal gland formation.
d. Organoid, insular, trabecular, or sheet-like growth pattern.
Reserved Copyrights - 2013

339

Pathology Questions

MCQs

1091. Which is not a characteristic feature of colon cancers that are


microsatellite unstable? (PIP 2010 C)
a. Crohn-like reaction.
b. Intratumoral lymphocytes.
c. Luminal or dirty necrosis.
d. Mucinous phenotype.
1092. Which tumor is characterized by diffuse cytokeratin positivity, positive
CD10 and negative HepPar 1 staining? (PIP 2010 C)
a. Adrenal cortical adenoma.
b. Adrinal cortical carcinoma.
c. Clear cell renal cell carcinoma.
d. Hepatocellular carcinoma.
e. Pheochromocytoma.
1093. Which tumor is characterized by sustentacular pattern of S-100 protein
immunostaining? (PIP 2010 C)
a. Adrenal cortical adenoma.
b. Adrinal cortical carcinoma.
c. Clear cell renal cell carcinoma.
d. Hepatocellular carcinoma.
e. Pheochromocytoma.
1094. Which

malignant

neoplasm

is

characterized

by

the

following

immunoprofile: Melan A(+), vimentin(+), inhibin(+), and Calretinin(+)?


(PIP 2010 C)
a. Adrenal cortical adenoma.
b. Adrinal cortical carcinoma.
c. Clear cell renal cell carcinoma.
d. Hepatocellular carcinoma.
e. Pheochromocytoma.

Reserved Copyrights - 2013

340

Pathology Questions

MCQs

1095. Ductal plate malformation is a bile duct abnormality which is afeature of:
(PIP 2010 D)
a. Alagille syndrome.
b. Congenital hepatic fibrosis.
c. Cystic fibrosis.
d. Extrahepatic biliary atresia.
e. All of the above.
1096. Hepatorenal fibrocystic diseases include all of the following except: (PIP
2010 D)
a. Alagille syndrome.
b. Autosomal dominant polycystic kidney disease.
c. Autosomal recessive polycystic kidney disease.
d. Meckel syndrome.
e. Nephrolithiasis.
1097. A typical feature of extrahepatic biliary atresia which is helpful in
distinguishing it from congenital hepatic fibrosis is: (PIP 2010 D)
a. Association with ciliopathies.
b. Reactive ductal proliferation.
c. Ductal plate malformation.
d. Hepatocellular swelling and giant cell transformation.
e. PKHD1 gene mutation.
1098. Which entity is characterized by hepatosplenomagaly, neurological
dysfunction, skeletal disorders, and enlarged macrophages infiltrating the
liver, spleen brain and bone marrow which have a fibrillary tissue-paper
appearance to the cytoplasm? (PIP 2010 D)
a. Gaucher disease.
b. Malakoplakia.
c. Niemann-pick disease.
d. Rosai-Dorfman disease.
e. Tay-Sachs disease.
Reserved Copyrights - 2013

341

Pathology Questions

MCQs

1099. Which entity is characterized by extensive accumulation of S-100 proteinpositive histiocytes within lymph nodes? (PIP 2010 D)
a. Gaucher disease.
b. Malakoplakia.
c. Niemann-pick disease.
d. Rosai-Dorfman disease.
e. Tay-Sachs disease.
1100. Which entity is diagnosed using a biochemichal test for sphingomyelinase
activity? (PIP 2010 D)
a. Gaucher disease.
b. Malakoplakia.
c. Niemann-pick disease.
d. Rosai-Dorfman disease.
e. Tay-Sachs disease.
1101. Which of the following is true regarding tumoral calcinosis? (PIP 2010 D)
a. Associated with abnormalities in calcium metabolism.
b. Lesions with identical histologic features may be seen in patients
with chronic renal disease and secondary hyperparathyroidism.
c. Patients most often present with a solitary lesion.
d. Radiographs

typically

demonstrate

associated

osteoporotic

changes or other bony abnormalities.


e. The disease usually has its onset in the sixth to seventh decade of
life.
1102. Which lesion is characterized by multiple tumor nodules (usuallyon the
arm or hand) composed histologically of bland mononuclear cells, short
spindle cells, and osteoblastic giant cells? (PIP 2010 D)
a. Aneurysmal bone cyst.
b. Dystrophic calcifications secondary to gluteal ischemic necrosis.
c. Giant cell reparative granuloma.
Reserved Copyrights - 2013

342

Pathology Questions

MCQs

d. Giant cell tumor of soft tissue.


e. Tumoral calcinosis.
1103. Which of the following typically shows reactivity with antibodies directed
against smooth muscle actin, desmin, CD34 and Bcl-2? (PIP 2010 D)
a. Angiosarcoma.
b. Fibroadenoma.
c. Pseudoangimoatous stromal hyperplasia (PASH).
d. Sclerosing adenosis.
1104. Which of the following is more typical of PASH than low-grade
angiosarcoma? (PIP 2010 D)
a. A positive reaction to CD31 and anti-factor VIII antibodies.
b. A tendency to infiltrate individual adipocytes.
c. Marked nuclear pleomorphism.
d. Slit-like lumina devoid of erytherocytes.
1105. If PASH is diagnosed on a core biopsy, the lesion: (PIP 2010 D)
a. May be followed clinically if the patient so desires.
b. Should be treated with lumpectomy and a sentinel lymph node
biopsy.
c. Should be treated with neoadjuvant chemotherapy before excision.
d. The lesion should be questioned as PASH, cannot be reliably
diagnosed on core biopsy.
1106. Which tumor is characterized by a small to medium sized blood vessels
with mural hyalinization? (PIP 2010 D)
a. Aggressive angiomyxoma.
b. Angimyofibroblastoma-like tumor of male genital tract.
c. Malignant fibrous histiocytoma (pleomorphic sarcoma).
d. Spindle cell lipoma.
e. Well-differentiated liposarcoma.

Reserved Copyrights - 2013

343

Pathology Questions

MCQs

1107. Which tumor can show areas of differentiation to high grade sarcoma?
(PIP 2010 D)
a. Aggressive angiomyxoma.
b. Angimyofibroblastoma-like tumor of male genital tract.
c. Malignant fibrous histiocytoma (pleomorphic sarcoma).
d. Spindle cell lipoma.
e. Well-differentiated liposarcoma.
1108. Which tumor is most often characterized by MDM2 gene amplification?
(PIP 2010 D)
a. Aggressive angiomyxoma.
b. Angimyofibroblastoma-like tumor of male genital tract.
c. Malignant fibrous histiocytoma (pleomorphic sarcoma).
d. Spindle cell lipoma.
e. Well-differentiated liposarcoma.
1109. Classical

Hodgkin

lymphoma

differs

from

nodular

lymphocyte

predominant Hodgkin lymphoma by which of the followingfeature(s)?


(PIP 2010 D)
a. Multilobulated neoplastic cells.
b. Neoplastic cells stain positive for CD30 and CD15.
c. Nodular growth pattern.
d. Sparse neoplastic cells in a background of mixed inflammatory
cells.
1110. Nodular sclerosis classical Hodgkin lymphoma (NSCHL) can be
distinguished from mediastinal large B-cell lymphoma by which of the
following features? (PIP 2010 D)
a. NSCHL is CD30 immunostain positive.
b. NSCHL is seldom seen in young patients, whereas PMBL is seen in
younger patients.
c. Only PMBL arises in a mediastinal location.

Reserved Copyrights - 2013

344

Pathology Questions

MCQs

d. Sclerotic bands in NSCHL separate nodules, while in PMBL they


tend to encircle individual large cells.
1111. Immunoglobulin Heavy Chain (IgH) gene rearrangement studies in
NSCHL are frequently negative. This is because: (PIP 2010 D)
a. NSCHL is not a B-cell malignancy and only B-cell lymphomas are
positive for IgH gene rearrangement studies.
b. NSCHL is not a clonal disorder, it is an inflammatory disease.
c. Neoplastic cells typically make up less than 1% of the sampled
specimen.
d. The IGH gene locus is lost in NSCHL.
1112. Translocation t(12;22)(q13;q12) AFT1 and EWS- is usually seen in
which neoplasm? (PIP 2010 D)
a. Clear cell sarcoma.
b. Desmoplastic small round cell tumor.
c. Lymphoblastic lymphoma.
d. Monophasic synovial sarcoma.
e. Neuroblastoma.
f. Primary renal Ewing sarcoma/PNET.
g. Rhabdoid tumor.
h. Rhabdomyosarcoma.
i.

Small cell carcinoma.

j.

Wilm's tumor, blastema predominant.

1113. This tumor is usually positive for FLI-1: (PIP 2010 D)


a. Clear cell sarcoma.
b. Desmoplastic small round cell tumor.
c. Lymphoblastic lymphoma.
d. Monophasic synovial sarcoma.
e. Neuroblastoma.
f. Primary renal Ewing sarcoma/PNET.
g. Rhabdoid tumor.
Reserved Copyrights - 2013

345

Pathology Questions

MCQs

h. Rhabdomyosarcoma.
i.

Small cell carcinoma.

j.

Wilm's tumor, blastema predominant.

1114. This tumor typically displays loss of INI 1 immunoexpression: (PIP 2010
D)
a. Clear cell sarcoma.
b. Desmoplastic small round cell tumor.
c. Lymphoblastic lymphoma.
d. Monophasic synovial sarcoma.
e. Neuroblastoma.
f. Primary renal Ewing sarcoma/PNET.
g. Rhabdoid tumor.
h. Rhabdomyosarcoma.
i.

Small cell carcinoma.

j.

Wilm's tumor, blastema predominant.

1115. Which entity is characterized by a mutation in the ATP78 protein encoded


for on chromosome 13? (PIP 2010 D)
a. Acetaminophen toxicity.
b. Alpha-1 antitrypsin deficiency.
c. Nodular regenerative hyperplasia.
d. Primary biliary cirrhosis.
e. Viral hepatitis.
f. Wilson disease.
1116. Which entity is primarily seen in middle age women and is characterized
by a positive antimitochondrial antibody, elevated alkaline phosphatase
and elevated IgM? (PIP 2010 D)
a. Acetaminophen toxicity.
b. Alpha-1 antitrypsin deficiency.
c. Nodular regenerative hyperplasia.
d. Primary biliary cirrhosis.
Reserved Copyrights - 2013

346

Pathology Questions

MCQs

e. Viral hepatitis.
f. Wilson disease.
1117. Which entity is associated with endoplasmic reticulum stress secondary
to accumulation of abnormally folded protein? (PIP 2010 D)
a. Acetaminophen toxicity.
b. Alpha-1 antitrypsin deficiency.
c. Nodular regenerative hyperplasia.
d. Primary biliary cirrhosis.
e. Viral hepatitis.
f. Wilson disease.
1118. Which entity has the highest genetic predisposition of allhematologic
neoplasms? (PIP 2010 D)
a. Bacillary angiomatosis.
b. Chronic lymphocytic leukemia/Small lymphocytic lymphoma.
c. Diffuse large B-cell lymphoma, not otherwise specified.
d. Extra-nodal marginal zone lymphoma of mucosal-associated
lymphoid tissue.
e. Hairy cell leukemia.
f. Nodular sclerosing classical Hodgkin lymphoma,
g. Splenic marginal zone lymphoma.
1119. Based on gene expression profiling, which entity has two prognostic
subtypes- germinal center B-cell like (GCB) and activated B-cell like
(ABC)? (PIP 2010 D)
a. Bacillary angiomatosis.
b. Chronic lymphocytic leukemia/Small lymphocytic lymphoma.
c. Diffuse large B-cell lymphoma, not otherwise specified.
d. Extra-nodal marginal zone lymphoma of mucosal-associated
lymphoid tissue.
e. Hairy cell leukemia.
f. Nodular sclerosing classical Hodgkin lymphoma,
Reserved Copyrights - 2013

347

Pathology Questions

MCQs

g. Splenic marginal zone lymphoma.


1120. Which entity is associated with t(11;18) translocation? (PIP 2010 D)
a. Bacillary angiomatosis.
b. Chronic lymphocytic leukemia/Small lymphocytic lymphoma.
c. Diffuse large B-cell lymphoma, not otherwise specified.
d. Extra-nodal marginal zone lymphoma of mucosal-associated
lymphoid tissue.
e. Hairy cell leukemia.
f. Nodular sclerosing classical Hodgkin lymphoma,
g. Splenic marginal zone lymphoma.
1121. Loss of INI 1 (hsNF5/Baf47) is seen in which other tumor besides renal
medullary carcinoma? (PIP 2011 A)
a. Collecting duct carcinoma.
b. Metastatic adenocarcinoma.
c. Renal cell carcinoma, unclassified.
d. Renal medullary carcinoma.
e. Rhabdoid tumor.
f. Urothelial carcinoma with glandular features involving the renal
pelvis.
1122. Patients with this type of tumor typically have sickle trait (Hb-AS or HbSC) or rarely, sickle cell anemia (Hb-SS): (PIP 2011 A)
a. Collecting duct carcinoma.
b. Metastatic adenocarcinoma.
c. Renal cell carcinoma, unclassified.
d. Renal medullary carcinoma.
e. Rhabdoid tumor.
f. Urothelial carcinoma with glandular features involving the renal
pelvis.

Reserved Copyrights - 2013

348

Pathology Questions

MCQs

1123. This tumor is usually positive for HMCK, EMA, CK7, ULEX-1 and INI 1:
(PIP 2011 A)
a. Collecting duct carcinoma.
b. Metastatic adenocarcinoma.
c. Renal cell carcinoma, unclassified.
d. Renal medullary carcinoma.
e. Rhabdoid tumor.
f. Urothelial carcinoma with glandular features involving the renal
pelvis.
1124. CASTLE can be distinguished from primary metastatic squamous cell
carcinoma of the thyroid by: (PIP 2011 A)
a. Membranous CD5 and CD117 positivity.
b. Epstein-Barr virus positivity.
c. Close

resemblance

of

CASTLE

to

lymphoepithelioma-like

carcinoma of the thymus.


d. p63 positivity.
e. Lymph node involvement.
1125. The immunohistochemical profile of CASTLE includes: (PIP 2011 A)
a. Keratin and p63 positivity.
b. CD5 and CD117 (C-kit) positivity.
c. Chromogranin and synaptophysin positivity.
d. Absence of CD20/CD45+ lymphocytes.
e. Presence of CD1a/TdT+ small lymphocytes in stroma.
1126. CASTLE

shares

with

lymphoepithelioma-like

carcinoma

of

the

nasopharynx all of the following EXCEPT: (PIP 2011 A)


a. Dense lymphoplasmacytic stromal infiltrates.
b. Propensity for lymph node metastasis.
c. Good response to treatment.
d. Uniform population of keratin-positive cells with prominent
nucleoli.
e. Association with Epstein-Barr virus.
Reserved Copyrights - 2013

349

Pathology Questions

MCQs

1127. Which of the following conditions is NOT associated with HPS? (PIP 2011
A)
a. Herpes virus.
b. Antiviral therapy.
c. Lymphoma.
d. Lupus erythematosis.
e. Mutations of the perforin gene.
f. Immunodeficiency disorders.
g. None of the above.
1128. How

does

one

differentiate

sinus

histiocytosis

with

massive

lymphadenopathy (SHML) from HPS? (PIP 2011 A)


a. SHML cells have much more cytologic atypia.
b. SHML cells typically contain more intact lymphocytes and plasma
cells.
c. HPS histiocytes are S100 positive.
d. SHML is associated with a dense eosinophilic infiltration.
1129. Which lesion can secondarily complicate benign and malignant tumors?
(PIP 2011 A)
a. Aneurysmal bone cyst.
b. Desmoplastic fibroma.
c. Fibrous dysplasia.
d. Fibrosarcoma of bone, low-grade.
e. Intraosseous osteosarcoma, low-grade.
f. Parosteal osteosarcoma.
1130. Which condition involves an activating mutation of the G protein signaling
pathway? (PIP 2011 A)
a. Aneurysmal bone cyst.
b. Desmoplastic fibroma.
c. Fibrous dysplasia.
d. Fibrosarcoma of bone, low-grade.
Reserved Copyrights - 2013

350

Pathology Questions

MCQs

e. Intraosseous osteosarcoma, low-grade.


f. Parosteal osteosarcoma.
1131. Which entity is characterized by fairly uniform spindle cells arrangedin a
herringbone pattern and aggressive growth pattern radiologically? (PIP
2011 A)
a. Aneurysmal bone cyst.
b. Desmoplastic fibroma.
c. Fibrous dysplasia.
d. Fibrosarcoma of bone, low-grade.
e. Intraosseous osteosarcoma, low-grade.
f. Parosteal osteosarcoma.
1132. Which lesion is characterized by round, epithelioid cells with duct-like
structures and stains with CAM5.2? (PIP 2011 A)
a. Angiosarcoma.
b. Glomus tumor.
c. Hemangiopericytoma/solitary fibrous tumor.
d. Hidradenoma.
e. Myopericytoma.
1133. Which lesion is characterized by round cells surrounding vessels, with
those cells showing a round, centrally placed nucleus? (PIP 2011 A)
a. Angiosarcoma.
b. Glomus tumor.
c. Hemangiopericytoma/solitary fibrous tumor.
d. Hidradenoma.
e. Myopericytoma.
1134. Which lesion may be immunoreactive for CD99? (PIP 2011 A)
a. Angiosarcoma.
b. Glomus tumor.
c. Hemangiopericytoma/solitary fibrous tumor.
Reserved Copyrights - 2013

351

Pathology Questions

MCQs

d. Hidradenoma.
e. Myopericytoma.
1135. Of the small lymphocytic processes that can involve the white pulp ofthe
spleen, which of the lesions characteristically has the highest proliferation
index (Ki-67) within the small lymphocytes? (PIP 2011 A)
a. Classical Hodgkin lymphoma.
b. Diffuse large B-cell lymphoma.
c. Hairy cell leukemia.
d. Mantle cell lymphoma.
e. Small lymphocytic lymphoma/chronic lymphocytic leukemmia.
f. Splenic diffuse red pulp small B-cell lymphoma.
g. Splenic marginal zone lymphoma.
1136. Tumor cells of this entity are usually characteristically sparse within the
nodules and are usually positive for CD30? (PIP 2011 A)
a. Classical Hodgkin lymphoma.
b. Diffuse large B-cell lymphoma.
c. Hairy cell leukemia.
d. Mantle cell lymphoma.
e. Small lymphocytic lymphoma/chronic lymphocytic leukemmia.
f. Splenic diffuse red pulp small B-cell lymphoma.
g. Splenic marginal zone lymphoma.
1137. The tumor cells of this B-cell leukemia/lymphoma are CD103 positive and
DBA.44 positive: (PIP 2011 A)
a. Classical Hodgkin lymphoma.
b. Diffuse large B-cell lymphoma.
c. Hairy cell leukemia.
d. Mantle cell lymphoma.
e. Small lymphocytic lymphoma/chronic lymphocytic leukemmia.
f. Splenic diffuse red pulp small B-cell lymphoma.
g. Splenic marginal zone lymphoma.
Reserved Copyrights - 2013

352

Pathology Questions

MCQs

1138. Which entity is characterized by a collection of endothelial-linedcysts


surrounded by a fibrous stroma containing smooth muscle fibers? (PIP
2011 A)
a. Florid cystic endosalpingiosis.
b. Lymphangioma.
c. Malignant peritoneal mesothelioma.
d. Multicystic peritoneal mesothelioma.
e. Pseudomyxoma peritonei.
1139. In addition to lymphangioma, in which other cystic lesion do D2-40
positive cells line the cystic structures? (PIP 2011 A)
a. Florid cystic endosalpingiosis.
b. Lymphangioma.
c. Malignant peritoneal mesothelioma.
d. Multicystic peritoneal mesothelioma.
e. Pseudomyxoma peritonei.
1140. Which entity typically occurs in women of reproductive age and may
contain cilia in the cyst lining? (PIP 2011 A)
a. Florid cystic endosalpingiosis.
b. Lymphangioma.
c. Malignant peritoneal mesothelioma.
d. Multicystic peritoneal mesothelioma.
e. Pseudomyxoma peritonei.
1141. Pseudomembranes may form in this disease process but will have a
paucity of neutrophils: (PIP 2011 A)
a. Amoebic colitis.
b. Clostidium dificille pseudomembranous colitis.
c. Graft versus host disease.
d. Inflammatory bowel disease with pseudomembrane formation.
e. Ischemic colitis with pseudomembrane formation.
Reserved Copyrights - 2013

353

Pathology Questions

MCQs

f. Radiation colitis.
g. Typhilitis.
1142. In which disease does examination of the colon reveal pseudomembranes
with siderophages and fibrosis of lamina propria? (PIP 2011 A)
a. Amoebic colitis.
b. Clostidium dificille pseudomembranous colitis.
c. Graft versus host disease.
d. Inflammatory bowel disease with pseudomembrane formation.
e. Ischemic colitis with pseudomembrane formation.
f. Radiation colitis.
g. Typhilitis.
1143. In which disease does examination of the colon reveal pseudomembranes
with flask-shaped ulceration containing histiocystoid structures that
exhibit erythrophagocytosis and strong cytoplasmic PAS reaction? (PIP
2011 A)
a. Amoebic colitis.
b. Clostidium dificille pseudomembranous colitis.
c. Graft versus host disease.
d. Inflammatory bowel disease with pseudomembrane formation.
e. Ischemic colitis with pseudomembrane formation.
f. Radiation colitis.
g. Typhilitis.
1144. Which entity is characterized by diffusely enlarged, often cavitated villi
with concentric trophoblastic proliferation and is associatedwith the
greatest risk for persistent trophoblastic disease? (PIP 2011 A)
a. Choriocarcinoma.
b. Complete hydatidiform mole.
c. Hydropic abortus.
d. Partial hydatidiform mole.

Reserved Copyrights - 2013

354

Pathology Questions

MCQs

1145. Which entity is associated with a lack of immunostaining for the


paternally imprinted, maternally expressed gene product, p57kip2 in
villous mesenchymal cells and villous cytotrophoblast?
a. Choriocarcinoma.
b. Complete hydatidiform mole.
c. Hydropic abortus.
d. Partial hydatidiform mole.
1146. Which entity can be associated with modestly increased beta-HCG levels,
a normal uterine ultrasound, an admixture of normal and hydropic villi,
concentric syncytiotrophoblastic hyperplasia and fetal syndactyly? (PIP
2011 A)
a. Choriocarcinoma.
b. Complete hydatidiform mole.
c. Hydropic abortus.
d. Partial hydatidiform mole.
1147. Which tumor is characteriz by positive staining for CD5 and CD117? (PIP
2011 B)
a. Follicular dendritic cell sarcoma.
b. Nodular sclerosis classical Hodgkin lymphoma.
c. Primary mediastinal large B-cell lymphoma.
d. Spindle cell thymoma.
e. Thymic carcinoma.
1148. Which tumor is typically defined by positive staining for CD21 and
CD35/23? (PIP 2011 B)
a. Follicular dendritic cell sarcoma.
b. Nodular sclerosis classical Hodgkin lymphoma.
c. Primary mediastinal large B-cell lymphoma.
d. Spindle cell thymoma.
e. Thymic carcinoma.

Reserved Copyrights - 2013

355

Pathology Questions

MCQs

1149. Which tumor is characterized by CD30 and CD20 positive cells with clear
cytoplasm in a background of delicate interstitial fibrosis? (PIP 2011 B)
a. Follicular dendritic cell sarcoma.
b. Nodular sclerosis classical Hodgkin lymphoma.
c. Primary mediastinal large B-cell lymphoma.
d. Spindle cell thymoma.
e. Thymic carcinoma.
1150. Mucinous and signet ring cell carcinomas together account for
approximately what percentage of colorectal adenocarcinoma? (PIP 2011
B)
a. 1%.
b. 10%.
c. 20%.
d. 25%.
1151. Mucinous colorectal adenocarcinoma is associated with which ONE of the
following? (PIP 2011 B)
a. A near 100% likelihood of hereditary non-polyposis colorectal
cancer (HNPCC).
b. A near 100% likelihood of high microsatellite instability (MSI-H).
c. An excellent response to chemotherapy.
d. A better prognosis compared to signet ring cell carcinoma.
1152. Which of the following is/are required for classification of a tumor as a
signet ring cell carcinoma? (PIP 2011 B)
a. More than 50% of the tumor is comprised of mucin.
b. More than 50% of the tumor cells are signet ring cells.
c. High microsatellite instability (MSI-H).
d. A family pedigree consistent with HNPCC.

Reserved Copyrights - 2013

356

Pathology Questions

MCQs

1153. While the capsule of C. neoformans typically stains fuchsia with


mucicarmine stain, capsule-deficient variants may be stained by: (PIP
2011 B)
a. Alcian blue.
b. Fontana-Masson.
c. Verhoeff-Van Gieson.
d. Masson trichrome.
1154. Malakoplakia is: (PIP 2011 B)
a. A granulomatous-type inflammatory response usually seen in the
urinary tract association with Gram-negative coliform bacilli.
b. Is typically associated with the formation of Michaelis-Gutmann
bodies.
c. Seen in the lungs associated with R. equi infection.
d. All of the above.
e. None of the above.
1155. C. neoformans: (PIP 2011 B)
a. Is a common soil contaminant spread through aerosolized dust.
b. Is spread through pigeon droppings.
c. Is usually a pathogen in immunocompromised patients but may
also afflict immunocompetent patients.
d. Has a world-wide distribution.
e. All of the above.
f. Non of the above.
1156. Which human papillomavirus type is the most common cause of recurrent
respiratory papillomatosis? (PIP 2011 B)
a. HPV 16.
b. HPV 18.
c. HPV 31.
d. HPV 33.
e. HPV 11.
Reserved Copyrights - 2013

357

Pathology Questions

MCQs

1157. Which is not typically a feature of recurrent respiratory papillomatosis?


(PIP 2011 B)
a. Fibrovascular cores covered in an orderly squamous epithelium.
b. A pushing, non-infiltrative border.
c. Increased mitotic figures and a desmoplastic stroma.
d. Presence of mild to moderate squamous dysplasia.
e. Multiple exophytic lesions.
1158. What is the most common clinical presentation for recurrent respiratory
papillomatosis? (PIP 2011 B)
a. Cavitary lung lesions seen on imaging.
b. Hoarseness or changes in voice.
c. Respiratory distress.
d. Presence of enlarged cervical lymph nodes.
e. Post-obstructive pneumonia.
1159. Which is the most common renal neoplasm in children 2 months of age?
(PIP 2011 B)
a. Clear cell sarcoma.
b. Congenital mesoblastic nephroma.
c. Rhabdoid tumor.
d. Wilms tumor.
e. Leiomyoma.
1160. Cellular mesoblastic nephromas often exhibit trisomy 11. Which other
tumor harbors a chromosomal abnormality of chromosome11? (PIP 2011
B)
a. Clear cell sarcoma.
b. Congenital mesoblastic nephroma.
c. Rhabdoid tumor.
d. Wilms tumor.
e. Leiomyoma.
Reserved Copyrights - 2013

358

Pathology Questions

MCQs

1161. Loss of INI 1 nuclear staining is associated with which pediatric tumor?
(PIP 2011 B)
a. Clear cell sarcoma.
b. Congenital mesoblastic nephroma.
c. Rhabdoid tumor.
d. Wilms tumor.
e. Leiomyoma.
1162. Which one of the following features can be helpful in distinguishing
cytoplasmic inclusions in GSD IV and Lafora disease? (PIP 2011 B)
a. Inclusions of Lafora disease are pale staining or basophilic.
b. Inclusions of Lafora disease are digested with diastase.
c. Inclusions of GSD IV are coarsely stained with colloidal iron.
d. Inclusions of GSD IV resist digestion with pectinase.
1163. Which

of

the

following

conditions

is

NOT

associated

with

swollenlymphocytes loaded with glycogen? (PIP 2011 B)


a. Urea cycle enzyme defects.
b. Fibrinogen storage diseases.
c. Glycogenic hepatopathy.
d. High dose steroids.
1164. Which of the following statements is correct about glycogen storage
diseases? (PIP 2011 B)
a. Swollen hepatocytes are common in GSD, type 0.
b. Adenomas occur in GSD, type I.
c. Lysosomal glycogen is seen in GSD, type III.
d. Ground glass inclusions are seen in GSD, type IX.
1165. Which lesion is most often associated with severe thrombocytopenia,
purpura,

and

consumptive

coagulopathy

(Kasabach-Merritt

phenomenon)?
Reserved Copyrights - 2013

359

Pathology Questions

MCQs

a. Infantile hemangioma.
b. Kaposiform hemangioendothelioma.
c. Angiosarcoma.
d. Venous malformation.
e. Neonatal hemangimatosis.
1166. Which of the following immunohistochemical markers is expected to be
positive in Kaposiform hemangioendothelioma? (PIP 2011 B)
a. HHV8.
b. Glut-1.
c. D2-40.
d. HMB45.
e. Myogenin.
1167. The biologic behavior of Kaposiform hemangioendothelioma istypically
characterized by: (PIP 2011 B)
a. Proliferative phase of growth over a period of months, followed by
slow involution during the first year of life.
b. Rapid near-complete involution in the first weeks of life.
c. Partial involution in the first weeks of life, followed by a prolonged
period of stability.
d. Progressive growth.
e. Distant metastasis.
1168. Which of the following spindle cell tumors of soft tissue is LEAST likely to
show a prominent hemangiopericytic pattern of growth? (PIP 2011 C)
a. Fibrosarcoma.
b. Synovial sarcoma.
c. Leiomyosarcoma.
d. Solitary fibrous tumor.
e. Malignant peripheral nerve sheath tumor.

Reserved Copyrights - 2013

360

Pathology Questions

MCQs

1169. Which of the following combinations of immunohistochemical stains are


most consistent with a diagnosis of synovial sarcoma? (PIP 2011 C)
a. Strong and diffuse pan-keratin positivity, CEA+, MOC31+ and
CK19+.
b. Focal keratin positivity, bcl-2+, calponin+, CD99+, EMA+, TLE-1+.
c. Focal keratin positivity, bcl-2+, S-100+, CD56+, CD10+, TLE-1
negative.
d. Negative keratin, bcl-2+, CD99+, CD34+, SMA negative, TLE-1
negative.
e. CK8/18+, SMA+, desmin+, calponin+, CD99 negative, bcl-2
negative.
1170. Which of the following molecular genetics results using fluorescence insitu hybridization (FISH) would be consistent with a diagnosis of synovial
sarcoma? (PIP 2011 C)
a. Demonstration of SS18-SSX1 or SS18-SSX2 fusion gene.
b. Demonstration of EWSR1-FLI 1 fusion gene.
c. Demonstration of FUS-DDIT3 fusion gene.
d. Demonstration of COL1A1-PDGFB fusion gene.
e. Demonstration of EWSR1-NR4A3 fusion gene.
1171. Which neoplasm frequently lacks evidence of urothelial differentiation on
immunohistochemical study for markers such as uroplakin III,
thrombomodulin or CK20? (PIP 2011 C)
a. Benign Brenner tumor.
b. Borderline (proliferative) Brenner tumor.
c. Malignant Brenner tumor.
d. Metastatic urothelial (bladder) carcinoma.
e. Transitional cell carcinoma of the ovary.
1172. Which is the correct classification for a primary ovarian tumor with areas
of urothelial, squamous or undifferentiated carcinoma found in
association with nests of benign urothelial cells?
Reserved Copyrights - 2013

361

Pathology Questions

MCQs

a. Benign Brenner tumor.


b. Borderline (proliferative) Brenner tumor.
c. Malignant Brenner tumor.
d. Metastatic urothelial (bladder) carcinoma.
e. Transitional cell carcinoma of the ovary.
1173. Which neoplasm should be suspected when the tumor is composed of
high grade urothelial cells that stain positive for uroplakin III,
thrombomodulin, and CK20, but does not have an adjacent benign or
borderline component of Brenner tumor? (PIP 2011 C)
a. Benign Brenner tumor.
b. Borderline (proliferative) Brenner tumor.
c. Malignant Brenner tumor.
d. Metastatic urothelial (bladder) carcinoma.
e. Transitional cell carcinoma of the ovary.
1174. Which entity is characterized by a poorly lobulated architecture, an
increased cell population per unit area with hyperchromatic nuclei,
increased binucleated cells, lack of osteoid formation and irregular
margins on imaging? (PIP 2011 C)
a. Chondroblastic osteosarcoma.
b. Chondrosarcoma, well-differentiated.
c. Differentiated chondrosarcoma.
d. Enchondroma.
e. Extraskeletal myxoid chondrosarcoma.
f. Fracture callus.
1175. Which cartilage producing tumor, can be identified by the characteristic
separation of hyaline cartilage lobules by hematopoietic elements? (PIP
2011 C)
a. Chondroblastic osteosarcoma.
b. Chondrosarcoma, well-differentiated.
c. Differentiated chondrosarcoma.
Reserved Copyrights - 2013

362

Pathology Questions

MCQs

d. Enchondroma.
e. Extraskeletal myxoid chondrosarcoma.
f. Fracture callus.
1176. Which

solid

tumor

demonstrates

reproducible

chromosomal

rearrangements involving 9q22, which results in the production of a


NR4A3 fusion gene? (PIP 2011 C)
a. Chondroblastic osteosarcoma.
b. Chondrosarcoma, well-differentiated.
c. Differentiated chondrosarcoma.
d. Enchondroma.
e. Extraskeletal myxoid chondrosarcoma.
f. Fracture callus.
1177. Which entity is characterized by a reciprocal translocation, t(12;22)
(q13;q12), resulting in EWS/ATF1 gene fusion? (PIP 2011 C)
a. Cellular schwannoma.
b. Clear cell sarcoma of soft tissue.
c. Fibrosarcoma.
d. Leiomyosarcoma.
e. Malignant peripheral nerve sheath tumor.
f. Synovial sarcoma.
1178. Which tumor is characterized by sweeping fascicles of irregularly shaped
spindle cells with lightly-stained indistinct cytoplasm and hyalinized
cords that resemble giant rosettes? (PIP 2011 C)
a. Cellular schwannoma.
b. Clear cell sarcoma of soft tissue.
c. Fibrosarcoma.
d. Leiomyosarcoma.
e. Malignant peripheral nerve sheath tumor.
f. Synovial sarcoma.

Reserved Copyrights - 2013

363

Pathology Questions

MCQs

1179. Which tumor shows positive staining for cytokeratins, vimentin, EMA,
CD99 and Bcl-2? (PIP 2011 C)
a. Cellular schwannoma.
b. Clear cell sarcoma of soft tissue.
c. Fibrosarcoma.
d. Leiomyosarcoma.
e. Malignant peripheral nerve sheath tumor.
f. Synovial sarcoma.
1180. A lymph node from an upper cervical biopsy in a young Asian male shows
increased eosinophils, some in large clusters along with plasma cells,
nodules separated by fibrocollagenous sclerosis and frequent binucleated
giant cells that are reactive for CD30 and CD15? (PIP 2011 C)
a. Angiolymphoid hyperplasia with eosinophilia.
b. Classical Hodgkin lymphoma.
c. Histiocytosis.
d. Kimura lymphadenopathy.
e. Systemic mastocytosis.
1181. A 3-year old child presents with bony lesions involving predominantly flat
bones (skull). Biopsy shows increased eosinophils, along with a spindle
cell infiltrate which is positive for CD117: (PIP 2011 C)
a. Angiolymphoid hyperplasia with eosinophilia.
b. Classical Hodgkin lymphoma.
c. Histiocytosis.
d. Kimura lymphadenopathy.
e. Systemic mastocytosis.
1182. Which of the above conditions present as a papular rash in a young
Cucasian female, with histopathology revealing soft tissue and dermal
eosinophilic infiltrate, proliferation of small capillaries with prominent
endothelial hypertrophy and no nodal involvement? (PIP 2011 C)
a. Angiolymphoid hyperplasia with eosinophilia.
Reserved Copyrights - 2013

364

Pathology Questions

MCQs

b. Classical Hodgkin lymphoma.


c. Histiocytosis.
d. Kimura lymphadenopathy.
e. Systemic mastocytosis.
1183. Which entity is characterized by multiple cysts lined by low-cuboidal
epithelial cells with clear cytoplasm and can be seen in association with
the von Hipple-Lindau syndrome? (PIP 2011 C)
a. Intraductal papillary mucinous neooplasm.
b. Lymphoepithelial cyst.
c. Metastatic renal cell carcinoma.
d. Microcystic serous cystadenoma.
e. Mucinous cystic neoplasm.
f. Pancreatic ductal adenocarcinoma.
1184. Which entity is almost entirely seen in women and is characterized by
cystic spaces lined by mucinous cells with a surrounding ovarian-type
stroma? (PIP 2011 C)
a. Intraductal papillary mucinous neooplasm.
b. Lymphoepithelial cyst.
c. Metastatic renal cell carcinoma.
d. Microcystic serous cystadenoma.
e. Mucinous cystic neoplasm.
f. Pancreatic ductal adenocarcinoma.
1185. Which cystic entity is often filled with keratinaceous debris? (PIP 2011 C)
a. Intraductal papillary mucinous neooplasm.
b. Lymphoepithelial cyst.
c. Metastatic renal cell carcinoma.
d. Microcystic serous cystadenoma.
e. Mucinous cystic neoplasm.
f. Pancreatic ductal adenocarcinoma.

Reserved Copyrights - 2013

365

Pathology Questions

MCQs

1186. A small dural lesion is resected and is composed of epithelioid cells with
prominent eosinophilic cytoplasm. Some cells have single vacuoles
resembling signet ring forms while other cells have bubbly, multivacuolated cytoplasm. These cells are present in a mucoid matrix. The
lesional cells express brachyury, EMA, cytokeratin and S-100 but do not
express D2-40 or GFAP. Which of the following is the best diagnosis? (PIP
2011 C)
a. Choroid glioma, WHO grade II.
b. Choroid meningioma, WHO grade II.
c. Choroid sarcoma.
d. Intradural chondroma.
e. Metastatic mucinous adenocarcinoma.
1187. A 45-year-old man presents with a radiographic circumscribed, noninfiltrating third ventricle tumor that is resected. It is composed of
epithelioid cells forming cords and nests within a mucinous stroma that
has a prominent lymphoplasmacytic infiltrate. Surrounding non-tumoral
tissue exhibits piloid gliosis. There is no mitotic activity and tumoral cells
express CD34 (membranous) and GFAP. Tumor cells do not express EMA,
cytokeratin, CD18 or brachyury. Which of the following is the best
diagnosis? (PIP 2011 C)
a. Choroid glioma, WHO grade II.
b. Choroid meningioma, WHO grade II.
c. Choroid sarcoma.
d. Intradural chondroma.
e. Metastatic mucinous adenocarcinoma.
1188. A 9-year-old girl presents with a dura-based 2.4 cm lesion that is resected.
It is composed of epithelioid cells some containing cytoplasmic vacuoles
forming loose nests and cords in a faintbasophilic mucoid background.
There is no mitotic activity or necrosis present. Tumor cells express D240, EMA and weakly express S-100 and cytokeratin. Tumor cells do not
express GFAP or brachyury. Thereis a mononuclear lymphoid infiltrate
Reserved Copyrights - 2013

366

Pathology Questions

MCQs

present that expresses CD79a and CD138. What additional clinical feature
might be expected to present in this patient pre-operatively? (PIP 2011 C)
a. Hypercalcemia.
b. Hypercoagulable state.
c. Hypoglycemia.
d. Microcytic hypochromic anemia.
e. Polycythemia.
1189. Which tumor is characterized by the presence of a malignant epithelial
component and a malignant mesenchymal component? (PIP 2011 C)
a. Carcinosarcoma.
b. Mllerian adenosarcoma.
c. Rhabdomyosarcoma.
d. Undifferentiated uterine sarcoma.
1190. Which tumor is characterized by the presence of a benign epithelial
component and a malignant mesenchymal component? (PIP 2011 C)
a. Carcinosarcoma.
b. Mllerian adenosarcoma.
c. Rhabdomyosarcoma.
d. Undifferentiated uterine sarcoma.
1191. Which tumor does not show any particular line of differentiation both by
morphology and immunophenotype? (PIP 2011 C)
a. Carcinosarcoma.
b. Mllerian adenosarcoma.
c. Rhabdomyosarcoma.
d. Undifferentiated uterine sarcoma.
1192. Which of the following best describes lymphocytic and collagenous
colitis? (PIP 2011 C)
a. Distorted mucosal architecture with variability in size, shape and
spacing of glands.
Reserved Copyrights - 2013

367

Pathology Questions

MCQs

b. Preserved mucosal architecture, inflamed lamina propria and


increased intraepithelial lymhocytes.
c. Fringed, fuzzy coating at the luminal side of the mucosa, with no
associated inflammation or architectural dissaray.
d. Superfisial mucosal necrosis with atrophic crypts, hyalinization of
the lamina propria and features of regeneration.
1193. Which special stain can be used to distinguish lymhocytic colitis from
collagenous colitis in equivocal cases? (PIP 2011 C)
a. Trichrome.
b. Warthin-starry.
c. GMS.
d. Gram.
1194. Which entity is typically associated with chronic watery diarrheaand
normal

or

near-normal

endoscopic

mucosal

appearnce

during

colonoscopy? (PIP 2011 C)


a. Ulcerative colitis.
b. Ischemic colitis.
c. Intestinal spirochetosis.
d. Collagenous colitis.
1195. Which tumor is characteristically poorly demarcated? (PIP 2011 D)
a. Aggressive angiomyxoma.
b. Angiomyofibroblastoma.
c. Cellular angiofibroma.
d. Solitary fibrous tumor.
1196. Which tumor is typically positive for desmin but not muscle-specific actin
or CD34? (PIP 2011 D)
a. Aggressive angiomyxoma.
b. Angiomyofibroblastoma.
c. Cellular angiofibroma.
Reserved Copyrights - 2013

368

Pathology Questions

MCQs

d. Solitary fibrous tumor.


1197. Which tumor is characterized by hemangiopericytoma-like vascular
pattern and CD34 positivity? (PIP 2011 D)
a. Aggressive angiomyxoma.
b. Angiomyofibroblastoma.
c. Cellular angiofibroma.
d. Solitary fibrous tumor.
1198. Which of the following choices best allows one to discriminate between
juvenile fibroadenoma and low-grade phyllodes tumor? (PIP 2011 D)
a. The rate of growth and assessment of a peri- versus intracanalicular growth pattern.
b. Type of margin.
c. Degree of mitotic activity.
d. Gross size.
e. Sex of the patient.
1199. High-grade phyllodes tumors have been linked to mutations in which of
the following? (PIP 2011 D)
a. p53.
b. BRAF.
c. N-myc.
d. p16 INK4A
e. BRCA-1.
1200. The most common clinical problem associated with low grade phyllodes
tumor is: (PIP 2011 D)
a. High risk of distant metastasis.
b. Chest wall invasion.
c. Local recurrence after excision.
d. Incidence of hypercalcemia (paraneoplastic syndrome).
e. Differentiation to high grade phyllodes tumor.
Reserved Copyrights - 2013

369

Pathology Questions

MCQs

1201. Which of the following pathologic processes is inherited in an autosomal


recessive manner? (PIP 2011 D)
a. Chronic myelogenous leukemia.
b. Gaucher disease.
c. Hairy cell leukemia.
d. Rosai-Dorfman disease.
e. Splenic lymphoma.
1202. What histochemical stain is used to differentiate Gaucher cells
fromPseudo-Gaucher cells? (PIP 2011 D)
a. Prussian blue.
b. Ziehl-Neelsen.
c. PAS.
d. Sudan black B.
1203. Which is a positive immunohistochemical marker for Gaucher cells? (PIP
2011 D)
a. S100.
b. CD68.
c. CD1a.
d. CD11b.
1204. Which is characteristic of ovarian sclerosing stromal tumors? (PIP 2011
D)
a. They are typically bilateral.
b. They typically occur in the fifth decade or later.
c. They typically occur in patients thirty years or younger.
d. They typically are hormonally active.
e. They typically co-exist with ovarian mucinous tumors.
1205. Which immunomarker is most typically positive in an ovarian sclerosing
tumor? (PIP 2011 D)
Reserved Copyrights - 2013

370

Pathology Questions

MCQs

a. AE1/AE3.
b. Inhibin.
c. Calretenin.
d. Desmin.
e. Cytokeratin 7.
1206. What is the most characteristic of the signet ring cells seen in some
sclerosing stromal tumors? (PIP 2011 D)
a. They contain mucin.
b. They contain lipid.
c. They contain glycogen.
d. They will express AE1/AE3 and cytokeratin 20.
e. They are strongly inhibin and desmin positive.
1207. A 45 year old otherwise healthy woman presents with a progressively
enlarging mass in her lower neck. Physical examination and ultrasound
show nodules within the thyroid and adjacent soft tissue. The patient
denies a family history of thyroiddisease. An FNA is performed on a neck
nodule, which is diagnosedas scanty cellularity, suspicious for
neuroendocrine neoplasm. At surgery, a 2.0 cm nodule located in the
middle one-third of the left lobe of the thyroid is excised.Given the age
and clinical history in this case, to which clinical group would our patient
most likely belong? (PIP 2011 D)
a. A sporadic case of medullary carcinoma lacking C-cell hyperplasia.
b. A familial case of medullary carcinoma without an associated
endocrinopathy.
c. Multiple neuroendocrine neoplasia 2A.
d. Multiple neuroendocrine neoplasia 2B.
1208. The majority of medullary thyroid carcinomas associated with MEN2A
and MEN2B contain mutations in which proto-oncogene? (PIP 2011 D)
a. RAS.
b. PTEN.
Reserved Copyrights - 2013

371

Pathology Questions

MCQs

c. RET.
d. P53.
e. Beta-catenin.
1209. Which staining pattern is most consistent with medullary thyroid
carcinoma? (PIP 2011 D)
a. TTF-1(-), Calcitonin(-), Low Molecular weight cytokeratin(+),
CEA(+).
b. TTF-1(+), Calcitonin(+), Low Molecular weight cytokeratin(-),
CEA(-).
c. TTF-1(-), Calcitonin(+), Low Molecular weight cytokeratin(-),
CEA(+).
d. TTF-1(+), Calcitonin(-), Low Molecular weight cytokeratin(+),
CEA(-).
e. TTF-1(+), Calcitonin(-), Low Molecular weight cytokeratin(+),
CEA(-).
1210. Which tumor is characterized morphologically by malignant glandular
and mesenchymal elements, and is frequently associatedwith metastases
and poor prognosis? (PIP 2011 D)
a. Gastrointestinal stromal tumor.
b. Leiomyosarcoma.
c. Sarcomatoid carcinoma.
d. Stromal sarcoma.
e. Stromal tumor of uncertain malignant potential.
1211. Which tumor is characterized by diffuse immunoreactivity for
CD117/KIT? (PIP 2011 D)
a. Gastrointestinal stromal tumor.
b. Leiomyosarcoma.
c. Sarcomatoid carcinoma.
d. Stromal sarcoma.
e. Stromal tumor of uncertain malignant potential.
Reserved Copyrights - 2013

372

Pathology Questions

MCQs

1212. Which tumor is characterized by the following histologic features and


immunophenotype: hypercellular, hyperchromatic spindle cell lesion with
mild to moderate cytologic atypia, no mitoses, no necrosis, and
vimentin(+), CD34(+), progesterone receptor(+)?(PIP 2011 D)
a. Gastrointestinal stromal tumor.
b. Leiomyosarcoma.
c. Sarcomatoid carcinoma.
d. Stromal sarcoma.
e. Stromal tumor of uncertain malignant potential.
1213. Which of the following neonatal lung diseases is characterized by nearcomplete lack of airspace development and its universally fatal in the first
hour of life? (PIP 2011 D)
a. Acinar dysplasia.
b. Alveolar capillary dysplasia.
c. Bronchopulmonary dysplasia.
d. Congenital alveolar dysplasia.
e. Pulmonary interstitial glycogenosis.
1214. Chronic congestive vasculopathy (pulmonary venous hypertension) may
result from all of the following cardiopulmonary lesions, EXCEPT: (PIP
2011 D)
a. Congenital cardiomyopathy.
b. Mitral valve stenosis.
c. Pulmonary vein stenosis.
d. Total anomalous pulmonary venous connection.
e. Ventricular septic defect.
1215. Alveolar capillary dysplasia is caused by mutations or deletions in which
of the following genes? (PIP 2011 D)
a. ABCA3.
b. FOXF1.
Reserved Copyrights - 2013

373

Pathology Questions

MCQs

c. SFTPB.
d. SHH.
e. TTF1.
1216. Which of the following statements is correct about hepatoblastoma? (PIP
2011 D)
a. Crowded fetal pattern typically shows mitoses > or = 20/hpf.
b. Macrotrabecular pattern typically shows 3-5 cells thick trabeculae.
c. Immature fibrous tissue, cartilage and osteoid are the most
common

mesenchymal

elements

in

the

mixed

epithelial

mesenchymal subtype.
d. Diffuse

glutamine

synthetase

on

immunohistochemistrydistinguishes it from HCC.


e. Absence of glypican-3 on immunohistochemistry distinguishes it
from HCC.
1217. Which of the following argues against the diagnosis of undifferentiated
enbryonal sarcoma? (PIP 2011 D)
a. Occurrence in 6-10 age group.
b. Marked nuclear pleomorphism with rhabdomyoblast-like cells.
c. Entrapped hepatocytes and bile ductules at periphery.
d. PAS-diastase positive globules in cytoplasm and stroma.
e. Diffuse expression of myogenin by immunohistochemistry.
1218. Which of the following favors hepatoblastoma over hepatocellular
carcinoma? (PIP 2011 D)
a. Age above 10 years.
b. Glycogen storage disease.
c. Elevated serum AFP.
d. Androgen therapy for Fanconi anemia.
e. Large tumor cells resembling mature histiocytes.

Reserved Copyrights - 2013

374

Pathology Questions

MCQs

1219. How should most tumors with primarily non-mucinous lepidic growth
and a focus of invasive carcinoma measuring < or = 5mm be classified in
the IASLC/ATS/ERS classification scheme: (PIP 2011 D)
a. Adenocarcinoma, predominantly lepidic type.
b. Adenocarcinoma, mixed type.
c. Brochioloalveolar carcinoma.
d. Minimally invasive adenocarcinoma.
1220. In

case

otherwise

meeting

criteria

for

minimally

invasive

adenocarcinoma (MIA), the diagnosis of MIA is excluded if which of the


following is/are present? (PIP 2011 D)
a. The invasive component consists of a papillary or micropapillary
pattern.
b. Multiple foci of invasion less than 5 mm are present, but the total
amount of invasive tumor adds up to more than 5mm.
c. Pleural and/or lymphovascular invasion is present.
d. Multiple primary tumors are present.
1221. The vast majority of tumors previously classified as mucinous
bronchioloalveolar carcinoma will be classifed as which of the following
in the IASLC/ATS/ERS classification? (PIP 2011 D)
a. Adenocarcinoma in situ.
b. Minimally invasive carcinoma.
c. Invasive mucinous carcinoma.
d. Colloid carcinoma.
1222. Which ovarian tumor is most often associated with pseudoprecocious
puberty? (PIP 2011 D)
a. Choriocarcinoma.
b. Dysgerminoma.
c. Embryonal carcinoma.
d. Juvenile granulosa cell tumor.
e. Yolk sac tumor.
Reserved Copyrights - 2013

375

Pathology Questions

MCQs

1223. Which ovarian tumor is associated with tunor-infiltratinglymphocytes


and epithelioid granulomas? (PIP 2011 D)
a. Choriocarcinoma.
b. Dysgerminoma.
c. Embryonal carcinoma.
d. Juvenile granulosa cell tumor.
e. Yolk sac tumor.
1224. An ovarian tumor has the following immunohistochemical profile;
PLAP(+), OCT4(+), CD30(+), cytokeratin(+) and CD117(-). What is the
most likely diagnosis? (PIP 2011 D)
a. Choriocarcinoma.
b. Dysgerminoma.
c. Embryonal carcinoma.
d. Juvenile granulosa cell tumor.
e. Yolk sac tumor.
1225. This lesion consists of cytologically bland spindle cells loosely arranged in
an edematous, myxoid stroma that contains numerous eosinophils. The
spindle cells concentically whorl around thin-walled vessels: (PIP 2012 A)
a. Gastrointestinal stromal tumor.
b. Inflammatory fibroid polyp.
c. Inflammatory myofibroblastic tumor.
d. Leiomyoma.
e. Leiomyosarcoma.
1226. This lesion is typically composed of long, well-oriented fascicles of
elongate spindle cells that exhibit monotonus, mitotically inactive, cigarshaped nuclei and may exhibit extensive intratumoral hyalinization: (PIP
2012 A)
a. Gastrointestinal stromal tumor.
b. Inflammatory fibroid polyp.
Reserved Copyrights - 2013

376

Pathology Questions

MCQs

c. Inflammatory myofibroblastic tumor.


d. Leiomyoma.
e. Leiomyosarcoma.
1227. A 61-year old man presents with iron deficiency, and a 6.5 cm gastric
mass. Histologically, the mass exhibits foci of coagulative necrosis and is
composed of spindle and epithelioid cells, some exhibiting signet-ring
type morphology. The proliferative index is approximately 20% and
tumor cells express CD34 and DOG1. Tumor cells do not express muscle
actin, CD117 (KIT), or pan-cytokeratin and they are mucicarmine
negative. What is the best diagnosis? (PIP 2012 A)
a. Gastrointestinal stromal tumor.
b. Inflammatory fibroid polyp.
c. Inflammatory myofibroblastic tumor.
d. Leiomyoma.
e. Leiomyosarcoma.
1228. Which mesenteric lesion tends to have histologic variability from patient
to patient but typically contains areas of fibrosis, fat necrosis and/or
chronic inflammation? (PIP 2012 A)
a. Gastrointestinal stromal tumor.
b. Low grade fibromyxoid sarcoma.
c. Mesenteric fibromatosis.
d. Sclerosing mesenteritis.
e. Solitary fibrous tumor.
1229. Which bland-appearing lesion is locally aggressive and can develop late
metastases? (PIP 2012 A)
a. Gastrointestinal stromal tumor.
b. Low-grade fibromyxoid sarcoma.
c. Mesenteric fibromatosis.
d. Sclerosing mesenteritis.
e. Solitary fibrous tumor.
Reserved Copyrights - 2013

377

Pathology Questions

MCQs

1230. Which mesenteric lesion often involves the bowel wall and shows nuclear
positivity for -catenin by immunohistochemistry? (PIP 2012 A)
a. Gastrointestinal stromal tumor.
b. Low-grade fibromyxoid sarcoma.
c. Mesenteric fibromatosis.
d. Sclerosing mesenteritis.
e. Solitary fibrous tumor.
1231. Which tumor is typically positive for demin and myogenin? (PIP 2012 A)
a. Infantile fibromatosis.
b. Infantile fibrosarcoma.
c. Infantile myofibromatosis.
d. Malignant peripheral nerve sheath tumor.
e. Spindle cell rhabdomyosarcoma.
f. Synovial sarcoma.
1232. Which tumor is typically characterized by a biphasic pattern on histology
and cytokeratin positivity? (PIP 2012 A)
a. Infantile fibromatosis.
b. Infantile fibrosarcoma.
c. Infantile myofibromatosis.
d. Malignant peripheral nerve sheath tumor.
e. Spindle cell rhabdomyosarcoma.
f. Synovial sarcoma.
1233. Which tumor shares a cytogenetic abnormality with cellular mesoblastic
nephroma? (PIP 2012 A)
a. Infantile fibromatosis.
b. Infantile fibrosarcoma.
c. Infantile myofibromatosis.
d. Malignant peripheral nerve sheath tumor.
e. Spindle cell rhabdomyosarcoma.
Reserved Copyrights - 2013

378

Pathology Questions

MCQs

f. Synovial sarcoma.
1234. Whcih of the following histologic findings are characteristic of sclerosing
mesenteritis? (PIP 2012 A)
a. Atypical stromal cells with hyperchromatic, enlarged, irregular
nuclei.
b. Fibrosis, chronic inflammation and fat necrosis.
c. Infiltration into the adjacent bowel wall.
d. Storiform or fascicular growth pattern of spindle cells.
e. Myxoid change and keloid-type collagen.
1235. Which of the following is CORRECT about sclerosing mesenteritis? (PIP
2012 A)
a. Usually presents in childhood or young adulthood.
b. Most cases present with intestinal obstruction.
c. Smooth muscle actin immunohistochemistry is often positive.
d. Association with systemic IgG4 disease is well established.
e. Surgery is the treatment of choice in most cases.
1236. Which of the following features favors mesenteric fibromatosis over
sclerosing mesenteritis? (PIP 2012 A)
a. Fat necrosis.
b. Infiltration into the adjacent bowel wall.
c. Intralesional lymphoid aggregates.
d. Membranous staining with -catenin.
e. Dystrophic calcification.
1237. Translocation (t(X;18)(p11;q11))-SYT-SXX1 is usually seen in which
tumor? (PIP 2012 A)
a. Desmoplastic small round cell tumor.
b. Ewing sarcoma/primitive neuroectodermal tumor.
c. Lymphoma.
d. Neuroblastoma.
Reserved Copyrights - 2013

379

Pathology Questions

MCQs

e. Rhabdomyosarcoma.
f. Small cell carcinoma.
g. Synovial sarcoma.
1238. FLI-1 and CD99 are expressed in which one of these tumors? (PIP 2012 A)
a. Desmoplastic small round cell tumor.
b. Ewing sarcoma/primitive neuroectodermal tumor.
c. Lymphoma.
d. Neuroblastoma.
e. Rhabdomyosarcoma.
f. Small cell carcinoma.
g. Synovial sarcoma.
1239. Which of the following markers is useful in identifying Mallory-Denk
hyaline? (PIP 2012 A)
a. Cytokeratin 8/18.
b. Congo red.
c. Inhibin.
d. Mucicarmine.
e. P53.
f. Trichrome.
1240. Which of the following drugs is most likely to be a cause of
phospholipidosis? (PIP 2012 A)
a. Acetaminophen.
b. Amiodarone.
c. Metronidazole.
d. Nicotine.
e. NSAIDS.
1241. Which of the following can be seen in alcoholic liver disease but not in
non-alcoholic fatty liver disease? (PIP 2012 A)
a. Central vein obliteration.
Reserved Copyrights - 2013

380

Pathology Questions

MCQs

b. Cirrhosis.
c. Mallory-Denk hyaline.
d. Pan-lobular macrovesicular steatosis.
e. Periportal fibrosis.
1242. Which of the following is most correct regarding primary amyloidosis?
(PIP 2012 A)
a. T(11;14) is very specific for patients with primary amyloidosis.
b. Most patients with plasma cell myeloma or MGUS will showsome
manifestation of amyloidosis.
c. Most cases of AL amyloidosis occur in patients with a plasma cell
dyscrasia.
d. The amyloidogenic protein is usually an intact immunoglobulin
with both light and heavy chains.
1243. Which of the following is most correct regarding splenic amyloidosis?
(PIP 2012 A)
a. Splenic amyloidosis is usually a manifestation of generalized
disease.
b. Red pulp involvement is commonly seen in systemic amyloidosis.
c. Splenic involvement is more common in systemic than primary
amyloidosis.
d. When rupture occurs, it is usually secondary to a traumatic event.
1244. Atypical features of PEComas include all of the following EXCEPT: (PIP
2012 A)
a. Size >5.0 cm.
b. Mitotic activity >1/50 HPF.
c. Pushing border.
d. High cellularity.
1245. Immunoreactivity to which type of antibody can aid indifferentiating
angiomyolipoma from leiomyosarcoma? (PIP 2012 A)
Reserved Copyrights - 2013

381

Pathology Questions

MCQs

a. Cytokeratin AE1/AE3.
b. Desmin.
c. EMA.
d. HMB-45.
e. Smooth muscle actin.
1246. Which of the following statements regarding renal angiomyolipomas in
tuberous sclerosis patients versus the general population is not true? (PIP
2012 A)
a. The incidence is much higher in tuberous sclerosis patients.
b. The mean age at diagnosis is lower in tuberous sclerosis patients.
c. The tumor is most often bilateral in tuberous sclerosis patients.
d. Renal failure due to angiomyolipoma is common in tuberous
sclerosis patients.
e. Renal angiomyolipoma metastasis is more common in tuberous
sclerosis patients.
1247. Which histologic feature has been shown to correlate with more
aggressive behavior? (PIP 2012 A)
a. Clear cell change of the smooth muscle cells.
b. Epithelioid cytology of the smooth muscle cells.
c. Presence of multinucleated giant cells.
d. Prominent vascular component.
e. Vascular thrombosis.
1248. Which of ther following entities is most commonly seen in youngchildren?
(PIP 2012 A)
a. Acinar cell carcinoma.
b. Pancreatic endocrine tumor.
c. Pancreatoblastoma.
d. Solid pseudopapillary neoplasm.
e. Well-differentiated ductal adenocarcioma.

Reserved Copyrights - 2013

382

Pathology Questions

MCQs

1249. Which entity is rare in men? (PIP 2012 A)


a. Acinar cell carcinoma.
b. Pancreatic endocrine tumor.
c. Pancreatoblastoma.
d. Solid pseudopapillary neoplasm.
e. Well-differentiated ductal adenocarcioma.
1250. Which adult tumor is immunohistochemically positive for pancreatic
enzymes trypsin and chemotrypsin? (PIP 2012 A)
a. Acinar cell carcinoma.
b. Pancreatic endocrine tumor.
c. Pancreatoblastoma.
d. Solid pseudopapillary neoplasm.
e. Well-differentiated ductal adenocarcioma.
1251. Which neoplasm expresses smooth muscle & muscle-specific actin and it
is associated with mutations to the ALK gene on chromosome 2: (PIP
2012 B)
a. Dysgerminoma
b. Epithelioid sarcoma
c. Inflammatory myofibroblastic tumor
d. Intra-abdominal desmoplastic small round cell tumor
e. Renal clear cell carcinoma
1252. Which tumor is associated with a reciprocal translocation of EWSR-WT1
with gene fusion t(11:22)(p13;q12): (PIP 2012 B)
a. Dysgerminoma
b. Epithelioid sarcoma
c. Inflammatory myofibroblastic tumor
d. Intra-abdominal desmoplastic small round cell tumor
e. Renal clear cell carcinoma

Reserved Copyrights - 2013

383

Pathology Questions

MCQs

1253. An infant has a suprarenal mass. The mass is confined to the adrenal
gland and does not cross the midline. The surgeon describes a gross total
excision, although microscopic margins are positive. There is no evidence
of metastasis in sampled lymph nodes. Bilateral bone marrow biopsies
and aspirates are negative for tumor and MIBG scan shows no distant
metastasis. What is the INSS stage: (PIP 2012 B)
a. Stage 1
b. Stage 2A
c. Stage 3
d. Stage 4
e. Stage 4S
1254. Which of the following would be a poor prognostic feature for
neuroblastoma: (PIP 2012 B)
a. Duplication of 1p
b. FISH for N-Myc 2 copies
c. FISH for N-Myc 16 copies
d. Patient age less than 6 months
e. Stage 4S
1255. According to the International Neuroblastoma Pathology Classification
system, which of the following scenarios would be classified as
unfavorable histology: (PIP 2012 B)
a. 4-month old boy neuroblastoma, poorly differentiated, low MKI
b. 8-month old girl neuroblastoma, differentiating, intermediate
MKI
c. 12-month old girl neuroblastoma, poorly differentiated, high MKI
d. 2-year old boy neuroblastoma, differentiating, low MKI
e. 7-year old boy ganglioneuroblastoma, intermixed type
1256. Which lesion can be found in Mazabraud syndrome: (PIP 2012 B)
a. Deep aggressive angiomyxoma
b. Intramuscular myxoma
Reserved Copyrights - 2013

384

Pathology Questions

MCQs

c. Low grade fibromyxoid sarcoma


d. Myxofibrosarcoma
e. Myxoid liposarcoma
f. Myxoid neurofibroma
1257. A translocation involving t(12:16)(q13;p11) is characteristic of which
lesion: (PIP 2012 B)
a. Deep aggressive angiomyxoma
b. Intramuscular myxoma
c. Low grade fibromyxoid sarcoma
d. Myxofibrosarcoma
e. Myxoid liposarcoma
f. Myxoid neurofibroma
1258. In which lesion would you expect to see giant collagen rosettes: (PIP 2012
B)
a. Deep aggressive angiomyxoma
b. Intramuscular myxoma
c. Low grade fibromyxoid sarcoma
d. Myxofibrosarcoma
e. Myxoid liposarcoma
f. Myxoid neurofibroma
1259. Using the AJCC 7th edition guidelines, what is the correct classification of
an urothelial carcinoma with deep muscularis propria invasion with
extension into the prostatic urethral without prostatic stromal invasion:
(PIP 2012 B)
a. T2a
b. T2b
c. T3a
d. T4a
e. T4b

Reserved Copyrights - 2013

385

Pathology Questions

MCQs

1260. Which prostate specific immunohistochemical stain has a finely dotted


perinuclear cytoplasmic staining pattern: (PIP 2012 B)
a. PSA
b. Prostein (p501S)
c. P63
d. PSMA
e. HMWCK
1261. Which of the following lesions is histologically characterized by
concentric arrangement of fibrocartilage around hyaline cartilage or
bone: (PIP 2012 B)
a. Synovial chondromatosis
b. Loose bodies
c. Osteoarthritis
d. Periosteal chondroma
1262. Which of the following histologic characteristics can be seen in
chondrosarcoma, but is usually not observed in synovial chondromatosis:
(PIP 2012 B)
a. Hypercellular hyaline cartilage
b. Ossification
c. Necrosis, increased cellularity and tumor liquefaction, invasion
into or from bone
d. Mild nuclear atypia
1263. Which of the following joints are most commonly affected by synovial
chondromatosis: (PIP 2012 B)
a. Elbow & knee
b. Temperomandibular joint
c. Ankle and sital interphalangeal joints
d. Proximal & distal interphalangeal joints

Reserved Copyrights - 2013

386

Pathology Questions

MCQs

1264. Which ovarian tumor is characterized by spindle cells with moderate to


severe nuclear atypia and >4 mitosis per 10 HPFs: (PIP 2012 B)
a. Cellular fibrothecoma
b. Cellular leiomyoma
c. Endometrial stromal sarcoma
d. Fibrosarcoma
e. GIST
1265. Which ovarian tumor is unlikely to recur unless associated with
adhesions or rupture: (PIP 2012 B)
a. Cellular fibrothecoma
b. Cellular leiomyoma
c. Endometrial stromal sarcoma
d. Fibrosarcoma
e. GIST
1266. Which ovarian tumor is most likely to display tongue-like projections at
its peripheral borders and CD10 immunoreactivity: (PIP 2012 B)
a. Cellular fibrothecoma
b. Cellular leiomyoma
c. Endometrial stromal sarcoma
d. Fibrosarcoma
e. GIST
1267. Which tumor is poorly circumscribed, comprised of bland spindle cells
and expresses -catenin: (PIP 2012 B)
a. Desmoid tumor
b. Malignant mesothelioma, desmoplastic type
c. Schwannoma
d. Solitary fibrous tumor
e. Synovial sarcoma

Reserved Copyrights - 2013

387

Pathology Questions

MCQs

1268. Which tumor may be associated with hypoglycemia or hypertrophic


pulmonary osteoarthropathy: (PIP 2012 B)
a. Desmoid tumor
b. Malignant mesothelioma, desmoplastic type
c. Schwannoma
d. Solitary fibrous tumor
e. Synovial sarcoma
1269. Which tumor may express CK & bcl-2 but is usually negative for CD34,
and is additionally characterized by t(X:18): (PIP 2012 B)
a. Desmoid tumor
b. Malignant mesothelioma, desmoplastic type
c. Schwannoma
d. Solitary fibrous tumor
e. Synovial sarcoma
1270. The presence of periportal PAS positive intracytoplasmic globules with
PiZZ present on isoelectric focusing is diagnostic for: (PIP 2012 B)
a. Alpha-1 antitrypsin deficiency
b. Wilsons disease
c. Hepatitis C
d. Primary biliary cirrhosis
1271. The most common cause of PAS positive intracytoplasmic globules
adjacent to the central vein is: (PIP 2012 B)
a. Hepatocellular carcinoma
b. Centrilobular congestion
c. Alpha-1 antitrypsin deficiency
d. Hepatitis C
1272. There is a high prevalence of what viral infection associated with alpha-1
antitrypsin deficiency: (PIP 2012 B)
a. EBV
Reserved Copyrights - 2013

388

Pathology Questions

MCQs

b. Hepatitis B
c. Hepatitis E
d. Hepatitis A
1273. Which entity is characterized by splenic red pulp involvement and is
positive for flowcytometry markers CD103, CD11c and CD25: (PIP 2012
B)
a. Hepatosplenic T-cell lymphoma
b. Sickle cell crisis
c. Hairy cell leukemia
d. Splenic marginal zone lymphoma
e. Gaucher disease
f. Classical Hodgkin lymphoma
g. Aggressive NK-cell lymphoma/leukemia
1274. Which entity is a hematopoietic malignancy of predominantly activated
cytotoxic cells, positive for perforin and granzyme: (PIP 2012 B)
a. Hepatosplenic T-cell lymphoma
b. Sickle cell crisis
c. Hairy cell leukemia
d. Splenic marginal zone lymphoma
e. Gaucher disease
f. Classical Hodgkin lymphoma
g. Aggressive NK-cell lymphoma/leukemia
1275. A patient with type B symptoms has nodular abnormalities in the spleen
grossly. A well-sampled (representative) flow cytometry analysis did not
reveal T-cell or B-cell abnormalities. Histology reveals scattered large binucleated and mononuclear cells with a prominent nucleolus. What is the
best possible diagnosis from the above list: (PIP 2012 B)
a. Hepatosplenic T-cell lymphoma
b. Sickle cell crisis
c. Hairy cell leukemia
Reserved Copyrights - 2013

389

Pathology Questions

MCQs

d. Splenic marginal zone lymphoma


e. Gaucher disease
f. Classical Hodgkin lymphoma
g. Aggressive NK-cell lymphoma/leukemia
1276. Which of the following best describes focal nodular hyperplasia of the
liver: (PIP 2012 B)
a. Multiple hepatic nodules ranging from 1 to 2 mm in size
b. Large nodule (1 to 30 cm) composed of clonal hepatocytes without
associated portal tracts
c. Well-demarcated nodule near the liver capsule, with a central
fibrosed region
d. Well-circumscribed nodule measuring up to 3 cm in a background
of cirrhosis
1277. Which entity has been associated with a risk of malignant transformation:
(PIP 2012 B)
a. Focal nodular hyperplasia
b. Hepatic adenoma
c. Nodular regenerative hyperplasia
d. Large benign regenerative nodule
1278. Which entity is typically associated with oral contraceptive use: (PIP 2012
B)
a. Focal nodular hyperplasia
b. Hepatic adenoma
c. Nodular regenerative hyperplasia
d. Large benign regenerative nodule
1279. Which of the following lesions is characterized by sclerotic germinal
centres, lollipop lesions and onion-skinning of mantle zones: (PIP 2012 C)
a. Follicular hyperplasia
b. Follicular lymphoma
Reserved Copyrights - 2013

390

Pathology Questions

MCQs

c. Hyaline vascular variant of Castleman disease


d. Manle cell lymphoma
e. Plasma cell variant of Castleman disease
f. Thymoma
1280. Which of the following lesions is characterized by an interfollicular
expansion of large sheets of mature-appearing plsams cells: (PIP 2012 C)
a. Follicular hyperplasia
b. Follicular lymphoma
c. Hyaline vascular variant of Castleman disease
d. Manle cell lymphoma
e. Plasma cell variant of Castleman disease
f. Thymoma
1281. Which of the following is characterizd by an atypical lymphoid infiltrate
that effaces lymph node architecture and is comprised of monomorphic
small to intermediate sized CD5 ositive B-cells with irregular nuclear
contours and has a balanced translocation involving the CCND1 (cyclin
D1) gene: (PIP 2012 C)
a. Follicular hyperplasia
b. Follicular lymphoma
c. Hyaline vascular variant of Castleman disease
d. Manle cell lymphoma
e. Plasma cell variant of Castleman disease
f. Thymoma
1282. Which entity is characterized by mutations in KIT or PDGFRA and is
usually positive for DOG1 by immunohistochemistry? (PIP 2012 C)
a. Epithelioid gastrointestinal stromal tumor
b. Mixed adenoneuroendocrine carcinoma
c. Neuroendocrine carcinoma
d. Neuroendocrine tumor

Reserved Copyrights - 2013

391

Pathology Questions

MCQs

1283. A tumor with monotonous neuroendocrine morphology shows strong


synaptophysin positivity by immunohistochemistry, no mitotic activity
and a Ki-67 index of 4%. What is the best diagnosis? (PIP 2012 C)
a. Epithelioid gastrointestinal stromal tumor
b. Mixed adenoneuroendocrine carcinoma
c. Neuroendocrine carcinoma, WHO grade 3
d. Neuroendocrine tumor, WHO grade 1
e. Neuroendocrine tumor, WHO grade 2
1284. In which entity should the pathologist count mitotic figures in 50 hihg
power fields as part of risk stratification? (PIP 2012 C)
a. Epithelioid gastrointestinal stromal tumor
b. Neuroendocrine tumor, WHO grade 1
c. Neuroendocrine tumor, WHO grade 2
d. All of the above
1285. Which tumor tumor is characterized by the presence of intracytoplasmic
PAS positive diastase resistant rhomboid cyrstals: (PIP 2012 C)
a. Alveolar soft part sarcoma
b. Granular cell tumor
c. Malignant rhabdoid tumor
d. Paraganglioma
e. Rhabdomyosarcoma
1286. Which tumor is characterized by loss of INI1 gene product? (PIP 2012 C)
a. Alveolar soft part sarcoma
b. Granular cell tumor
c. Malignant rhabdoid tumor
d. Paraganglioma
e. Rhabdomyosarcoma
1287. Which tumor is differentiated from alveolar soft part sarcoma on the basis
of positivity for chromogranin and synaptophysin: (PIP 2012 C)
Reserved Copyrights - 2013

392

Pathology Questions

MCQs

a. Alveolar soft part sarcoma


b. Granular cell tumor
c. Malignatn rhabdoid tumor
d. Paraganglioma
e. Rhabdomyosarcoma
1288. Which of the following features favors mesenchymal hamartoma over
infantile hemangioma: (PIP 2012 C)
a. Branching bile duct structures
b. Cavernous vascular channels
c. Entrapped hepatocytes and bile ducts
d. Fibromyxoid stroma
e. Immunoreactivity for CD31
1289. Which of the following is correct about undifferentiated embryonal
sarcoma: (PIP 2012 C)
a. Cytogenetic changes similar to mesenchymal hamartoma.
b. Diffuse

expression

of

desmin

and

myogenin

on

immunohistochemistry.
c. Mild nuclear pleomorphism.
d. PAS-positive diastase sensitive globules in cytoplasm and stroma.
e. Peak incidence in 0 3 year age group
1290. Which of the following features favors mesenchymal hamartoma over
mixed epithelial-mesenchymal subtype of hepatoblastoma: (PIP 2012 C)
a. Absence of bile duct elements.
b. Glandular elements with mucinous epithelium.
c. Markedly elevated AFP
d. Normal appearing hepatocytes
e. Osteoid formation in stroma.
1291. Which immunohistochemical panel below is likely to be positive in
oncocytoma: (PIP 2012 C)
a. CD10, RCC, racemase
Reserved Copyrights - 2013

393

Pathology Questions

MCQs

b. CK7, Ksp-cadherin
c. KIT, S100A1, PAX2
d. Vimentin, CD10, racemase
1292. Which of the following is not typical of chromophobe renal cell
carcinoma?(PIP 2012 C)
a. Abundant mitochondria
b. Binucleation
c. Colloidal iron positivity
d. Perinuclear halos
e. Solid growth of neoplastic cells.
1293. A renal tumor composed of cells with abundant granular cytoplasm
demonstrates immunoreactivity with antibodies against HMB45 while
shows no reactivity with antibodies against S100A1 or PAX2. The likely
diagnosis is: (PIP 2012 C)
a. Chromophobe renal cell carcinoma
b. Epithelioid aniomyolipoma
c. Oncocytoma
d. Papillary renal cell carcinoma
1294. Which of the following is least likely to be associated with villous synovial
hypertrophy: (PIP 2012 C)
a. Articular prosthesis
b. Hemosiderotic synovitis
c. Pigmented villonodular synovitis
d. Rheumatoid synovitis
1295. Which of the following is most indicative of a malignant pigmented
villonoduar synovitis: (PIP 2012 C)
a. Bone infiltration and destruction of large segments of bone.
b. Increased mitoses by itself.

Reserved Copyrights - 2013

394

Pathology Questions

MCQs

c. Marked nuclear atypia, zonal necrosis, sarcomatoid growth pattrn


and > 20 mitoses per 10 high power fields.
d. Osteoclast-like giant cell reaction.
1296. Which of the following findings are consistent with a pigmented
villonodular synovitis. (PIP 2012 C)
a. Lobulated architecture with osteoclast-like giant cells.
b. Villous architecture with foreign body macrophages.
c. Villous architecture with osteoclast-type giant cells and xanthoma
cells.
d. Villous architecture with pigment limited to the surface of the
synovium.
1297. Which of the following immunophenotypes is most compatible with a
diagnosis of Burkitt lymphoma. (PIP 2012 C)
a. CD20+ve, Bcl-2+ve, CD10-ve, Bcl-6-ve, CD43+ve
b. CD20+ve, Bcl-2-ve, CD10+ve, Bcl-6+ve, CD43+ve
c. CD20+ve, Bcl-2+ve, CD10+ve, Bcl-6-ve, CD43-ve
d. CD20+ve, Bcl-2-ve, CD10-ve, Bcl-6-ve, CD43+ve
1298. Which statement is ost correct concerning Burkitt lymphoma? (PIP 2012
C)
a. Burkitt lymphoma commonly presents as acute leukemia.
b. Burkitt lymphoma consists of large transformed cells with nuclear
pleomorphism.
c. Cases that do not morphologically resemble classic Burkitt
lymphoma can share the genetic signature of Burkitt lymphoma.
d. The most common genetic abnormality seen in t(14:18).
1299. Which entity is charachterized by the translocation t(X:17): (PIP 2012 C)
a. Alveolar soft part sarcoma
b. Metastatic malignant melanoma
c. Metastatic renal cell carcinoma
d. Paraganglioma
Reserved Copyrights - 2013

395

Pathology Questions

MCQs

e. Well-differentiated neuroendocrine carcinoma


1300. Which entity may exhibit mutations in mitochondrial succinate
dehydrogenase subunit genes: (PIP 2012 C)
a. Alveolar soft part sarcoma
b. Metastatic malignant melanoma
c. Metastatic renal cell carcinoma
d. Paraganglioma
e. Well-differentiated neuroendocrine carcinoma
1301. Which of the following is correct about PanNET? (PIP 2012 C)
a. Cytologically bland tumors may show lymph node metastasis.
b. Low grade and high-grade neuroendocrive carcinomas are
distinguished by size.
c. Most insulinomas are metastatic at presentation
d. Most tumors larger than 2 cm are malignant.
1302. Which immunostain panel is useful in diagnosing solid pseudopapillary
tumor? (PIP 2012 C)
a. Amylase, lipase, trypsin
b. -catenin, E-cadherin, progesterone receptor
c. Estrogen receptor, somatostatin, CD10
d. Keratin, chromogranin, Mib-1
1303. Which tumor is commonly associated with mutations in exon 3 of the
beta-catenin gene? (PIP 2012 D)
a. Pancreatic acinar cell carcinoma
b. Pancreatic ductal adenocarcinoma
c. Pancreatic pseudocyst
d. Pancreatoblastoma
e. Solid pseudopapillary neoplasm
f. Pancreatic neuroendocrine tumor

Reserved Copyrights - 2013

396

Pathology Questions

MCQs

1304. Which tumor typically occurs in patients under 10 years of age and
displays squamoid corpuscles on microscopy? (PIP 2012 D)
a. Pancreatic acinar cell carcinoma
b. Pancreatic ductal adenocarcinoma
c. Pancreatic pseudocyst
d. Pancreatoblastoma
e. Solid pseudopapillary neoplasm
f. Pancreatic neuroendocrine tumor
1305. Which tumor stains strongly for chromogranin and displays a
membranous staining pattern with CD99? (PIP 2012 D)
a. Pancreatic acinar cell carcinoma
b. Pancreatic ductal adenocarcinoma
c. Pancreatic pseudocyst
d. Pancreatoblastoma
e. Solid pseudopapillary neoplasm
f. Pancreatic neuroendocrine tumor
1306. Which of the following tumors arising has the greatest potential for
malignant transformation? (PIP 2012 D)
a. Cutaneous neurofibroma
b. Optic pathway glioma
c. Palisaded encapsulated neuroma
d. Plexiform neurofibroma
e. Plexiform schwannoma
1307. All of the following are true of malignant peripheral nerve sheath tumor
except? (PIP 2012 D)
a. Compared to the spindle cell variant, the epithelioid variant shows
more focal and less intense S100 positivity.
b. Elongated wavy nuclei help identify neural differentiation.
c. The sciatic nerve is commonly involved.
d. They typically have uniform cellularity throught the tumor.
Reserved Copyrights - 2013

397

Pathology Questions

MCQs

e. When associated with NF1 these lesions occur at young ages.


1308. Which of the following is true regarding peineuromas? (PIP 2012 D)
a. Lesional cells show EMA positivity.
b. Recent evidence suggests they may be a reactive process.
c. They

are

homogeneous

in

their

histologic

and

clinical

presentation.
d. They commonly undergo malignant transformation.
e. They represent a neoplastic expansion of the most inner portion of
the peripheral nerve.
1309. Which of the following statements about molar pregnancy is true? (PIP
2012 D)
a. By immunohistochemistry, p57 is typically positive in partial
moles and negative in complete moles.
b. Complete hydatidiform mole is typically associated with a
complete well-formed fetus.
c. Partial hydatidiform mole typically shows no associated fetal
parts.
d. The placental karyotype of complete hydatidiform mole is typically
triploid.
e. The placental karyotype of partial hydatidiform mole is typically
diploid.
1310. Which of the following lesions is a consequence of fetal thrombotic
vasculopathy? (PIP 2012 D)
a. Complete hydatidiform mole.
b. Hemorrhagic endovasculitis.
c. Mesenchymal dysplasia.
d. Partial hydatidiform mole.
e. Villitis of unknown etiology.
1311. Which of the following genetic syndromes is associated with placental
mesenchymal dysplasia? (PIP 2012 D)
Reserved Copyrights - 2013

398

Pathology Questions

MCQs

a. Beckwith-Wiedemann syndrome.
b. Down syndrome.
c. Gonadal dysgenesis.
d. Trisomy 18.
e. Turner syndrome mosaic.
1312. Which of the following immunophenotypic findings is most typical of
mantle cell lymphoma? (PIP 2012 D)
a. CD5+ CD19+ CD20+ CD23- IgM+
b. CD5+ CD19+ CD20+ CD23+ CD38+
c. CD5- CD19- CD20+ CD23- IgM+
d. CD5- CD19- CD20+ CD23+ IgD+
1313. Which of the following chromosomal translocations leading to
overexpression of cyclin D1 is associated with mantle cell lymphoma?
(PIP 2012 D)
a. t(8:14)
b. t(11:14)
c. t(14:18)
d. t(3:14)
e. t(2:5)
1314. Which of the following statements concerning splenic involvement by Bcell lymphomas is true? (PIP 2012 D)
a. All B-cell lymphomas involving the spleen are characteristically
positive for CD23.
b. Hairy cell leukemia/lymphoma of the spleen is an aggressive B cell
lymphoma
c. Mantle cell lymphomas are typically distinguished from chronic
lymphocytic leukemia/small lymphocytic lymphoma by the
absence of CD23 immunoreactivity.
d. Splenic marginal zone lymphoma are characterized by a recurring
t(8:14) translocation.
Reserved Copyrights - 2013

399

Pathology Questions

MCQs

e. Splenic marginal zone lymphoma may be discriminated from hairy


cell leukemia/lymphoma involvement of the spleen by the
presence of CD123 and CD11c in SMZL tumor cells.
1315. All of the following histologic features may be seen in hibernoma except?
(PIP 2012 D)
a. Cytoplasmic cross striations.
b. Granular eosinophilic cells.
c. Myxoid stroma.
d. Numerous univacuolated white fat cells.
e. Spindle cells.
1316. Which of the following is most characteristic of the vacuolated cells of
hibernoma? (PIP 2012 D)
a. They contain glycogen.
b. They contain lipid.
c. They contain mucin.
d. They express CD34.
e. They express desmin.
1317. Hyperchromatic, scalloped nuclei are most characteristic of which of the
following soft tissue tumors? (PIP 2012 D)
a. Adult rhabdomyoma.
b. Granular cell tumor.
c. Hibernoma.
d. Lipoma.
e. Liposarcoma.
1318. Which tumor typically expresses CD117 (KIT) and Oct4 and is associated
with abnormalities of chromosome 12? (PIP 2012 D)
a. Clear cell carcinoma.
b. Diffuse large B-cell lymphoma.
c. Dysgerminoma.
Reserved Copyrights - 2013

400

Pathology Questions

MCQs

d. Metastatic renal clear cell carcinoma.


1319. Which neoplasm is most commonly associated with endometriosis? (PIP
2012 D)
a. Clear cell carcinoma of the ovary.
b. Diffuse large B-cell lymphoma.
c. Dysgerminoma.
d. Metastatic renal clear cell carcinoma.
1320. Which tumor is more commonly associated with gonodal dysgenesis
syndromes such as testicular feminization (46XY)? (PIP 2012 D)
a. Clear cell carcinoma.
b. Diffuse large B-cell lymphoma.
c. Dysgerminoma.
d. Metastatic renal clear cell carcinoma.
1321. Which disorder is a diffuse interstitial process characterized by a
polyclonal population of B-cells and T-cells and is most often seen in the
setting of collagen vascular disease or immunodeficiency syndromes?
(PIP 2012 D)
a. Extranodal marginal zone B-cell lymphoma.
b. Lymphocytic interstitial pneumonia.
c. Lymphomatoid granulomatosis.
d. Mantle cell lymphoma.
e. Nodular lymphoid hyperplasia.
1322. Which disorder forms a mass lesion with architectural effacement and is
morphologically well circumscribed, lacks lympho-epithelial lesions or
pleural invasion, has prominent germinal centre formation and
interfollicular plasma cells? (PIP 2012 D)
a. Extranodal marginal zone B-cell lymphoma.
b. Lymphocytic interstitial pneumonia.
c. Lymphomatoid granulomatosis.
Reserved Copyrights - 2013

401

Pathology Questions

MCQs

d. Mantle cell lymphoma.


e. Nodular lymphoid hyperplasia.
1323. Which disorder is EBV driven and characterized by a mixture of atypical
B-cells and non-neoplastic T-cells arragned in angiocentric pattern? (PIP
2012 D)
a. Extranodal marginal zone B-cell lymphoma.
b. Lymphocytic interstitial pneumonia.
c. Lymphomatoid granulomatosis.
d. Mantle cell lymphoma.
e. Nodular lymphoid hyperplasia.
1324. The three characteristic features of fibrolamellar hepatocellular
carcinoma are: (PIP 2012 D)
a. Eosinophilic granular hepatocytes, prominent hepatocyte nucleoli
and lamellar fiborsis.
b. Eosinophilic granular hepatocytes, lamellar fibrosis and abundant
mitoses.
c. Lamellar fibrosis, pseudoglandular formation and eosinophilic
granular hepatocytes.
d. Mucin

production,

lamellar

fibrosis

and

pseudoglandular

formation.
e. Tumor necrosis, prominent hepatocyte nucleoli and lamellar
fibrosis.
1325. The presence of a central scar in fibrolamellar carcinoma seen by
computer tomography abdominal imaging will raise the possibility of
which other entity? (PIP 2012 D)
a. Focal nodular hyperplasia.
b. Metastatic adenocarcinoma.
c. Metastatic melanoma.
d. Scirrhous variant of conventional hepatocellular carcinoma.

Reserved Copyrights - 2013

402

Pathology Questions

MCQs

1326. A tumor mass attached to the adrenal gland was excised and
histologically it showed mature adipocytes. There was a single focus of
neutrophils, restricted to a congested blood vessel. What is the most
compatible diagnosis? (PIP 2012 D)
a. Extramedullary hematopoiesis.
b. Lipoma.
c. Liposarcoma.
d. Myeloid sarcoma.
e. Myelolipoma.
1327. A 56-year-old patient presents with a recent onset of bluish swelling in
the fat pad of her chin; she has fever, petechiae and malaise. Biopsy of the
swelling reveals immature atypical cells with Aur rods and rare granules,
which stain positive for CD34. What is the likely diagnosis? (PIP 2012 D)
a. Extramedullary hematopoiesis.
b. Lipoma.
c. Liposarcoma.
d. Myeloid sarcoma.
e. Myelolipoma.
1328. Which entity is composed of adipocytes and myeloid cells that, according
to recent evidence, belong to the same clone? (PIP 2012 D)
a. Extramedullary hematopoiesis.
b. Lipoma.
c. Liposarcoma.
d. Myeloid sarcoma.
e. Myelolipoma.
1329. Which of the following best describes angiosarcoma of the liver? (PIP
2012 D)
a. Multiple tumor nodules alternating with blood filled cavities.
b. Spindle cell tumor cells with intracytoplasmic capillary lumens.
c. Trabecular arrangements of malignant endothelial cells.
Reserved Copyrights - 2013

403

Pathology Questions

MCQs

d. Tumor nodules composed of malignant spindled cells associated


with a fibrotic stroma.
1330. Which entity has been associated with arsenic exposure? (PIP 2012 D)
a. Angiosarcoma.
b. Epithelioid hemangioendothelioma.
c. Metastatic sarcoma.
d. Poorly differentiated hepatocellular carcinoma.
1331. All of the following statements are associated with epithelioid
hemangioendothelioma (EHE) of the liver except? (PIP 2012 D)
a. EHE is typically associated with bilateral hepatic lobe involvement.
b. EHE may express both CD31 and keratin.
c. Females are more frequently affected by EHE as compared to
males.
d. Surgical resection is not effective treatment for EHE.
1332. Which of the following DNA abnormalities would support a diagnosis of
melanoma over melanocytic nevus? (PIP 2013 A)
a. Gains of 11p
b. GNAQ gene mutation
c. Multiple DNA gains and losses
d. t(12:22)(q13:q12)
e. V600E BRAF gene mutation
1333. Which of the following parameters is used for pathologic tumor staging in
melanoma? (PIP 2013 A)
a. Clarks level of invasion
b. Mitotic count per mm2
c. Presence of regression
d. Tumor infiltrating lymphocytes
e. Tumor largest diameter

Reserved Copyrights - 2013

404

Pathology Questions

MCQs

1334. What is the main reason for evaluating BRAF gene status in melanoma?
a. Presence of V600E BRAF mutation is associated with resistance to
Vemurafenib treatment.
b. Presence of V600E BRAF mutation is associated with resistance to
Ipilimumab treatment.
c. Presence of V600E BRAF mutation is associated with tumor
response to Vemurafenib treatment.
d. Presence of V600E BRAF mutation is associated with tumor
response to Ipilimumab treatment.
e. Serves as a diagnostic test for melanoma
1335. Which tumor is characterized by an immature mesenchymal component
and -catenin mutations? (PIP 2013 A)
a. Carcinosarcoma
b. Pleomorphic carcinoma
c. Pulmonary blastoma
d. Squamous cell carcinoma
1336. Which tumor is defined in the current WHO classification as a biphasic
tumor with heterologous differentiation such as bone, cartilage or skeletal
muscle? (PIP 2013 A)
a. Carcinosarcoma
b. Pleomorphic carcinoma
c. Pulmonary blastoma
d. Squamous cell carcinoma
1337. Chordoma can typically be distinguished from chondrosarcoma by which
of the following immunohistochemical stains? (PIP 2013 A)
a. Brachyury, D2-40 and pancytokeratin
b. Galectin-3, pancytokeratin and S100
c. GFAP, pancytokeratin and CD99
d. S100, chromogranin and CK7
e. S100, pancytokeratin and GFAP
Reserved Copyrights - 2013

405

Pathology Questions

MCQs

1338. Which of the following statements concerning chordoma is false? (PIP


2013 A)
a. Chordomas classically demonstrate physaliphorous cells on
histologic examination.
b. Chordomas typically affect younger adults and have no risk for
recurrence.
c. Chordomas

may

be

distinguished

from

myxopapillary

ependymoma by immunohistochemical staining characteristics.


d. The candidate gene locus for chordoma has been mapped to
chromosome 1p.
e. The primary treatment approach for chordoma is en bloc excision
with negative margins.
1339. Which of the following entities is characterized by splenic white pulp
involvement and by flow cytometry shows a population that is positive
for markders CD19 and CD20 with light chain restriction? (PIP 2013 A)
a. Classical Hodgkin lymphoma
b. Primary myelofibrosis
c. Splenic extramedullary hematopoiesis secondary to primary
myelofibrosis
d. Splenic marginal zone lymphoma
1340. A patient with HIV infection shows symptoms suggestive of bone marrow
failure and pancytopenia. Large binucleate cells with prominent
eosinophilic nucleoli are seen in the bone marrow biopsy and within
spleen. The large atypical cells are CD30 positive and CD15 negative.
What is the diagnosis? (PIP 2013 A)
a. Classical Hodgkin lymphoma
b. Hairy cell leukemia
c. Primary myelofibrosis
d. Splenic marginal zone lymphoma

Reserved Copyrights - 2013

406

Pathology Questions

MCQs

1341. A characteristic morphology seen in primary myelofibrosis which is less


common in essential thrombocytosis or chronic myeloid leukemia is the
following: (PIP 2013 A)
a. Megakaryocytes showing compact cloud like nucleus with
condensed chromatin.
b. Enlarged megakaryocytes with branched nuclear lobes.
c. Mononuclear megakaryocytes in tight clusters.
d. Megakaryocytes with crinkledtissue paper cytoplasm.
1342. Which of the following best describes undifferentiated embryonal
sarcoma of liver? (PIP 2013 A)
a. Multiple tumor nodules formed by small undifferentiated cells.
b. Primitive small tumor cells, resembling fetal liver.
c. Polymorphous spindle, ovoid and anaplastic tumor cells with
intracytoplasmic globules.
d. Tumor nodules composed of malignant spindled cells associated
with cytoplasmic vacuoles.
1343. Which entity shares 19q abnormalities with some cases of embryonal
sarcoma, suggesting a possible shared pathway? (PIP 2013 A)
a. Bile duct hamartoma
b. Hepatobiliary rhabdomyosarcoma
c. Hepatoblastoma
d. Mesenchymal hamartoma
1344. Positive staining for which immunohistochemical marker(s) can be seen
in undifferentiated embryonal sarcoma of the liver? (PIP 2013 A)
a. Cytokeratin
b. Desmin
c. Vimentin
d. All of the above
e. None of the above

Reserved Copyrights - 2013

407

Pathology Questions

MCQs

1345. Which of the following is false of goblet cell carcinoids? (PIP 2013 A)
Immunohistochemical stains are positive for CEA.
a. Metastasis to the ovary can resemble poorly differentiated
adenocarcinoma.
b. The growth pattern in the appendix is submucosal.
c. They are more common in males.
d. They have both glandular and endocrine components.
1346. Which of the following findings is more suggestive of an ovarian primary
than a metastasis? (PIP 2013 A)
a. Bilaterality
b. Expansile pattern of growth
c. Nodular pattern of growth
d. Tumor on ovarian surface
e. Young age at presentation
1347. The organism responsible for causing Chagas disease is: (PIP 2013 A)
a. Cytomegalovirus
b. Epstein Barr virus
c. Listeria monocytogenes
d. Tinea cruris
e. Trypanosoma cruzi
1348. The chronic phase of T. cruzi infection is characterized by all of the
following except: (PIP 2013 A)
a. Cardiac aneurysm
b. Dilated colon
c. Few intracellular amastigote forms
d. Parasitemia
e. Spans many years duration
1349. Immunosuppression is a risk factor for systemic infection by which of the
following: (PIP 2013 A)
Reserved Copyrights - 2013

408

Pathology Questions

MCQs

a. Candida
b. Cytomegalovirus
c. Toxoplasma gondii
d. Trypanosoma cruzi
e. All of the above
1350. Regarding myogenin immunohistochemial staining, which of the
following

patterns

correlates

with

embryonal

and

alveolar

rhabdomyosarcoma staining, respectively? (PIP 2013 A)


a. Embryonal rhabdomyosarcoma shows cytoplasmic staining and
alveolar rhabdomyosarcoma shows strong diffuse nuclear staining.
b. Embryonal

rhabdomyosarcoma

is

positive

and

alveolar

rhabdomyosarcoma is negative for myogenin.


c. Embryonal rhabdomyosarcoma shows membranous staining and
alveolar rhabdomyosarcoma shows cytoplasmic staining.
d. Embryonal rhabdomyosarcoma shows variable nuclear staining
and alveolar rhabdomyosarcoma shows strong diffuse nuclear
staining.
1351. Which of the following genetic changes is most commonly found in
alveolar rhabdomyosarcoma? (PIP 2013 A)
a. N-MYC
b. t(1:13)
c. t(2:13)
d. t(11:22)
1352. Which of the following is not a favorable prognostic factor in
rhabdomyosarcoma? (PIP 2013 A)
a. Age less than 10 years
b. Orbital location
c. Parameningeal location
d. Size less than 5.0 cm

Reserved Copyrights - 2013

409

Pathology Questions

MCQs

1353. Which staining pattern is most consistent with chromophobe RCC? (PIP
2013 A)
a. Vimentin - , Hales colloidal iron + (diffuse), CK7 + (diffuse), CD10 ,
CD117 +
b. Vimentin -, Hales colloidal iron + (focal), CK7 + (focal), CD10 +,
CD117 +
c. Vimentin -, Hales colloidal iron -, CK7 -, CD10 -, CD117
d. Vimentin +, Hales colloidal iron -, CK7 -, CD10 +, CD117
1354. Which histologic feature has been associated with metastases and local
recurrence?
a. Abundant microvesicles on ultrastructural examination.
b. Binucleation
c. Fuhrman nuclear grade
d. Sarcomatoid transformation
1355. Which of the following statements regarding hereditary chromophobe
RCC is not true?
a. Aggressive behavior has been associated with the hereditary form
of this tumor.
b. Birt-Hogg-Dube syndrome is linked to FLCN gene mutation
mapped to chromosome 17p11.2
c. Birt-Hogg-Dube syndrome is an autosomal dominant condition
characterized y bening cutaneous tumors, pulmonary cysts and
multifocal and bilateral renal cancer.
d. The most common renal tumor seen in Birt-Hogg-Dube syndrome
is the oncocytic hybrid tumor.
1356. Which finding gross or microscopic finding is least suggestive of extraovarian origin for an ovarian mucinous tumor? (PIP 2013 B)
a. Bilateral masses
b. Foci of necrosis
c. Signet ring cells
Reserved Copyrights - 2013

410

Pathology Questions

MCQs

d. Unilateral mass
1357. Which antibody or molecular finding is not typical of primary ovarian
mucinous carcinoma? (PIP 2013 B)
a. Expression of CDX2
b. Expression of Cytokeratin 7
c. Presence of BRCA1 or BRCA2 mutations
d. Presence of KRAS mutations
1358. Which of the following neoplasms is commonly found in association with
an ovarian mucinous tumor? (PIP 2013 B)
a. Brenner tumor
b. Leiomyoma
c. Mesonephric duct remnant
d. Serous cystadenoma
1359. All of these features are compatible with sclerosing extramedullary
hematopoietic tumor except: (PIP 2013 B)
a. Association with myeloproliferative neoplasms
b. Favorable clinical course
c. Multiple tumors
d. Splenomegaly
1360. All of these features help differentiate sclerosing extramedullary
hematopoietic tumor from myelolipoma except: (PIP 2013 B)
a. Atypical megakaryocytes
b. Non-adrenal location
c. Presence of anemia
d. Thick collagen bands
1361. Which tumor typically displays peri-nuclear dot-like cytokeratin 20
positivity? (PIP 2013 B)
a. Melanoma
Reserved Copyrights - 2013

411

Pathology Questions

MCQs

b. Merkel cell carcinoma


c. Mesothelioma
d. Prostate adenocarcinoma
e. Small cell carcinoma of the lung
1362. What is the definition of nodal micrometastasis of Merkel cell carcinoma?
(PIP 2013 B)
a. Clinically identifiable lymph node metastasis confirmed by
histological examination.
b. Clinically negative lymph node with histological evidence of
Merkel cell carcinoma.
c. Focus of metastatic carcinoma that can be identified only by
immunohistochemical studies.
d. Presence of fewer than 300 neoplastic cells in a lymph node.
e. Small than 2.0 cm focus of metastatic carcinoma.
1363. Which feature is typically not found in pseudomembranous colitis? (PIP
2013 B)
a. Hyalinization of the lamina propria
b. Inflammatory luminal debris
c. Neutrophilic inflammation
d. Signet ring cells
1364. Which is the most common predisposing factor leading to Clostridium
difficile colitis? (PIP 2013 B)
a. Advanced age
b. Feeding tube placement
c. Gastrointestinal surgery
d. Intravenous antibiotics
e. Oral antibiotics
1365. Which combination of findings meets WHO diagnostic criteria for PeutzJeghers syndrome? (PIP 2013 B)
Reserved Copyrights - 2013

412

Pathology Questions

MCQs

a. Three or more histologically confirmed Peutz-Jeghers polyps


b. Any

number

of

Peutz-Jeghers

polyps

and

characteristic

mucocutaneous pigmentation
c. Any number of Peutz-Jeghers polyps with a family history of
Peutz-Jeghers syndrome
d. Characteristic mucocutaneous pigmentation with a family history
of Peutz-Jeghers syndrome
e. All of the above
1366. What is the lifetime cancer risk in Peutz-Jeghers syndrome? (PIP 2013 B)
a. 5%
b. 18%
c. 53%
d. 93%
1367. Plasma cell villitis is most characteristic of: (PIP 2013 B)
a. Cytomegalovirus infection
b. Herpes simplex virus infection
c. Listeriosis
d. Toxoplasmosis
e. Varicella zoster virus infection
1368. Chronic intervillositis, organisms in the maternal red blood cells and
pigment deposition are typical features of: (PIP 2013 B)
a. Cytomegalovirus infection
b. Mycobacterium tuberculosis infection
c. Parvovirus B19 infection
d. Plasmodium falciparum infection
e. Treponema pallidum infection
1369. The pattern of placentitis typically associated with varicella zoster virus
infection is: (PIP 2013 B)
a. Acute chorioamnionitis
Reserved Copyrights - 2013

413

Pathology Questions

MCQs

b. Acute necrotizing funisitis


c. Acute villitis
d. Chronic intervillositis
e. Chronic villitis with multinucleate giant cells
1370. Which tumors can have staghorn vasculature?
a. Melanoma, carcinoma, lymphoma
b. Merkel cell carcinoma, Ewing, rhabdomyosarcoma
c. Mesothelioma
d. Prostate adenocarcinoma
e. Synovial sarcoma, mesenchymal chondrosarcoma, solitary fibrous
tumor
1371. What are the criteria for low-grade malignancy in solitary fibrous tumor?
a. Clinically identifiable lymph node metastasis confirmed by
imaging
b. Cytologically bland monotonous uniformity
c. Focus of lymph node metastasis that can be identified only by
CD34 stain
d. Large size (>5 cm), mitoses >4/10 hpf, necrosis
e. Peripheral location
1372. Which of the following statements about angiosarcoma of the breast
secondary to radiation therapy is false? (PIP 2013 B)
a. At presentation, tumors are often large and locally aggressive
b. It is more common in premenopausal women than in
postmenopausal women
c. It typically has a shorter latency period for development after
radiation therapy than other sites
d. Prognosis is related to the degree of differentiation of the tumor
e. Tumors may present with cutaneous changes similar to primary
angiosarcoma

Reserved Copyrights - 2013

414

Pathology Questions

MCQs

1373. Which of the following statements about angiosarcoma is false? (PIP 2013
B)
a. Angiosarcoma

may

show

well

differentiated

and

poorly

differentiated areas within the same tumor


b. Angiosarcoma should show positive staining for CD31, CD34 and
factor VIII, regardless of degree of differentiation
c. Classic angiosarcoma is characterized by infiltrative anastomosing
vascular channels with varying degrees of nuclear size and
pleomorphism
1374. Which of the following statements about epithelioid angiosarcoma is
false? (PIP 2013 B)
a. Carcinoma should be considered in the differential diagnosis for
epithelioid angiosarcoma due to the presence of sheets of
eipithelioid

cells

and

signet

ring

cells

that

can

mimic

intracytoplasmic lumina
b. Cytokeratin staining can be used to rule out epithelioid
angiosarcoma
c. Epithelioid angiosarcoma may be predominantly vascular or
predominantly epithelioid in morphology
d. Staining for CD34 can be used to rule out other cellular neoplasms
in the differential, including melanoma, mesothelioma, and
anaplastic lymphoma

Reserved Copyrights - 2013

415

Potrebbero piacerti anche